Greenhouse Gas warming doesn’t break the second law — Comments Overflow

Here are the first 500 comments on this thread (plus some orphans*). To make a new comment go to the original page.

Why greenhouse gas warming doesn’t break the second law of thermodynamics

*Orphan comments were recent ones attached to an older comment. When the older comment was moved to this thread they lost their position, and needed to be moved here. Sorry for any disruption to the flow. – Jo

7.8 out of 10 based on 4 ratings

546 comments to Greenhouse Gas warming doesn’t break the second law — Comments Overflow

  • #
    Brian G Valentine

    Second law does not say that the time-averaged heat flow will allow the cooler reservoir to become COOLER and another reservoir to become WARMER, not without the expenditure of work, anyway.

    Greenhouse gas warming requires the stratosphere to cool, and the troposphere to warm. Molecular conduction is still present.

    Conduction isn’t very large, but it acts over the time greenhouse gases are supposed to warm the troposphere (depending on the magnitude of the “climate sensitivity”).

    I’ve been through this a million times. I’ve convinced no one convinced the second law is violated and no one has convinced me it isn’t.

    Stalemate

    20

  • #
    Brian G Valentine

    Meant to say, “convinced no one convinced the second law is not violated” by greenhouse gas warming.

    20

  • #

    BS!

    First argument:

    1. ANY heat absorbed by the cold object that is coming from the warm object is no longer in the warm object.

    2. The warm object is colder by that amount.

    3. Any of the head from 1 that radiates back to the warm object simply REPLACES the prior lost heat.

    4. The warm object simply returns to its original temperature.

    Conclusion: back radiation does not and cannot warm a warm object beyond its original temperature. In fact it is always less because the “back” radiation is re-emitted spherically rather than totally reflected back to the warm body.

    Every other incident source of heat on the warm object is sourced from an EXTERNAL warmer object even if it is via the so called colder body.

    Second argument:

    1. If an energy level in a body is filled, it cannot absorb energy of the same level.

    2. The “colder” energy levels of a warmer object are filled and cannot absorb any more colder energy. The energy MUST be warmer.

    3. The “warmer” energy levels of a colder object are largely unpopulated and thus cannot be the source of warming energy energy WITHOUT an external source of warming energy.

    Thus a colder object cannot head a warmer object.

    The authors thesis is absurd.

    20

    • #
      Jose_X

      >> First argument:
      >> 1. ANY heat absorbed by the cold object that is coming from the warm object is no longer in the warm object.

      >> 4. The warm object simply returns to its original temperature.

      You left out such an important point. The sun does not stop adding energy to the earth+atmosphere system! Back radiation is a process which stalls energy in the system that was introduced much earlier in time by the sun. At the same time as this “old” energy is being circulated, the sun pumps more and more and more energy. This is the way an oven works. At equilibrium (with electricity providing the same steady current and dissipating heat matching that), you have a high temp held inside because of the high level of stored energy that accumulated slowly during the time that dissipation was less than energy in. Temperature is basically just average energy per molecule. You can have high temp at equilibrium if you can manage to keep a lot of energy in one region. Insulation is the key. Now, let me add, how do we achieve temperatures on earth even higher than the sun’s surface temp? We collect energy until we have all the amount we need packed into a small really hot space. Ever use reflective foil to cook something faster? That’s because the radiation that would have left, was redirected back. The total energy around the food is higher, rather than having most of that energy off in the air or space doing something else for someone else.

      >> “back” radiation is re-emitted spherically rather than totally reflected back to the warm body.

      That loss is why the temp on earth doesn’t keep growing to infinity; however, please don’t assume the sun stops adding energy to the system. If the sun did stop, then back radiation would be a way to cool the system down to a very low temperature, yes. With the sun on, however, back radiation both heats to a higher temp, and then provides enough cooling at that higher temp (at that high gradient) so that the system doesn’t reach a melting point.

      >> 3. The “warmer” energy levels of a colder object are largely unpopulated and thus cannot be the source of warming energy energy WITHOUT an external source of warming energy.

      Yes, that external source is called the sun. It works its magic 24x7x365. It’s not planning to close down for at least a billion years, I think.

      30

      • #
        Paul

        Yes, that external source is called the sun. It works its magic 24x7x365. It’s not planning to close down for at least a billion years, I think

        There is a fallacy in that statement.

        The sun may radiate its energy continuously, but it does not impact on the surface of the earth uniformly as if the sphere of the earth could be reduced to a two-dimensional circular surface, constantly radiated by the sun at half its intensity. No, the earth is a sphere and the surface radiated by the sun is constantly changing with the rotation of the earth, producing the diurnal experience of day and night with which we are all familiar. The performance of the atmosphere above both the radiated hemisphere and an unradiated hemisphere need to be explained rather than an atmosphere above some mythical zone that receives constant radiation at half the sun’s intensity!

        The atmosphere above the radiated hemisphere acts as an efficient cooler keeping the surface temperatures much lower than they would otherwise be as can be seen by comparing them with the temperatures on the radiated surface of the moon. There is nothing more certain than that ‘greenhouse’ gasses are not warming the surface in this phase, since the entire effect of the atmosphere is to cool the surface. And these processes are dominated by conduction, convection, and latent heat of vaporisation. Radiation, while it is minimally present at the surface, is of little significance being absorbed and converted into heat energy within a few tens of meters.

        The atmosphere above the hemisphere that is in the earth’s own shadow acts as an efficient insulator keeping the surface much warmer than it would otherwise be. It does this regardless of the presence of so-called ‘greenhouse gasses’, since the entire atmosphere is warm and continues to receive heat from the surface during the night. The surface of the moon, in contrast, without any atmosphere goes from intense heat to near absolute zero just stepping into the shade even on the irradiated surface.

        The role that atmospheric carbon dioxide plays in the above process is minimal, being entirely negated by convection during the time of irradiation by the sun. At night, as the atmosphere radiates heat out to space, with no augmentation from the sun, the rate at which it does so is determined by the radiative properties of the now still air. However it is mainly water vapour in the atmosphere that is absorbing radiation, contributing up to 95% of the so-called ‘greenhouse’ effect, with atmospheric carbon dioxide and other trace gasses contributing the remaining 5% or so.

        Since the amount of atmospheric water vapour is determined by independent air temperatures, cloud formation, etc, there is virtually no role left for atmospheric carbon dioxide to impact life on the surface of the earth except in its essential role as plant food, for which it has no substitute.

        Basically, the more there of it there is up there the better for everyone and everything it will be.

        Paul

        10

        • #
          Jose_X

          >> The sun may radiate its energy continuously, but it does not impact on the surface of the earth uniformly

          Conservation of energy.

          Regardless of how the earth is irradiated, the energy is arriving and it has to go somewhere… and it does.. most of the earth is hospitable, day or night. The average value has meaning because it allows us to compare it to radiation lost back out into space.

          >> There is nothing more certain than that ‘greenhouse’ gasses are not warming the surface in this phase, since the entire effect of the atmosphere is to cool the surface

          The atmosphere’s ability to cool depends on its temperature. GHG keep the temperature higher than it otherwise would be. GHG interfere with what would otherwise be a greater flow of radiation heat into space. Nothing you said about convection is inconsistent with this. We get both effects.

          >> Radiation, while it is minimally present at the surface

          Did you make that up or are you using data to back it?

          Try these graphs showing “downward longwave radiation”
          http://scienceofdoom.files.wordpress.com/2010/07/dlr-many-stations-wild-2001-499px.png?w=500
          or a few graphs which I looked up for Point Reyes, California, August 11, 12, 13 2005
          http://www.archive.arm.gov/quicklooks/2005/pye/pyeskyrad60sM1.b1/pyeskyrad60sM1.b1.200508/pyeskyrad60sM1.b1.20050813.000000.png
          http://www.archive.arm.gov/quicklooks/2005/pye/pyeskyrad60sM1.b1/pyeskyrad60sM1.b1.200508/pyeskyrad60sM1.b1.20050812.000000.png
          http://www.archive.arm.gov/quicklooks/2005/pye/pyeskyrad60sM1.b1/pyeskyrad60sM1.b1.200508/pyeskyrad60sM1.b1.20050813.000000.png
          (to see more days, just modify the hyperlink.. they have records for 2005 and 2006 but maybe not afterward).
          I go into detail trying to explain some of these (or related) graphs here: http://bbickmore.wordpress.com/2011/07/26/just-put-the-model-down-roy/#comment-8132

          You will note that values around 350 for back radiation (day or night) is not negligible when you consider that the sun provides not even 1000 around this part of the earth in the peak daytime hours.

          >> It does this regardless of the presence of so-called ‘greenhouse gasses’, since the entire atmosphere is warm and continues to receive heat from the surface during the night. The surface of the moon, in contrast…

          This isn’t inconsistent with ghg effect. GHG make the effect larger. You aren’t arguing against ghg.

          >> The role that atmospheric carbon dioxide plays in the above process is minimal

          You aren’t backing this up with math… well, I suppose you are invoking the Postma paper (I started looking at it).

          >> However it is mainly water vapour in the atmosphere that is absorbing radiation, contributing up to 95% of the so-called ‘greenhouse’ effect, with atmospheric carbon dioxide and other trace gasses contributing the remaining 5% or so

          Can you cite reference for this? I have finished reading (but not totally analyzing) a paper by someone with a low opinion of mainstream climate science who nevertheless appears to agree (and uses that in his model) that CO2 contribution to ghg effect is about 1/4 of total (at least during the day). See http://en.wikipedia.org/wiki/Greenhouse_gas for some values. The low end they mention is not a reflection of how much is absorbed, but includes overlap.

          >> Since the amount of atmospheric water vapour is determined by independent air temperatures

          You assume the independence between greenhouse gases and temperature. You didn’t offer evidence that there is no connection although I will read the Postma paper. There are many papers and physics textbooks you can read that argue ghg is significant.

          10

  • #
    Brian G Valentine

    The worst argument I’ve heard, is “the second law doesn’t apply because the Earth isn’t an isolated system.”

    I also regret that there are a number of very bad thermo textbooks out these that could easily be misinterpreted to lead to that conclusion as well.

    10

  • #
    RJ

    This article is wrong. A cold object can not heat a warmer one. Read a book like slaying the climate dragon if you want to know why

    And statements like this are silly

    however now the warmer walls of the room radiate more energy back to you than did the cold outside

    The air inside is warmer so the rate of cooling is less. The room does not warm you. Just like clothes do not warm someone. Clothes has no energy so just reduce the cooling rate. The warmth comes form the body as earth’s warmth comes from the Sun not the colder atmosphere.

    10

    • #
      Jose_X

      >> The air inside is warmer so the rate of cooling is less.

      This statement is correct. The rate is less inside because of a lower temp gradient (the higher the temp difference, the greater the rate of cooling); however, that law does not contradict the quote you provided of walls warming you.

      Your rate observation comes from a macro theory. The “walls heating me back” comes from a micro theory, which itself can explain the macro observation (quantitatively).

      Energy is redistributed in the room by already hot gas molecules moving around (diffusion and advection), by molecules “hitting” each other, and by molecules releasing energy packets (photons) which fly off and are absorbed by nearby molecules. These are micro effects that add up to produce the “hot-to-cold” experiences we are familiar with.

      Let’s consider how cold can warm hot objects at the microscopic level, at the same time as hot objects are warming cold objects at a greater net rate.

      With radiation, we do have every solid object emitting lots of photons, with hotter objects simply emitting more of them than colder objects.

      With convection, we do have hot molecules moving to where cold molecules used to be and also cold molecules moving to where hot molecules used to be while losing or gaining energy along the way (essentially working towards equilibrium, although a gradient might be preserved if we have a hot reservoir and cold reservoir that essentially preserve their temperatures).

      With conduction and whenever “contact” occurs among molecules, we also have microscopic units of energy moving about from hot to cold and cold to hot.

      To get an idea of the cold-to-hot effect from a momentary impact, consider this analogy. A billiard ball strikes another ball hard and in a line. The ball that was moving transfers most of its energy (and momentum) to the ball that was static. Afterward, the initially “hot” moving ball turns cold (as it stops on the table) while the other ball leaves with more energy than the stopped ball has.

      To get an idea of a cold-to-hot effect from steady contact (as we might “see” within in a solid object), consider an analogy of many balls lying on the floor of a room and each attached by springs to various neighbor balls. If we film the chaotic motion of the balls pulling at each other, we will find cases where a ball was moving slowly and then came to a standstill as it was pressed from the relevant opposite sides in just the right way. This short term effect would be of a cold ball getting even colder with a net gain in heat by its two+ neighbors who were already hotter than it.

      To get an idea of cold-to-hot effect from radiation, consider an analogy of many players throwing super bouncy balls at each other within a small room. “Hotter” players are the ones that throw faster. At any given 1 second of time, however, we might see a cold player not being hit while a hot player gets hit by a few balls (with one or more perhaps having come from a cold player). Of course, over the long run, the cold player will get hit more times than it throws balls (assuming every ball thrown eventually hits a player).

      In short, at the microscopic level, hot warms cold and cold warms hot, with hot warming cold at a greater rate. On a macro scale, where many interactions occur, we experience just the net statistical result of all of the microscopic reactions; we experience hot warming cold.

      It’s important to recognize that the micro effects exist, since micro effects are needed to explain different physical phenomenon, and we don’t want to get caught in the trap of thinking cold warming hot is always impossible.

      10

  • #
    RJ

    Another point

    The highest temperature on any day can never be increased by GHG backradiation from the atmosphere.

    Otherwise new energy would have to be created by CO2 (or water vapour) which is impossible. Energy would have to leave earth and more would have to come back with a higher energy rating to further heat earth.

    GHG’s especially water vapour do reduce the rate of cooling so can increase the average temperatures over a day or period. But that’s all. The cooling rate is reduce nothing more.

    10

    • #
      Jose_X

      The highest temperature of a given day can be increased by ghg.

      New energy is not created. Energy is recycled. The sun pours an endless stream of energy. Any time we can corral some more of it on earth before it flies off into outer space, we increase the energy density somewhere on earth.

      10

  • #
    RJ

    Imagine three blocks of metal side by side. They are 11°C, 10°C, and 9°C. If heat never flows from cooler blocks to warmer blocks, think about what happens to the photons coming off the atoms in the middle of the medium temperature block between the other two. Can you imagine all those photons going “right“, and not ever going “left”, because they “know” that way is towards a cooler block? (How would they?!)

    Regarding this comment. If a 11 degree block is put beside a 9 degree block. In no way can the 11 blocks temperature rise about 11 degrees. If the heat source was turned off the 11 block might cool more slowly.

    However the 11 degree block could further warm the 9 degree block above 9 degrees. Warm can heat colder objects but colder can not heat warmer ones.

    10

  • #
    Brian G Valentine

    Technically, strictly, greenhouse gases don’t “warm” the planet, but they slow the cooling

    Careful, we have the heat flux

    q = (sigma)[(eps1)T1**4 – (eps2)T2**4]

    as the heat flux between the lower atmosphere at temperature T1 and upper atmosphere at temperature T2,

    if T1 increases and T2 decreases, then the heat transfer rate q has to increase

    10

  • #

    I’ve added in another paragraph, and a link to the Postma paper in paragraph 1.

    People are being caught by semantics. Technically, strictly, greenhouse gases don’t “warm” the planet (as in, they don’t supply additional heat energy) but they slow the cooling, which for all pragmatic purposes leaves the planet warmer that it would have been without them. It’s a bit like saying a blanket doesn’t warm you in bed. Sure, it’s got no internal heat source, and it won’t add any heat energy that you didn’t already have, but you sure feel cold without one.

    Instead of calling it “global warming”, I guess they could have called it “less-global-cooling”. I can’t see it catching on.

    10

    • #
      Jose_X

      “Warming” is used naturally to communicate that the temperature is going up. [Yes, something it’s the easy explanation 😀 ]

      There is an actual heating effect by the atmosphere much as a cold mirror can help further heat a simmering piece of meat by redirecting and concentrating radiation back towards it. http://www.youtube.com/watch?v=iR60VbMh8VM

      The atmosphere doesn’t reflect radiation back, but it’s a similar net result.

      It’s also accurate to say the atmosphere slows down cooling. But this slowdown at ordinary (no-ghg) temperatures leads to higher temperatures, which then speed up the rate of cooling, until a balance is established at a higher temperature.

      10

      • #
        Paul

        It’s also accurate to say the atmosphere slows down cooling. But this slowdown at ordinary (no-ghg) temperatures leads to higher temperatures, which then speed up the rate of cooling, until a balance is established at a higher temperature.

        No, it is not accurate to say this because it assumes that the earth is the equivalent to a two-dimentional circle receiving constant radiation at half the sun’s actual value!

        As I have posted elsewhere on this thread, during the day the temperature of the air near the surface is dependent on conduction, convection and the latent heat of vaporization. Radiation plays virtually no part in the process under the sun’s radiation. It is one of cooling, pure and simple.

        Then at night, when the radiative properties of still air do come into play, there is just no way that equilibrium is at a higher temperature. The only possible effect, by night, is the slowing of the loss of heat from the surface. Therefore there is no such thing as a higher equilibrium temperature.

        How much the surface temperatures might be affected by an increase in atmospheric carbon dioxide, given these facts, is obviously vanishingly small. There can be no possible ‘multiplier effect’ due to ‘positive feed-backs’ and the possibility of catastrophic global warming is absolutely zero. QED

        Paul

        10

        • #
          Jose_X

          >> No, it is not accurate to say this because it assumes that the earth is the equivalent to a two-dimentional circle receiving constant radiation at half the sun’s actual value!

          You don’t need that assumption to explain greenhouse effect. The greenhouse effect doesn’t require 24 hours of sunshine. The average value you are criticizing is perhaps for its use in some simple models (and that’s potentially a good criticism).

          To use an oven analogy, you can turn an oven on and off and it won’t change the fact that the heat inside dissipates slowly through the insulated walls. We can analyze that oven by looking at an average value or not, but the effect from insulation doesn’t stop existing. While I think you don’t credit ghg with a useful purpose in warming the planet further, I don’t see why you would think that those who argue for a greenhouse effect require the assumption that the earth has a constant radiation at average intensity.

          >> As I have posted elsewhere on this thread

          I already replied to your other comment making similar arguments. I provided graphs of radiation measurements to show they are not negligible as you stated there.

          Your convection and other arguments aren’t inconsistent with ghg, just like having a belly ache doesn’t preclude you from also having a headache so that overall you are worse off.

          10

  • #
    RJ

    Jo

    But the wording is important. There are people who do think the planet can be warmed above what the sun would warm the planet. Not just reducing the rate of cooling but also further heating so that the maximum possible temperature is higher than just due to the sun.

    Or that ice warms people in a igloo by backradiation for example. This GHG backradiation theory is on shakey ground yet people like Monckton still seem to accept it without question.

    Most of the increase in average temperatures is likely caused by convection and conduction (and advection) not GHG backradiation. At least the support for GHG backradiation theory is not as strong now as it was.

    10

    • #
      KinkyKeith

      Hi RJ

      I’m with you on that comment about “This GHG back-radiation theory” being unreal.

      No such animal. Btw Lord Monckton, who I saw in Newcastle, is a mathematician.

      The accumulation of energy would not be possible if there was no atmosphere.

      All of the ground energy would be quickly radiated to space.

      I agree that the atmospheric gases acquire energy from the emitted Ground IR and delay its exit to space.

      The concentration of energy in the atmosphere makes life more comfortable for us and stops the ground losing more energy overnight until a refill arrives by solar post next morning.

      10

  • #
    RJ

    And I thought the Postma paper was excellent.

    Although a number of sceptics (Luke warmers)don’t agree. As I found out on a wattsup thread when I posted a section from it.

    10

  • #
    RJ

    And BTW. This website is brilliant. I have used many articles and graphs from it in my battles with warmists.

    10

  • #
    Bryan

    I think some people are mixing up electromagnetic radiation with Heat.
    Physics text books make clear the thermodynamic meaning of heat.
    ………………………………………………
    From University Physics by Harris Benson page 382
    Modern definition of Heat
    Heat is energy transferred between two bodies as a consequence of a difference in temperature between them.
    ……………………………………………..
    University Physics Young and Freedman

    Energy transfer that takes place sole because of a temperature difference is called heat flow or heat flow transfer and energy transferred in this way is called heat page 470

    Heat always flows from a hot body to a cooler body never the reverse. pg 559
    …………………………………………………………………………………
    Heat is the net energy difference between the two photon streams.
    Heat also has the thermodynamic capacity to do work that is to change into another energy form like the moving pistons on a steam engine.
    The word heat also has a colloquial meaning which varies “like a curry has a lot of heat”.
    But in a discussion on science we must use the thermodynamic meaning of heat.
    Any physics student who said heat can travel spontaneously from a cold surface to a warmer surface would fail their exam.

    10

    • #
      Jose_X

      >> Any physics student who said heat can travel spontaneously from a cold surface to a warmer surface would fail their exam.

      Thermodynamics is a macroscopic theory. Macroscopically, we observe the net result of many microscopic reactions.

      At the microscopic level “colder” molecules can transfer energy to “warmer” ones during any given reaction. We certainly don’t observe these reactions with a thermometer outside, but knowing how molecules can exchange energy is important to explaining some complex macroscopic phenomenon.

      Note that the Second Law of Thermodynamics is about total entropy of the universe.. not about heat or temperature.

      Note that a cold mirror can “heat up” a hotdog. The sun provides enough energy for us to cook almost anything, including our atmosphere (and us) if the conditions were to be set up just right.

      10

  • #
    Brian G Valentine

    There is no way to make this greenhouse effect from gas in the atmosphere work. This was pointed out by the American physicist Robert W Wood in 1913 and it continues to be the folklore to this day.

    Only because people want it to work; and if it can, then build a model of it and sell your perpetual motion machine.

    10

  • #
    Nullius in Verba

    The mechanism proposed doesn’t conflict with the second law, but it’s still not quite correct because it ignores convection. If extra heat were to build up near the surface, then because hot air rises it would float away. Temperatures in the troposphere are limited by convection.

    There’s another illustrative example – a pond full of water. Suppose the bottom of the pond is painted black. The sun shining down passes through the water and is absorbed at the bottom. The atoms re-radiate the infrared, but being covered with water the radiation is almost immediately absorbed. Water is such a powerful absorber, that virtually none can penetrate more than a few millimetres. The bottom few millimetres radiate it back, of course, in all directions – including up. So the next layer above does the same, only it is only getting half the energy from the layer below. The next level above gets half the energy it radiates, and so on.

    Since the top surface of the pond must radiate all the energy the pond absorbs, and as the radiation *doubles* every few millimetres you descend, the bottom of the pond a couple of metres down must be very hot indeed! If the temperature distribution of water in ponds was determined by radiative balance, you could never get more than a few centimetres depth before the water boiled!

    Radiation certainly *exists* in water – you can measure it – but it doesn’t decide the temperature distribution because there are other mechanisms for heat transfer besides radiation. Both convection and conduction keep the water cool. With conduction alone, it would take a few metres of water before it boiled. Solar ponds work this way, and can get up to 80-90 C at the bottom. Only with convection can we explain the more typical temperatures in water.

    Greenhouse gases in the atmosphere *do* very definitely have an effect on the surface temperature, making it warmer than it otherwise would be. But they do so by increasing the average altitude of emission to space, which is the level that settles at the effective radiative temperature for the given heat input. The reason the surface is warmer than this level is that convection’s ability to carry heat away is limited by the adiabatic lapse rate. Back-radiation exists, but it does not determine the surface temperature. It’s effect is cancelled out.

    See also here. http://judithcurry.com/2010/11/30/physics-of-the-atmospheric-greenhouse-effect/#comment-16901

    10

  • #

    There is a third argument.

    The so called Greenhouse Effect, even if it existed, CANNOT break the second law of thermodynamics nor any other law that rules the physical universe. That is a totally sufficient argument within itself.

    The false notion of a Greenhouse Effect arises out of the fact that the earth’s surface is observed to be higher that it is expected to be. Simply coming up with a complicated mechanism that can be adjusted to fit the data (alternatively adjusting the data to fit the expectation) proves absolutely nothing about the real world.

    The discrepancy is to be resolved by accepting the facts and identifying the underlying causal physical phenomenon and not random expectations. The expectation is nothing but fantasy at best and fraud at worst.

    It is only the slight of hand of equivocated terms, evaded context, and ignored underlying mechanism that allows some people to pretend otherwise. Especially to pretend they don’t violate the laws and then to proceed to present violent and numerous violations of them in opaque language and irrelevant mathematics.

    10

  • #
    Lynn Clark

    I am now convinced that the so-called “atmospheric greenhouse effect” does not actually exist as Harry Dale Huffman shows in his concise post, Venus: No Greenhouse Effect. Postma also briefly discusses this on pages 22-23 of his paper. The temperature at any given pressure level of any planet with an atmosphere is largely, if not entirely, determined by the adiabatic lapse rate. Any effect that CO2 has on temperature is minuscule (as you stated) and is completely overwhelmed by other factors.

    10

  • #
    Brian G Valentine

    I will say, that “denying” (or refuting) the “atmospheric greenhouse effect” gives some global warmers the opportunity to identify “crank” denialists (“He doesn’t even believe the greenhouse effect exists ha ha ha”), only because of a highly simplified picture that is evoked to refute the notion

    (“CO2 and water vapor are greenhouse gases, right? So their presence causes a greenhouse effect, QED”)

    CO2 and water vapor certainly absorb in the IR, but this property cannot be used to transfer heat from a cooler reservoir to a warmer one without work expended.

    11

  • #
    Nullius in Verba

    “but this property cannot be used to transfer heat from a cooler reservoir to a warmer one without work expended.”

    *Net* heat transfer, yes.

    (And incidentally, if differential heating causes convection to drive air around the Hadley cells, this can do work. It can drive the successive compression and expansion of a gas moving around a circuit, like in a refrigerator.)

    10

  • #
    GorkMork

    The example with the person indoor/outdoors in the above article is wrong.

    Second law of thermodynamics sez that heat always moves from the warmer body into the cooler one until they reach an equilibrium.

    Put a person in a cool atmosphere and his bodyheat is constantly carried away by the cold gas particles hitting him.
    If the bodyheat gets removed faster than the body can replenish it by burning energy, then the person get cooled down.

    Put a person in a warmer atmosphere and his generated bodyheat can easily keep with the slower heatloss.

    The walls of your house does not keep you warm because they reflect radiation, they keep you warm because they keep the gases in your house from wafting off and taking your precious, precious heat with them.

    Also , space is NOT cold. A temperature is an property only a physical object can possess. A particle with a lot of kinetic energy has a lot of heat, a particle with very little energy has very little heat.
    If there is no particle there is no energy and therefore no heat and no temperature.

    Heat can move by conduction, convection, or radiation.
    In a vacuum heat can only move by radiation.
    This means that an object in a vacuum is in nearly perfect insulation and looses heat very, very slowly.

    There is absolutely no way our planet can loose heat any slower unless someone rewrites the law of physics.
    Of course plenty of people seems to be trying just that…

    10

  • #
    Brian G Valentine

    The stratosphere is certainly stratified, and so any property related to convection is minimal – leaving molecular conduction pretty much the only means to transfer heat between the stratosphere and troposphere.

    I hope you don’t think I’m trying to refute phenomena related to natural convection or instability related to buoyancy.

    I’ve been around this carousel too many times already, time to hop off. Gerhard Gerlich eventually hopped off noting that a “global climate” has no physical meaning to begin with. He’s right about that.

    10

  • #
    RJ

    MH

    I think Dr Spencer has written fiction in this article. He has not done this experiment it seems but is guessing.

    Here’s one of the comment on this article. Hopefully some of the other posters above will comment on this experiment as well

    Indeed, Clausius was a smart guy. He said it all in a single sentence. :

    “Again as regards the ordinary radiation of heat, it is of course well known that not only do bodies radiate to cold, but also that cold bodies radiate to hot; nevertheless the general result of this simultaneous double exchange of heat always consist, as is established by experience, in an increase of the heat in the colder body at the expense of the hotter.”
    Rudolf Clausius “ The Mechanical Theory of Heat “ 1879

    10

  • #
    Nullius in Verba

    “Put a person in a warmer atmosphere and his generated bodyheat can easily keep with the slower heatloss.”

    Warmer than what? Body temperature? Or outside?

    The same argument applies – instead of between the body and the walls, it is between the body and the air. If you put a person at 37 C in air at 20 C, he will feel warmer than a person in air at -40 C. But (net) heat does not flow from the air at 20 C to the warmer body at 37 C, so where does the body’s feeling of warmth in a warm room come from?

    10

  • #
    michael hammer

    I have just been reading through the 23 replies since this article was posted and it seems there is still a common misconception. Before I start to explain further let me state I am very much a sceptic, I do not believe that increasing CO2 will warm the surface to a dangerous or even a particularly noticeable level and I have posted on that theme several times in the past. However, as a scientist I am concerned to get the science right. There are excellent scientific reasons why the theory of CAGW is wrong, we don’t need to use false arguments and leave ourselves open to justified criticism. OK – on with the further explanation;

    The Earth’s surface receives energy from the sun. This energy causes the Earth’s surface temperature to rise. It will continue to rise until equilibrium is reached. That means until the earth is losing as much energy as it gains. Since the Earth as a whole is a body floating in space it cannot lose energy by conduction and convection – both those require a material substance to transport the heat. That means the equilibrium is established by radiative energy gain from the sun balanced by radiative energy loss to space.

    Now changing the Earth’s temperature means upsetting that equilibrium. Clearly if the sun radiates more energy to us energy input rises causing the temperature to rise. However the temperature will also rise if we in some way reduce the energy loss to space.

    All objects which are warmer than absolute zero (and all are) radiate energy in all directions. These objects are not intelligent, they have no way of knowing what other objects are in their immediate vicinity and modifying their radiation pattern accordingly, they simply radiate in all directions. The amount of energy radiated per square meter of surface depends ONLY on the temperature of the object and on the surface emissivity which is a property of the surface itself. A black surface has a high emissivity (close to 1) a mirror surface has a low emissivity (close to 0). If you cannot accept this I am sorry but it is science that has been well known and more than adequately proven for many many decades. Much of our functioning technology is based on this being true.

    If this is the case, how come a hot object, say a pot of custard just off the stove, cools faster if put in the fridge compared to leaving it out on the bench? If the energy radiated by the object does not depend on the surroundings then the rate of cooling should be the same in both cases. The reason it isn’t is because the surroundings are also above absolute zero and therefore also radiate energy in all directions. Some of that energy strikes the custard and warms it. Of course the custard is hotter than the surroundings so it radiate more energy than it receives and thus still cools but the warmer the surroundings the more energy the custard receives and thus the smaller the difference hence the slower it cools.

    In the case of the Earth, without an atmosphere it would be radiating to space which is at a temperature of -269C. If the atmosphere is transparent (which is another way of saying it does not absorb any of this radiation ie: it has zero absorptivity and thus also zero emissivity) it does not impede this radiation. On the other hand, if the atmosphere is opaque at some wavelengths (ie: it has a high absorptivity and thus also a high emissivity) it means the atmosphere absorbs the surface radiation at those wavelengths and in turn radiates back to the surface at those wavelengths at a level dependent on the temperature of the atmosphere. Since the atmosphere is much warmer than outer space the back radiation is much more which would reduce the net heat loss. In response the surface will warm (increasing the outward radiation per unit area) unitl a new equilibrium is reached.

    CO2 simply makes the atmosphere opaque to radiation at 15 microns and the net impact of that is to reduce the energy loss to space at that wavelength.

    10

  • #
    michael hammer

    comment 3.

    Your entire argument is based on a misrepresentation of what the word warming means. Of course a cold object cannot make a warmer object still warmer in the absence of any other heat sources. But in the case of the Earth there is a major heat source called the sun. The “warming” comes about becuase the outflow of energy is impeded. In the same way if a dam filling with water from a river and at the same time leaking water out through the dam wall eventually the water level will rech an equilibrium where inflow equals rate of leakage. If the river stops flowing I cannot riase the water level in the dam without using an active pump but while the river is flowing I can certainly raise the level in the lake simply by plugging one of the holes. That is what CO2 does, it partially plugs the leak of energy to space at 15 microns.

    10

  • #
    michael hammer

    comment 5 RJ; you say “The air inside is warmer so the rate of cooling is less.” Exactly, couldn’t have said it better myself and for the Earth given there is continuous heat flow into the planet from the sun the CO2 makes the rate of cooling less. So what happens if the rate of energy intput does not change but the rate of cooling is less?

    10

  • #
    Bryan

    michael hammer says

    ….”CO2 simply makes the atmosphere opaque to radiation at 15 microns and the net impact of that is to reduce the energy loss to space at that wavelength.”….
    Yes I think most would agree with that.
    The radiation from the colder surface in this case atmospheric CO2, is like the radiative component of insulation.
    It slows down the heat loss from the warmer Earth surface.
    Backradiation is no different to backconduction or backconvection although the later two are rarely mentioned.

    As RJ says above

    Indeed, Clausius was a smart guy. He said it all in a single sentence. :

    “Again as regards the ordinary radiation of heat, it is of course well known that not only do bodies radiate to cold, but also that cold bodies radiate to hot; nevertheless the general result of this simultaneous double exchange of heat always consist, as is established by experience, in an increase of the heat in the colder body at the expense of the hotter.”
    Rudolf Clausius “ The Mechanical Theory of Heat “ 1879

    This is now better known as the Second Law of Thermodynamics.

    10

  • #
    Siliggy

    Brave post Jo.
    If we consider that without the sun the earth would cool to a very low temperature, then the system can be compared to a tank of water with the heat flowing in raising the water level and the heat flowing out causing the level to fall. If CO2 is like a tap in the outflow from this tank then closing off the tap would cause the level in the tank to increase. The thing is that this tank has other taps and leaks and other variations on the inflow(albedo etc)and even other sources of inflow(radioactive decay metorites cosmic radiation etc etc)! As the tank fills the flow out by all these other paths will increase. Also CO2 will block incoming radiation at it’s absorbtion frequencies. The incoming radiation at those frequencies is lower but NOT zero.
    To test any theory i like to check the extremes. If it works at the very hot and very cold extremes it is likely to work the whole way.
    So what happens if CO2 reaches the point that 100% of heat radiated from the surface at the CO2 absobtion frequencies is reabsorbed? Answer: Heat leaves the planet at other frequencies.
    So the theory does not work at that extreme end.
    Regardless of the 2nd law postulations heat is not trapped because it has another way to escape. It can only be delayed in the rate of escape. The only effect of CO2 is not a change in the rate of escape. If the amount of CO2 in the atmosphere increases the absorbtion and radiation at a distance from the surface then the radiating surface area of the planet at these frequencies has been increased.
    Now the other extreme. If CO2 in the atmosphere was zero it cannot cause cooling below that point. So CO2 can only have a small effect over a narrow range and humans have only changed this small effect in a very small way. Sadly not enough to prevent the coming cooling.

    10

  • #
    michael hammer

    comment 8. The second term of your equation is exactly the back radiation. However you assume T2 changes due to changes in the temperature of the upper atmosphere. That is not correct. To take an extreme case, what is happening is that at 15 microns the atmosphere is changing from transparent to opaque so T2 changes from the temperature of outer space -269C to the temperature of the upper atmosphere -53C. Thats a pretty big change. I said above “to take an extreme case”. Thats becuase there is anyway far more than enough CO2 in the atmosphere to make the atmosphere completley opaque at 15 microns. Adding more CO2 simply makes the wavelength range over which the atmosphere is opaque (about 14.5 microns to 15.5 microns) very slightly greater.

    10

  • #
    michael hammer

    comment 28 Your explanation of a tank of water is spot on. Your point that there are many leaks from the tank of which the emission at 15 microns is only 1 is also completley correct. To follow your analogy however if one blocks one of the leaks (by turning off the CO2 tap) then the rate of leakage though the other leaks must slightly increase to maintain balance. For that to happen the level of water in the tank must rise slightly so as to increase the pressure on the other leaks so as to increase their flow slightly.

    By the way, as an aside CO2 does not stop radiation to space at 15 microns. Certainly all surface emission to space at this wavelength is blocked (that would happen even at less than 0.5 ppm of CO2 in the atmosphere) but it is replaced by radiation to space from the top of the CO2 column. That means in effect the radiation temperature to space at 15 microns is reduced from about 288 kelvin to about 220 kelvin and that is very very clearly seen in Earth’s radiation pattern as seen from space (eg: the Nimbus data).

    10

  • #
    michael hammer

    comment 22 As Bryan already said in comment 22 This is now better known as the Second Law of Thermodynamics.

    10

  • #
    michael hammer

    comment 20 GorkMork you say

    “Heat can move by conduction, convection, or radiation.
    In a vacuum heat can only move by radiation.
    This means that an object in a vacuum is in nearly perfect insulation and looses heat very, very slowly.”

    Your first line correctly states that there are 3 mechanisms of heat transfer
    Your second line correctly states that in a vacuum only the third is possible
    Your third line implies that radiation does not occur and is rubbish. An object in a vacuum can lose heat very rapidly through radiation. The Earth is a ball completely surrounded by vacuum and the entire energy imput from the sun is lost by radiation. For that matter. the energy from the sun reaches us via radiation. If you stand near a fire almost all the warming you feel is through radiative heat transfer. You don’t believe me? Try putting a thin sheet of metal between you and the fire or even sheet of paper and note the immediate impact. Do you really think a sheet of paper is impeding the conduction of heat?

    10

  • #
    Binny

    Bottom line greenhouse gases don’t increase warming,they slow cooling.
    Somehow it’s hard to make that statement sound alarming.

    10

  • #
    John Watt

    Good to see the discussion back on the physics fundamentals. Maybe now we are ready to tackle Nicol’s explanation of the Greenhouse Effect and the his dismantling of the fallacy that more CO2 means higher temperatures.

    10

  • #
    Brian G Valentine

    If I make the statement, “Heat transfer rates increase when the temperature (difference or gradient) increases” and make no reference to “greenhouse effect,” you would simply nod your head in agreement.

    When the subject “greenhouse effect” is discussed, then hand-waving and circular arguments are invoked to derive a conclusion that is desired and not physically possible.

    There is something in the mind that wants “atmospheric greenhouse effect” present in the physical world and casuistry is invoked if need be to validate this need.

    Sorry!

    10

  • #
    Binny

    michael hammer:@30
    I guess at the end of the day, the $80 billion question is how big is that CO2 tap in relation to all the other taps. And how much will the level of the tank raise if that CO2 tap is turned off.
    Sceptics say not much, alarmists say the tank will overflow.

    10

  • #
    Louis Hissink

    Argue all you want, it’s all semantics and rhetoric and irrelevant – so where is the physical experiment validating the idea that a cooler radiating gas can raise the thermal state of an object hotter than it?

    10

  • #

    yeah, when you are outside it is your radiative energy loss that is making you lose energy./sarc… Has absolutely nothing to do with the physical temperature of the air around you. Nope. Nice try again, I just wonder why on earth all you good people eventually get hooked into these sorts of things.

    Yes, the greenhouse gases cause some level of increase in the temperature of the earth. But it is not due to the radiation they emit. It is due to the radiation they absorb. They absorb energy, become more energetic, bounce into other air molecules, and then lose the energetic boost when they re-emit the radiation.

    Find a place where you can solidly freeze the walls or warm them up to 100degF, while simultaneously keeping the physical temperature in the middle at precisely one set temperature. Perhaps using wind walls to prevent the physical transfer of energy from one side to the other, just leaving radiative forcing. Measure the temperature inside a radiation blocking sphere and outside. Tell me the results of this with appropriate controls, and I will let you know if I believe your radiate theory.

    10

  • #
    michael hammer

    post 36 Binny; Your comment is absolutely spot on. My calculations suggest that in the absence of any feedbacks doubling CO2 would increase the temperature by about 1C. Other values I have seen range from about 0.8C to 1.2C which is entirely consistent. Most warmist and sceptical scientists seem to agree on this range. Where all the controversy comes in is the nature of the feedbacks. Warmists claim massive positive feedback raising that figure to 3C or even 6C. Sceptical sceintists claim negative feedback reducing the total rise to less than 0.5C (an irrelevant and largely unmreasurable result). I have looked at the basis of the warmists claims of positive feedback and there are massive contradictions and conflicts in their arguments. (lets start with the missing hotspot in the upper tropoosphere). It is also worth noting that essentially all natural systems whcih have remained “stable” over long periods of time (and the climate certanly has remained stable despite its gentle fluctuations with changing forcings) exhibit strong negative feedback.

    10

  • #
    MattB

    Credit to you Jo for throwing this one in the mix.

    10

  • #
    michael hammer

    comment 37 Louis I am amazed that you cliam the underlying science is “all semantics and rhetoric and irrelevant”. It is the entire issue.

    10

  • #
    MattB

    p.s. the fact that AGW theory does not contravene the 2nd law of thermodynamics is not a reason to think AGW theory is correct… but certainly I feel that arguments based on AGW breaking the 2nd law are fairly desperate (or potentially groundbreaking I guess).

    10

  • #
    Bob Massey

    I always wondered why my Coffee got colder quicker in colder weather ..thanks All

    10

  • #
    MattB

    Astoneril in #38… seriously … WTF?

    10

  • #
    Kevin

    Jo Nova wrote;

    “but they slow the cooling, which for all pragmatic purposes leaves the planet warmer that it would have been without them.”

    Well, yes and no. Since the energy input into any part of the system varies from blazing sunlight to moon and star light every 24 hours any delay caused by “slowing the cooling” would have to amount to an appreciable portion of these 24 hours to “leave the planet warmer”.

    So let’s do a quick calculation. The time for IR radiation to travel from the surface of the Earth to the top of the atmosphere; a few (perhaps as much as ten) milliseconds. Light (which IR radiation is) travels at a wicked high speed (~3X10^8 meters per second). So if this IR radiation is absorbed/reflected/absorbed/reflected many many times by “greenhouse gases” it might (I repeat MIGHT) be delayed by a few hundred milliseconds. Please note that after each “BOUNCE” back towards the surface of the Earth the IR radiation loses something more than 50% of the total energy present (this is fixed by the geometry and is not determined by any computer model).

    So how many milliseconds are there in 24 hours you might ask. There are exactly 86.4 million milliseconds in each day.

    So the magic properties of “greenhouse gases” can indeed slow the flow of heat through the atmosphere by a few hundred (or in the worst case maybe a few thousand) milliseconds, HOW DOES THIS CREATE A “HIGHER EQUILIBRIUM TEMPERATURE” ???????????????

    All the time that this energy is being “back conducted” towards the surface of the Earth, the Earth is cooling continuously, and most of the energy is leaving for the cold vacuum of space at the speed of light.

    Cheers, Kevin.

    10

  • #
    Colin Henderson

    The first person to create a membrane that will move heat against a gradient will immediately become a kazillionaire, and will have solved all our energy problems. Is there CO2 insulation that move heat into your house during the winter and out during the summer?

    10

  • #
    RobJM

    While the greenhouse effect is not in violation of thermodynamics, water vapour positive feedback is due to free energy dynamics.
    Equilibrium represents the lowest free energy state and a system at equilibrium will resist any forcing due to increasing entropy.
    Basically equilibrium represents the bottom of the hill or valley and any work done to move the system out of the valley will involve losses and so the potential energy gained will be less than the energy expended. Negative feed back is a consequence of increasing entropy.
    The only way to get positive feedback is by draining energy from a “new” power source (0th law thermodynamics) or through internal cannibalisation where the net free energy is minimalized. Effectively in fluid dynamics a positive feedback occurs when one energy type such as sensible heat energy changes at a non linear rate at the expense of another free energy type like kinetic or latent heat energy.
    This is where water vapour positive is in violation, it claims that an increase in temp will also cause an increase in latent heat. This can’t happen unless there was an even greater loss in free energy from somewhere else.
    Free energy dynamics explains climate variability while maintaining negative feedback. Different climate states simply represent states with different proportions of the different energy types. On the whole the total free energy is minimised.
    In addition water vapour feedback represents a reaction and so if it could cause warming it must cause warming if the reaction occurs. It cant wait around for 6billion years!

    10

  • #
    RobJM

    Kevin at #45
    The greenhouse effect(86%saturated) basically intercepts and redirects 50% of absorbed photons back towards the ground. effectively the earth would only emit 57% of the energy it absorbed so it heats up (or bypasses with evaporative/convective cooling) until it reaches equilibrium.

    10

  • #
    Brian G Valentine

    If Jo Anne’s web log exists to prevent needless suffering of average people from the actions of zealous politicians and wrong-headed greenie groups, then I’m all for it, even if I don’t agree with all the material presented therein.

    10

  • #
    michael hammer

    comment 45 Kevin; I don’t agree with all the detail of your calculations but lets agree the delay is completely insignificant. You wrote “So if this IR radiation is absorbed/reflected/absorbed/reflected many many times by “greenhouse gases” This is not quite right. It is certanly absorbed but then new photons at the same wavelength are emitted due to the temperature of the absorber. Note the absorption depends only on the absorptivity but the emission depends on the emissivity (exactly the same as the absorptivity) AND the temperature of the emitter. Thus a very cold gas could absorb almost everything but emit very little while the same gas if very hot could still absorb almost everything but emit more than it absorbs. The point is that 15 micron energy is absorbed and emitted repeatedly up the atmospheric column. Becuase of this, only emissions (at 15 microns) from the very top of the atmospheric column escape to space (emissions from lower down are re-absorbed by the gas above). Thus the emission to space at 15 microns is coming from a body at the temperature of the upper atmosphere (about 220K) instead of the temperature of the surface 288K. Since the emikssion intensity is lower from a body at 220K than one at 288K the net emission to space at 15 microns is reduced and that difference is in effect energy retained by the GHG.

    10

  • #

    If you want to get the true basic physics straight, you (and the rest of the scientific world) need to read, understand, and accept my article,

    Venus: No Greenhouse Effect

    The result is overwhelming and definitive: Venus, with 96.5% carbon dioxide (CO2), or about 2,412 “doublings” of CO2 over the 0.04% CO2 concentration in the Earth’s atmosphere, is not raised by so much as 1°C, much less by 2,412 °C, compared at points of equal pressure in the two atmospheres (which is the “secret” to a proper comparison of the Venus/Earth atmospheric temperatures, that has eluded the incompetent promulgators of the greenhouse effect hypothesis for nearly 20 years, since the detailed Venus data was obtained). Lukewarmers (who believe there is a greenhouse effect, but only a small one) are clearly wrong, in the face of the Venus/Earth comparison, properly done. You should also read the comments following my blog article, and the preceding posts that discuss the real physics involved in the absorption and emission of infrared radiation in the atmosphere. My simple Venus/Earth comparison should be front-page news worldwide, and it should be the end of the present “consensus” and the hysterical legislation that is being enacted because of it. All those like Mr. Hammer who continue to promulgate false physics, need to face the facts, not continue to bandy wrong-headed theory. At this point, everyone needs to be re-educated on the “basic physics”, because a whole generation has been taught only empty dogma. Again, this is not theory on my part, this is the definitive experimental fact,of the Venus/Earth comparison, which has been neglected for nearly 20 years by incompetent scientists like James Hansen, who should have done it that long ago.

    10

  • #
    Louis Hissink

    Michael Hammer @ # 41

    I never claimed the underlying science was rehtoric etc, merely that the discussion here is all rhetoric and argument and that in the final analysis science demands physical validation of a particular hypothesis.

    If Prof. Gert Venter’s conclusions from his greenhouse experiments are valid, that introducing CO2 into the greenhouse environment causes a drop in temperature, thus forcing him to conclude that CO2 isn’t a greenhouse gas, then it is pretty interesting that no one in the AGW camp can point to a physical experiment refuting this; one can be quite sure if there were such experimental data that we would hear no end of it.

    This due to the apparent lack of solid in situ empirical data, rhetoric and semantics are used instead to convince us that CO2 is a greenhouse gas.

    10

  • #
    Binny

    Brian G Valentine:@49
    “If everyone is thinking alike, someone isn’t thinking.” — George Patton

    10

  • #
    michael hammer

    Louis at comment 52. There is solid empirical data. Look at the plot of earth emissions as seen from space (the Nimbus plots). They show an equivalent radiation temperature in the atmospheric window equivalent to that of the surface. This indicates emission to space directly from the surface. At 15 microns however the equivalent emission temperature drops abruptly to 220K -the temperature of the tropopause. Below 8 microns the equivalent emission temperature is again that of the tropopause. Above 15 microns it shows a very rapidly varying equivalent emission temperature due to the close absorption line spacing from water (the so called picket fence analogy.

    Further more, put a quartz cylinder of CO2 into an fourier transform infra red instrument and plot absorption versus wavelength. There is a sharp peak around 15 microns. How does that come about if CO2 is not absorbing at that wavelength ie: if it is not a green house gas. This has been documented beyond the slightest doubt. We need to purge our instruments with dry air to avoid unacceptable energy loss from water vapour absorption at many wavelengths.

    P.S. why the equivalent emssion temeprature at 15 microns should be that of the tropopasue rather than the stratosphere is a very interesting issue which again conflicts with some of the established wisdom but that is another issue too involved to go into here.

    10

  • #
    Bob Massey

    Brian, your comment at #45..

    “There is something in the mind that wants “atmospheric greenhouse effect” present in the physical world and casuistry is invoked if need be to validate this need.”

    hit the nail on the head

    10

  • #
    Bob Massey

    Brian my apologies typo should have been #35

    10

  • #
    michael hammer

    comment 51 Harry dale Hoffman. The pressure on Venus is about 90 bar – 90 times that of earth. The composition is indeed about97% CO2 so the concentration of CO2 is about 90 * 0.97/0.04 times that of earth or about 2200 times higher. That corresponds to log(2200)/log(2) = 11 doublings not 2412 doublings as you claim.

    I agree the current “consensus” is a poor basis for the legislation being proposed however I disagree that I am promulgating false physics. In fact what I am describing are concepts that have been tested and proven so frequently (in ways not tainted by AGW) that they have become part of extremely well established physics. Whole industries are based on this knowledge and would not work if this physics was wrong. Ultimately I believe in reality over theory.

    10

    • #
      Truthseeker

      Michael Hammer, you seem to have missed the important part of Dr Huffman’s analysis, and that is that he is comparing the atmospheric temperatures at the same air pressure (1000 mb to 200 mb). The fact that the surface air pressure of Venus is 90 times that of Earth is irrelevant to the argument that Dr Huffman is making.

      The 2412 degrees C comment Dr Huffman made is in relation to 96.5% CO2 versus 0.04% CO2 (96.4 / 0.04 = 2412.5)and was not connected to air pressure in any way.

      10

  • #
    BobC

    Colin Henderson @46:
    May 14th, 2011 at 10:58 am
    The first person to create a membrane that will move heat against a gradient will immediately become a kazillionaire, and will have solved all our energy problems. Is there CO2 insulation that move heat into your house during the winter and out during the summer?

    There’s one more criteria: It has to move a “significant” amount of heat.

    There are a number of devices/phenomena that already can be shown to violate the 2nd law of thermodynamics, but they don’t do it at a “significant” level:

    1) Water electrolyzers can, if run at low enough voltages, extract work from a single heat reservoir — that is, they can decrease entopy. Check out what it means to run below the “thermoneutral voltage“.

    2) A p-n diode (a solar cell), immersed in a uniform heat environment with Planck’s distribution of photon energies, will selectively turn high energy photons into electricity which can be taken out of the reservior and used to produce work — leaving the reservoir cooler. In effect, a p-n junction is a “Maxwell’s demon”.

    3) On slightly shakier empirical ground, I have seen molecular sieves that apparently had a diode effect, and have been told by a researcher in outdoor clothing that the newest breathable waterproofing has a diode effect for the transmission of water vapor. These devices move gases against a pressure gradient which is conceptually the same as moving heat against a heat gradient. (They are also Maxwell demons.)

    Unfortunately, the last two phenomena are barely measurable, and the first — while easily measurable and non-controversial among people who engineer electrolyzers — is still way too small to be useful.

    I’m not particularly surprised, since the second law was designed to explain steam engines. It’s promotion to a “Law of Nature” is largely theoretical.

    10

  • #
    BobC

    “A cooler body can reduce net heat loss from a warmer one”?

    That’s what my down parka does in Montana at -40C.

    10

  • #
    Bulldust

    Thanks Michael Hammer … you have clarified a few concepts for me. It takes a fair bit of talent to explain complex concepts clearly and succinctly. While some of Jo’s analogies might not represent the science perfectly, the point is that they try to communicate complex issues to the less educated readers. It isn’t difficult to read between the lines and arrive at the truth, but some always like to nit pick individual words to cast doubt on the entire argument.

    10

  • #
    memoryvault

    One of the favourite weapons of politicians, cultists and others with a personal agenda is as follows – in debate when confronted with hard facts that can’t be refuted, simply change the argument. This is one of those cases and it is discouraging to see Jo and so many other erstwhile good people wasting so much time and energy on it.

    There never was a debate about whether the “theory” of AGW contradicted the Laws of Thermodynamics until a group of warmist trolls invented it a year or so ago in the comments section of an article by James Delingpole in the Telegraph. And they invented it NOT to “prove” AGW didn’t contravene Thermodynamics, but rather, to AVOID debating the issue at hand.

    The issue at the time was “the missing heat” aka “Trenberth’s Travesty”. This is the “heat” Kevin Trenberth referred to in his Climategate email – “The fact is we can’t account for the lack of warming at the moment and it is a travesty that we can’t”.

    At the time of the debate on Delingpole Trenberth and the other Team Warmistas had given up on finding the “missing heat” in the non-existent tropical tropospheric hotspot, and instead had floated the idea that maybe it was “hiding” in the “deep oceans” (where, in typical fashion, it couldn’t actually be measured).

    The point made in the comments by a skeptic was the atmosphere cannot “heat” the oceans (as had been claimed by one of the trolls); the net transfer HAD to be the other way – from the oceans to the atmosphere. Net energy transfer from the atmosphere to the oceans would be in contravention of the Second Law of Thermodynamics (not to mention observable fact – eg evaporation).

    Rather than debate this, the trolls simply swung it around into an argument as to whether the “theory” of AGW contravened the Second Law. After that the whole thing degenerated into lengthy discussions about “metal blocks” and “positive radiative forcings” and “custard and fridges” and much the same cr#p as fills most of the 60 plus comments above this one.

    The warmista brigade must be laughing themselves senseless as otherwise well-meaning folk on our side of the fence tie ourselves in knots over a non-issue.

    Why on earth would any self-respecting, reasonably informed skeptic bother wasting any time arguing about whether the “theory” of AGW contravenes the Laws of Thermodynamics?

    Other fairy stories contradict the laws of physics and chemistry all the time without stirring up any controversy at all.

    10

  • #
    brc

    This has been banging around in my head for a while actually.

    As Jo says, it’s the terminology that is all wrong. Whomever came up with ‘greenhouse gas’ has committed a crime against us all, and then the rest of climate science and politics has become a wholesale massacre of language from that point onwards. ‘Gas Blanket’ is a much better term, but then that applies to the atmosphere as a whole. Without it, we’d be like the moon which is +100 on the sunny side and -100 on the dark side. IF people talked more about ‘atmospheric composition’ and less about ‘greenhouse gas’ things would go much more smoothly.

    Oh, and another thing : all this talk about hot/cold is where people go wrong. As there is no such thing as ‘cold’, what we should be talking about is ‘more heat’ and ‘less heat’. Nothing on earth is zero kelvin.

    10

  • #

    Here we get back to the fundamental issue: failure to communicate the science.

    The greenhouse gas theory does break basic laws of physics. Ergo, it is counter-intuitive. Which is why people have trouble ‘understanding’ it. A greenhouse is a closed system. The atmosphere is not. When you heat a gas, basic laws of physics dictates that the gas will expand. When the atmosphere becomes more energetic(by trapping outgoing infrared radiation) it will expand out into space.

    The temperature range at the surface of a planet is purely a function of that planet’s gravity and atmosphere bouyancy(composition of gases). Unless mankind can radically change the mass of the earth or perform industrial alchemy on the atmosphere, the planets surface temperature will always remain within the range that it has been in for the last hundred million years or so.

    Trying to argue that the earth is like a pressure cooker is ridiculous.

    10

  • #
    Brian G Valentine

    In fact what I am describing are concepts that have been tested and proven so frequently (in ways not tainted by AGW) that they have become part of extremely well established physics.

    Horses hit. You are describing methods to make a perpetual motion machine work by a spectral decomposition of radiant heat. That is the sum of your contribution today.

    10

  • #
    michael hammer

    comment 67, you seem to be drawing an exceptionally long bow in suggesting anything I said leads to a perpetual motion machine. It doesn’t although some of the comments made by others do (eg: comment #47). Since you have chosen such an extreme (and abusive) stand maybe you might like to justify it to me and others by explaining exactly how what I have said leads to a perpetual motion machine. Just for the record, as far as I am concerned perpetual motion machines are an impossibility.

    10

  • #
    Siliggy

    michael hammer:
    May 14th, 2011 at 1:08 pm
    Whole industries are based on this knowledge and would not work if this physics was wrong.

    An example of cooler radiation causing warmer to become even warmer for an industry is the common TV satellite dish. A simple antenna cannot be used for satellite TV because the signal is too weak as recieved by the antenna from the satellite and because the wanted signal is lower than the background temperature of unwanted signals created by heat both in the antenna and from other directions. The parabolic dish acts as a reflector that directs many small weak(cold) signals from the one distant direction to one point. The signal at this one point is guided to the antenna and becomes stronger with the addition of the many small wanted signals than from its own noise temperature at that frequency. Stronger enough for the signal to be fed away from it to the system and there distinguished from the colder random noise.

    10

  • #
    Damian Allen

    Falsification Of The Atmospheric Co2 Greenhouse Effects Within The Frame Of Physics………

    http://www.worldscinet.com/ijmpb/23/2303/S021797920904984X.html

    10

  • #
    Siliggy

    michael hammer:
    May 14th, 2011 at 9:30 am
    comment 28 Your explanation of a tank of water is spot on.

    Thankyou!

    For that to happen the level of water in the tank must rise slightly so as to increase the pressure on the other leaks so as to increase their flow slightly.

    Yes. Which is why i said:
    “As the tank fills the flow out by all these other paths will increase.”

    By the way, as an aside CO2 does not stop radiation to space at 15 microns. Certainly all surface emission to space at this wavelength is blocked (that would happen even at less than 0.5 ppm of CO2 in the atmosphere) but it is replaced by radiation to space from the top of the CO2 column.

    We are talking two different shapes.
    Column vs near sphere.
    If the 100% blocked point is as you say 0.5PPM then
    “the radiating surface area of the planet at these frequencies has been increased.”
    Compare two spheres both are at same temperature but one is larger. The larger one has more of the Meters squared part of W/m^2. So the volume of heat being radiated becomes larger as CO2 increases. Also with the poles being cold and the equator being warm radiation from the equator would be larger than at the poles. The earth is not a sphere so the surface area as seen from above the poles is larger than that seen from above the equator and the surface area of the equator is greater than than an equivalent width line around through the poles. So the sun (above the equator) warms (at it’s radiation spectoral range) a smaller area and the outgoing radiation occurs from a larger area than the a division by 4 to represent the ratio between the effective planetary area for absorption of solar radiation and the planetary area emitting longwave radiation.
    The ratio between incoming M^2 and outgoing M^2 is not constant. An equatorial upper troposphere hot spot (if it existed) would increase this error of the in out ratio.

    10

  • #
    RJ

    Michael Hammer at 25

    There are excellent scientific reasons why the theory of CAGW is wrong, we don’t need to use false arguments and leave ourselves open to justified criticism. OK – on with the further explanation;

    I don’t agree with this in total. Yes there are many ways to challenge CAGW and the remedies. As the opposition is doing very effectively in Australia (challenging a pointless CO2 tax even if CAGW is valid). But there is no reason whatsoever why the underlying AGW backradiation science should not also be debated. I believe now that it’s likely that the basic AGW science is also completely flawed. Let’s debate this and use this as another weapon where appropriate. Not hide behind but the AGW (just not CAGW) science is settled. This is an alarmists tactic.

    Since the Earth as a whole is a body floating in space it cannot lose energy by conduction and convection – both those require a material substance to transport the heat.

    Agree. But the surface is different to outer space. Most of the energy from the surface leaves by conduction and convection (and advection) not radiation. How much of each is unknown though but the slayer book claims the radiation component is very small.

    These objects are not intelligent, they have no way of knowing what other objects are in their immediate vicinity and modifying their radiation pattern accordingly, they simply radiate in all directions

    Yes but so what. The IR radiation from a cooler object CAN NOT then heat the hotter object. Like a teacher can increase knowledge of a child. But the child can’t then use the same knowledge to increase the teachers knowledge further. The child though can increase knowledge (heat) of another child (object) etc.

    Some of that energy strikes the custard and warms it.

    No. The custard just cools at either a slower or faster rate due to conduction and convection. An object can’t be warmed by IR radiation from a cooler body. It can receive the radiation but it will not have a warming impact.

    10

  • #

    No. The custard just cools at either a slower or faster rate due to conduction and convection. An object can’t be warmed by IR radiation from a cooler body. It can receive the radiation but it will not have a warming impact.

    RJ. I’m sure you didn’t mean that as it came out… how would an object know if the incoming IR beam was from the sun or from a GHG molecule so it could “ignore” the latter?

    An IR beam is an IR beam. It heats if it hits.

    Memoryvault #62: No. This argument did not start on Delingpoles blog. It has been doing the rounds behind the scenes for as long as I’ve been in the skeptical community.

    10

  • #
    Bryan

    Those advocates of the “greenhouse theory” should get real.

    If the radiative effects of H2O and CO2 are supposed to “heat” the atmosphere to 33K higher than “it would otherwise be” then they need to show significant radiative heating in a volume of the atmosphere.

    This they cannot do.

    Instead they distort equations and say look “there’s the proof”.
    Another method is to say that a spectrograph “proves it”.
    If we went to the installer of our home central heating system with a complaint that it wasn’t working would we be satisfied with a couple of equations as an excuse.
    Or if he produced a picture of a boiler would that satisfy us?

    Call the bluff of “greenhouse theory” advocates!
    Ask for proof of radiative heating in a volume of the atmosphere.

    They will be stumped!

    The fact that the temperature profile of the troposphere can be derived from thermodynamics without reference to radiation sums it up.
    A good place for the “greenhouse theory” advocates to start would be the radiative heating of a space the size of a real greenhouse.
    Lets get some real figures for say 30 cubic metres of air at STP on an average day.

    10

  • #
    RobJM

    RJ at #72 “It can receive the radiation but it will not have a warming impact”
    Dude, You cant destroy energy, it has to go somewhere, like a chemical bond, or in the case of IR into molecular motion.
    The whole point is that thermodynamics does not apply to quantum scale interactions.
    The quantum scale mechanism of the greenhouse effect is back radiation,
    The thermodynamic scale mechanism is as a radiative insulator.
    The two things are not mutually exclusive!

    10

  • #
    Morris Minor

    Yes, I am one the number of people claiming ‘back-radiation’ would contravene the second law of thermodynamics. I have lost count of the number of people believing in ‘back-radiation’. Has anyone observed, measured or detected this radiation? If so why dont we use it as a source of energy? It is after all claimed to deliver 300 Watts per sq meter downward (all day .. would be more useful than solar which is zero at night).

    The emissive power of Body 1 relative to Body 2 should be written:-
    E = s (T1^4 – T2^4) where T1 is the temperature of Body 1 and T2 the temperature of Body 2.
    Take a simpler example where T1 = T2. Between the surfaces there is no emission or absorption. I anticipate that some will say that one body will emit sT1^4 toward the other body and absorb sT2^4 from the other body… but what does that mean? Will a body absorb a photon and emit one of the same energy simultaneously? Well, maybe, but you could never observe this. A photon may be reflected but how would you know the difference? Both the Stefan-Boltzmann Law and the Second Law of Thermodynamics are statistical models. They do not predict the path of a single photon.

    It appears that a small misconception in the application of the Stefan-Boltzmann law has led to the invention of so-called back-radiation

    An analogy: consider two tanks of water both with water level H relative to the ground.
    The simplified flow rate from a pipe at the base of the tanks is given by:-
    F = k.H
    Where:
    k is a constant which allows for radius and length of pipe, viscosity etc.
    H is the Head or water level above our reference point (the ground).

    What would happen if we joined these tanks together by attaching both pipes to the other tank? According to the concept of back-radiation , F would flow from Tank1 into Tank2 and F would flow in the reverse direction from Tank2 into Tank1 simultaneously.

    If we put a flow meter in a pipe we could measure the flow between the tanks. The flow would be zero. This is analogous to back radiation. Does it really exist?

    Back radiation has not been detected, why continue with this mis-construed concept?.

    10

  • #
    memoryvault

    Jo says @ #74

    “An IR beam is an IR beam. If it hits it heats”.

    Jo, I have an enormous amount of respect for you and your work here. I come and read the articles and posts every day. It is because of that respect that I have bitten my tongue and until now refrained from commenting on the actual article, or the posts that have followed.

    However, your above comment cannot be left entirely unchallenged. Since it appears to be in his line of work, you might like to get Mr Hammer to write a short article on how and why a GAS spectrophotometer works the way it does.

    As for your comment on my post, I repeat, what does it matter?

    Fairy stories are allowed to contravene the Laws of Nature. Otherwise there would be no fire-breathing dragons, pixie dust, or sleeping beauties who can only be awakened by the kiss of a Prince.

    The “theory” of AGW is no different. New fairy story, same old set set of non-rules.

    10

  • #
    Ted Middleton

    (“CO2 and water vapor are greenhouse gases, right? So their presence causes a greenhouse effect, QED”)

    No!! they are gasses with infrared absorption bandwidths that absorb and re-radiate within those bandwidths. A greenhouse works by preventing convection; CO2 and H20 in the atmosphere do not exclude convection.

    They do reduce the rate of cooling, but the physics of the process is distinct from the physics of a greenhouse. Yes I’m being a bit picky here but isn’t it time we adhered to an accurate application of physics.

    10

  • #
    John Brookes

    Yo Hammer! Hey Hammer!

    There may be areas in which climate science is less than perfect – but this is not one of them.

    If you wish to refute the conclusions of the IPCC, and you use the “it can’t happen because of the 2nd law of thermodynamics” argument, then no one should listen to anything else you say on the subject – at least not without care. Ultimately, you’ll hurt your own cause, although I suppose you can fool some of the people all of the time….

    That the surface of the earth is warmer than it would be without water vapour and CO2 in the atmosphere is not disputed outside of the blogosphere (which seems to have its very own methods of producing heat).

    [Err, Brooksie, you missed it, Hammer and I agree with the IPCC on this one point… — JN]

    10

  • #
    bananabender

    The Greenhouse Effect doesn’t exist. Never has. Never will. It is pointless to even discuss it.

    You won’t find any proper spectroscopy textbook even mentioning the alleged and totally imaginary Greenhouse Effect (except to imply it is total BS).

    Far from warming the Earth the atmosphere actually massively reduces the surface temperature via convection and the transfer of energy to high altitudes which then allows energy to directly radiate into space.


    What really happens:

    The Sun heats the surface and the energy is then transferred to the atmosphere primarily by the evaporation and condensation of water with minor contributions from conduction. The heated air convects energy to the very top of the atmosphere where it radiates into the vacuum of space. End of story.

    No back-radiation or warming effect by delaying heat loss ever occurs due to “greenhouse” gases.

    The temperature of any planet is a molecular-kinetic effect primarily determined by the pressure according to the Ideal Gas Law pV=nRT. If we increase or lower the atmospheric pressure we automatically heat or cool the atmosphere. This is why the atmosphere cools as altitude increases – the adiabatic lapse rate.

    In effect the atmosphere acts as a giant piston compressing and heating the air below. This is just like the compression stroke of a diesel engine which heats the air sufficiently to ignite the fuel. Compressing the atmosphere via gravity increases the pressure and kinetic energy which in turn causes heating.

    Because Earth’s atmosphere has a low density convection causes a great deal of mixing which tends to cool the surface and heat the upper atmosphere. Without convection the surface of the Earth would average closer to 80C not 15C. In fact this is exactly what happens to car parked in the sun with the windows closed. The inside of the car doesn’t heat due to any magical greenhouse effect. It heats because the air is trapped inside.

    The temperature of Earth’s atmosphere would be similar regardless of the gaseous composition – whether it was pure nitrogen or pure methane.

    Mars, Venus and Earth all have very similar temperatures at the same barometric pressures despite totally different atmospheric composition and a huge difference in the incident solar radiative flux. As long as there is enough thermal energy from the Sun to keep the atmosphere as a gas there will be some atmospheric warming effect. Mars is colder than expected simply because the atmosphere is essentially non-existent (0.001 bar) and Venus is hotter than expected mainly because the atmospheric pressure is very high (90 bar – 5x as high as in diesel engine) and the substantial reduction in convection due to high atmospheric density.

    No radiative physics is even needed to explain the heating of the atmosphere.

    10

  • #
    Nigel Brereton

    This from an electrical engineer? Next we will be told that electrons flow from positive to negative making the negative charge greater.

    Heat energy travels from hot to cold bodies, the amount of transfer depends on the magnitude of the difference or the potential. the greater the potential the greater the flow. When you are outdoors on a cold night the potential is large so you radiate a lot of heat. When you move indoors the potential is less so you radiate less. There is no back flow from the colder bodies to the warmer bodies, heat enrgy will only flow in one direction how much flows depends upon the difference in temperature.
    The greenhouse effect is apparent only in the relationship that the potential is lessened by an atmosphere in the same manner as a blanket on a cold night, the transfer of heat from your body warms the air between you and the blanket which is a poor conductor of heat thus the potential is reduced and the heat transfer reduces which is why you feel that you get warmer after the time it takes to equilibriate the temperature. If there was no atmosphere the potential would be larger thus allowing a greater transfer of heat energy.

    10

  • #
    memoryvault

    Bananabender @ #79

    Yeah.

    What he said.

    10

  • #
    RJ

    RJ. I’m sure you didn’t mean that as it came out… how would an object know if the incoming IR beam was from the sun or from a GHG molecule so it could “ignore” the latter?

    An IR beam is an IR beam. It heats if it hits.

    How does an object know? Are you asking why doesn’t energy (IR radiation) leaving a colder object heat a warmer one. As I understand it’s because the wave length is longer. A photon can not raise the temperature of an object it is hitting if it is an equal or higher energy level.

    I thought this was accepted science that AGW has tried to push to one side. But science is not my area of expertise

    Here’s a quote from chapter 5 of the slayers book

    For more than a century now, the theory of an atmospheric greenhouse effect gained ground only because academic eggheads were losing contact with reality, having never grasped basic physics.

    Isn’t this the issue on this topic. Is the AGW theory based on flawed science. Can a colder object heat a warmer object in certain circumstances. Or not.

    More from the slayers book. Ch 15

    A brand new Law of Physics here, where parts within a system can behave contrary to the 2nd law but the whole obeys. Only in climate science can such chicanery etc.

    These are valid comments or questions that should not be brushed under the carpet with the science is settled claim, as some on here are doing.

    10

  • #
    David

    Best comment series to date – Jo a credit to all above!

    Your following is growing at a rate faster than the IPCC membership!

    I can’t stop reading these comments! WELL DONE!

    10

  • #
    Bryan

    Not all photons are created equal.
    The photons from the Sun have short wavelengths and have high energy.
    An ice cube also radiates photons
    They have much longer wavelengths and much less energy.
    We would not expect to be heated by ice!
    However an ice cube at -50C could be heated by an ice cube at -5C.

    10

  • #

    Michael Hammer @58:

    A slip of the tongue on my part (I managed to leave out how many doublings amounts to a factor of 2,400 increase), which typically is then pounced upon to try to ignore the main point: Venus is not heated by even 1 degree, much less 11 degrees, by its increased CO2 content. And you may believe in “reality” over theory, but what you are arguing IS theory, and what I presented in the Venus/Earth comparison is the overwhelming reality. And no one here, even Jo, seems to have faced that reality, judging by the fact that you just keep going over the same ground instead of confronting the simple truth of the Venus/Earth temperatures: There is no greenhouse effect whatsoever, as promulgated by the “consensus”. IR absorption and emission in the atmosphere is merely one pathway for the transfer of heat, which is governed primarily by the temperature lapse rate, which is set by gravity and the hydrostatic condition of the troposphere, as long recognized in the Standard Atmosphere. Heat will take whichever path is available to it (radiative, conductive, convective) to get to the same distribution, governed by the lapse rate. Increased CO2 can only increase the efficiency of heat transfer within that governing structure, it cannot change that structure. You refuse to face facts, not “alternative” theory, when you disregard the definitive results of the Venus/Earth comparison, properly done. This shows how hopeless it is to argue with all like you, who are deluded in your religiously-held conviction of what is the real “physics”. I’m a physicist, only an independent one, and I’m telling everyone that what passes for the “basic, settled physics” in climate science is not true, and is easily disproved as I have done, by showing the clear results of comparing two detailed planetary atmospheres. You will never convince me those results can be ignored, or do not mean what they clearly mean. But science demands that you try, and verify them for yourself. Instead, you pretend (to yourself, first) that you have the answer, regardless of definitive evidence counter to it. You are throwing away the first principles of scientific investigation, and teaching false physics. The atmosphere does not work as you believe, and you are only furthering the mass delusion in climate science, and the political tyranny derived from it.

    10

  • #
    MattB

    Jo is infinitely correct on this one. The photon from the cooler object does not make the warmer object warmer, as the warmer object is already emitting more photons towards the cooler object that it receives from the cooler object. If the cooler object was a bit warmer, then the warmer object would cool more slowly as it would be getting more photons back. They don’t magically bounce off just because they come from a cooler object.

    10

  • #
    Alistair

    What the author is describing is physically correct and is encapsulated by the oldest Law in Radiative heat transfer:

    ‘A body emits and absorbs radiant energy at equal rates when it is in equilibrium with its surroundings. Its temperature then remains constant. If the body is not at the same temperature as its surroundings there is a net flow of energy between the surroundings and the body because of unequal emission and absorption.’

    To this you have to add Kirchhoff’s Law which is that at thermal equilibrium, emissivity and absorptivity are equal.

    So, in the atmosphere you have radiant heat transfer but it is balanced by an equal and opposite flux in the reverse direction. Add to this that to calculate absorptivity and emissivity is complex and you have to go top Hottell Diagrams or a computational equivalent.

    The problem in climate science is that back radiation is an artefact of incorrect maths by Arthur Milne in 1922. Go to Miskolczi’s 20056 paper to learn why and be sure your physics is first class. The measured radiative flux in the atmosphere has been confused with this imaginary effect.

    GHG heating occurs but it’s much lower than claimed.

    10

  • #
    BLouis79

    The logical flaw in the “greenhouse” theory starts way back when people thought greenhouses worked like that – as radiation blockers. If the glass greenhouse mechanism had been right from the start, the CO2 greenhouse never would have existed.

    That the CO2 greenhouse persists is based on a serious logical flaw that the observable Tyndall gas effect on IR absorption is somehow independent of the second law. Gerlich and Tscheuschner are not alone as physicists who think this is unphysical. I have not found a climate scientist who has been able to pin down the physical mechanism by which they theorise CO2 can emit IR radiation to result in net heat flow up a thermal gradient. I have not found any empirical evidence that this is possible. Without a better explanation of the physical mechanism, it is untestable in the laboratory. (This is good for warmists.)

    A “Slaying the Greenhouse Dragon” author explains why he thinks CO2 can’t even act as “insulation”. The fact that radiation travels at the speed of light makes radiative “insulation” a nonsense. I think it is nonsense to even contemplate the notion that “insulation” has any impact on an equilibrium heat flow state. “Insulation” requires work to maintain a temperature gradient. Without work, the equilibrium state is isothermal.

    Talk of photons is irrelevant and confusing, since net heat flow is determined by temperature and radiation is only one mechanism of heat transfer.

    However, when someone is prepared to document the mechanism by which IR photon flow can be made to transfer net heat from cooler to warmer without the addition of work and demonstrate it in the physics lab, I’m all ears.

    10

  • #
    BLouis79

    On a related topic, there is a big difference between temperature and heat energy. The thermosphere is reportedly several hundred degrees kelvin, yet several hundred degrees in a near vacuum becomes near irrelevant in terms of heat energy. It may even be the result of instrument effects and not even be real. Apparently, astronauts and spacecraft do not melt in the thermosphere. Interested readers can read more on orbit temperature here http://tinyurl.com/lx776c.

    10

  • #
    Climate realist number9

    Forget entropy and you lose understanding. The domestic bedroom and kitchen examples above supporting the greenhouse effect theory are sillier than the theory itself – all wrong. The first and second laws of thermodynamics are violated by a theory that says back scattered infrared radiative transfer can cause global surface warming. The temperature of the earth is a measure of the heat equilibrium state reached over millennia between the earth and the input of radiation from the sun. The earth has an atmosphere and the ideal gas law adequately explains the temperature at the surface of the earth – temperature rises as the gases of the atmosphere are compressed against the surface of the earth by gravity. This causes more thermal energy (heat) in the atmosphere close to the earth surface and the observed adiabatic lapse of temperature with increasing altitude. A point here is that the nitrogen and oxygen – the dominant atmospheric gases but ones that do not absorb radiated infrared energy – hold much of the earth’s atmospheric thermal energy due to the compressing effect of gravity.

    10

  • #
    BLouis79

    On a related topic, there is a big difference between temperature and heat energy. The thermosphere is reportedly several hundred degrees kelvin, yet several hundred degrees in a near vacuum becomes near irrelevant in terms of heat energy content. It may even be the result of instrument effects and not even be real. Apparently, astronauts and spacecraft do not melt in the thermosphere. Interested readers can read more on orbit temperature here http://tinyurl.com/lx776c.

    10

  • #
    Nullius in Verba

    “I have not found a climate scientist who has been able to pin down the physical mechanism by which they theorise CO2 can emit IR radiation to result in net heat flow up a thermal gradient.”

    No climate scientist proposes that net heat flows up a thermal gradient. That is not the theory. That is not what they say. It’s a simple misunderstanding.

    It’s partly due to the poor way that climate scientists have explained the theory to the public, but the continued insistence that the GHE involves net flow from cold to hot, even after other sceptics have explained that it doesn’t, is unhelpful. By all means if you don’t understand how it works or why this is the case, then keep on asking questions, but there’s no point in arguing that the GHE claims are false because their proponents say that net heat flows up thermal gradients when they actually say – clearly and distinctly – that they don’t.

    There is no net transfer of heat from cold to hot by backradiation. Backradiation is not a ‘net’ transfer, it is one half of a two-way exchange that is in the forwards direction. If you go into a shop, buy a $3.40 item with a $5 note, the shop will give you $1.60 in change. Is it accurate to say then that the shop is paying you to take their goods away?! They’re giving you $1.60 in money, physically handing it over. Does this not violate the fundamental laws of economics? But by counting the change as a transaction entire, it makes the same error as treating backradiation as an entire energy transaction in isolation.

    The backradiation argument is wrong for a different reason, which confuses things. It does obey the second law, but it ignores the effect of alternative means of heat transfer.

    10

  • #
    Leonard Weinstein

    Jo and Michael,
    Two things:
    1) You both made misstatements, and your entire problem hinges on the definitions of heat transfer and energy transfer. You both are being careless and confusing them. The second law relates to heat transfer (one way), and the photon transfer is energy (bidirectional). Please get that straight.
    2) The effect of atmospheric greenhouse gases is to cause a reduction in direct radiation from the surface to space. However, water evaporation, surface convective heat transfer (with buoyancy) and atmospheric convection take over the difference of raising the remainder of energy to space to finely be radiated out. THE ATMOSPHERIC LAPSE RATE DOES THE REST TO CAUSE SURFACE HEATING. The larger surface radiation up (due to the surface being hotter) and back radiation from absorbing gases are a consequence of this effect, not a cause.

    10

  • #
    thRealUniverse

    WRONG! einsteins

    10

  • #
    Brian G Valentine

    explaining exactly how what I have said leads to a perpetual motion machine.

    Glad to oblige. You have derived a method to extract heat from a cooler reservoir (stratosphere) to move it to a warmer reservoir (troposphere) without expending work.

    This principle could be used to construct a heat engine that gives heat (for “nothing”) that could be used to operate a heat engine cycle. The heat that must be rejected by this cycle could be returned to the cooler reservoir (stratosphere) because it is now at a lower temperature than the initial temperature.

    Doesn’t matter how small the amount of heat it is, in principle at least, one could build a heat engine large enough to perform the amount of work one wanted.

    Sorry about that, and I’m not the only individual present amongst this crown who believe Gerlich and Tscheuschner had it right.

    10

  • #
    Brian G Valentine

    “crowd” not “crown”

    10

  • #

    memoryvault, thanks for visiting in good spirit. I appreciate it.

    What does it matter? Believe me, I’m all ears for new flaws in the IPCC version of reality, but we need to be bulletproof to beat them. Only those with buckets of cash can afford to spin fairyfloss, call it silk, and get away with it. It’ll sound grandiose, but I want to get the science right.

    And yes, Michael has offered to write more. We wanted to wait and see what the comments thread produced.

    10

  • #

    All,

    I am breaking down the post at Jo’s site and responding to individual points, with some additions.

    “Behind the scenes some skeptics are suggesting that CO2 can’t warm us because the atmosphere is colder than the planet, and it would break the 2nd Law of Thermodynamics (see Postma*, for example, p 6 – 7).”

    Actually I am not just talking about the 2nd Law…I am talking about the laws of heat transfer in total. It was an early typo which was missed when I mentioned the 2nd Law. The paper which is posted now correctly makes it clear that I am referencing the entire set of laws of heat transfer.

    “The 2nd Law of Thermodynamics applies to net flows of heat, not to each individual photon, and it does not prevent some heat flowing from a cooler body to a warm one.”

    The laws of heat transfer DO forbid a cold body from raising the temperature of a warmer body. This is a much more physically unambiguous clarification. “Flows of heat” is ambiguous and confusing. It is obviously correct to say that a cold body does not raise the temperature of a warmer body. “Flows of heat” is ambiguous and not important, in terms of what has the ability to raise who’s temperature.

    “Imagine three blocks of metal side by side. They are 11°C, 10°C, and 9°C. Think about what happens to the photons coming off the atoms in the middle of the medium temperature block between the other two. If heat never flows from cooler blocks to warmer blocks, all those photons have to go “right“, and not ever go “left”, because they “know” that way is towards a cooler block? (How would they?!)”

    There is no restriction, obviously, on which direction the photons can travel. But there IS a restriction on which body can raise the temperature of its neighbours. Only the hotter body can raise the temperature of a colder neighbour. As Dr. Nasif Nahle makes clear, a hotter body has a higher populated number of energy microstates. A colder body has a lower number of populated microstates. There is only one direction in which temperature can change on the respective bodies. Discussion of “net heat flow” is ambiguous and misses the point. For example, simply discussing heat, an iron slug and aluminum slug can have the exact same temperature, but the Iron will have a heck of a lot more heat. Most people would conclude the latter statement to mean the Iron slug is hotter. It is not…it is the same temperature. The Iron simply has a higher thermal capacity. Discussion of heat is too ambiguous for most people. Discussion of temperature is more clear. A cold body cannot raise the temperature of a warm body…”heat flow” is less important a concept in this more physically clear context.

    “The photons go both ways (actually every way, in 3D). There are more coming from the 11°C block to the 10°C block, sure, but the the 10°C block is sending ‘em back to the 11°C block too. So heat is flowing from cold to hot. It happens all the time. Net heat is flowing always hot to cold. But some heat is going the other way, every day, everywhere, bar possibly a black hole.”

    Lionell Griffith explained this very clearly. Again, heat flow is meaningless in the context above, when what is important is who is capable of raising who’s temperature.

    ” 1. ANY heat absorbed by the cold object that is coming from the warm object is no longer in the warm object.
    2. The warm object is colder by that amount.
    3. Any of the head from 1 that radiates back to the warm object simply REPLACES the prior lost heat.
    4. The warm object simply returns to its original temperature.
    Conclusion: back radiation does not and cannot warm a warm object beyond its original temperature. In fact it is always less because the “back” radiation is re-emitted spherically rather than totally reflected back to the warm body.”

    “People are being caught by semantics.”

    I agree Jo! This ambiguous distinction between “heat flow” and the physically unambiguous term of temperature is a perfect example. The hard truth is that something cold cannot raise the temperature of something warm. Instead of “heat flow” think of temperature flow – temperature can only flow from hot to cold.

    “Technically, strictly, greenhouse gases don’t “warm” the planet (as in, they don’t supply additional heat energy), but they slow the cooling, which for all pragmatic purposes leaves the planet warmer that it would have been without them.”

    This statement is fraught with contradiction. Is it technical or is it real? I mean this is very important! If GHG’s, as you correctly point out, cannot raise the planet’s temperature because they are not actually sources of additional energy, then how do we arrange some logic in which they do exactly what we just said they cannot do?

    “It’s a bit like saying a blanket doesn’t warm you in bed. Sure, it’s got no internal heat source, and it won’t add any heat energy that you didn’t already have, but you sure feel cold without one.” – Jo

    But Jo you know darned well that preventing convection has NOTHING to do with the radiative greenhouse theory! Preventing convective cooling is exactly what a real greenhouse does and you know that it has nothing to do with trapping radiation. This statement is just a little unfair!

    Besides, it is unfortunate that I find people are becoming so confused in the semantics between heat flow and actual temperature change. The whole rest of my paper dealt with the really big elephant in the room: that the +15C ground temperature CAN NOT be actually characteristic of the actual planetary temperature, and that the actual planetary temperature IS the expected -18C; and that -18C is NOT actually equal to the average solar insolation on the planet, i.e., dividing the Solar Flux by a factor of four, diluting its intensity to only -18C.

    In the first case, we can take a lesson from solar astrophysics. The bottom of the photosphere in the Sun, the part we actually see, is about 9000K. The top of the photosphere is about 4000K. Yet the temperature we measure of the photosphere is actually about 5800K. That is similar to the situation on the Earth. The bottom of the atmosphere is warmer, due to actual solar insolation reaching potentially as high as 121C, and because that solar insolation only impinges on one side of the planet, not both day and night side (i.e. division by four), and the top of the atmosphere is colder. The average of the system is therefore found in between, and is actually equivalent (NOT EQUAL) to an output of -18C. In other words, energy is conserved, but the energy flux density, or temperature, is doing lots of different things, including sustaining a bottom of atmosphere temperature of +15C (if that is even an accurate assessment). And this all occurs via the simple physics of thermal capacity and heat retention, without need to postulate a radiative greenhouse effect. The only conclusion one can make, in the face of the empirical reality that the atmosphere is always cooler than the maximum solar insolation upon the surface, is that the atmosphere cools the surface, and takes heat away from it.

    I was going to respond to Michael Hammer’s post, but I’ve actually answered him in all the above, plus he repeatedly makes the unfortunate mistake of comparing convective cooling to radiative effects. It is most unfortunate.

    Lionell Griffith and Harry Dale Huffman nicely sum up the real, unambiguated science.

    “PS from Jo: Just so we’re clear here, I think CO2 molecules absorb Infra Red and have some warming effect, but I think feedbacks from clouds or humidity keep those effects so small that the total effect of adding more CO2 is minor, and not worth taking any action over. (There’s more info on feedbacks in these posts).”

    Their ability to absorb IR only connects to their ability to transfer heat from hot to cold and also their thermal capacity. Obviously the situation is that the temperature on the ground is provided by the Sun, and the atmosphere keeps it cooler than the Solar potential…CO2 does not provide any additional energy in such a way as that it could raise the temperature of the already warmer ground.

    Don’t forget, everything else in the atmosphere is also “back-radiating” IR via thermal collision and emission – the N2, the O2, all of it radiates IR, not just CO2 or GHG’s. I have seen warmists claim that only GHG’s emit thermal IR and without them no radiative loss from the atmosphere could occur. Such people are complete frauds with no minds for physics whatsoever. Every single collision occurring between ANY gas species in the atmosphere results in an emission of IR EM energy and represents a loss in energy, and therefore cooling, of the ensemble.

    Curious that there is no discussion on the back-IR heating contribution from these other gases. That’s because the entire concept is a fraud, and “they” just want to specifically tax CO2 and reduce our energy usage and quality of life, etc. And make reams of money in doing so.

    I’ll answer this one below because it is a perfect example of semantic ambiguity.

    “*The Postma example correctly shows that two ice cubes at 0°C will not heat one cube above zero, but it’s all about context. If the surrounding air is even colder, being next to an ice cube will keep you warmer longer”.

    But does it raise your temperature? No! You and the ice cube will only cool. You won’t live longer by hugging the ice cube. You can try I guess.

    “Even ice can be a “blanket”. Being wrapped in ice would slow heat loss if you happen to be on a rock in outer space”

    Does slowing heat loss make you raise in temperature? No, your temperature would still decrease.

    “Eskimoes stay warmer in an igloo. (Yes there are lots of reasons why, but the point remains… if you reduce your heat loss you stay warmer.)”

    Again, confusing convective insulation with radiative loss. You shouldn’t do that, it isn’t nice. The “reasons why”, which you conveniently neglect to include, make all the difference in the world! Take a real botanist’s greenhouse for example: It warms by preventing convective heat loss. “Trapping radiation” in fact has nothing to do with why a real greenhouse gets warm inside. And it makes ALL the difference in the world when you understand the difference. The “radiative trapping greenhouse postulate” is in fact based on a theory which a real greenhouse is incapable of doing, and which it doesn’t do.

    There is no experimental (nor theoretical, in truth) framework which supports the idea of radiative self-amplification of temperature after insolation from another hotter source. I will spare details, but we will be experimentally proving this simple reality in due course. There is no such thing as conductive self-amplification of temperature; nor is there such a things as radiative self-amplification of temperature. Both conduction and radiation are modes of heat transfer and obey said laws of heat transfer. Radiation cannot conveniently get around those laws, as many try to claim.

    The logic of reality of the situation, devoid of semantics, is thus:

    In the GHG theory, a too-low of solar insolation is calculated as the input by dividing the solar flux by four, and thus impinging it upon all sides of the Earth. The resulting dilution of insolated solar energy flux density necessitates a postulate of temperature self-amplification to explain why warmer temperatures than -18C exist anywhere. Thus the GH effect is created.

    In the thermodynamic theory, the physical insolation is recognized as being only over one hemisphere, as the true average, and with a maximum insolation under the solar zenith. The temperature this should produce over the sunlit hemisphere is +30C as a SYSTEM temperature (and remember, the ground temperature SHOULD be warmer than the system temperature). The temperature this should produce under the solar zenith is in the vicinity of +90C for the system. Why is, in fact, the ground air temperature always cooler than the insolation? How can a radiative greenhouse effect exist as a fundamental physical/physics property of gases and the atmosphere when this supposed effect is apparently NEGATIVE under solar insolation when there is in fact MORE thermal IR from the ground to augment and amplify the effect? Where is the +33C GH effect when the Sun is warming me to +35C in the summer time all by its own insolation?

    There is nothing wrong with having a ground temperature of +15C, and the attempt to qualify this as characteristic of the planetary temperature is a hijack of logic and physics. But because humans live upon the ground it is easy to trick them into thinking so.

    The planetary system output of energy IS equal to the input energy, and has an equivalent value of -18C energy flux density, though this has nothing to do with the actual energy flux density (i.e. temperature) of the input, or necessarily anywhere you might find upon the ground. There is in fact MORE than enough solar insolation to sustain the ground temperature of +15C.

    The atmosphere keeps the planet cool under the Sun, and then loses heat slowly over night. Temperatures are thus modulated and smoothed as compared to having no atmosphere at all. The bottom of atmosphere temperature average is a result of standard adiabatic and thermal capacity effects. To increase said temperature would require an increase in atmospheric depth and density. The effects of changes of a negligible component of the atmosphere will have negligible effect.

    Cheers,

    Joe Postma

    10

    • #
      Truthseeker

      Jose_X, you pose the question; Why would it be difficult to believe that this CO2 radiation leads to a higher temp equilibrium?

      Let Harry Huffman’s very clear analysis of actual data comparing Earth (0.04% CO2) to Venus (96.4% CO2) answer that for you.

      10

      • #
        Jose_X

        Truthseeker, I am still analyzing how to derive the pressure of a gas at a certain height. I suspect the reason those values for pressure and temp are similar between earth and venus is because Venus and the Earth have similar radii and masses. [???] This may lead perhaps to similar concentrations of gas at a given pressure. pv=nrt (or any version of the ideal gas law) would then imply, if n/v are about the same, that pressures and temperatures would also have to correspond.

        Now, I noticed that the argument you pointed to fails to consider albedo. That discussion point came up a lot in the comments. The theory by Huffman is that albedo doesn’t matter, so CO2 and other gases must be absorbing the sun’s radiation, I presume he accepts. If you believe that, I then ask you and him the same set of questions I asked Postma, and we can ask many more questions now since there appears to be disagreement generally with what spectroscopy of gases reveals and what quantum mechanics explains.

        And remember something else. We know the details of the atmosphere of Venus because of theories we trust and apply to information we get from certain types of instruments we have created. So trying to go too much against established science is very likely to lead to contradictions, perhaps even in the data you take for granted. [Also, Stefan-Boltzmann comes from Planck blackbody distribution, which is closely related to findings in spectroscopy (that appear to argue against anyone who thinks the sun’s radiation is absorbed at TOA).]

        10

        • #
          Truthseeker

          Jose_X, it is nice to have someone trying to talk to the argument and not the person. You say

          This may lead perhaps to similar concentrations of gas at a given pressure. pv=nrt (or any version of the ideal gas law) would then imply, if n/v are about the same, that pressures and temperatures would also have to correspond.

          I do not think that Huffman or myself could agree more. The underlying theory being questioned here is that CO2 raises the ambient temperature more than other gases such as O2 or N2. Looking at the analysis, there are precisely four variables and a result. Those variables are the Sun’s output, the distance from the Sun, atmospheric composition and atmospheric pressure. The result is the average temperature in K. Given that the Sun’s output is common and the air pressure is common to both, which of the other two variables is determining the atmospheric temperature? The answer is the distance from the Sun and not the atmospheric composition. Therefore CO2 does not affect atmospheric temperatures. Albedo is not relevant. QED.

          I am not a physicist, but my take on the whole spectroscopy vs thermodynamics is that IR radiation is used as a proxy for temperature and that may not be valid. “Hot” objects may give off IR radiation but that does not mean that IR radiation necessarily makes objects warmer. Finding spectroscopic effects of different gases is not the same as measuring temperature and it may be invalid to assume that one causes the other. Even if I am wrong about this, that does not invalidate Venus/Earth analysis that Huffman has done.

          10

          • #
            blouis79

            Sorry, I don’t agree albedo is not relevant. I agree molecular composition of atmosphere is probably irrelevant as far as Stefan-Boltzmann equation is concerned. I have not seen any evidence that changing the composition of a blackbody changes its radiative equilibrium temperature.

            I am also having trouble finding *any* scientific evidence of the disposition of IR radiation “not transmitted” by a “greenhouse” gas. Tyndall only properly determined the fact of IR not being transmitted. He has nothing to say in where it went, though assumes absorbed.

            Possibilities for which the relative proportions have not been elucidated:
            1. Absorbed into higher electron state without temperature rise.
            2. Absorbed into higher electron state then reemitted in random directions without temperature rise.
            3. Absorbed resulting in temperature rise.

            As far as I can figure:
            a. people have misinterpreted Tyndall’s findings for centuries
            b. people have misinterpreted Stefan-Boltzmann and Plank as applied to atmospheric temperatures by using, as it suits the argument, absorption/emission at planetary suface and absorption/emission at the top of atmosphere. According to Postma, there is no need for a “greenhouse” effect to explain earth temperature if one considers absorption/emission of the integrated planetary/atmospheric surface as detectable from space
            c. according to Gerlich and Tscheuschner, there is no basis for a “greenhouse effect” in theoretical physics

            It shouldn’t take too many years before the real science of atmosphere is understood and verified in theoretical and experimental physics.

            Then maybe we can move to a real problem (forgetting the CO2 red herring) – anything that generates waste heat on earth (liberation of any source of potential energy including chemical [coal] and nuclear) causes warming on earth. This argument is well founded in basic physics and is only ignored by climate scientists who don’t understand the physics of energy, mass, specific heat and temperature. Bo Nordell has quantified the effect.

            10

            • #
              Jose_X

              >> Sorry, I don’t agree albedo is not relevant.

              I think this means we agree it is relevant. [just checking]

              >> I have not seen any evidence that changing the composition of a blackbody changes its radiative equilibrium temperature.

              If the blackbody is the solid planet only, then the atmosphere doesn’t change the composition but effectively adds insulation. That leads to higher surface temperatures.

              If the atmosphere is part of the blackbody, then we can use the example of removing the sweat glands (changing the composition of your skin) and add insulation material to it (lots of fat). Your internal body (right underneath your skin — which is like the atmosphere in this case) will get hotter. And it can lead to death much more easily.

              Consider also that biochemical reactions inside your body provide a near constant and continuous heat source just as the sun serves that purpose to the earth.

              >> 1. Absorbed into higher electron state without temperature rise.

              So this leads to radiation (aka “back radiation”) because we can’t accumulate higher and higher electron state forever.. not to mention that the physical laws dictate spontaneous emission on average after a period of time. OR this leads to temperature rise later on.

              In other words, 1 leads necessarily to 2 or to 3.

              >> According to Postma, there is no need for a “greenhouse” effect to explain earth temperature if one considers absorption/emission of the integrated planetary/atmospheric surface as detectable from space

              I’ll have to read his paper. I know most scientists disagree.

              >> according to Gerlich and Tscheuschner

              I think at least some of those works have been criticized significantly, but do you have a link to something specific I can read for $0?

              >> It shouldn’t take too many years before the real science of atmosphere is understood and verified in theoretical and experimental physics.

              [With a little sarcasm] The process will be sped up if Postma and others do an analysis of the current papers and text books that have been accepted by the wider scientific community.

              I don’t think you are keeping count properly.

              >> This argument is well founded in basic physics and is only ignored by climate scientists who don’t understand the physics of energy, mass, specific heat and temperature.

              I think this is another one of those gut feelings.

              Sorry, I don’t mean to be distracting, but I haven’t seen any disproving. I’ll take a look at the Postma paper later.

              10

              • #
                Jose_X

                >> I’ll have to read his paper. I know most scientists disagree.

                I should have proofread. No, I don’t know what most scientists believe, but if it disagrees with accepted science in significant ways, I suspect most scientists that study this matter will disagree with its results.

                10

              • #
                blouis79

                #616@Jose_X

                When people misundertand the basics of physics, the argument can be settled in theoretical and experimental physics. What we get from climate scientists is lots of observations and words without a sound basis in physical theory. When people have too much time invested in erroneous thinking, it is hard for them to admit to error. Good science will prevail in time.

                Gerlich and Tscheuschner
                arxiv.org/pdf/0707.1161

                Bo Nordell
                http://www.ltu.se/research/subjects/Vattenteknik-Fornyelsebar-energi-Hydrologi/Forskningsprojekt/Global-Warming-1.5035?l=en

                There is no value is analysing mountains of textbooks based on fallacy. The core is the original sources on which false assumptions are based. Disproving original sources makes the whole house of cards fall down. Gerlich and Tscheuschner do analyse all prevailing greenhouse theories. G&T and Postma are the *wider* scientific community – people in science following scientific principles who are not wedded to climate sciences misperceptions of physics and atmospheres.

                Climate scientists are what people *think* is the wider scientific community, but they are not.

                Atomic energy state changes have not much to do with mass effects (insulative properties). Insulation will not ever change equilibrium temperature in a cycling system or system out of equilibrium, it can only reduce excursions from the equilibrium in a cycling system or delay equilibrium in a system out of equilibrium. These effects are easily demonstrated in “kitchen science”.

                10

              • #
                KinkyKeith

                Hi blouis79

                Have just downloaded a copy of Gerlich and Tscheuschner’s paper and scanned the index. It looks interesting. Thanks.

                10

              • #
                Jose_X

                I’ll take a look at that G&T paper (and the Postma one).

                I think the part that is confusing to you (and I say this because I have asked myself a similar question) is that you recognize that a barrier placed between a hot object (earth) and a cold object (space) cannot warm the hot object more than it already is but merely slow down the rate of heat flow.

                I agree. But you are ignoring that the Sun is not turned off. The sun keeps adding more and more heat to the earth. We aren’t simply allowing the Earth to cool. When you drive the earth+atmosphere with steady power you get a different equilibrium point than driving the earth-atmosphere with that same steady power.

                It’s like a tub with water. You are visualizing that adding more CO2 could be at most like narrowing the drain. The water will flow more slowly away, but it will not flow backwards and fill the tub. However, the actual planet has the sun turned on. We have the faucet on and the water stable at some level. If we now add more CO2 and narrow the drain, the water level will rise to a new equilibrium value. As the water level rises, more pressure is exerted below, increasing the flow rate until it equals what it was before at the lower level with the wider drain. The analogy makes great sense.

                Getting back to ovens.. An oven with heat inside is unplugged. The insulation basically slows the rate of heat loss. Good insulation won’t result in higher oven temp but slower loss only. Now, we plug the oven in. This time good insulation will result in a higher temperature than bad insulation. The oven will reach equilibrium temperature (which could be very high) based on how good is the insulation. Super ideal insulation would result in the oven getting hotter and hotter until things start melting or failing in some way. So the key is that by not turning off the electricity (aka the sun) we can drive the oven (earth) until it gets extremely hot if we use excellent insulation. This is ordinary physics and something we can experience a bit in the kitchen (if you bypass the thermometer control). Greenhouse effect is that. It’s great insulation that leads to earth that cooks hotter under the raging sun.

                [Another analogy I used earlier involves mirrors… perhaps used to cook hotdogs by re-diverting heat that otherwise would have gone elsewhere (into the ground, aka, into space for the atmosphere case).]

                10

        • #
          Jose_X

          Truthseeker, I had to go somewhere for a while, but I had a chance to think back on this problem. It really is an odd problem and got me by surprise before.

          First, though, let me reply to one thing you said,
          >> I do not think that Huffman or myself could agree more.

          I don’t think you understood what I was trying to say. I meant to say that I think the relationship he spotted (either is coincidental or) depends on variables that are different from the sun and the Stefan-Boltzmann calculation. If there is a simple reason why that pattern holds (eg, maybe it exists by coincidence because of similarities in size between venus and earth) and has nothing to do with Stefan Boltzmann, then maybe there is no motivation to attack greenhouse effect and push in S-B to where it doesn’t belong. That was my point, that perhaps his table value can be explained even without invoking greenhouse effect since gravity effects on particles might lead to similar concentration levels, and pv=nrt would then dictate necessarily that p and t match up.

          OK, let me now present another fail of the Huffman theory (besides what I mentioned already about albedo and such).

          The fail is seen as follows.

          He claims “since the radiating temperature of an isolated body in space varies as the fourth-root of the power incident upon it, by the Stefan-Boltzmann law.”

          Now, I can’t know exactly what laws he believes are valid and which are not, but I’ll assume he believes that temperature is fixed for a given body for a given power flux.

          But there is no dependency in the Stefan-Boltzmann formula on pressure! So why did he tie in the pressure relationship across atmospheres when stefan-boltzmann doesn’t provide room to add in pressure considerations?

          Let me ask a related question. The “body in space” has a constant temp allegedly defined by that s-b law. He stated something along those lines. Now, at what point is the s-b result supposed to hold? On earth, apparently, it holds at ground level, but on venus, that relationship held somewhere up in the atmosphere. So Huffman is going to have to probably have the s-b in his model apply at a given pressure since he favored that interpretation. He will have to modify the s-b formula or semantics in order to have a pressure dependency. If he doesn’t do that, we will reach inconsistencies. Let’s take a look at an example.

          Let’s keep the earth and venus right along their current orbits and hence at the same radiation flux from the sun. However, on earth, let’s pump our atmosphere with so much gas so as to double its pressure at ground level (let’s say we keep burning hydrocarbons and anything else we can get our hands on).

          We reach a contradiction.

          The stefan-boltzmann argument has not changed for either planet (same sun, same orbit, etc,), so we find that the formula produces the same temperature results as it did before. Yet here is where we have to decide where this temperature from s-b applies. If we attach it again to the earth’s surface or lowest atmosphere layer touching it, then we violate the relationship with venus. We now have the same s-b temp result but at 2 atm on earth. And if we hunt deeper into Venus’ atmosphere for the 2atm location, we will find a higher temp and power flux even though the earth calculation didn’t change. So we violate that earth venus relationship. Huffman’s pretty table is destroyed.

          On the other hand, if we say that s-b applies to an outside gas layer at some pressure, then what pressure is that supposed to be? s-b doesn’t mention pressure. What if we didn’t know what venus was like, where do we attach that s-b result?

          So it seems that his formula for temperature needs to add in a dependency on pressure to break the ambiguity.

          Also, keep in mind for later that power was derived only from sun’s distance and the orbit geometry. Huffman’s simple model makes no mention of other sources of power.

          Now, what pressure dependency can solve our ambiguity? Remember, we can’t pick a specific location such as the ground solid surface since we just modified the earth atmosphere to 2atm so to break the relationship with venus if we do pick the ground layer. [Eg, we’d have the same 287.4 calculation as before but this time at 2000 mbar and that would conflict surely with venus’ atmosphere temp at 2000 mbar.]

          What if the planet has no atmosphere? Then where do we peg that temperature result from the s-b calculation?

          Let’s pretend that the s-b calculation applies only to molecules that border a gas of pressure 1 atm. Well, can this formula predict any other temperature up and down that atmosphere or on the planet’s ground surface? I don’t know? Huffman should try to come up with a formula or else his theory will be weaker than what scientists today already have. Between a limited theory that only applies to 1 atm bounded surfaces and a different theory that can predict much more, I prefer the powerful one. Welcome back, greenhouse effect.

          OK, so we’ll talk to Huffman later for the details so that his s-b formula doesn’t just work at 1 atm and can derive values at other pressures.

          Let’s now look at what I suspect is a problem with his table.

          Take a look at the table of height, pressure, and temperature for venus http://en.wikipedia.org/wiki/Atmosphere_of_Venus . We can see that as the pressure drops tremendously the temp more slowly but also drops. However, the earth’s atmosphere does not follow this pattern. At very low pressures high on earth’s atmosphere, we have the temp rise again and then get very very hot (eg, see earth Thermosphere temperature and pressure profiles). I haven’t done a point by point comparison, but I suspect the pattern completely breaks down. Note that in the ozone section of our atmosphere the temp is higher, matching the predictions of ozone absorption of sun radiation in the uv range. I doubt this same profile exists on venus. So we have formulas and theories which explain our atmosphere, and I doubt the earth’s atmosphere matches venus up and down. Hence, I think Huffman still has a lot of work to do to make his formula useful in any real way.

          In other words, Huffman hasn’t really given a formula that works on venus and earth. He stated a few values on a table where a particular ratio almost holds. He didn’t list all the values. If he had, we likely would see very large divergences, rendering his novel theory rather incomplete. [and this besides the fact that his s-b needs to be tied to a pressure dependency first or else we can break the table my mucking around with earth pressure.]

          Let’s look at another issue.

          This rule that s-b only applies at 1 atm (with further dependencies to be provided by Huffman at a later date) contradicts all the physics and evidence we have on earth! On earth, at the very same pressure we have varying temperatures all year long. In fact, at the poles in winter we have a very different temperature at that same 1 atm than we have along the equator.

          So while Huffman’s theory predicts one value for a given sun-radiation level (remember how much of a big deal he made about the distance from the sun?), it fails totally on the surface of the earth where clearly we have weather patterns that are anything but one fixed static s-b value for the whole surface.

          Huffman’s theory has to be de-linked from the sun if he wants to make it practically. It needs to focus on flux and not on distance from the sun.

          But of course breaking the link from the sun sort of sinks the whole motivation for the model. After all, he refused to analyze the details of the atmosphere and simply said that all that really mattered was the radiation coming from the sun. So Huffman chose already to ignore the details of weather in order to assume that at a given pressure, we always have a single temperature.

          Now, I did say that Huffman might enlarge s-b to depend on pressure and in the process come up with a very intricate formula that works at many different combinations of pressure and temperature.

          However, looking at the venus table, we see there is no room there for the same temperature to exist at different pressures, generally, yet clearly on earth we have created the same temperature at many different pressures. For example, for a gas that behaves quasi ideally, pv=nrt means we can increase pressure and volume at the same rate (without changing the number of particles) in order to keep temperature constant.

          So Huffman argued that the key point was that at different pressures certain related temperatures would be found. Big fail because on earth those two values are independent not dependent. We can break that table all day and night long. Temperature and pressure are not dependent.

          What is my main point? ..that Huffman decided to tie together two things that aren’t really connected. If he wants us to believe temp and pressure are connected, he is going to have to re-derive a lot of physics first. [not to mention that he needs to reconcile the rest of the missing table values.]

          Note as well: All of this madness came to be apparently because, as he stated, “the entrenched incompetence in science today” is perhaps partly due to “incompetent climate scientist (which 97% of them demonstrably are)”. Huffman, “a competent physicist”, clearly knows what the incompetents don’t, that “there is no greenhouse effect at all.”

          So Huffman is just sure.. He is so sure that he confidently waves off almost all climate scientists as incompetent. He states that clearly there is no greenhouse effect. Yet, all he can come up with is a very contorted and limited theory to explain something that likely already had a simple explanation, while leaving himself facing a bunch of contradictions unless he starts removing and redeveloping more formulas and models to replace what physicists have usefully been using for years.

          Please don’t be surprised if I still think the questions I posed to Postma have not been answered by Huffman.

          And hopefully Huffman will re-evaluate his theories, reconsider his surety about the wrongness of the atmosphere greenhouse effect, and consider that maybe he is not in the best position to judge all 97% climate scientists.

          10

          • #
            Paul

            See my reply in #600 above.

            For some reason just adding a reply gives an earlier number than the latest post.

            Paul

            10

          • #
            Truthseeker

            Jose_X, I would like to thank you for such a detailed and considered reply, dealing with the argument as it is and not some straw-man version of it. I will at this point say that trying to argue on Huffman’s behalf without a solid grounding in physics is going to be a waste of both of our times. I have posted a comment on Huffman’s blog with a link to this comment and will leave him to reply or not as he sees fit.

            10

            • #
              Truthseeker

              Jose_X, Huffman has replied on his blog. If you want to continue this discussion, please do so at the source which is here (scroll down to the latest comment).

              10

              • #
                Jose_X

                >> If you want to continue this discussion, please do so at the source

                I don’t know if I will open one of the required accounts in order to post. Many sites allows the public to post without having to register. [I am not blaming him.]

                I appreciate you providing the links, but we are still on topic here, and I am posting where both I and he are allowed to post right now (maybe I will post at his blog if I open an account later).

                I think Harry (Hoffman) might want to start reading my comment from the bottom up one paragraph at a time since it might be easier that way [sorry for not writing a clear comment].

                Reading his reply to me, he appears to confirm what I stated that his analysis doesn’t derive the actual temperature or actual radiation level. He uses the fourth root result to state he knows the ratio of planet atmosphere temperatures at equal pressures. He doesn’t know what those values are, just that if he is given two different planets and measurements, that such a calculated ratio will hold.

                But even in this he is incorrect, and no I certainly didn’t ignore his result that his formulation is dependent only on the sun’s distance — which makes his methods rather limited .. and incorrect:

                First, he is missing data for 100 mb, 50 mb, 20 mb, etc. I think he would find the values to diverge even more. In contrast, radiative theories of gases (eg, as applied to ozone) help explain why our atmosphere gets hotter in some places even as the pressure continues to drop.

                Second and more fundamentally, I stated that on earth we have many different locations, each with a different temperature from the other, even though those spots are at basically the same distance from the sun and at the same pressure of about 1000 mbar.

                His methods imply the same temperature for two spots on earth at the same pressure and distance from the sun. This is not what we observe.

                For him to be right, he not only has to explain this gross oversight, but we would have to overturn centuries of understanding that pressure and temperature can vary independently from each other (for gases, we have the volume and molecule count knobs to allow temp and pressure to diverge from each other).

                He did not address what I wrote (and I know it was confusing, so I am not blaming anyone). I understand you don’t want to referee, but he is welcomed to post a reply here as he did over there, so I can reply directly. Joanne has been very welcoming over here, but I encountered a barrier over there (and I am not blaming him for the barrier, just saying that the most welcoming spot at this point in time is right over here). If he doesn’t mind waiting, I might get an account later on and post over there.

                Thanks again Truthseeker, and, Harry, I have nothing against you, but I think your methods going against so much established science are wrong. If you read only this paragraph, please answer how your methods are consistent with the diverse temperature readings of the air that we have on earth at roughly equal distances from the sun at the same pressures.

                10

              • #
                KinkyKeith

                Hi Jose

                It seems that I am not alone in my assessments posted between 598 and 603 on the tone of your scientific comments.

                Harry Dale Huffman seems to agree in that he describes comment by you as follows:

                “That, and everything he “deduces” from it, is nonsense. ”

                You can try to talk science by copying stuff and editing it but it stands out to a real scientist as though a lawyer was trying to pose as a scientist.

                Environmental Lawyer??

                Gotcha.

                Go Away.

                10

              • #
                Jose_X

                KinkyKeith,
                Why don’t you comment directly on what part you think is inaccurate. It doesn’t help me if you just lob generalized claims at me.

                I’ll repeat a link from one of the last comments I posted, so that you see that many top notch scientists, at least as communicated by their universities indirectly on the web, would not agree with Harry Huffman’s dissing of accepted science and of the “greenhouse effect” in our atmosphere. http://green.harvard.edu/greenhousegas

                10

      • #
        Jose_X

        Thanks for posting a link there. I did not have a blogger account at the moment I tried to post, so was not able to post there.

        10

    • #
      Jose_X

      Joseph Postma, I have argued otherwise before, but I now think instead that both a greenhouse of glass and one made of a material that is transparent to IR (and to visible light) will have similar levels of IR radiation inside them (and hence temperatures). This seems possible even if glass absorbs large chunks of IR while the other material (like thin polyethylene) does not.

      The glass blocks the ground IR from leaving and blocks the “back” radiation outside from entering. This glass would absorb fair quantities of IR but then would re-radiate both outside and inside. So while thin polyethylene would allow much of this IR to pass through, glass would instead block/absorb at each boundary (in/out) but then would itself re-radiate both in and out to roughly make up for the radiation it was “blocking”. Glass does have a high emissivity at least in some regions where it absorbs IR (eg, 4-8 micrometer wavelength range). If what glass absorbs it can make up in re-radiation, then we will get a similar effect as with the polyethylene.

      Now, a greenhouse is different than a normal house, so there must be a difference between glass and thin transparent polyethylene (even though each has a different effect on IR than does the other) on the one hand and other “opaque” materials like concrete on the other. The difference — and this is why it makes sense to call the atmosphere heating effect a greenhouse effect — is that glass/polyethylene are transparent to some shortwave radiation while concrete and most other substances are not; thus, the inside of the greenhouse gets more total radiation than does a normal house. The inside of a greenhouse gets ordinary IR radiation that the inside of a normal house would also get, but the greenhouse also gets extra radiation at higher frequencies that do come from the sun but are not re-radiated by earth temp objects as one finds inside an ordinary house (like walls, glass, grass, water, plants, furniture, floors, etc). Atmospheric convection would shift heat around to balance out temperatures inside a greenhouse and inside a normal house with their respective surroundings.. unless you block convection. If we block convection in a greenhouse and in an ordinary house, we will have the greenhouse balance out at the higher temperature. Note that we can’t block conduction effects and convection effects on the outside surface of any house structure. These convection effects work to bring the greenhouse temp back towards average, but the shortwave energy coming in and the rate of conduction+convection outflow is such to stabilize the temperature a little higher than average.

      I suspect the inside of a dark car gets very hot because besides the glass shortwave transmission/greenhouse effect, we also have strong conduction effect on the metal exterior of the car bringing heat into the car internals initially faster than convection from air outside cools that same surface (note that radiation would be both towards the outside and towards the inside). As energy accumulates, the temperature rises inside. Eventually, the inside is hot enough that conduction through the metal works slow enough so that convection from outside air matches this rate and the temperature reaches a stable point. Apparently this effect in cars can lead to higher temps inside them than you find inside a typical greenhouse.

      10

      • #
        Jose_X

        I should add that besides convection and conduction at the outside surface of a greenhouse “fighting” to bring the temperature of the greenhouse towards equilibrium, the slightly higher temp itself is likely where the balance comes in. The higher temp implies higher IR (blackbody) flux, so this extra radiation (carried out as radiation by the glass indirectly or by polyethylene transparently) eventually matches the shortwave being absorbed (give or take). When this extra flux going out matches the extra shortwave in, we get stable temperature.

        10

    • #
      Jose_X

      Thanks Joanne for not removing my comment. I rewrote that section a few times and didn’t realize it still came across negatively. “Denier” is a label that I think I can reasonably apply to someone who is not willing to do the work necessary to understand something and refuses to agree with those who have put in much more time. It is very human to be this way (to be a denier). We can’t all do all the work to understand every theory man has put forward. To be a denier is not a sin. It means you go on faith (as we do on most things) paying attention to other beliefs and views you hold in to help you make a decision about that item you are allegedly denying. It is absolutely true and human that most of us don’t do all the legwork and do go on faith a lot. Also, I was not calling Paul a denier. Paul truly appears to be trying to work through this. I put “deniers” both in parenthesis and in quotations. I too am trying to work through this global warming stuff.

      I do sympathize with Postma, Paul, and anyone else not getting back to this conversation. If Joanne wants, I can try to put together a summary of some of these points if she feels another thread discussion on this is warranted. [I will try to read Postma’s paper later and would likely give opinions on it directly to Postma.]

      PS: I probably do get a little aggressive (or animated) in some parts. When I debate with someone, at some point I tend to mention that I might get a little unruly. I don’t want my tone to be misunderstood for disrespect. [I argue with and cuss/taunt/scream/etc at myself periodically when I am alone.] I have a physics background, but I am trying to learn this stuff like many people here. Some probably want to cause chaos, but most people believe a fair amount of what they write. In fact, on this very page, I changed my opinion about an important element of greenhouse gases. I then went back to a different webpage/conversation and updated my views to basically agree with an important point where I had disagreed. It was about an experiment done to confirm Wood’s 1909 result that a box with glass blocking doesn’t do much for temperature vs. a box with rock salt (or IR transparent covering) blocking. I had suggested the experiment did not test for the effect by cutting short the time of wait and possibly not testing one or two particular scenarios. I’ll review my comments there later, but I now have no problem accepting the main result of that experiment (although the “conclusion” of the paper went too far as it generalized to atmosphere greenhouse effect I think). Again, thanks for the forum, and I hope in the end we can all come to understanding of the best theories, whatever they be. If global warming were to be believed to be a real threat, I imagine most people at that point would think about their future generations and the comfy planet we have had handed down to us as a gift. If the global warming concern is misplaced, who wouldn’t want to breathe the sigh of relief? [There are always exceptions, of course.]

      10

  • #
    Brian G Valentine

    And yes, Michael has offered to write more.

    That’s good. British Parliament (lower House anyway) can get pretty “raucous” at times, which is the way truth is brought to light, ever since the time of the Hellenes.

    10

  • #
    Nullius in Verba

    “You have derived a method to extract heat from a cooler reservoir (stratosphere) to move it to a warmer reservoir (troposphere) without expending work.”

    First, the backradiation doesn’t come from the stratosphere, it comes primarily from the upper troposphere. Secondly, the net transfer – the combination of radiation up from the hot surface and the radiation down from the cold air – is still upwards. The net heat flow is definitely, certainly, and unequivocally upwards. The heat goes up.

    You cannot build a heat engine based on backradiation, because you cannot avoid getting “forwardradiation” as well. No more than you can fund your lifestyle from the change you receive in shops. Backradiation is only a part of the story.

    10

  • #
    Brian G Valentine

    Secondly, the net transfer – the combination of radiation up from the hot surface and the radiation down from the cold air – is still upwards.

    Should that be so, the troposphere will cool.

    No sale.

    : )

    10

  • #
    Nullius in Verba

    “Should that be so, the troposphere will cool.”

    No, the net heat flow is upwards, so the surface cools and the troposphere warms. (But they don’t change temperature because separately the surface is being warmed by the sun and the upper troposphere is being cooled by radiation to space.)

    10

  • #
    Leonard Weinstein

    Harry Dale Huffman:
    I have seen comments on Venus, and the claim that the adiabatic lapse rate fully accounts for the high surface temperature. That is not correct. The adiabatic lapse rate is a temperature GRADIENT. If there were no greenhouse gases present to raise the location of outgoing radiation going to space, which sets the location of the temperature at that altitude, Venus would be cool. BOTH LAPSE RATE AND ABSORBING GASES ARE NECESSARY FOR THE HIGH TEMPERATURE OF VENUS. Nullius in Verba has given the clearest description of the atmospheric greenhouse gas effect I have read (somewhere in this discussion), but I and others gave similar points, and people do not seem to get it.

    10

  • #
    Brian G Valentine

    and the upper troposphere is being cooled by radiation to space.

    It ALL is cooled by radiation, Nullius, and I would go along with your argument 110% if the stratosphere and troposphere were insulated from each other.

    Alas they are not, and that being so, we must accept the TRAGIC conclusion that we live in a world without a “greenhouse effect.”

    Which may not be so bad after all, but we must accept the religious zeal of some greenie groups as a reality.

    I think life would be a lot better for a lot of people without them.

    10

  • #
    Nullius in Verba

    “It ALL is cooled by radiation, Nullius, and I would go along with your argument 110% if the stratosphere and troposphere were insulated from each other.”

    The stratosphere and troposphere are not insulated from one another, but heat transfer between them is limited – partly because convection ceases at the top of the troposphere, and partly because there is no thermal gradient at the tropopause. The lapse rate reduces to zero here, and then reverses (becomes negative) in the stratosphere. The temperature increases with height, and the enhanced greenhouse effect actually leads to cooling of the stratosphere.

    But the stratosphere is irrelevant to the issue. 90% of the atmosphere is in the troposphere, and apart from ozone absorbing the incoming UV, the stratosphere is not really involved. All the stuff we’re talking about goes on down in the troposphere.

    10

  • #
    Brian G Valentine

    On the topic of Venus:

    I believe that many people have missed a big chunk of the story, insofar as the sulphuric acid clouds are a source of considerable heat for Venus.

    I believe that the sulphuric acid droplets come from the vapor phase reaction of sulphur trioxide and water, which is a highly exothermic reaction, and the the sulphuric acid is decomposed by Venusian volcanic heat on the surface to sulphur dioxide and water, the dioxide catalyzed to the trioxide in the vapor phase on dust particles present in Venus’s atmosphere.

    The vapor pressure of the droplets of the clouds is stabilized by the dust impurity, when the droplets coalesce to form larger droplets, they fall to the surface (and are subsequently decomposed to the dioxide and water again).

    In this manner, Venusian volcanic heat becomes a permanent part of Venus’s atmosphere, as it were.

    Certainly the atmospheric temperature profile of Venus is consistent with this picture.

    10

  • #
    Alistair

    Leonard Weinstein refers to Venus, the lapse rate being a temperature gradient but that the absolute level of temperature requires greenhouse heating.

    This is an interesting point because no light reaches the surface of Venus so if you define the greenhouse effect as the trapping of IR emitted from the surface, then Venus’ high temperature is not due to the greenhouse effect. Rather, it is caused by the absorption of energy in the atmosphere and the distribution of that thermal energy is set by the lapse rate: http://theendofthemystery.blogspot.com/2010/11/venus-no-greenhouse-effect.html.

    This paper shows that Venus and the Earth’s atmospheric temperature distribution is set exactly by lapse rate AND the difference of insolation.

    10

  • #
    Brian G Valentine

    For not accepting the atmospheric Greenhouse effect, I’ve been called

    – a “blithering idiot” (Eli Rabbit)

    – a “turkey” (Tamino)

    – a “know-nothing fool” (Deltoid)

    It ain’t all that easy to be a “no greenhouse effect denialist,” believe me.

    10

  • #
    Nullius in Verba

    “insofar as the sulphuric acid clouds are a source of considerable heat for Venus”

    Sure. But the temperature profile is adequately explained by the fact that there are opaque clouds at 50-80 km altitude, combined with a 8 C/km adiabatic lapse rate in a convective atmosphere. Just as the condensation of water vapour alters Earth’s lapse rate from 10 C/km to about 6 C/km, so the reactions of sulphuric acid will affect that of Venus. But the basic mechanism is the same.

    Outgoing radiation from the surface is absorbed by the opaque clouds and re-emitted more slowly at a lower temperature. On Venus even more than Earth, backradiation is almost exactly cancelled by forward radiation, and has no net effect.

    10

  • #
    Nullius in Verba

    “For not accepting the atmospheric Greenhouse effect, I’ve been called”

    My sympathies. I’m not calling you any names here. Understanding didn’t come easily.

    It’s not easy for any of us.

    10

  • #

    Jo:

    The blanket you refer to is an insulator, you supply the heat, just like clouds at night don’t allow the earth to cool off as fast.

    This common sense cooling effect of water vapor was proved following the 9-11 terrorist attacks. Atmospheric scientists studied the effect of water vapor on temperature in the wake of the attacks. The Federal Aviation Administration (FAA) prohibited commercial aviation over the United States for three days following the attacks and this presented a unique opportunity to study the temperature of the earth without airplanes and their contrails.

    Dr. David Travis, an atmospheric scientist at the University of Wisconsin, along with two other scientists, looked at how temperatures for those three days compared to other days when planes were flying. They analyzed maximum and minimum temperature data from about 4,000 weather stations throughout the conterminous (48 states) United States for the period 1971–2000, and compared those to the conditions that prevailed during the three-day aircraft grounding period and the three days when planes were flying before and after the grounding period. This research effort was sponsored by grants from the National Science Foundation.

    They found that the average daily temperature range between highs and lows was 1.1 degrees C higher during September 11-14 (without air traffic – no contrails)compared to September 8-11 and September 14-17 with normal air traffic.

    The data proved that contrails (water vapor) have a net cooling effect. You cannot just look at a nighttime effect only, like the IPCC climatologists and meteorologists have done, both day and night must be included to determine the overall effect. Water vapor, CO2 and particulates in the atmosphere all reflect as well as scatter some radiant energy back to outer space and this causes cooling.

    Here is a simple test, go outside when the sun is shining, see how warm you feel when you are in the direct sunlight and compare that with how warm you feel when a cloud goes overhead and you are in the shade of the cloud. Of course you feel cooler in the shade of the cloud; a child knows this. So Dr. Travis confirmed this with scientific analysis of real data that most people on this planet already know.

    Everyone also knows that cloud cover at night (more insulation) prevents the earth from cooling off as fast as it does when there are no clouds. However, on a relatively clear night if a cloud goes overhead you cannot feel any warming effect of the cloud, so this insulating effect is shown to be minimal compared to the daytime effect.

    The IPCC adopted the work completed by Kiehl and Trenberth of the National Center for Atmospheric Research, Boulder Colorado to show how radiative forcing from greenhouse gases causes the earth to warm. Here is a statement from that paper:

    The long wave radiative forcing of the climate system for both clear [125 W/m2 (watts/square meter)] and cloudy (155 W/m2) conditions are discussed. We find that for the clear sky case the contribution due to water vapor to the total long wave radiative forcing is 75 W/m2, while for carbon dioxide it is 32 W/m2.

    Really, when the water vapor concentration in the atmosphere averages around 1 volume % (or 10,000 ppmv) and carbon dioxide concentration is less than 400 ppmv? The CO2 concentration is only 4% of the water vapor concentration. In the Hottel and Egbert correlation the only difference between water vapor and carbon dioxide regarding the radiation effect is their partial pressures. Partial pressures of gases are proportional to their volumetric concentrations. Based on this and using the water vapor effect as a basis at 75 W/m2 then the CO2 effect would be 3 W/m2, not the 32 W/m2 stated. But all of this is wrong anyway both water vapor and CO2 cool the earth.

    The first problem with the Kiehl and Trenberth graph is a serious violation of the First Law of Thermodynamics. Completing a balance only on the energy entering and leaving the earth, one sees on the middle to left side of the graph that 161 W/m2 hits the earth from the sun. The earth then loses 97.9 W/m2 to thermal and evaporation/transpiration losses with 0.9 W/m2 being retained by the earth. This leaves only 63.1 W/m2 not accounted for. However, on the right side of the earth it shows 396 W/m2 leaving the earth. If you only have 63.1 W/m2 available, no more than that can leave the earth. Well you say an additional 333 W/m2 in back radiation from the clouds hits the earth and this then provides the balance. Mathematically you can do anything you like of course, as long as input equals output, but where is the reality in such a computation?

    The only problem here is, where did this added energy come from? It came from the bogus 396 W/m2 shown radiating from the earth’s surface to the clouds. If what was presented here were true, for every unit of energy in, one would get back an additional (396 ÷ 63.1) = 6.28 units of energy. The First Law of Thermodynamics states that energy can be transformed from one form to another but cannot be created nor destroyed. Therefore, the graph violates the First Law of Thermodynamics.

    It also violates the Second Law of Thermodynamics by showing energy radiating from cooler clouds back to a warmer earth. One version of the second law states no process is possible where the sole result is the transfer of heat from a body of lower temperature to a body of higher temperature. Maybe they are still counting on the non-existent hot spot in the greenhouse signature to accomplish this. Real data shows the IPCC computer models, both for predicting the effect of CO2 on the earth’s temperature and the climate signature, are misconstrued.

    As a final rebuttal of the influence of carbon dioxide over the climate, the alleged IPCC greenhouse effect is a non-existent effect. No greenhouse, whether made from glass, plastic, cardboard or steel will reach a higher inside temperature due to the magic of re-radiated IR energy. If it did, engineers would have long ago been able to design power stations made from air, mirrors and glass, extracting more energy out of it than was put into it – if only!

    http://www.tech-know.eu/uploads/Greenhouse_Gases_Cool_Earth.pdf

    10

  • #
    Brian G Valentine

    Well, thanks for your sympathies, and the subject is so emotional to some people, I wouldn’t blame you if you did call me names. Frankly the existence of a greenhouse effect is some people’s reason to get up in the morning, as we all recognize.

    I do enjoy and appreciate the opportunity to debate to reach the unvarnished truth, wherever that leads us.

    Life would be no fun if we all agreed on everything.

    10

  • #
    Nullius in Verba

    “The Federal Aviation Administration (FAA) prohibited commercial aviation over the United States for three days following the attacks”

    I remember that study. If I remember correctly, the diurnal temperature range was almost on the 95% border of the normal spread, and if you hadn’t been looking for it, would not have stood out as anything unusual. They also reported that the local weather at the time made this sort of range more likely, so attribution was ambiguous.

    “Completing a balance only on the energy entering and leaving the earth”

    By “the earth” do you mean the surface of the Earth, or the Earth-atmosphere system?

    The net radiative longwave transfer is 396-333 = 63 W/m^2 upwards, just as is required for balance at the surface. The numbers are just how much each part radiates towards the other. That the actual radiation being emitted largely cancels out doesn’t violate the first law. Take a slice through any opaque material at constant temperature, and the radiation emitted by one side across the surface towards the other is exactly balanced by the radiation emitted by the other side back. Nothing changes as a result, so we usually ignore it, but it goes on all the time.

    10

  • #
    BobC

    @ Brian G Valentine (#109)

    Given the people who are calling you an idiot, that is probably a compliment!

    I like all your arguments except the one from the 2nd law of thermodynamics, as it is essentially circular (as are all “laws of physics”):

    1) “A” is a law of physics that all phenomena must obey
    2) Phenomena “B” apparently violates law “A”
    3) Therefore, phenomena “B” doesn’t really exist.

    The proper view is that laws of physics are all provisional — when you find something that “violates” one, it must be demoted. To do otherwise is to try to simplify the world by increasing your ignorance — a very human tendency, but not a valid response from an empiricist’s point of view.

    It is true (as you say @97) that violation of the 2nd law of thermodynamics implies the existence of perpetual motion machines of the second kind — that is, machines that cannot create energy, but only recycle it by decreasing entropy.

    Like you, I find the potential existence of such machines bizarre — however, I find lots of things about the observed universe bizarre 🙂

    There is, however, a widely used electro-chemical machine that is credibly observed to violate the 2nd law of thermo (linked in my post #59) — I refer to modern solid-electrolyte water electrolyzers operating below the thermoneutral voltage.
    Note that the limiting electrical efficiency of an electrolyzer, as voltage goes to zero, is 130% — at that point, 30% of the energy to split water is coming from the ambient heat, with no requirement for a lower temperature reservoir as required by the 2nd law.

    This is a very well established fact — so I see no alternative but to demote the 2nd law to a useful rule for analyzing heat engines, which is where it started out.

    (At least some) electro-quantum machines obey different laws.

    10

  • #

    The stratosphere and troposphere are not insulated from one another, but heat transfer between them is limited – partly because convection ceases at the top of the troposphere, and partly because there is no thermal gradient at the tropopause. The lapse rate reduces to zero here, and then reverses (becomes negative) in the stratosphere. The temperature increases with height, and the enhanced greenhouse effect actually leads to cooling of the stratosphere.

    Heat transfer is not limited, it is simply irregular. Most of the heat transfer comes in the form of breaking gravity waves which dissipate entropy in the troposphere in the form of momentum. Also, the tropopause is not at a fixed height, it will rise when energy in the atmosphere increases.

    I’m a bit sceptical of the claim that increasing energy within a system will create a cooling effect. That doesn’t pass my bull$#!t detector. But, I don’t know enough about the wherefore and whys of that claim to refute it.

    10

  • #
    Leonard Weinstein

    Alistair:
    You need to go to Science of Doom’s blog site and find the long discussion he and I had just on that issue. First, some light does reach the ground, but it is small. I contend that even if none does, the adiabatic mixing of the atmosphere maintains the adiabatic lapse rate, and the high elevation of outgoing radiation plus lapse rate do the rest. However, the dense cloud cover complicated the issue. Outgoing radiation has a combination of cloud to space and upper troposphere to space to set the average location of outgoing radiation. Nevertheless, even without any clouds, the result would probably be close to the same.

    10

  • #
    Nullius in Verba

    “There is, however, a widely used electro-chemical machine that is credibly observed to violate the 2nd law of thermo”

    The increase in entropy at the power source exceeds the decrease in entropy involved in the cooling. It no more violates the 2nd law than a Peltier cooler, or a refrigerator.

    10

  • #
    Nullius in Verba

    “’m a bit sceptical of the claim that increasing energy within a system will create a cooling effect.”

    I’m not claiming it does.

    The warming/cooling effect is created by the raising of the altitude of emission direct to space. At any given level, you get temperatures that you previously saw some distance below you. i.e. the entire temperature profile is shifted upwards. If it’s warmer below you, as in in the troposphere – then you get warming. If it’s cooler below you, as in the stratosphere – you get cooling.

    Like I said, the profile within the atmosphere has a lot more to do with the adiabatic lapse rate than radiative energy balance. If the lapse rate goes negative, then the sign of the temperature change changes too.

    10

  • #
    Bryan

    This topic has been about the most misunderstood parts of science ever encountered.
    It has however given rise to some comic exchanges.
    Eli Rabbit(Halpern) and the gang of six published an attack on the Gerlich and Tscheuschner paper.
    Halpern decided that because G&T said that heat did not move from a cold surface to a hotter surface this meant that G&T said that a colder surface could not radiate to a hotter one.
    Had Halpern looked properly at the G&T paper with several diagrams showing a two way radiative interaction he would have been saved from a major embarrassment.
    However because G&T did not reply for months the Halpern misunderstanding persisted.
    So much so that some G&T supporters accepted Halperns distortion and began to defend the no radiation gaff.
    Of course all G&T had to say when they finally replied was “where did we say that”

    Leonard Weinstein gets it correct above when he says

    ..”You both made misstatements, and your entire problem hinges on the definitions of heat transfer and energy transfer. You both are being careless and confusing them. The second law relates to heat transfer (one way), and the photon transfer is energy (bidirectional). Please get that straight.”…….

    10

  • #
    Alistair

    BobC: I forget the exact law, but when you plot log current vs potential in electrolysis, the thermoneutral voltage is the extrapolation of the linear part to the abscissa. You still get ionic current below that but it asymptotes to zero.

    So, what you have is the effect of the Maxwell-Boltzmann distribution extreme, not some magic deviation from the 2nd Law.

    To do the latter, you have to be clever in electrochemistry: I patented an effect in 2003 which is truly astonishing physics.

    10

  • #
    BobC

    You aren’t thinking carefully about this, Nullius:

    Nullius in Verba:
    May 15th, 2011 at 2:47 am

    “There is, however, a widely used electro-chemical machine that is credibly observed to violate the 2nd law of thermo”

    The increase in entropy at the power source exceeds the decrease in entropy involved in the cooling. It no more violates the 2nd law than a Peltier cooler, or a refrigerator.

    1) An electrolyzer can have (up to) 130% electrical efficiency — the rest of the energy to produce H2 and O2 from water comes from the ambient heat.

    2) A hydrogen-oxygen fuel cell can easily achieve 96% efficiency at producing electricity from converting H2 and O2 into water.

    You can easily combine these two elements in a closed loop, both within the single heat reservoir. The combination of the two can produce excess electricity (as long as the electrolyzer is run far enough below the thermoneutral voltage so as to obtain > 104% electrical efficiency). The excess electricity can be taken out of the reservoir (via wires, or EM waves) and do useful work elsewhere, as the reservoir cools down.

    This is really pretty obvious. Let’s not get into an argument over this on Jo’s website. If you’re really interested, I hereby give Jo authorization to give you my email.

    10

  • #
    BobC

    Alistair: Solid-Oxide Electrolytic Cells (SOEC) are routinely run at high temperatures, below the thermoneutral voltage in order to get more H2 and O2 for a given amount of electricity. (Heat energy is cheaper than the equivalent amount of electrical energy.)

    See Here:

    The process of water decomposition in a SOEC runs at an acceptable rate under an average cell voltage of 1.1 V, which corresponds to the specific power inputs 2.63 kWh/nm3H2. This is less than 75% of the theoretical electric energy input. The deficient energy for water decomposition is supplied in the form of heat.

    The thermodynamic efficiency of an electrolytic cell is defined as the energy of the hydrogen produced divided by the electric energy consumed by the cell. Thus, the efficiency of the best low temperature electrolytic cell is only 87%, while an SOEC has an efficiency of up to 135%.

    Alistair:
    May 15th, 2011 at 3:32 am

    BobC: I forget the exact law, but when you plot log current vs potential in electrolysis, the thermoneutral voltage is the extrapolation of the linear part to the abscissa. You still get ionic current below that but it asymptotes to zero.

    So, what you have is the effect of the Maxwell-Boltzmann distribution extreme, not some magic deviation from the 2nd Law.

    There is no “magic” involved in the second law having a limited domain of validity — just empirical fact.

    10

  • #
    Alistair

    BobC: SOFCs have high efficiency because entropic effects don’t have so much of an effect.

    Few people understand electrochemistry. I don’t claim to have special knowledge but to get unusual effects you have to apply irreversible thermodynamics or something else. my own work was producing entropy waves by cooperative tunnelling of protons. The patent was an electrochemical cell having two anodes [for a period of a few ns]. Otherwise it’s Nernst, Maxwell Boltzmann etc.

    10

  • #
    Lionell Griffith

    It is always possible to get a greater than 100% efficiency of energy transformation if you don’t account for ALL of the energy in all of its forms required to make the transformation in your computation of efficiency. That is cheating any way you look at it.

    That is like running an automobile on an AAA battery and ignoring the 258 CC V8 that is actually doing the work.

    10

  • #
    Brian G Valentine

    SOFCs have high efficiency because entropic effects don’t have so much of an effect.

    That’s true. The theoretical available work from the fuel cell is

    Delta(G) = Delta(H) – T DELTA(S)

    For the reforming of methane -> hydrogen + CO -> oxidation to CO2 and water, the DELTA(H) is high, although DELTA(S) is low (although increases with temperature, fortunately not all that much so the fuel cell can operate at a high temperature).

    I like them, although they are pretty expensive (noble metal reformers) and they don’t seem to stand up to cycling, which they have to do a lot of (since they are usually back up to something else)

    10

  • #
    Alan Siddons

    Joanne’s interpretation is that GHGs inhibit radiational cooling. That’s the blanket theory. In this view, the sun brings the surface to a certain temperature and GHGs act to sustain it, thus increasing the average temperature by limiting how fast the nighttime earth can cool. The principal problem with this, of course, is that the earth is seen to emit all of the energy it gets from the sun, i.e., there’s no sign of a blanketing effect.

    This, of course, is profoundly embarrassing to greenhouse theorists — which is why the actual theory isn’t what Joanne describes. Theorists will admit yes, the earth appears to be naked. But that’s only an illusion, they’ll argue. In fact, the GHG blanket does indeed block the exit of light, cloaking the earth and making it darker as seen from space. But since this blanket gets warm, it emits infrared back to the earth, the result of which is to heat the planet so much that it radiates the SAME AMOUNT as it would without the blanket.

    Ergo, the earth without a greenhouse effect emits about 239 W/m² to space. And the earth WITH a greenhouse effect emits about 239 W/m² to space.

    Reminds me of a joke. Gunslinger brags about how fast he can draw and stands still like a statue. Then he asks, “Wanna see it again?”

    Anyway, although greenhouse proponents often start with a blanket analogy, they’re soon compelled make it an electric blanket, thereby contradicting their initial premise. Rather than inhibit cooling like passive insulators, their GHGs are active heat sources. These heat sources warm the earth below while simultaneously making it appear that the earth isn’t being warmed at all. Any introduction to greenhouse theory will prove this. Introductory models have 239 W/m² emitted upwards from the surface and 239 W/m² returning, thus bringing the total to 478, enough to bring the surface temperature to 303 Kelvin. Once this impossible point is conceded, determining the “correct” temperature from back-radiation is only a minor detail. Lindzen, for instance, estimates the average surface temperature from this extra radiation as a blistering 350 K.

    But the average temperature of the earth’s atmosphere is far lower than the surface, and it remains that a cooler body cannot heat a warmer body. Greenhouse theory does violate the 2nd Law. It’s not a matter of semantics.

    Hammer’s post is a sad indication that after everybody else has finally abandoned the self-contradictory intricacies of Greenhouse Theory, AGW skeptics will be standing among its last defenders.

    10

  • #
    Roy Hogue

    Ad infinitum! 🙂

    This debate is utterly fascinating. That so many with such good credentials disagree so completely is amazing. Or at least would be if you don’t realize that it’s all theory with no way to perform an experiment that would prove it or falsify it. The devil as they say is in just such little details.

    I know which position I find convincing but I’m at the point where I realize the whole debate is only academic. The problem is this — it is now abundantly clear that CO2 is not the driver of temperature/weather/climate on this planet. It controls none of those and we all know it. Something else is in the driver’s seat.

    By all means continue the debate, I read every bit of it. But perhaps we should spend our time hitting hard on the evidence that CO2 just doesn’t matter enough to bother with?

    Thank you Jo for getting it out in the open that the Earth isn’t being warmed; only the rate of cooling is slowed down. After I realized what “global warming” really is, both these positions became more easily understood. The 2nd law is not violated — no new heat is created. Co2 may well be doing its thing. But so far we can’t even tell if it is. And there is the sun after all.

    We have to keep hitting the world with the overwhelming evidence that CO2 is not the climate driver.

    10

  • #
    Brian G Valentine

    Actually, Alan, the “greenhouse” analogy with a blanket isn’t too bad at all.

    – The idea “smothers” the flame of progress, making it impossible to conduct any industry in the world

    – Brings about a depressing mood thinking about it or in conversation (“wet blanket”)

    – Provides security to those who have no use for humans or human activity

    10

  • #
    Thierry

    Hello everybody.

    This whole article from Hammer is really pointless. Nor he has carefully read G&T papars (the orignal and their response to halpern) neither he really understands the second principle.

    If a cold body could warm a warmer one, why don’t everybody is rushing to invente and build a heat cavity (black body) to trap the energy from up above ? This could be an endless source of energy for the world….

    Where are the experimental validations of the greenhouse effect ? None exists.

    As well as the blocking of cooling. Why does CO2 only block outgoing radiation and not downcoming radiation from the sun ?

    This whole story is ridiculous.

    10

  • #
    Bulldust

    Peronally I am open to arguments that the GHG effect exists or not … it matters not a whit to me because every line of reasoning beyond that by the politicians, advocates, and scientists is flawed anyway (on the CAGW side). The feedbacks don’t work, the economic arguments don’t work, the tax proposed in Austrealia will change nothing (other than a transfer of wealth) and the politicians are wrong (i.e. the PM going to an election promising no tax and now pushing for one). In the grand scheme of things we should get the physics right, but it pales compared to the importance of not crippling an otherwise sound economy.

    Don’t hobble the economy and we will all have more money and time to discuss the finer points of physics without wasting time on things that should not be.

    10

  • #
    Siliggy

    Does this simple experiment settle the issue?
    Can radiation from a colder object cause a warmer object to warm more?Experiment:
    In a temperature controlled room connect an incandescent lamp to an accurate and stable Voltage regulated DC supply in series with a current meter. The lamp needs to be a higher colour temperature than any other light in the room.
    Record the DC current through the lamp.
    Now switch on a second identical lamp next to this one. The second lamp is to be running from a lower regulated Voltage so that it is colder than the first lamp.
    Does the current measured for the first lamp decrease?
    A decrease in current would be due to the positive temperature coefficient of the first lamp indicating warming.

    To improve this experiment seperate the two lamps by about a meter to cause conduction and convection effects to be slow.
    Switch the second lamp on and off at 1 second intervals.
    Does the current through the first lamp show the 2Hz signal?

    10

  • #
    Siliggy

    Ooops 1/2 Hz.

    10

  • #
    BLouis79

    @Nullius in Verba (#94) “No climate scientist proposes that net heat flows up a thermal gradient. That is not the theory. That is not what they say. “

    If that’s not it then, what exactly do they say, since I have been unable to find one who can explain exactly the physical process (and defining laws) behind the “backradiation” or “insulation” theories.

    Simply, if “backradiation” exists but the net heat transfer effect is nil, then it is of no effect. Secondly if it acts as radiation “insulation”, how many milliseconds does it last at the speed of light?

    A simple lab experiment:
    1. IR laser, shining at a
    2. chamber to put gas in, with
    3. instruments to measure temperature at various points and IR scatter

    Please explain:
    How much % energy will be absorbed by CO2?
    How much % energy will be scattered by absorption/radiation?
    How much % energy will be scattered in the direction of the source (backscatter)?

    By what physical process this happens?
    What formulae are use to calculate the answers?

    10

  • #
    David, UK

    @ Lionell Griffith: Sorry my friend, but for the first time since I started visiting this site several months ago, I have clicked on the old “thumbs-down” icon below your comment.

    I’ll skip to the end of your comment: “Thus a colder object cannot heat a warmer object. The author’s thesis is absurd.”

    But the author wasn’t suggesting for a moment that a colder object can heat a warmer object. He was simply stating the fact that energy from both the relatively colder object and the relatively warmer object travel in all directions. Obviously the warmer of the two objects emits more energy and so the net movement is from warmer to colder. Now, no one is saying that CO2 literally “warms” the planet. That’s the job of the Sun. But CO2 and any other matter capable of absorbing energy emitted from the earth (that originally came from the sun) is equally capable of emitting it BACK, thus making the earth’s net energy loss ever so slightly less.

    HOWEVER, in the infinitely complex climate system, I draw the line at concluding that therefore if we add a smidgen of CO2 to the already abundant mass of the stuff it will lead to runaway catastrophic global warming. That really is BS. Especially as CO2 becomes ever less effective at capturing heat energy the more abundant it becomes. Think of the example Jo gave of keeping warm under a blanket. Said blanket absorbs some of your body heat and emits some of it back, thus allowing your body to work less hard to stay warm (or get warmer). Add another blanket or two and you may get warmer still. But keep adding more blankets and the effect of each blanket is less than before until you reach saturation level where adding blankets makes no difference at all. CO2 levels are already at such a level that the stuff we are adding makes very very little difference. Not to mention all the other forcings and feedbacks that likely collectively keep our climate quite stable (otherwise how on earth did we manage to last this long without the seas boiling away at any one of numerous occasions when CO2 levels have been higher).

    10

  • #
    RJ

    david UK

    Said blanket absorbs some of your body heat and emits some of it back

    It does not. Blankets simple slow down the rate of cooling

    And what is this net movement.

    the net movement is from warmer to colder

    Are you suggesting that a proton from a cold object can in fact heat a warmer object as long as the opposite is happening at the same time. This just does not seem logical or scientific to me. Either heat moves from hot to cold or it moves in both directions. Hot to cold is the correct option IMHO. Net does not enter the picture.

    10

  • #
    RJ

    Sorry photon

    10

  • #
    Nullius in Verba

    “If that’s not it then, what exactly do they say, since I have been unable to find one who can explain exactly the physical process (and defining laws) behind the “backradiation” or “insulation” theories.”

    The “backradiation” theory is what I call the “shells” model. It assumes that heat is only transferred by radiation, and therefore does not apply in a convective atmosphere, but is a self-consistent and thermodynamically valid model.

    The usual way it is introduced is to imagine the atmosphere divided into concentric shells, each one unit optical depth in thickness – so it can be treated as opaque (to IR). The system as a whole is heated by w Watts of incoming sunlight at the bottom of the atmosphere. The system is in equilibrium, steady state, so the outermost shell must radiate w Watts outward into space, and must be at the right temperature to make this happen. Because the radiation is isotropic, the shell radiates the same amount inwards. To balance this and not change temperature, the outermost shell must receive 2w Watts input which it can only get from the shell below. The next shell in therefore radiates 2w up, and 2w down. It’s receiving 1w from the shell above, so must be receiving 3w Watts from the shell below to balance its output. And so on. The next shells radiate 4w, 5w, 6w, etc. up and down, the power increasing linearly with optical depth, and hence the temperature rises as the fourth root of optical depth. Since optical depth is proportional to density, and density increases approximately exponentially as you descend (inverse of pressure), so the temperature rises exponentially as you descend below the top of the atmosphere. If you plug the numbers in for Earth, the average surface temperature would be about 60 C.

    However, at no point does net heat flow from cold to hot. For any shell radiating n units of power up and down, it receives n-1 units from the shell above, and n+1 from the shell below. Thus the *net* flow is 1 unit from the shell below, passing upwards to the shell above, hotter to colder. You *never* get heat coming down without even *more* going up.

    Another way of looking at it is to think of it as a relay where heat is passed from layer to layer, but *because* heat can only flow hotter to colder, each relay has to be hotter than the next one in the chain to get the energy to flow across it. Otherwise it will just hang around where it is, building up like a sand pile until the slope is steep enough.

    The shells model is a nice introduction to the historical concept as developed by Fourier and Tyndall, but is obviously falsified by reality – as Manabe realised back in the 1960s. (It’s too hot and the wrong profile.) They developed a new model in which convection dominated, but still taught the shells model as the basis with convection being a “small correction” to the model. It’s this that has caused much of the confusion.

    The problem is even more acute in a pool of water, as I mentioned above. But it’s difficult trying to argue both for and against the model simultaneously – no, it doesn’t violate the second law, but it isn’t true either. That’s just confusing. So my apologies for that.

    10

  • #
    Roy Hogue

    RJ @138,

    The fallacy is this:

    The colder body doesn’t know it’s near a warmer or a colder one. It just has a rule that says radiate periodically in some random direction. And so some photons go off in the direction of the warmer body.

    The warmer body doesn’t know if it’s near a colder body or a hotter one. Indeed, neither one knows the other even exists, much less whether it’s warmer or colder. But the rule for an incoming photon is, if it’s at a wavelength I can absorb I must absorb it. And so it does.

    But there’s yet another rule — the higher your temperature the more often you must radiate a photon. The warmer body sends out more than it receives, the colder body receives more than it sends out. Net transfer is from warmer to colder and “net” does indeed count. Remember, each body operates without knowledge of the other. Incoming radiation, absorb it. Radiate as a function of your absolute temperature. That’s all they know how to do.

    10

  • #
    MaxL

    Lionell Griffith@3

    3. Any of the heat from 1 that radiates back to the warm object simply REPLACES the prior lost heat.

    Is this not the same process that Michael Hammer is referring to?
    If so then surely the statement: “A cooler body can reduce net heat loss from a warmer one”, means Jo’s phrase of “less-global-cooling” is quite accurate.

    10

  • #
    michael hammer

    Wow get involved with other things for a few hours and suddenly there are 50+ new comments. To try and answer a few;

    comment 71 – yes the radius is larger by about 20km in 6400 km but the temperature difference 220K at the tropopause cf 288k at the surface is a far larger effect

    comment 72 – this post is exactly about debating the science which is what you are saying we should be doing. You say radiation is a very small component of surface energy loss. On the contrary, I would claim that radiation is the dominant mechanism of radiation loss from the surface. Followed by latent heat via convention. Your analogy of a teacher and child and one way informati0on flow is flawed. If a photon strikes an object that photon is either reflected or absorbed (whether it is absorbed depends only on the absorptivity of the object and for the earths surface in the IR this is high). If absorbed its energy is transferred to the object.

    comment 77 – Your equation is right. The T2 term is the back radiation term. The difference is that with a transperent atmosphere T2 is the temperature of outer space ie: 4K. With an oapque atmosphere it is the temperature of the atmosphere whcih is far higher than 4K. The effect of CO2 is to make the atmosphere opaque at 15 microns. E in your equation is not the emissive power, it is the energy loss per unit area. Clearly if T2 is higher then E will be lower. If E is lower the surface is losing less energy. If the energy input from the sun remains the same and the surface loses less energy then it would seem to be obvious that the equilbrium is disturbed and the surface temperature will rise until T1 has increased enough to restore E to balance the incoming energy.

    comment 80 – As Jo said, you entirely missed the point. I am not disagreeing with IPCC on the second law of thermodynamics issue, I am agreeing with them

    comment 82 – see comments referenced to 72 and 77 above

    comment 84 – see comment referenced to 72 above

    comment 91 – temperature is a measure of the energy per molecule. A very high tempertaure means the energy per mooecule is very large. However as you correctly suggest, if the density of molecules is very low the total heat energy per unit volume can still be low even if the temperature is high.

    10

  • #
    michael hammer

    I have to say I am getting a bit tired of repeating myself. Let me try to put it another way. If you lie in bed without a blanket you lose a lot of energy and feel cold. If you now cover yourself with a blanket, the blanket reduces your energy loss. With a reduced energy loss you get less cold than you otherwise would have. Whether you consider “get less cold” to be semantically equivalent to saying “you will be warmer than you otherwise would have been” is up to you but I point out that general useage would say the blanket warms you. This is despite the fact that the blanket is colder than you are. No the blanket being colder does not transfer NET heat from itself to you, it merely reduces the energy loss allowing your internal heat generation to raise your temperature more. If you put a blanket over a piece of cold steel it does not make the steel warmer.

    An exactly analogous situation exists with respect to Earth. There is an external energy input notably the sun. An opaque atmosphere reduces the energy loss from the surface to space which allows the energy input from the sun to raise the temperature slightly. The effect of more CO2 is to very slightly increase the range of wavelengths around 15 microns at which the atmosphere is opaque.

    The mechanism by which this energy loss is reduced cannot be by reducing the heat radiated by the surface because the atmosphere cannot influence the emissivity of the surface. Rather it acts by returning some of the energy radiated back to the surface. This is the back radiation.

    If you want the analogy with a blanket to be more accurate consider the survival blankets which are simply a silvered sheet of thin plastic. Clearly the thin plastic has neglegible impact on conduction. It could act by reducing convection but then again it does not need to be silvered to do that. A transparent sheet of plastic would do that just as well yet a transparent sheet of plastic does not work anywhere near as well as a silvered sheet. The silvered sheet works so well becuase the shiny surface has very low absorptivity and emissivity so it loses very little energy by radiation. There is still a differrence in that the silvered surface reflects the energy back onto your body rather than via an absorption and then emission process but the overall impact is very similar.

    10

  • #
    michael hammer

    comment 97 Brian valentine – you have not understood correctly what I have been saying and you explanation of a perpetual motion machine is completely wrong. I can put the nett effect a different way. Situation 1 is a hot object (Earth’s surface) radiating directly to a very cold object (outer space at 4K). Situation 2 is to interpose an opaque object between the hot and cold objects. This opaque object by absorbing energy from the hot object and radiating to the cold object will come to an equilibriuum temperature somewhere between the hot and cold objects. Since all the energy radiated by the hot object is intercepted by the opaque intermediate object, the environment for the hot object is changed from that of outer space to that of the opaque object. Its like moving from being outside on a cold winters night to moving inside a warmer room. The hot object will lose less heat becuase of its warmer surroundings. If it loses less heat, the incoming solar energy will heat it to a slightly higher temperature. The impact of more CO2 is to make the atmosphere opaque over a very slightly larger range of wavelengths.

    That does not imply in any way shape or form NETT heat transfer from the cooler opaque layer to the warm central object which could then be used in a perpetual motion machine. You seem to be assuming warming means there must be NETT heat flow from colder to hotter. NOT SO! Simply reducing heat outflow from the hot object upsets the equilibium allowing the incoming solar energy to further raise the temperature.

    The mechanism by which this heat outflow is reduced is NOT by reducing the radiation of the hot object bt rather by returing some of that radiated energy back to the hot object. Such return could be done by reflecting some of the energy eg: a mirror or it could be acheived by back radiation which is the case for the atmosphere.

    10

  • #
    Joe V.

    RJ@#10

    This GHG backradiation theory is on shakey ground yet people like Monckton still seem to accept it without question.

    I don’t think it’s so much that Monckton necessarily accepts it, but he tends to give warmists the benefit of any doubt “solum ad argumentum” before demolishing them with their own arguments.

    10

  • #

    MaxL @ 142:

    Stop acting like a troll and reproduce exactly what I said rather than intentionally misrepresenting it. Put that extracted statement in the full context of my post.

    I will have no further conversation with you until you demonstrate a sincere effort to understand what was said. I don’t expect agreement but I do demand a demonstrated understanding which you have as yet failed to show.

    10

  • #
    Joe V.

    Jo@#9

    Instead of calling it “global warming”, I guess they could have called it “less-global-cooling”. I can’t see it catching on.

    Yes, it’s an interesting idea. Though for the same amount of actual heating (by the Sun), if there is less cooling, then the net effect is still warming. So you’re dead right, it ain’t gonna catch on .

    10

  • #
    MaxL

    Lionell,

    First argument:
    1. ANY heat absorbed by the cold object that is coming from the warm object is no longer in the warm object.
    2. The warm object is colder by that amount.
    3. Any of the head from 1 that radiates back to the warm object simply REPLACES the prior lost heat.
    4. The warm object simply returns to its original temperature.
    Conclusion: back radiation does not and cannot warm a warm object beyond its original temperature. In fact it is always less because the “back” radiation is re-emitted spherically rather than totally reflected back to the warm body.
    Every other incident source of heat on the warm object is sourced from an EXTERNAL warmer object even if it is via the so called colder body.

    I agree with everything you said in your argument, and I agree with your conclusion.
    I was simply asking for clarification on point 3, as to whether it is the same process that Michael has described.

    10

  • #

    MaxL,

    Like I said, I don’t expect agreement but I demand understanding. Taking 3 out of context totally misrepresents what I said and supports your willful misunderstanding. Read what I said again and try very hard to understand what I actually said.

    The only clarification I am willing to add is “head” is a mistyped “heat” which you apparently already understood.

    10

  • #
    BobC

    Roy Hogue @ 128:
    May 15th, 2011 at 5:39 am

    Ad infinitum! 🙂

    This debate is utterly fascinating. That so many with such good credentials disagree so completely is amazing. Or at least would be if you don’t realize that it’s all theory with no way to perform an experiment that would prove it or falsify it. The devil as they say is in just such little details.

    The history of science shows that it is far easier to explain a phenomena when it is well known, than it is to predict anything that hasn’t been measured. The problem with GHG effects is that they can’t easily be measured, so there is no “ground truth”.

    In 1962 the US set off a 1.4 Megaton bomb at 250 mi altitude about 1000 miles SW of Hawaii (Project Starfish Prime). The bomb created an Electromagnetic Pulse sufficient to knock out power grids and equipment in Hawaii, and blew a hole in the Van Allen belts while injecting enough high energy electrons into the Earth’s magnetic field to disable 1/3 of the existing satellites in low Earth orbit.

    Although all of these effects can now be easily be explained by undergraduate physics, none of them were predicted by the PhD physicists who designed the experiment.

    This history does not give one confidence in the ability of climate models to predict future climate. When the future arrives, no doubt the scientists will have a ready explanation (or several) for how we got there.

    10

  • #

    I’ve added two addendums to the post (Hammers answer at #140 and this below.)

    Here’s why talk about whether its convection or conduction vs radiative cooling is irrelevant

    The blanket analogy is perfect because we are discussing whether it’s possible for a cooler item to induce (somehow) an increase in temperature of a warmer item. NOTE: The cooler item has no internal heat source, but the warmer item (Earth or body) does have energy added in. The method of heat transfer is irrelevant. (Talk of two ice cubes misses the point unless one cube is heated by the sun, or burns fat. )

    The point is that YES, obviously in the real world, blankets keep us warm. Pink batts “lift the temperature of your home in cold weather”. They don’t do it by supplying energy, they do it by blocking energy loss. The cooler item is not supplying a single new joule of energy, but there another mechanism of increasing an objects temperature. It’s called insulation. It’s a reality we all know and use every single day. Why deny it?

    Can commenters move on from repeating the truism that a colder object can’t make a warmer one even warmer without supplying energy? We all know that, but it applies to a closed theoretical system with no extra source of energy. In the systems with blankets/people and the sun/earth there IS an extra source of energy, at least until you’re dead or the sun burns out.

    10

    • #
      Duster

      Jo,

      While there is some merit in the ‘insulator’ analogy, there is also a fundamental failure in the parallelism of the analogy. If you are in bed, with a blanket over you, and most heat you lose to the room is related to the rapidity with which circulating air can carry off heat from the outer surface of the blanket, that still doesn’t really parallel what we experience outside.

      The real parallel is how warm would a rock (I considered using a dead body as an example, but that seems a bit gruesome) get, if covered by a blanket in a room at 70 deg. F. The answer is that it would warm to room temperature at maximum, because that is just how warm the blanket would get. The only heat source the body has is the ambient temperature of the room, regardless of how many blankets are piled on. By replacing the blanket with a transparent fluid gel, that is absorptive of LWIR, and which convects and flows heating the outer surface, you could probably claim a true parallel bahviour between the climate and the analogy. If energy is received through the transparent blanket at a number of wavelengths, but can only clear it from a limited window, that would possibly result in slight heating of the rock surface above ambient.

      10

      • #
        Jose_X

        >> ‘insulator’ analogy, there is also a fundamental failure …most heat you lose to the room is related to the rapidity with which circulating air

        With respect to some uses of insulator analogies I (and probably many others) have used in this discussion..

        “Insulator” is independent of how the insulation works. It’s defined by the fact it slows energy transfer.. enabling a heated object to remain at a higher equilibrium temperature even through heat flows to an environment beyond the insulator. This means the atmosphere is an insulator to the planet.

        The analogies each compare the planet to a different type of insulation and how in each case the equilibrium temperature is higher because of the insulative effect. This is an important parallel since there was a claim that it made no sense for insulation (a slowing of energy transfer) to result in a higher equilibrium temperature.

        For an analogy to fail, it is not sufficient (or relevant) to find a parallel that cannot be drawn. Rather, you would have to show that the relevant parallel(s) (which may simply be a single parallel) upon which the analogy is based is not true. .. The insulation analogies do not fail that test (or if you believe they do, please state how they fail their key parallel).

        >> The real parallel is how warm would a rock

        You then describe another potential analogy, albeit perhaps a less intuitive one than the sweater or oven analogy if this new analogy must invoke an unexplained reference to the key point that makes the greenhouse effect itself different from most other insulation and which presumably the reader may not entirely understand (and perhaps the reason for which the simpler analogies were created). [blackbody physics and related effects.]

        To repeat a key point from the oven and sweater analogies, these bodies, just like the planet, are being heated independently of the environment. The “how” doesn’t matter in order to provide the intuition that this insulation on this heated body will then lead to a higher equilibrium temperature (vs the no insulation cases).

        Note what is happening in all cases, the rate of energy loss is initially balanced with energy gain. Insulation slows energy loss but doesn’t much slow energy gain (to first approximation). For a while this imbalance means energy accumulates, raising the temperature. That rise in temperature results in greater energy loss. What we have: a rise in temp does happen and it stops once the energy loss has again balanced that gain.

        10

  • #

    Okay, I’m game enough to put this one out there, because I really DO want to know.
    When I talk with average people who have barely a passing interest in Climate Change/Global Warming, I’m surprised at how little they really do know. One of the things that does surprise me is they don’t really know about the fall in temperature with altitude, and when I mention that it’s around 5C with every 1000 Metres, they look at me like I’m crazy, and this is where my question comes in, and it has to do with a colder area and a warmer area.
    Of the Greenhouse gases that make up in total 2% only of the whole Atmosphere, water vapour is by far the largest, at 1.95% while CO2 makes up only 0.039% of the total Atmosphere.
    If we take water vapour as clouds, and water vapour is H2O, and we can see the clouds ‘floating’ in the sky, I can explain that as that water Vapour is not all that much heavier than the ‘air’ that those clouds are ‘floating’ in.
    If so, then what of CO2, which in effect is almost three times heavier than H2O.
    Now, burn coal and you make CO2, which, coming from the critical furnace is really hot. Anything gaseous that is hot rises.
    However, with that altitude vs Temperature of 5C/1000 Metres, then the CO2 cools quite rapidly.
    Now, being almost three times heavier than the ‘air’ it is in, it theoretically should fall back towards the surface of the Planet.
    We see those clouds floating and if CO2 is almost three times heavier, why should that CO2 just keep rising, as we are told that it is ‘way out there’ forming a surrounding blanket, and warming the Earth.
    With that water vapour so much larger in content, 51 times greater than for CO2, should that water vapour now act as a greater influence than the CO2?
    I know I’ll be shot down in flames here, but that’s okay, because I really do want to know.

    Again, sorry to take so much space on something as trivial as this.

    Tony.

    10

  • #
    BobC

    Lionell Griffith:
    May 15th, 2011 at 4:43 am

    It is always possible to get a greater than 100% efficiency of energy transformation if you don’t account for ALL of the energy in all of its forms required to make the transformation in your computation of efficiency. That is cheating any way you look at it.

    You’re missing the point, Lionell (unusual for you): All the energy involved in splitting water IS taken into account in my description of a solid-oxide electrolytic cell — 75% of the energy to produce the H2, O2 output from water comes from electricity, and the other 25% is from heat.

    The point is that the heat (which produces 25% of the chemical potential energy output) is absorbed directly from the ambient surroundings and does not have to flow between a hot and cold reservoir as it would in a mechanical heat engine (and as the second law of thermodynamics requires). Therefore, the electrolyzer uses heat from a single temperature heat reservoir to produce significant work, in violation of the second law.

    I’m not talking about a few peer-reviewed experiments that measured a weak effect and await replication — these are industrial processes which are optimized to save millions of dollars per year in the production of industrial quantities of gas (by substituting heat energy for electrical in the conversion process).

    The second law definitely describes the behavior of heat engines. It definitely doesn’t accurately describe the behavior of solid-oxide electrolyte cells. It is an open question (IMO) whether it correctly describes the behavior of GHG, since it clearly isn’t a “universal law”.

    10

  • #
    Reed Coray

    My two cents.
    If a small, blackbody (or graybody) object whose surface temperature is everywhere TO Kelvin is placed inside the vacuum of a large enclosure whose walls (a) completely surround the small object and (b) are at a uniform temperature TW Kelvin (where TO is greater than or equal to TW), the small object rate-of-cooling is directly proportional to the difference in the fourth powers of TO and TW. The rate-of-cooling is the rate at which internal thermal energy of the small object is depleted. As the internal thermal energy of the small object is depleted, the small object’s surface temperature drops. If TO = TW, the internal thermal energy of the small object doesn’t change; and the surface temperature remains constant at TO = TW. If TO > TW, the internal energy of the small object is depleted, which produces a lowering of the small object’s surface temperature. In the absence of work being converted into thermal energy, the temperature of the small object will eventually equal the temperature of the enclosure walls.

    Now consider a small object internal to which a motor converts work into thermal energy. For example, a motor may be producing relative motion between surfaces that are in contact. The friction between these surfaces converts mechanical energy into heat at an assumed fixed rate of W Watts. Assume further that the internal thermal conduction and convection properties of this small object are such that the outside surface temperature of the small object is at all times everywhere the same. Place this object in an enclosure whose walls are maintained everywhere at TW Kelvin. When the small object’s outside surface temperature is such that the depletion rate of energy is equal to W, a state of energy-rate equilibrium will be reached and the temperature of the small object’s outside surface will not change.

    Now consider two Scenarios. In Scenario 1, let TW1 = 0 Kelvin be the temperature of the enclosure walls. Scenario 1 is equivalent to placing the small object in the vacuum of cold space at 0 Kelvin. For Scenario 1, the energy-rate equilibrium temperature, TO1, of the small object’s surface will be such that the fourth power of TO1 is proportional to W–i.e., the rate of thermal energy output by the internal motor. Thus,

    TO1^4 = C*W, where C is the constant of proportionality.

    In Scenario 2, let TW2 be the temperature of the enclosure walls, where 0 < TW2 < TO1. This is equivalent to "placing" a second object whose surface temperature is greater than 0 but less than TO1 next to the object at temperature TO1, where "placing" implies "completely surrounding." Let TO2 be the Scenario 2 energy-rate equilibrium temperature of the small object's surface. For Scenario 2, the energy-rate equilibrium temperature, TO2, of the small object's surface will be such that the difference between (i) the fourth power of TO2 and (ii) the fourth power of TW2 will be proportional to W. That is,

    TO2^4 – TW2^4=C*W, where C is the same constant of proportionality as Scenario 1.

    Thus

    TO2^4 = C*W + TW2^4 = TO1^4 + TW2^4.

    This implies that the fourth power of TO2 is greater than the fourth power of TO1, which since temperatures in Kelvin are non-negative, implies TO2 is greater than TO1. Thus, we have the situation where by placing an object (object 2) whose temperature is less that TO1 next to an object (object 1) whose temperature is equal to TO1 and internal to which mechanical energy is being converted to thermal energy, the temperature of object 1 will increase. People who argue that a "cooler object" placed next to a "warmer object" cannot result in an increase in the temperature of the "warmer object" had better find an error in the above logic.

    I believe what such people forget is that the second law of thermodynamics says that in the absence of work being performed, heat (energy) cannot flow from a "colder" object to a "warmer" object. [In both Scenario 1 and Scenario 2, in the absence of work being performed internal to the small object, the "energy-rate equilibrium" temperature of the small object will be the temperature of the enclosure walls.] The ban against the flow of heat from a colder object to a warmer object doesn't apply when work is performed. We all know that refrigerators exist. The internal thermal energy of the cooler air inside a refrigerator is being transferred to the warmer air outside the refrigerator. The second law of thermodynamics states that to make this happen, work must be performed–as it is by the refrigerator's compressor.
    The issue of whether or not the presence of a "colder object" can "warm" a "warmer object" is, in my opinion, a matter of semantics. I like to think the presence of the colder object may result in an increase in the temperature of the warmer object not because heat flows from the colder object to the warmer object; but because to achieve energy-rate equilibrium in the presence of the colder object, the mechanical energy that is converted to heat may produce a higher "warmer object" temperature.

    10

  • #

    I have to say I am getting a bit tired of repeating myself. Let me try to put it another way. If you lie in bed without a blanket you lose a lot of energy and feel cold. If you now cover yourself with a blanket, the blanket reduces your energy loss. With a reduced energy loss you get less cold than you otherwise would have. Whether you consider “get less cold” to be semantically equivalent to saying “you will be warmer than you otherwise would have been” is up to you but I point out that general useage would say the blanket warms you. This is despite the fact that the blanket is colder than you are. No the blanket being colder does not transfer NET heat from itself to you, it merely reduces the energy loss allowing your internal heat generation to raise your temperature more. If you put a blanket over a piece of cold steel it does not make the steel warmer.

    Michael,

    The blanket analogy is incorrect. A blanket is a solid barrier which physically prevents the warm gas between you and the blanket from escaping or de-pressurizing. In the depths of winter when you are cold in your bed, you try to find gaps in your bed covers to seal off, so that heat does not escape.

    There is no solid object covering the atmosphere. Hence those properties which you have described do not exist. The only solids in our atmosphere are clouds, which we know keep the whether warm at night. But, we already know that clouds have a net cooling effect. The atmosphere is a gas and all planets show that at the same atmospheric pressure the temperature is roughly the same. The small variations are due to atmosphere composition.

    The more I study climate science, the more outraged I am that basic laws of physics are tossed out the window. I don’t believe the 2nd law of thermal dynamics is broken because the arguments about how the climate works are invalid. Empirical evidence does not support the greenhouse(blanket) theory. Common sense tells you that there is no solid object ‘trapping heat’ within the atmosphere.

    C02 emissions do have a temperature increase effect due to the fact that we are increasing the size of our atmosphere. It is the weight of that extra gas which will create the temperature increase, not the IR absorption properties. So, if we put an extra tenth of one percent of gases into our atmosphere, we can expect to increase the absolute temperature of our atmosphere at the surface by a tenth of one percent at the most(along with pressure). Of course, we decrease the solid mass of the planet by doing so, resulting in less gravity to create that exact temperature increase.

    The better analogy for the planet’s heating from external sources is, say you light a campfire out in the bush. As long as you are close to the fire you are warm. If you walk away you get cold. If the fire runs out of fuel you can cold but, if you stoke it or throw some wood on it you will get warm again. Catch-phrase of the sceptics: “it’s the sun stupid”.

    REPLY” No. See my comment #149 — JN

    10

  • #
    BobC

    TonyfromOz:
    Now, being almost three times heavier than the ‘air’ it is in, it [CO2] theoretically should fall back towards the surface of the Planet.

    In the absence of convection currents or turbulence, CO2 will stratify. In the Great Rift Valley in Africa, there are CO2 seeps in the ground which will, overnight, form pools of CO2 that are relative stable until the Sun rises and causes convection currents (and winds) to mix the CO2 into the atmosphere. Animals and people who walk into these pools can be asphyxiated.
    (See here.)

    Generally, however, CO2 is well-mixed into the atmosphere by convection currents and wind turbulence and doesn’t stratify to any significant extent. This is because the rate at which stratification takes place is much slower than the average rate of mixing in the Earth’s atmosphere.

    10

  • #

    Oh, I should also point out that, when we talk about climate variations which are internal to the planet, we are indeed talking about the planet’s actual blanket: the oceans. That is where we need to direct our attention, not on the atmosphere.

    10

  • #

    I agree that this “discussion” is getting tiresome and much to repetitive. However, one more time….

    A greenhouse works because it constrains convection inside an inclosure that does not conduct heat well. In fact you can make a quite satisfactory greenhouse by making the solar window of a material that is quite transparent to IR (fused NaCl, fused SiO2, and very thin Mylar sheeting are examples) and with the remainder of the greenhouse more or less air tight. Back radiation, if it occurs at all, contributes nothing to the warming of the inside of the greenhouse (as explained in my post at #3).

    A blanket or insulation works because it interposes thin layers of material that conducts heat poorly with constrained small volumes of air. In a very real sense, you have many “greenhouses” in series, each genuinely trapping heat within each tiny convective cell. As you move from the warm side to the cold side, each successive tiny cell is minutely cooler. Heat can not escape at all rapidly through such a material. Especially if you have hundreds to thousands of tiny “greenhouses” in series.

    Convection of air within the atmosphere is free to obey the gas laws from top to bottom, cool the surface, and transport heat outward so it can be radiated into space. Water vapor is free to perform its cooling and heat dumping function by absorbing and releasing latent heat by phase change. There is nothing in the atmosphere that traps heat.

    Now space is as close as we can get to a black body. It’s temperature is about 3 degrees kelvin – much colder than any part of the atmosphere so there is no impediment for its receiving the earth’s emitted radiation.

    Again, The so called back radiation has vanishingly little to do with the temperature of the earth’s surface per argument #3.

    The atmosphere as a greenhouse OR the atmosphere as a blanket have no foundation in reality. They are not even close to being appropriate or useful metaphors. They are not even close enough to reality to be false. They are worse than false because they are misleading to the point of being fraudulent.

    It is time to look at the real world, gather real data, not cook the books, and do some actual science for a change rather than leaping to conclusions based upon convenient fantasies.

    10

  • #

    Joseph Postma himself has replied with a detailed response — see comment #97 (it was accidentally caught in the spam filter and delayed until I realized it was hidden.)

    Michael Hammers reply #140 and mine #148 are both applicable.

    10

  • #
    bananabender

    @TonyfromOz:
    May 15th, 2011 at 2:15 pm

    The relative density of gases generally doesn’t make much difference because Brownian motion and convection will eventually mix them very well. The only exception is at very high altitudes where atmospheric density is very low and gravitational sorting becomes more important. Hydrogen can even escape the Earth’s gravity and drift off into space.

    10

  • #
    bananabender

    The only way a cooler object can heat a warmer one is if quantum mechanics doesn’t exist and corpuscular energy travels via a luminous aether. This is exactly what Arrhenius believed and how he derived his Greenhouse Effect. Angstrom showed him the error of his ways and he recanted. In the 1960s environmental activists rediscovered the work of Arrhenius and ignored that of Angstrom.

    10

  • #
    cementafriend

    I have not read all posts so I may have missed something.
    Firstly, I agree with the comments by Alan Siddons and I am sorry to disagree with Jo Nova. One can not says that an insulating material heats anything. Chambers technical dictionary (which would be accepted in a court of law) has this definition “heat (phys) that which when given to a body raises its temperature and when taken from a body lowers its temperature. Heat is a form of energy”. An isulating material around a hot body transfering heat to the surrounds can never heat (ie increase) the temperature of the hot body.
    There are some here who misinterrupt refrigeration. Work is not done to move heat. Work is done to move a fluid and to create a vacuum (compressors and pumps). The expansion of the fluid reduces its temperature to below that of the surrounds inside the refrigerator. The temperature difference is the driver of the heat transfer. There is a big doubt that photons exist see this from Nobel Physics prize winner WE Lamb http://www-3.unipv.it/fis/tamq/Anti-photon.pdf and this from an experienced engineer Xavier Borg http://www.worldsci.org/pdf/abstracts/abstracts_5711.pdf.
    Heat is transfer by electromagnetic waves which can cancelled or enhanced as in lasers.
    Tony from Oz. The adiabatic lapse rate is 9.8K/1000m. The reduction of temperature is due to expansion as one moves away from the surface. The environmental lapse is lower from the condensation of water vapor. The average rate is taken as 6.5K/1000m but it does vary from time to time depending on the water vapor content. If there was no movement of the atmosphere and with time there would be a layer of air that had more CO2 close to the surface. Measurement eg Kreutz (1941) have shown that in low wind the CO2 close to the ground is higher than at 14m. However, air movement mixing gases so that above about 1000m to the top of the atmosphere at about 8000m the concentration of CO2 is fairly even.
    Finally, it should be said that engineers particularly Prof. Hoyt Hottel at MIT have made measurements of the radiation absorption of CO2 and H2O vapor in heat exchangers and furnances and prepapred graphs and equations to calculate the gas emissivity/absorptivity. Obviously, climate scientist do not read engineering text or if they do they twist information for their own adgenda.
    The absorption of radiant energy from the earths surface by CO2 is insignificant. The main form of heat transfer from the surface to the atmosphere is by convection and evaporation (ie phase change)of water. The changes in the lapse rate is an indicator of the importance of evaporation and condensation.
    Miskolczi has analysed some 60 years of measurements from radiosondes and satellites and found that the optical density of the atmosphere has remained constant and that neither changes in CO2 or H2O vapor have an effect on atmospheric temperatures.
    Michael Hammer I respect your knowledge but your interpretation does not fit with measurements. Thought bubbles are no use in explaining facts. I suggets you read the following http://climategate.nl/wp-content/uploads/2011/02/CO2_and_climate_v7.pdf

    10

  • #

    I’m yet to see a defender of the greenhouse theory explain it using an example that doesn’t use a solid object as the insulator. Gases and solids have different properties. Gases expand when they become more energetic, solids don’t(generally). The flaw is the idea that heat is ‘trapped’. Our atmosphere is a heat conveyor, not insulator. The atmosphere will only get hotter when more energy is added to the system.

    10

  • #

    Here’s another analogy: Energy flows through the system continuously like water through a river.

    If we add a dam, the dam provides not one new drop of water, but it raises the level of the water available in the river at that point. The blanket, the GHG gases are like dams on energy flow. The temperature builds up behind them because energy is continuously being pushed into the system.

    A lot of people are making this much more complex than it needs to be. Get back to the basics. There are two ways to raise temperature in a system of continuous flow. Either increase the input or slow the output.

    10

  • #

    The earth receives 239W/m2 of energy from the sun. There is no other energy source. The Earth warms to what ever level that it is going to warm to while radiating out 239w/m2 outward. Now, some of this radiation is absorbed by the atmosphere in water and other gases that absorb some parts of the non visible light spectrum. There is a finite amount of each wavelength of this radiation, once it is absorbed by the atmosphere, any additional absorbing is close to meaningless as there is only and always will be just 239W/m2 of energy.

    So why is the Earth temperature at surface warmer than the black body ideal? Air Pressure. The air moves up, it cools, the air moves down it warms. Yes, once you inflate a tire the temperature goes down, but that is a closed single pressure environment. Have you looked at the HVAC requirements of huge tall rooms such as you find in international airports, go ahead and tell me that the temperature will just level out, hell they have had clouds forming in some of those places! They are totally climate controlled. According to the theory involving air pressure and temperature, it fills the role that warmists lay on greenhouse gases. It even explains the temperature on idyllic Venus with no need for greenhouse gasses filling the void. The air temperature of the entire atmosphere must be calculated to come up with the factual black body temperature of the Earth not just the first ten feet of the atmosphere. It would be like checking the temperature of the pot of water on the stove by checking the first mm of water off the bottom of the pot. Or in a closer to reality analogy, it would be like measuring the temperature of a massive airport room 100 feet in height at one specific point and saying that is the energy level of the entire rooms atmosphere. If CO2 is such a great insulator, I am wondering why we do not fill these huge rooms with lots of CO2 in order to reduce energy consumption.

    10

  • #

    Jo, again you’ve used an analogy in which the insulator is a solid. The increase of C02 in our atmosphere simply means that more of the conveyance is done by C02 rather than other gases which would otherwise perform that function. But, the conveyance rate is still the same. Hot air, regardless of composition, rises.

    10

  • #

    Thanks Bob C at comment 153
    Okay then, we know that CO2 is in a liquid state between -57C and -78C.
    Let’s take ambient surface temperature as 20C.
    So then, as that CO2 rises from the surface, it stays in its gaseous state, even though much heavier than the air it is ‘in’.
    We know that the temperature changes 5C for every 1000 Metres.
    As soon as that CO2 reaches 16,000 metres, it should then change to a liquid form, in much the same as water vapour exists in clouds.
    If it stratifies, into, (and for want of a better word) cloud like formations, what makes CO2 more dangerous a ‘blanket’ that stops the escape of warmth than water vapour which exists at a level 51 times greater than for CO2?
    However, this still doesn’t explain the ‘heavier than air’ thing, and that being the case, then surely most of the emitted CO2 would tend to fall back to the surface where it would be ‘fixed’ into anything that is green and grows, when the Sun comes up and ‘turns on’ the process for those growing green things.
    Tony.

    10

  • #
    BobC

    TonyfromOz:
    We know that the temperature changes 5C for every 1000 Metres.
    As soon as that CO2 reaches 16,000 metres, it should then change to a liquid form, in much the same as water vapour exists in clouds.

    Check out the phase diagram of CO2. This diagram shows under what temperature and pressure conditions CO2 can exist as a solid, liquid, or gas. (Here’s an easier one to read.)

    Some things to note:

    1) CO2 can’t exist as a liquid if the pressure is less than 5.1 atmospheres. This is why solid CO2 (dry ice) doesn’t ‘melt’, but changes directly into the gas phase.

    2) The transition temperature between CO2 as a gas and a solid drops rapidly with decreasing pressure. At 16,000 m altitude, the pressure is about one tenth that of sea level. At that pressure, CO2 remains a gas until the temperature is below -100C, significantly colder than your calculated -60C.

    So, there isn’t anywhere in the atmosphere that CO2 can exist as anything but a gas.

    As far as mixing vs. separating goes: Water is heaver than oil. Does this mean you can’t mix them? No, just put them in a blender and turn it on — they’ll become very well mixed very quickly! If you take the mixture out of the blender and let it set for a few hours, it will start to separate, but it won’t separate while in the turned-on blender. This is analogous to CO2 in the atmosphere — the mixing rate is much higher than the separation rate.

    10

  • #

    Thanks Bob,
    This liquid/gas transition is more helpful for me than you may realise, because it has a lot to do (for me anyway)with the ridiculous fallacy of Carbon Capture and Sequestration.
    That aside, what of the ‘heavier than air’ thing, and I do apologise for prolonging this, but I really do want to know, or is that tied in with the mixing vs separating?
    Tony.

    10

  • #
    michael hammer

    TonyfromOz A good question. Water vapour is lighter than air 18 versus 29 and rises however clouds are not water vapour, they are tiny droplets of liquid water which is why you can see them. When the droplets are small enough they will remain suspended in the air even though they ae of far higher density than the air around them. But when the droplets start to coalesce and form larger droplets they become too heavy to remain suspended and they fall (generally collecting even more droplets and growing still larger as they do so) as rain. CO2 is indeed heaver than air 44 versus 29 or about 1.5 times as dense but just like the tiny water droplets which are even more dense than CO2 they remain suspended being well mixed by brownian motion and air turbulence. The temperature at which CO2 condenses depends on the pressure. As the pressure falls the condensation temperature also falls. At about 30 bar (30 time the atmospheric pressure at sea level) CO2 is a liquid at room temperature. In fact at sealevel atmospheric pressure CO2 freezes before it liquifies which is why dry ice left on a kitchen bench changes directly from solid to gas (ablation) without ever going through the liquid phase (which is why its called DRY ice of course). Thus even were the temperature in the upper atmosphere to fall below -57C the pressure is far to low for the CO2 to condense or freeze out. In fact the coldest point in the atmosphere is the tropopause at about 220k or -53C.

    There is enough turbulence to stop the CO2 settling. In the stratosphere where turbulence is much less things could be different and that is one area where I disgree with established wisdom (they say the stratosphere is well mixed wth respect to CO2 while I have significant evidence to the contrary) but that is another story.

    10

  • #

    Tony, Where did you get those numbers for Co2 liquid state? They are not correct.

    10

  • #

    Michael,
    thanks a lot.
    That is the first time I’ve ever been able to get someone to explain it to me in a manner I can actually understand, and I guess that opens me up as looking a little foolish, but I really wanted to know how CO2 could stay suspended, being as heavy as it is, with respect to the air it is ‘in’.
    I still think that even now I have one extra bit to add to the ‘information base’, it still leaves so many questions without answers.
    It still doesn’t explain why ‘warmists’ are so ‘fixed’ on CO2 when that concentration of water vapour is 51 times greater.
    Tony.

    10

  • #

    Mike.
    I believe CO2 ‘liquefies’ at 217K, which is minus 57C, and is a solid, dry ice, at 195K, which is minus 78C.
    When released at room temperature, it changes directly from dry ice to the gaseous form.
    Is this incorrect?
    Tony.

    10

  • #
    Bryan

    Joseph Postma

    Great post here and a great paper.
    The confusion caused by diagrams like KT97 where day and night effects are minced into a meaningless hash of numbers has been addressed.
    Radiative heat transfer is not dealt with at great length in most undergraduate courses and this has provided IPCC advocates a chance to try to bamboozle the rest of us.
    Your experience in astrophysics where radiative transfer dominates is a great help.
    Your initial paper is very clear and helpful.
    I checked through your assumptions and equations and it all stacked up.
    In your next paper could you give a bit more space to the background behind the calculation of -59C shown on page 25.
    I got the same value but was a bit uncertain about the assumptions I was making.

    10

  • #
    RJ

    Joseph Postma at 97

    There is no experimental (nor theoretical, in truth) framework which supports the idea of radiative self-amplification of temperature after insolation from another hotter source. I will spare details, but we will be experimentally proving this simple reality in due course. There is no such thing as conductive self-amplification of temperature; nor is there such a things as radiative self-amplification of temperature. Both conduction and radiation are modes of heat transfer and obey said laws of heat transfer. Radiation cannot conveniently get around those laws, as many try to claim.

    I look forward to this experiment. I have often though it should not be too hard to do this. For example an oil filed heater in a vacuum surrounded by O2 (contained in plastic that does not trap IR). Then replace the O2 with CO2. Does the oil temperature rise or not due to back radiation. (I can almost guarantee it would not but…)

    And thanks for this outstanding reply. And to Jo for highlighting it as I missed it initially.

    10

  • #
    michael hammer

    joseph postma; I read your comments. Without intending any disrespect whatsoever, it has been my experience that once someone convinces themselves absolutely in respect to a position no amount of explanation will be accepted. There is always a rationalization which seems convincing. Yes I realize you could just as well direct this statement to me but that does not change things. It means that if we each try to keep explaining to the other we will get nowhere. A better approach that science has used many times in the past is to see how well each theory explains observable phenomena.

    With this in mind consider the “space blanket”. This is a thin foil of mylar plastic which has been metallised (ie: given a highly reflective surface). It is proven to be very effective at keeping people warm in cold environments. Yes you can argue that it does so by preventing convection but clearly an unsilvered transparent foil would stop convection just as well. Silvering the foil makes it far more effective in keeping someone warm even though the foil which is in contact with the cold outside environment is far colder than the person. According to your theory why would that be the case?

    You also continuously raise the issue of convection and conduction versus radiation. In that case consider the glass thermos flask. In this device a vacuum is created between two concentric glass vessels. The vacuum blocks conduction and convection leaving only radiation. The outside surface of the inner glass vessel and the inner surface of the outer glass vessel are both silvered to create mirror surfaces. When the thermos flask is filled with near boiling water the outer glass vessel is clearly colder being at essentially room temperature. Why do the manufacturers go to the trouble of silvering the surfaces – an expensive process? Would the thermos flask work as well without the silvering?

    10

  • #
    BLouis79

    The radiative invisible insulation theory is completely unphysical. Please provide empirical evidence of this effect in a lab. (See my earlier post on the IR laser experiment #132.)

    Any person can demonstrate simply at home the relative contributions of radiation, conduction, convection. Experiment with any open flame – candle or gas burner. See how close you can put your finger to the side of the flame. See how close you can put you finger to the top of the flame. Put a conductive (not radiative) barrier – sheet of glass between your finger and the flame and see how close you can now get to the flame.

    So what is all that heat absorbing CO2 doing in the flame, not even radiating much heat from the hot flame towards the cooler stove.

    10

  • #
    cohenite

    Very informative post; however, greenhouse vs pressure as the determinator of the temperature profile on Earth, Mars and Venus remains unresolved [I note that Leonard Weinstein has commented; he and William Gibson had a mammoth discussion with SoD on this point]; what is resolved is the difference between heat and energy transfer, hopefully.

    Some observations:

    1 The greenhouse temperature of 33K has been mentioned; the significance of this is that greenhouse/pressure smooths out temperature extremes; the average temperature of an atmosphere free Earth would be 255k, with an atmosphere 288K, but the extremes on the atmosphere free Earth are much greater.

    2 The prediction of a CO2 driven increased greenhouse effect has been that these extremes of temperature would shrink with a rising minimum temperature and a decreasing diurnal temperature range; Anthony Watts’ long awaited paper on the subsurface temperature project has been released:

    http://pielkeclimatesci.files.wordpress.com/2011/05/r-3671.pdf

    Their findings are: “poor siting leading to an overestimate of minimum temperature trends and an underestimate of maximum temperature trends, resulting in particular in a substantial difference in estimates of the diurnal temperature range trends.”

    In otherwords the opposite of what one would expect with an increase in the greenhouse effect.

    3 Ocean heat content is now the preferred indice for measuring the greenhouse effect with Hansen scrambling to cover the fact that the GCMs have overestimated the heat uptake by the oceans and the transfer of heat to the ocean deep; see:

    http://www.abc.net.au/unleashed/1031292.html

    One of the issues about the greenhouse effect which, as I far as I know, is unresolved is how the agent of greenhouse, backradiation, can heat the oceans in the first place when the energy of DLW is insufficient to penetrate the oceans:

    http://hockeyschtick.blogspot.com/2010/12/new-paper-solar-uv-activity-increased.html

    10

  • #
    Alistair

    The only place where CO2 stratifies is in the thermosphere where intermolecular collisions are rare. Elsewhere, mixing occurs almost perfectly as normal gas kinetics take over. For that same reason, it is only in the upper atmosphere that a CO2 molecule can spend enough time in the excited vibrational state to emit that energy as a photon with the same energy that arrived.

    The quid pro quo is that when ‘climate scientists’ claim the phenomenon of ‘back radiation’, they are deluded for three reasons:

    1. Band absorption measured at the top of the atmosphere represents energy absorbed close to the Earth’s surface, quickly transformed to general kinetic energy leading to a slight increase in average temperature and convection.

    2, Direct radiation from the surface is only a small proportion of the heat transfer, the rest being convection and thermo-transpiration.

    3. Consider the following thought experiment. The sand on a breezy beach is at 30°C. You erect a wind-break, sand temperature rises to 45°C to maintain the same radiative + convective heat transfer. Assuming 0.85 emissivity, 85% capture in the atmosphere and half is ‘back radiation’, you’ve created extra ‘forcing’ 23 times the total extra forcing from GHG increase in the industrial age 1.6W/m^2]. That is ludicrous. There is no change in total energy.

    So, in the absence of any compensatory factor, extra GHG warming might give a slight rise in the tropopause and through the lapse rate, a similar increase of surface temperature. The compensation is the reduction in upper atmosphere water vapour to allow easier radiative heat transport, maintaining constant IR optical depth, so constant GHG warming independent of atmosphere composition.

    10

  • #
    michael hammer

    TonyfromOZ; No one who recognises what they don’t know and asks for clarification can ever be considered foolish. Indeed such action is an indication of wisdom. Your question about CO2 versus water vapour is actually a very profound one and the explanation is somewhat involved.

    Firstly; people keep claiming that CO2 can’t have much effect because the concentration in the atmosphere is so low. Wishful thinking but simply not true. A simple way of looking at things is that the CO2 makes the atmosphere opaque at some wavelengths. So how much CO2 does it take to make the atmosphere opaque. Consider a glass of water. Beetroot juice added to the water will make the water turn red. How much beetroot juice does it take? Actually quite a bit. One drop will barely even give a tinge of colour. Now instead of beetroot juice try red food dye. Even one drop in a glass will turn the water deep pink. The amount required depends on the “colouring power” of what you add. In the case of CO2 even 0.5 ppm over the depth of the atmosphere is enough to make the atmosphere opaque at the peak of the absorption line. As the concentraion is further increased the range of wavelengths over which the atmosphere becomes opaque slowly widens – this is called line broadening and it is the reason the effect of adding more CO2 is logarithmic.

    Secondly why focus on CO2. Couple of reasons. Firstly of couse we can’t do anything about water vapour its “natural” (of course so is almost all of the CO2 but what the hell). But more significantly, each green house gas does not make the atmosphere opaque at all wavelengths only some. Water makes it opaque below 8 microns and increasingly above about 16 microns. In the region between ie 8-16 microns the atmosphere would otherwise be transparent and thus the surface can radiate to space without impediment in that wavelength range. If CO2 absorbed at below 8 microns or above 16 it would be completely irrelevant since water already has made the atmosphere opaque. However CO2 absorbs at 15 microns which is just on the edge of the atmospheric window and thus it slightly narrows the atmospheric window. Thus its effect is to some degree cumulative with that of water.

    Don’t forget this CO2 hysteria has been going on for 20+ years. If the theory was completley in conflict with known laws of physics it would have been debunked by now. The issue is more subtle. The reality is that CO2 is a “green house” gas and increasing CO2 will cause some warming. The $64 million (or is that $640 billion) question is how much. Warmists claim 3-6 C which would be significant. They get to this number by assuming massive positive feedback in the climate system. In reality the basis of their claims of positive feedback are not borne out by experimental data nor are their model outputs. Further, pretty well every naturally stable system displays massive negative feedback and there is increasing evidence that the climate system is no exception. If that is the case the claimed 3-6C rise is a massive wild exaggeration and the true number is less than 0.5C. Scuh a rise is of no consequence whatseoever and certainly does not need the political action being campaigned for.

    10

  • #
    RJ

    &176

    The reality is that CO2 is a “green house” gas and increasing CO2 will cause some warming. The $64 million (or is that $640 billion) question is how much. Warmists claim 3-6 C which would be significant.

    Not the reality. One theory. Otherwise you are doing what you accuse Joseph of doing about at

    Without intending any disrespect whatsoever, it has been my experience that once someone convinces themsleves absolutely in respect to a position no amount of explanation will be accepted.

    If you have such a fixed viewpoint any follow up thread by you will be pointless. This has happened on Wattsup with a series by Ira. He held fast with his viewpoint and did almost nothing to reflect the uncertainty from the previous threads

    10

  • #
    RJ

    And re my previous post. I somehow posted before it was finished. I enjoy this thread and appreciate all your posts but hope that the uncertainty can be reflected in any future threads if there are any

    Otherwise this debate (which I find brilliant even if this might be a bit sad) will not move forward. Further thread will generate lots of comments again but not the progress and clarity that it could

    10

  • #
    bananabender

    The Earth is surrounded by the vacuum of space which is already a perfect insulator. It is therefore absolutely impossible for the atmosphere to further reduce heat losses.

    Every single joule of energy that hits the Earth is eventually re-radiated into space whether a picosecond or a thousand years later. If it didn’t would simply continue to heat and eventually vaporise.

    The only thing the atmosphere/ocean system does is redistribute energy in a temporal-spatial manner via physical processes such as ocean currents, convection and wind. It doesn’t warm or cool the Earth in any manner whatsoever. To do so would violate the principle of Conservation of Energy.

    Current climate simply reflects past energy inputs from the Sun transformed via chaotic non-linear processes.

    10

  • #

    The reality is that CO2 is a “green house” gas

    I content that, the reality is C02 absorbs IR in points on the radiation spectrum which are not absorbed by other gases which already exist in the atmosphere. C02 behaving like a greenhouse? I’m yet to see any evidence for that. Any surface level increase in conveyance is converted to IR through friction. The total amount of heat radiated from the atmosphere remains the same.

    10

  • #

    Some quotes of Mike Hammer in post 176.

    1) ” In the case of CO2 even 0.5 ppm over the depth of the atmosphere is enough to make the atmosphere opaque at the peak of the absorption line. As the concentraion is further increased the range of wavelengths over which the atmosphere becomes opaque slowly widens – this is called line broadening and it is the reason the effect of adding more CO2 is logarithmic. ”

    In a closed flask sample that maybe, but in the real, open system of the atmosphere, increased CO2 increases scattering, because it is a gas that is radiatively able to cool itself. Which is NOT the same thing as heating.

    2) ” Firstly; people keep claiming that CO2 can’t have much effect because the concentration in the atmosphere is so low. Wishful thinking but simply not true. ”

    But, the climate models are absolutely DEPENDENT upon an amplification of this supposed* (logarithmic) warming effect of CO2 (which is too low on it’s own for the modelled projections of temp increase), BY THE AMPLIFICATION AS MODELED BY WATER VAPOUR.
    * = Derived from MODTRAN, HITTRAN, LOWTRAN MODELLING…From closed flask measurements….
    This is WHY all GCM climate models model a tropospheric, tropical “hot spot”, by amplification of supposed CO2 warming BY water vapour.
    http://i53.photobucket.com/albums/g43/DerekJohn_photos/climate%20stuff%202/gissfingerprint-SMALL.jpg
    It is also one possible explanation FOR THE FACT THAT THE MODELED HOT SPOT IS MISSING,
    if CO2 does not warm, then what is there to amplify by water vapour?????

    3) ” Secondly why focus on CO2. Couple of reasons. Firstly of course we can’t do anything about water vapour its “natural” (of course so is almost all of the CO2 but what the hell). ”

    As is almost all of atmospheric CO2 concentration variability – but what the hell.
    Just WHO is in denial here…..
    And yes, IPCC et al does “amplify” human “contributions” ridiculously. Whatever they actually are,
    rather than the bureaucratically convenient figures reported at present.
    Are you aware of the Bern carbon cycling model, and that it includes the term Ao,
    which equals, 21.3% of any human activity emitted CO2 today, and every day,
    WILL NEVER LEAVE THE ATMOSPHERE……..
    Pure bull dung, but that is what they calculate using the Ao term.
    http://homepage.ntlworld.com/jdrake/Questioning_Climate/_sgg/mhm1_1.htm
    NB – The line in the above plot at 21.3% ONLY stops at a 1,000 years, BECAUSE, that is where IPCC stops calculating,
    it would still be at that level, of 21.3%, 10,000, 100,000, add infinitum years, USING THE Ao TERM.

    4) ” Water makes it opaque below 8 microns and increasingly above about 16 microns. ”

    A crass misrepresentation of the width and strength of water vapour compared to CO2 “strength”.
    See,
    http://i53.photobucket.com/albums/g43/DerekJohn_photos/Does%20CO2%20trap%20heat/DoesCO2trapheat-001.jpg
    and, taking into account concentrations..
    http://i53.photobucket.com/albums/g43/DerekJohn_photos/Does%20CO2%20trap%20heat/DoesCO2trapheat-002.jpg

    5) ” Don’t forget this CO2 hysteria has been going on for 20+ years. If the theory was completley in conflict with known laws of physics it would have been debunked by now. ”

    It was debunked just after Arrhenius 1897, if only people would care to actually read the literature. Hence Arrhenius 1906, in which he made some slight amends..

    6) ” The reality is that CO2 is a “green house” gas and increasing CO2 will cause some warming. ”

    In a closed flask “system”, maybe, in the real, open to space, and mixed system, no. It is physically impossible for CO2 to “trap” heat,
    basic physics tells us that, undeniably. CO2 merely scatters heat, aiding it’s escape to space,
    thereby increasing the atmosphere’s cooling ABILITY..Particularly in the (far lower pressure) stratosphere.

    The “biggest” AGW “principles” supporters and believers appear to be “skeptics” that
    do not question those very AGW principles, they merely quibble the figures of.
    And, in general these same “skeptics” do not even manage that too well, if at all either,
    because of their also “believing” the unphysical “physics” that AGW is built upon.

    10

  • #
    michael hammer

    RJ comment 177: I am not sure I understand what you are saying. What I was suggesting was that nothing I say seems to change Postma’s outlook and equally nothing he says changes mine. At some point it becomes pointless to “explain at” each other. In such a case one has to try a different way of breaking the deadlock. One of the ways often used in science is to take a known proven observation and see how each veiwpoint accounts for it. That is what I did in comment #172. It seems to me that the two examples I gave were significant. If Postma (or of course you for that matter) have a simlar “proven beyond doubt” example which you think conflicts with my viewpoint lets debate it.

    Waffle at 180; You stated CO2 absorbs at some infra red wavelengths. So long as the absorption wavelength is somwhere between about 6 and 50 microns (the wavelengths at which there is significant radiation from Earth to space)that is a definition of a green house gas. I am very fully aware that the mechanism is completely different from what actually happens in a green house and the common term is thus a poor one but as far as I am concerned it is simply a widely used convenient handle by which to refer to the subject. Please don’t discount the arguments simply because I use the widely accepted label. I am not and never have clamed that the mechanism bears any relationshiop to what actually happens in a green house.

    10

  • #

    The second law of thermodynamics is about entropy.

    Calculating entropy is simple. Here is an example of entropy calculations in two cases – for a planetary surface with a warmer background temperature vs a colder background temperature:

    The Three Body Problem.

    All the many many people who imagine that the atmosphere cannot increase the surface temperature have to do is to prove that entropy decreases in the warmer surface case. The second law of thermodynamics says that entropy cannot decrease.

    The fact that no one has done this in that article just demonstrates that all of the “claimers” don’t really understand the second law of thermodynamics.

    Take a look at the comments. No recalculation of entropy. No pointing out of the flaws.

    Instead – the argument by “I have some vague idea that I understand this second law but can’t be bothered to actually calculate the result to prove you wrong

    So, lots of people here who think that the second law of thermodynamics forbids this process – go ahead and redo the entropy calculation. It should be so easy..

    δS = δQ / T, where δS = change in entropy, δQ = change in energy, T = temperature.

    10

  • #
    cohenite

    If the greenhouse concept as enunciated is accepted there still remains considerable doubt over the role of CO2 in that process; for instance in the overlapping part of the spectrum CO2 reduces the emissivity and therefore heating capacity of H2O: Nasif Nahle discusses this effect here:

    http://jennifermarohasy.com/blog/2011/04/determining-the-total-emissivity-of-a-mixture-of-gases-containing-overlapping-absorption-bands/

    His view is supported by these papers:

    http://meetingorganizer.copernicus.org/EGU2011/EGU2011-4505-1.pdf

    http://thesis.library.caltech.edu/2809/1/Lapp_m_1960.pdf

    And clearly shown by a graph of the absorption spectra of H2O and CO2 in the overlapping wavelength:

    http://scienceofdoom.files.wordpress.com/2010/07/spectra-h2o-co2-15um.png

    10

  • #
    Bryan

    The insulating blanket model.

    An ordinary blanket being almost a continuous surface is a far better radiator(continuous spectrum) than a gas(line spectra).
    So you don’t need a reflecting surface for this model.
    However it will restrict convection so not such a good atmosphere analogy there.
    Perhaps a string vest beloved of your grandad fits the bill better.
    Anyway as I see it more IR radiating gas particles could slightly increase the insulating properties of the atmosphere making for slightly warmer night Earth surface temperatures.
    However during the day the same atmospheric CO2 and H2O block some of the Suns infra red making for slightly cooler Earth surface temperatures.
    So the insulating blanket model cuts both ways!

    10

  • #
    Alistair

    ScienceofDoom: the 2nd law of thermodynamics can be interpreted as defining that a thermodynamic system always operates to maximise the rate of increase of entropy.

    In the case of the Earth’s atmosphere, that means GHG warming due to extra CO2 is minimised, in principle to zero by the processes in the atmosphere which already reduce theoretical warming [insolation plus the supposed ‘blanket’ of GHG warming] in the absence of convection of near 77K [Lindzen’s figure] to 33K.

    So, the null hypothesis question for those who claim CO2 increase will lead to increased temperature is: “Prove, using detailed heat transfer calculations, why the existing mechanism which maintains constant lapse rate can allow any surface temperature increase.

    For those who are interested, the IPCC consider the position of the tropopause to be fixed and surface temperature rises. That is thermodynamically impossible because it implies an increase of lapse rate.

    00

  • #

    Alistair:

    the 2nd law of thermodynamics can be interpreted as defining that a thermodynamic system always operates to maximise the rate of increase of entropy.

    You can say that if you like but it isn’t the 2nd law of thermodynamics. The 2nd law says that entropy can never reduce. And that’s it.

    So I think you need to prove your claim because everything subsequent in your comment seems to develop from that.

    For example, if we said (as a wild hypothesis) your “interpretation” was incorrect, would any of your other statements be valid?

    00

  • #
    Joe Lalonde

    Jo,

    The laws of thermodynamics is incorrect! There was a consensus of scientists to generate science laws as a starting point in science. Unfortunately, scientists made this all binding and unbreakable to any new technology or new science.
    The biggest thing that is missed is pressure changes and centrifugal force which is planetary rotation and motion.

    Currently two things are missed in formulas and general science.
    One that this planet is an orb which is biggest at the center and smaller going to the poles and second, they miss the planet rotates which means everything is in motion and energy is stored to generate this.

    00

  • #
    Bryan

    For SoD and others.

    Two objects A and B exchange heat Q in a closed system.
    We will not initially say if temperature Ta is bigger than Tb, however to get started say the heat Q moves from A to B in time dt

    Entropy lost by A = dSa =-Qdt/Ta
    Entropy gained by by B = dSb = Qdt/Tb

    The second law tells us dSa+dSb > 0

    -Qdt/Ta + Qdt/Tb > 0

    This is only true if Ta > Tb

    In other words in (the second law of thermodynamics) Heat only flows spontaneously from a higher temperature object to a lower temperature object never the reverse

    00

  • #
    Joe Lalonde

    Jo,

    The density difference of cold gases is that they compress more molecules into the same space compared with warm gases that vibrate and expand.
    There is evidence that our atmosphere has stretched due to heating over thousands of years with growth up on mountain sides never seen before.
    Once the atmosphere is stretched, it takes a great deal longer for it to return to what it was and generates less pressure at sea level. This in turn has allowed salt changes on the planets ocean surfaces by centrifugal force and density. This inhibits solar radiation and generates a cooler ocean. Cooler air is being pulled back to the equator with the difference in the stretched atmosphere to allow the space to be filled as it cools.

    00

  • #

    Bryan,

    It’s the three body problem – the sun, the earth’s surface and the atmosphere. You were there and never provided an alternative entropy calculation.

    With the sun, the earth and an background radiation that is a) colder, then b) warmer – as explained in The Three Body Problem, which equation do you think is incorrect?

    00

  • #
    Joe Lalonde

    Bryan #189,

    Where was this experiment done? At the poles or equator? What is the land height in the atmosphere or was this done at sea level? Does this formula work for all the planet or just one specific point on this planet?

    The planet is far more complex than a simple formula can reproduce as the planet is largest at the equator and smaller as it goes to the poles.
    Never mentioned the rotation of the planet as well as what pressure?

    00

  • #
    BLouis79

    OMG Joseph Postma is a genius. Well he probably thinks not since he is just an astrophysicist and the atmosphere paper is something he could have written as first year astrophysicist, he explains it all so simply. The “Thermodynamic atmosphere effect” paper totally explains that the earth is as it should be, no more, no less. The fact that the “atmospheric greenhouse theory” is unphysical is a side issue. The “atmospheric greenhouse theory” is not required to explain earth’s temperature.

    Paper via:
    http://climaterealists.com/index.php?id=7457
    or directly:
    http://www.tech-know.eu/uploads/Understanding_the_Atmosphere_Effect.pdf

    Gerlich and Tscheuschner are right, the greenhouse theory is falsified. The “Thermodynamic atmosphere effect” explains the world as we know it. Thank god for science. Now can we just kill the silly carbon tax.

    00

  • #
    Alistair

    ScienceofDoom: I disagree.

    Consider the addition of heat energy to the Earth by the sun; call it daybreak. The energy absorbed by the soil etc. as the sun’s rays hit is converted into heat. That drives convection, thermo-transpiration and radiation in the ratio 59% [convection+thermo-transpiration]: 41% IR radiation. Of the latter, 15% escapes directly to space [IR optical depth 1.86] so the real convection + thermo-transpiration is 94%. That drives weather. Recently, I have cycled against Easterly winds driven by the warming of the European mainland a few hours earlier.

    So what we have is two factors, one, IR to space is by definition the maximum possible rate of entropy production. The wind from Europe dies down when the sun sets there but during the time it blows, its instantaneous velocity is always in the direction of maximum pressure gradient, again the maximum rate of entropy production. As for the latent heat energy, that goes towards maintaining lapse rate at thermodynamic equilibrium by the fastest possible route.

    QED. The 2nd Law specifies the concept of entropy and that when an open thermodynamic system is perturbed, its production rate is always maximised. For this reason, the concept of ‘back radiation’ [an artefact of Milne’s incorrect mathematics in 1922] is bunkum because even if it did exist, its purpose would be to reduce the rate of entropy production.

    00

  • #
    RJ

    @ 194

    That drives convection, thermo-transpiration and radiation in the ratio 59% [convection+thermo-transpiration]: 41% IR radiation. Of the latter, 15% escapes directly to space [IR optical depth 1.86] so the real convection + thermo-transpiration is 94%

    Where do these figures come from. And how does 59% jump to 94%

    I would have thought (correct me if my logic is faulty) that a Thermos shows that it is much easier for heat to leave the surface by conduction / convection / advection rather than radiation. Logically this makes sense to me. If heat is delayed due to a vacuum (where the main option is radiation)then surely heat will escape in the easier form at the surface. And it will move up in this way until the upper atmosphere when the only option is radiation.

    00

  • #
    Joe Lalonde

    Alistair # 194,

    Most of solar radiation is bent radiation by the distance it passes through the atmosphere. This planet rotates and only certain points are exactly lines up for a second at a time. The rest has to travel through differing degrees of atmosphere by the tilting and rotation.
    When heated gases expand by vibration NOT included in the laws.
    This planet has motion of rotation as well as the solar system is in forward motion. This means that water ALWAYS has energy stored.

    00

  • #
    RJ

    & 193

    Gerlich and Tscheuschner are right, the greenhouse theory is falsified. The “Thermodynamic atmosphere effect” explains the world as we know it. Thank god for science. Now can we just kill the silly carbon tax.

    Kill off the carbon tax and cap and trade for good throughout the world.

    00

  • #
    Bryan

    ScienceofDoom

    It has been known for some time that the second law might not hold for a three body situation.
    Two sources and one sink.
    However I think all are agreed that during daytime the energy path is from the Sun to the Earth system.(A two body problem)
    At night where the backradiation enthusiasts claim most of a “greenhouse effect” we again have a two body system.
    The atmosphere and the Earth surface.
    Why complicate the physics?

    00

  • #
    michael hammer

    Alister comment 194. Firstly where do you get the relative figures of 59% convection etc and 41% IR radiation. I disagree with this ratio. My calculations suggest the radiation is a significantly greater fraction. Secondly where do you get the figure of 15% escapes directly to space. The concept of an overall optical depth which you apparently use is meaningless because it changes so rapidly with wavelength. If you look at the spectrum from the Nimbus satellite it shows emission from the surface directly to space between 8 and 14 microns (this is the so called atmospheric window and the Nimbus plot shows the equivalent emission temperature in this range matches the surface temperature). Now if one integrates Planks law from 8 to 14 microns (which is exactly what I did) the total emission is around 160 watts/sqM out of a total of 235 watts/sqM emitted by earth as a whole or about 66%. That is a great deal more than 15%. This issue is one of the complaints I have with the IPCC models. They are based on Trenberth’s model of the atmosphere which claims 40 watts/sqM directly from the ground plus a further 30 watts/sqM from cloud tops for a total black body emission of only 70 watts/sqM or about 30%. It claims the remaining emission is all modulated by GHG effects. I think this greatly overestimates the GHG impact and that matters a good deal. It casts questions over the models.

    00

  • #
    Joe Lalonde

    Bryan @ 198,

    “Why complicate the physics?”

    Simple answer is for accuracy.
    Most of science is based on just generalized theories that totally fall apart when you start to actually include some intelligence as well as new technology.
    This planet is a highly complex system of interacting actions and reactions. Just science forgotten that their is a mechanical component to understanding science and motion.

    00

  • #
    Paul S

    As I understand it, global warming theory holds that small temp increases due to C02 lead to positive feedback via other, more potent greenhouse gases, especially water. The paleoclimate had many times the current concentration of C02, and as much water was present then as know. That being the case, why did no runaway greenhouse effect occur at that time? Given that the same essential parameters existed then as now, surely 4400ppm C02 (or thereabouts) during the Jurassic should have lead to massive increases in global temperature. The theory should have retrospective, as well as prospective, predictive value.

    00

  • #
    RJ

    @199

    My calculations suggest the radiation is a significantly greater fraction.

    How did you do this calculation. And does it meet the common sense test

    A space ship has a hard time dumping internal heat to the surrounding vacuum of space. Based on this and the Thermos example above. Do you agree

    1 That logically it makes more sense that radiative cooling is a minor part of surface heat loss. As heat would leave in the easiest way.

    2 There are widely varying viewpoints on this and it needs more research.

    00

  • #
    Alistair

    RJ: of the IR radiation, 85% is absorbed in the atmosphere, the rest goes directly to space [you get that from Beer’s law with optical depth tau=1.86: I=Io exp-tau].

    The split of the energy transfer is generally accepted. My aim was to show IR is the minor heat transport mechanism and because most is absorbed in the atmosphere as extra kinetic energy/temperature, the idea of ‘back radiation’ is diluted from the very start.

    Your thermos flask idea is a diversion. I spent years measuring heat transfer in metallurgical plants and ambient radiation is always minor unless you stop convection. I looked at AR4 yesterday and the science does look at convective heat transfer so there is good work in the report. However, the lefties who wrote the summaries leave it out.

    ‘Back radiation’ is a mathematical artefact identified in Miskolczi’s 2006 paper. ‘DLR’ is the practical manifestation of Prevost’s Theory of Interchanges. Add in the other error, so-called ‘cloud albedo effect’ cooling supposed to hide imaginary high-feedback CO2-AGW is itself a product of incorrect physics, and there’s a real problem.

    Correct that error [the failure to incorporate substantial direct backscattering at upper cloud surfaces shown by Mie to be strongly dependent on droplet size,] and instead of cooling, aerosols make thicker clouds transmit more light. You prove this by looking at rain clouds – larger droplets means lower transmission/higher albedo. No-one in climate science or the physics [Sagan, Chandrasekar] looked out the bloody window!

    So, the IPCC’s case is a crock of ordure. Because cloud heating is another [self-limiting as albedo tends to 0.5] AGW that probably explains recent ocean heating as the result of Asian industrialisation, there is absolutely no proof of any net CO2-AGW. Also, the cloud physics explains palaeo-climate far better than CO2, including all the short-lived attempts to jump out of the ice age you can never get with the 800 year CO2 delay!

    It really is a mess. I have informed people in the subject they’ve got the basic optical physics very wrong. Trenberth, who only a couple of months’ ago was railing at ‘deniers’ recently announced AR5 will be very different because errors in cloud physics will be corrected. I suspect those in charge are dumping everything on NASA for having deceived climate science in 2004 when it published fake science, apparently to keep AR4 on track: http://geo.arc.nasa.gov/sgg/singh/winners4.html

    The ‘surface reflection’ claim, which is substituted for Twomey’s correct physics he warned did not apply to thicker clouds, was in my view a ‘bait and switch’ using NASA authority to deceive the rest of climate science, and they are now reacting to this con.

    00

  • #

    Sorry, Mike Hammer. I admit I laughed out loud when I read your comment posted at #26 in which you stated,

    “The “warming” comes about becuase the outflow of energy is impeded. In the same way if a dam filling with water from a river and at the same time leaking water out through the dam wall eventually the water level will rech an equilibrium where inflow equals rate of leakage.”

    Mike, do you actually believe that the same mechanism does not also impede the inflow that you say “dams” the output? Wake up, Mike. If you actually believe CO2 absorption and emission of IR only impedes energy leaving the earth and does not also impede solar radiation entering by the same token (in=out, or bang!), then I’m a Chinaman!

    00

    • #
      BobC

      John O’Sullivan
      May 15, 2011 at 10:29 pm · Reply
      Sorry, Mike Hammer. I admit I laughed out loud when I read your comment posted at #26 in which you stated,

      “The “warming” comes about becuase the outflow of energy is impeded. In the same way if a dam filling with water from a river and at the same time leaking water out through the dam wall eventually the water level will rech an equilibrium where inflow equals rate of leakage.”

      Mike, do you actually believe that the same mechanism does not also impede the inflow that you say “dams” the output? Wake up, Mike. If you actually believe CO2 absorption and emission of IR only impedes energy leaving the earth and does not also impede solar radiation entering by the same token (in=out, or bang!), then I’m a Chinaman!

      John, I don’t know your nationality, but:

      1) The proported “Greenhouse Effect” of CO2 can only work on wavelengths absorbed by CO2 — that is, in infrared. This is where the Earth radiates it’s peak power back to space.

      2) The peak output power of the Sun (and hence wavelength band where the Earth receives the most energy) is centered in the visible , ~0.5 microns, where CO2 is transparent.

      See this diagram of blackbody curves — note that at 300 deg K (the temperature of the Earth), the peak is at 10 microns and at 5800 deg K (the temperature of the Sun), the peak is at 0.5 microns.

      So, yes — the greenhouse effect would have a much larger effect on the energy loss mechanism of the Earth (radiation at ~10 micron wavelength) than it would on the inflow (absorption of visible radiation, ~0.5 micron peak).

      00

  • #

    […] last, but not least, Jo Nova explains why greenhouse gases do not break the 2nd law of […]

    00

  • #

    Err. John. The incoming radiation is a different spectrum to the outgoing. More UV in, but infra red out. Co2 doesn’t absorb the UV on the way in.

    00

  • #
    Steve Case

    Over here:

    http://antigreen.blogspot.com/2011/05/does-global-warming-theory-contravene.html

    Is this argument:

    “the earth without a greenhouse effect emits about 239 W/m² to space. And the earth WITH a greenhouse effect emits about 239 W/m² to space.” Implying that if there’s no difference heat isn’t going to build up, i.e., no warming.

    I think this dawned on Hansen, the IPCC or some one in that camp a ways back becuase Dr. Trenberth came out with a new heat budget chart

    http://www.skepticalscience.com/pics/Global_Energy_Flows.jpg

    Replacing the old one:

    http://www.ipcc.ch/publications_and_data/ar4/wg1/en/fig/faq-1-1-figure-1.jpeg

    And the big difference between the two is the addition of “Net absorbed 0.9/Wm² on the new chart.

    The old chart balanced, the new one doesn’t.

    Well anyway, If the sun emits 1364.8 Wm² and they can measure an imbalence of 0.9Wm² that’s five place accuracy, after all of the chaotic things that go on in our atmosphere, I’m impressed. In my experience five place accuracy is difficult to achieve even in a temperature controlled metrology lab.

    00

  • #
    Alistair

    The CO2 will absorb the IR produced by Ozone, which does absorb UV!

    00

  • #

    To Jo Nova @ # 71:
    Jo Nova: “An IR beam is an IR beam. It heats if it hits.”

    I hate to contradict such a respected fellow skeptic, Jo, but you’re flat wrong. Fundamentally, I fear you misunderstand that energy is not heat and heat is not temperature. Infra energy red transmission is not necessarily transmission of heat (heat implies work thus causing increase in temperature).

    You are incorrect to infer that the more IR beams that hit us the more heat we receive. Heat is a process and temperature is the consequence of the output of the work from the heating process. Transmission of IR is not, in itself, work output from a heating process. Transmission of IR is entirely independent of the heating process. Your interpretation may be shown to be a logical fallacy (see below) and is contrary to the laws of thermodynamics.

    First, all objects above Absolute Zero, which is equivalent to −273 °C in air, emit IR (basically everything!) and, I agree, the larger the object the more the IR “hits” we receive. However, if your statement was right then when I stand by a huge block of ice it will warm me more than if I stood by a smaller block of ice because, in your universe, the increase in IR from the larger ice block hitting makes me warmer as per ”It heats if it hits.” Right? Wrong!

    We know you’re wrong because when I stand next to a large block of ice (emitting more IR) I get colder much quicker than if stand by a smaller block of ice (less IR). Better still, no ice at all! Try it.

    The reason I don’t get warmer, but colder, when I stand by the bigger block of ice, despite being hit by all that extra IR, is neatly characterized by Professor Nasif Nahle who better characterizes the actual thermodynamic mechanism like this:

    “A cooler body has vacancies for more heat while the warmer body doesn’t.”

    Nahle’s characterization works better than yours because with his we understand that a bigger ice block has more ‘vacancies’ to accept warmth from my body than a smaller one. Thus your version of “if the IR beam hits it heats” is not only overly simplistic, it’s palpably bogus.

    What Nahle and the rest of us Slayers are hoping you will understand, as a valued Aussie skeptic we much respect, is that CO2 IR cannot, of itself, cause heating. When you comprehend the minutiae of the physics you better understand why extra CO2 IR cannot heat the atmosphere. Also, please remember that temperature is a number related to energy, but it is not energy itself. Temperature is a number that is related to the average kinetic energy of the molecules of a substance.

    Please read that last sentence carefully. It does not say that temperature is kinetic energy, nor does it state exactly what is the relation between temperature and kinetic energy. We would like to discuss this further with you. The Slayers are at your service for more detailed scientific explanations but for now some independent clarification can be found here:

    http://zonalandeducation.com/mstm/physics/mechanics/energy/heatAndTemperature/heatAndTemperature.html

    00

    • #
      KinkyKeith

      Hi John,

      I was a little uncertain of the purpose of your comments below and believe that they illustrate what to me is the main problem with the “science” of CO2 based AGW:

      1. “When you comprehend the minutiae of the physics you better understand why extra CO2 IR cannot heat the atmosphere.”

      and

      2. Mike, do you actually believe that the same mechanism does not also impede the inflow that you say “dams” the output? Wake up, Mike.”

      First I’m not sure why you think CO2 can’t heat the atmosphere; it can absorb ground IR and transfer accumulated energy very rapidly to surrounding gases via collision or radiation.
      It is true that MORE CO2 will have almost NO effect on the total energy collected from ground IR by CO2 and therefore no more heating.

      Any comment on Item 2 is pointless because you either understand or you don’t.

      For those who do understand the difference between incoming UV and the outgoing IR no explanation is needed.

      The main problem with the “science” of CO2 based AGW, as I see it, is that scientists are trying to sort it out.

      Unfortunately this is an ENGINEERING and THERMODYNAMIC problem.

      Engineers would break the analysis down into many smaller problems and establish a number of zones of interest and boundary layers in the atmosphere both vertically and horizontally and look at the problem again. (I KNOW THIS HAS ALREADY BEEN DONE BUT I THINK MORE ADVICE SHOULD HAVE BEEN SOUGHT BY THOSE DOING IT).

      They would assess that YOU CANNOT PRODUCE A COMPLETE SCIENTIFIC DESCRIPTION OF THE FUNCTIONING OF THE ATMOSPHERE because you can’t measure everything you need.

      Scientists who say you can are Delusional. We are dealing with an extraordinarily complex system but that doesn’t mean we can’t research and understand certain parts of it.

      All you can do is isolate small pockets of interest such as the CO2 – AGW issue and attack it while treating the rest of the factors as a constant “black box”.

      The physics and engineering all say that man made CO2 is totally irrelevant to the heating of the atmosphere but that the remainder, natural origin CO2 may have some influence.

      A more useful restatement of Nahle’s comment might be that heat will flow down a temperature gradient and the rate of flow will be proportional to the gradient. So heat moving from the surface to say 5 km up will have a temperature gradient represented by say 25 deg C to 0 deg C or a delta of 25 C deg. From there (5 km) to “space” (vac) the gradient will be 0 deg C to – 271 deg C or a delta of 271 C deg.

      In which zone will heat leave the earth fastest?

      00

      • #
        Jose_X

        John O’Sullivan >> However, if your statement was right then when I stand by a huge block of ice it will warm me more than if I stood by a smaller block of ice because, in your universe, the increase in IR from the larger ice block hitting makes me warmer

        You do get less radiation from something small than something large, each at the same temperatures (consider a very small grill vs a gigantic one).

        You are missing a few important things from the analysis.

        1) In an atmosphere, radiation is not the only heating mechanism. Temperature is a result of all the heat flows.

        2) There is probably something behind the ice blocks that have higher radiation than the ice itself (even ignoring the air radiation and convection, which we wouldn’t ignore in real life). For example, if the ice was in front of a warmer wall or warmer table indoors, then the large block of ice would block off more of that warmth radiating from the wall that the small ice would block.

        You certainly get more overall radiation next to the small ice because the radiation you get is the sum of that from the ice and from what the ice does not obscure. A large ice will obscure more warmer things that would otherwise radiate more in your direction.

        John >> It is true that MORE CO2 will have almost NO effect on the total energy collected from ground IR by CO2 and therefore no more heating.

        I say, wrong. More molecules mean more total energy if we are at the same temperature.

        What we get with doubling CO2 concentrations is that whatever was absorbed at a given height before (say, some quantity X of planetary radiation absorbed by CO2 by the 100 meter altitude mark), now can be absorbed closer to the ground, (resulting ultimately in more back radiation). This follows from Beer Lambert law (and from related laws and supporting experiments) rather easily. Double the concentration and at half the path length you get the same attenuation.

        To consider an example, your visibility will be cut approximately in half (or at least will be less) in fog or through cloudy water if the concentration of obscuring particles in the air/water doubles.

        >> They would assess that YOU CANNOT PRODUCE A COMPLETE SCIENTIFIC DESCRIPTION OF THE FUNCTIONING OF THE ATMOSPHERE because you can’t measure everything you need.

        We can’t measure the full functioning or produce a complete scientific description (in practice) of __anything__ on earth, so what is your point? ..that decent useful models can’t exist ever? This is what you are effectively saying whether you realize it or not.

        00

  • #
    John Brookes

    Wow! Its like trying to herd cats, isn’t it?

    A long time ago, I asked (somewhat naively) what AGW skeptics agreed on wrt climate science. The answer appears to be “nothing”.

    Jo & Michael Hammer have a pretty good understanding, probably better than mine. However many others are abysmal. I don’t think they are particularly stupid, just willfully obtuse.

    There is no harm in understanding a few things. You don’t give up your individuality or freedom by conceding that there are plenty of things which really smart people (much smarter than anyone on this blog) have painstakingly discovered which are true, and that you have much too little skill or expertise to argue otherwise.

    So if you don’t understand something, ask questions. If you don’t understand the answer, and you really want to, you could try reading some physics text books.

    00

  • #
    Climate realist number9

    Solid oxide electrolytic cells (SOEC) are in the same category as the Horvarth hydrogen engine (1970’s Qld and BjelkePetersen) and cold fusion – in the end there might be flaws in the claims about them. Be very wary of claims for exceptions to the laws of thermodynamics. Physical laws might break down in subatomic and cosmic extremes – it is hard to accept that the atmospheric arrangements of a simple planet orbiting an ordinary sun in a black body universe is in that category just now. Argue what you may, entropy is what it is.

    00

    • #
      BobC

      Climate realist number9
      May 16, 2011 at 12:56 am · Reply
      Solid oxide electrolytic cells (SOEC) are in the same category as the Horvarth hydrogen engine (1970′s Qld and BjelkePetersen) and cold fusion

      Um…SOECs are a commercially available technology. You can buy one and try it out.

      00

  • #

    cementafriend was caught in the spam filter. The comment has been released and appeared at #159.

    John:

    “if your statement was right then when I stand by a huge block of ice it will warm me more than if I stood by a smaller block of ice because, in your universe, the increase in IR from the larger ice block hitting makes me warmer as per ”It heats if it hits.” Right? Wrong!”

    No. It’s a nonsense example. Warm you compared to what? Are you in a room of 20C air or in deep space at -273. It’s not a useful thought experiment. There are too many unknowns.

    Manners and humility among friends will earn you more respect.

    00

  • #

    To Jo @ #207.

    Err, Jo, you’re cherry picking your facts. GHE exponents fail to explain that the atmosphere functions to block IR on the way in, too. In fact, almost no solar radiation makes it to the surface.

    Light comes in a lot of different wavelengths and different atmospheric gasses are opaque and transparent to different wavelengths of light. But ALL gases absorb and emit IR, not just CO2 that is a miniscule 0.04% of the atmosphere.

    So are you actually suggesting CO2 is a unique gas that ‘traps” energy? You could just as fallaciously argue that adding extra Oxygen molecules will increase atmospheric IR absorption.

    The IPCC and GHE supporters wrongly argue that the blocking mechanism only works one way, like a tap, trapping heat inside the atmosphere. But the blocking mechanism in reality operates on both the inflow and the outflow otherwise if input didn’t equal output then we’d go boom!

    Consider albedo, GHE believers don’t like albedo and its not in the ‘theory’. The IPCC refused to address Earth’s albedo the effects that blocks energy entering Earth. To make their ‘theory’ more plausible they ignored 13 of the 16 factors driving climate that would otherwise invalidate claims about CO2. See IPCC Report (2007) ‘2.9.1 Uncertainties in Radiative Forcing.’
    http://www.ipcc.ch/publications_and_data/ar4/wg1/en/ch2s2-9-1.html

    So with almost no solar radiation making it to the surface the ‘back radiation’ element of the GHE is also discredited. Adiabatic pressure is the key not CO2 because CO2, like other gases, works to inhibit warming during the day and delay cooling at night and thus is a moderating, not a heating factor.

    00

  • #

    John: I give up on us having a meaningful conversation. I don’t think we are talking the same language.

    “In fact, almost no solar radiation makes it to the surface. “

    I see solar radiation with my own eyes. In fact it’s so strong, it hurts if I look straight at it.

    00

  • #

    To Jo Nova @ # 213:
    Jo, please do not ignore the point I made that energy is not heat and heat is not temperature. Infra red energy transmission is not necessarily transmission of heat (heat implies work thus causing increase in temperature). But your statement about IR is proven to be false under the Laws where you say that ”It heats if it hits.”
    Transmission of IR is entirely independent of the heating process. Please explain your understanding of kinetic energy. Because your statement ”It heats if it hits” not only mischaracterizes the relationship between infrared radiation, temperature and kinetic energy, it is scientifically wrong. BTW, Which part of my comment lacked manners and humility? I did not intend to insult you, but if I did then I apologize.
    Many thanks.

    00

  • #
    Alistair

    Climate realist number9: 20 years ago, my old physical chemistry teacher and I had a brain storming session about how he could put one of my inventions into his metal/ceramic SOFC concept. On 20th September last year, we had the public announcement of the first production devices. 3,000 are being installed in the UK as a mass trial. About 2 months’ ago, Honda started marketing the domestic version of their automotive proton conductor system in Germany. And Australia has its own BlueGen, an all ceramic system with 60% efficiency from hydrogen, less for methane.

    As for cold fusion, I worked with Martin Fleischman in 1987. The idea was buried by the military: in 2007, the Atlanta NRL published proof that you get what they call Low temperature Nuclear Reactions. The internet buzz is that a 1 MW system will be unveiled in September this year in Italy with claims you get nickel converted to 30% copper: a powder cathode with pulsed power?

    Don’t mock it if you don’t know it. As for the radiation arguments: remember Kirchhoff’s law and emissivity and absorptivity of a gas mixture have to be determined from Hottel Diagrams or their computational equivalent. Also, recent work shows CO2 with O2 has a much reduced, possibly negative global warming potential. Net CO2-AGW could even be negative!

    The game has barely started now we know the IPCC was for 30 years scientific smoke and mirrors and by 2004 was a busted flush: to keep AR4 going, an inside group decided to bluff with fake physics [‘cloud albedo effect’ cooling is actually heating for thick clouds as can be easily proved just by looking at how clouds darken underneath as droplets coarsen – the two stream approximations [Sagan, Chhandrasekar] are wrong for thicker clouds].

    00

  • #
    Siliggy

    John O’Sullivan:
    May 16th, 2011 at 1:06 am
    To Jo @ #207.
    …..”We know you’re wrong because when I stand next to a large block of ice (emitting more IR) I get colder much quicker than if stand by a smaller block of ice (less IR). Better still, no ice at all! Try it.”….

    That would be a block of iced steel at perhaps 600 degrees C right?
    So are you cold next to a big block of iced steel at 600 degrees C but warm next to a small one and even warmer out in the vacuum of space!
    Oh right i get it.

    00

  • #
    Roy Hogue

    Wow! Its like trying to herd cats, isn’t it?

    A long time ago, I asked (somewhat naively) what AGW skeptics agreed on wrt climate science. The answer appears to be “nothing”.

    Jo & Michael Hammer have a pretty good understanding, probably better than mine. However many others are abysmal. I don’t think they are particularly stupid, just willfully obtuse.

    There is no harm in understanding a few things. You don’t give up your individuality or freedom by conceding that there are plenty of things which really smart people (much smarter than anyone on this blog) have painstakingly discovered which are true, and that you have much too little skill or expertise to argue otherwise.

    So if you don’t understand something, ask questions. If you don’t understand the answer, and you really want to, you could try reading some physics text books.

    John Brookes trots out his timeworn proof by authority one more time.

    Please John, give it up. You sound like a broken record. Worse, you hardly ever engage in any of the debate on the science involved. And if you do it’s at a pretty trivial level. Why? Is it because you’re scared to death that you’d embarrass yourself?

    00

  • #
    Alistair

    Climate modelling and the CO2-AGW scam exist solely because of two scientific mistakes. The key question is ‘is there any CO2-AGW at all’.

    00

  • #

    to Jo @ # 213:

    Jo: “No. It’s a nonsense example. Warm you compared to what? Are you in a room of 20C air or in deep space at –273. It’s not a useful thought experiment. There are too many unknowns.”

    Jo, Let’s stick to an earthly room filled with air at 20C to test your statement, “An IR beam is an IR beam. It heats if it hits.”

    I’m making a comparison of two events whereby in one room I’m standing with a large block of ice (bigger object = lots more IR hits) and then in another room with a smaller block of ice (smaller object = fewer IR hits). According to your “It heats if it hits” characterization I ought to be warmer standing in the room next to the bigger block of ice because it’s emitting more IR than the smaller block of ice, right?

    In the above scenario I think we see your explanation about heating from increased IR falls down. Temp will flow only from a warmer to a cooler body. By the same token such a supposition in the GHE is also wrong because any added IR radiation from CO2 is not using such energy to create additional heat. All the (non-heating) IR energy received readily escapes to where there is no warmth i.e. into space.

    00

  • #
    Alistair

    What Jo doesn’t factor in is the view factor in radiative heat transfer calculations. BTW there is recent experimental proof that ‘back radiation’ isn’t real. It’s a Dutch PhD thesis where the student shinned up and down an 800 foot high radio mast to do the radiative flux measurements.

    It’s all about IR absorption near the surface of the Earth and its conversion into sensible heat. Then it becomes Prevost interchange in a temperature gradient defined by buoyancy-induced convective heat transport. Lindzen knows full well that the atmosphere at the end of the Hadley cells is a patchwork quilt of hotter, damper air ascending and colder, dryer air descending.

    It’s far more subtle than most people can comprehend.

    00

  • #
    Nullius in Verba

    “According to your “It heats if it hits” characterization I ought to be warmer standing in the room next to the bigger block of ice because it’s emitting more IR than the smaller block of ice, right?”

    Nope. You get radiation from both the room and the ice. If the ice covers a bigger angular area, then you get more radiation from the ice but a lot less from the room, which is now covering a smaller angular area. Since the room is warmer than the ice, replacing your view of the room with ice lowers the total IR you are receiving.

    On the other hand, if you were surrounded by outer space at 4 K, then replacing cold space with ice at 273 K would increase IR considerably, and a bigger/closer block more so. Of course, if you are in a room at 0 C the same as the ice, then whether the ice is big or small makes little or no difference.

    00

  • #
    Siliggy

    John O’Sullivan:
    May 16th, 2011 at 2:12 am
    …”I’m making a comparison of two events whereby in one room I’m standing with a large block of ice (bigger object = lots more IR hits) and then in another room with a smaller block of ice (smaller object = fewer IR hits). According to your “It heats if it hits” characterization I ought to be warmer standing in the room next to the bigger block of ice because it’s emitting more IR than the smaller block of ice, right?….

    “Right”
    No wrong! By standing next to the ice(water ice this time) you have blocked radiation from the warmer room which is the alternative in your other room.

    Note the room is warmer than the ice but colder than you.

    Note that in BOTH rooms the radiating sources of IR are from objects that are colder than your body temperature.

    When you stand next to the large block of ice you get less “if it hits it heats” from the room because you are hiding behind the ice from the room’s radiation.
    So your example fails to prove Jo wrong but instead proves her to be correct.
    Well done!
    Lance Pidgeon

    00

  • #
    David, UK

    RJ:
    May 15th, 2011 at 9:10 am

    david UK

    [A blanket] does not [absorb some of your body heat and emit some of it back]. Blankets simple slow down the rate of cooling.

    Yes, but how exactly? Does it hold up a stop sign? No, it absorbs some of the heat from your body, and the heat energy is emitted in all directions, including back to your body.

    And what is this net movement? The net movement is from warmer to colder

    Yes! That’s exactly right! Where’s the argument there?

    Are you suggesting that a proton from a cold object can in fact heat a warmer object as long as the opposite is happening at the same time. This just does not seem logical or scientific to me.

    No of course not, I never suggested that, and never would suggest that. You’re right – that would be scientific nonsense. A colder object will never heat a warmer object. *Net* movement is always from warmer to colder.

    Either heat moves from hot to cold or it moves in both directions. Hot to cold is the correct option IMHO.

    “Both” is the wrong word. “ALL” is more accurate. Energy is emitted in all directions.

    Net does not enter the picture.

    Well ok, if you say so! But I suggest you read up on some basic laws of physics! 🙂

    Can I just say, I’m really grateful to Jo for raising this (and Anthony Watts has also had a similar post on WUWT which was good). Misunderstanding the second law of thermodynamics allows the AGW bedwetters to characterise us as morons. It’s good to have some education. The way some people are arguing you would think our whole case rests on a re-writing of a fundamental physical law! It does not. It rests on the fact that the *alarmists* have no argument but for their continually tweaked models, fudged data, screaming ad hominem, and tax-funded propaganda. Time after time they prove their scientific illiteracy with their skewed logic and wild assumptions (that old cliché “the models cannot account for the current warming without factoring in anthropogenic CO2 – therefore it must be CO2 wot dunnit” is a fave of mine). I urge those who still don’t get the 2nd law (as presented in this post) to read it again, please. 😐

    00

  • #

    To Nullius and Siliggy @ # 223 and 224.

    Congratulations guys, I’ll award you both first prizes in sophistry.

    00

  • #
    Kasmir

    I’m no alarmist, but CO2 absorption characteristics are a fact.

    The apparent paradox lies is resolved by looking at the behavior of the atmosphere, which must also obey the Ideal Gas Law. Temperature at the surface is thus higher than the Black Body temperature of the earth. CO2 still acts to change the heat distribution as measured by the temperature lapse rate; the distribution of heat within the atmosphere is changes so relatively more heat is retained in the lower atmosphere and less in the upper. Radiative losses balance gains exactly, but the profile of where within a vertical section of the atmosphere radiative losses are occurring changes. Hence the measured stratospheric cooling, which exactly offsets the lower atmospheric heating caused by additional CO2. There’s no paradox.

    So additional CO2 does indeed heat the lower atmosphere and cool the upper. That really isn’t disputable. What *is* disputable is how the huge amount of water available to the atmosphere responds to the changes in temperature forcings with altitude that the additional CO2 is creating. Water vapor is of course itself a powerful greenhouse, but water aerosols (clouds) have a strong positive albedo. Add in turbulent vertical convection and phase transitions (rain) responding to nearly chaotically created local heat engines and it becomes complex to the point where it’s unmodelable by GCM’s. Given a CO2 forcing redistribution, how much additional water vapor is created vs how much additional clouds at what level of the atmosphere represents completely unsettled science. That’s what the argument is about.

    00

  • #

    Kasmir Post 227 wrote,
    ” Given a CO2 forcing redistribution, how much additional water vapor is created vs how much additional clouds at what level of the atmosphere represents completely unsettled science. That’s what the argument is about. ”

    You have fallen into the “we” only debate the figures, or whether at a push, they are the wrong sign trap.

    Jo Nova in ADDENDUM #2
    ” The cooler item is not supplying a single new joule of energy, but there another mechanism of increasing an objects temperature. It’s called insulation. It’s a reality we all know and use every single day. Why deny it? ”

    Insulation does not increase an objects temperature, insulation reduces the objects losses (by one or more of the THREE possible ways heat transfer can occur, in the given examples).
    ie,
    1) Thermal radiation losses.
    2) Conduction and convection of sensible heat.
    AND,
    3) Losses due to the latent heat of water vapourisation, which most forget, as water vapour makes air lighter ALSO causes / further enhances convection. Very effective it is too at cooling, hence “we” sweat”. OK, OK, so Jo “perspires”.

    So, the insulated object remains warmer than it would otherwise be (ie without insulation) for longer.
    By reducing conduction and convection of both sensible and latent heat.
    radiative losses at these temps / time scales are minor im comparison, so I have effectively ignored them..

    In short the blanket greatly reduces convection losses, mostly of sensible heat, which means the released heat from the object raises the air temperature under the “blanket”, this in turn reduces the temperature difference to the object and so convection reduces.
    Quite simply the released heat of the object under the blanket “hangs around longer” – that’s all.
    Clouds do much the same thing (reduce convection), hence a cloudy night is warmer than a clear night.

    BTW – Please don’t forget the earth has a second massive heat source, as well as the sun,
    it’s called the earth’s core, which is damned hot.
    The oceans cover 71% of this planet’s surface, the ocean floors are thin (5 to 10 kms),
    the upper mantle is probably about 900C and, water vapour has a high specific heat value….

    But if you’d rather quibble the figures from closed flask samples, as modelled, for an atmospheric constituent of 0.04%,
    with no proven effects in the open atmosphere upon climate, then go ahead..

    00

  • #

    Michael @183:

    Waffle at 180; You stated CO2 absorbs at some infra red wavelengths. So long as the absorption wavelength is somwhere between about 6 and 50 microns (the wavelengths at which there is significant radiation from Earth to space)that is a definition of a green house gas. I am very fully aware that the mechanism is completely different from what actually happens in a green house and the common term is thus a poor one but as far as I am concerned it is simply a widely used convenient handle by which to refer to the subject. Please don’t discount the arguments simply because I use the widely accepted label. I am not and never have clamed that the mechanism bears any relationshiop to what actually happens in a green house.

    My understanding of basic physics is that, any increase in C02 concentration in the atmosphere will change the average relative elevation in which the C02 tends to stratisfy. This is a result of more heat being taken up by way of conveyance. This doesn’t change the temperature of the atmosphere. Fluid dynamics demands that less bouyant gases repalace C02’s average elevation at the investigated height. At no time does the temperature profile of the atmosphere change, only the composition profile.

    We’re yet to see experimental data to prove or disprove that claim but, this is what the basic laws of physics tell me. I guess that the greenhouse gas theory rests on the assumption that the earth is dissipating heat at it’s maximum rate. This assumption is dangerous to rest a theory on, and an entirely unproven one.

    00

  • #

    I should caveat the term dissipating to mean energy conveyance, not IR radiation. 🙂

    00

  • #

    John Brookes @ 211:

    Jo & Michael Hammer have a pretty good understanding, probably better than mine. However many others are abysmal. I don’t think they are particularly stupid, just willfully obtuse.

    This is the difference between the sceptics and the true believers, John. I would not be an unbeliever today if it weren’t for my readiness to argue a different point of view. My approach to most things is to find an expert and argue against their position(or, take my own position and argue against it).

    I agree that Jo and Michael know so, so much more than I do about the subject than I but, argument from authority is not the point of the exercise. It’s about prosecuting a case. Something you will not find on RealClimate or Deltoid or Climate Progress, etc.

    The greenhouse gas theory MUST be held to account. What does it mean? I go back to the failure to communicate the science issue. Greenhouse gas theory means a second mortgage to the true believers, to the sceptics it has to be defined and measurable. An argument must be constructed and defended. Simple.

    What is a greenhouse gas? To me(atm) it is an oxymoron, like military intelligence. Smoke and mirrors to fool the likes of you.

    00

  • #
    stephen richards

    I wrote a wonderful piece this morning in reply. Posted it and lost it. There are lote of issues with this post many of which stem from a very poor thought experiment devised by an engineer and not a physicist. I have seen time and time again this confusion between classical and quantum physics in the atmosphere. Let me try to clear my head because it is a long time since I had to use this physics.

    The 3 ‘laws’ of thermodynamics (= movement of heat)are from classical physics. They were promulgated at a point in time when convective and conductive heat were thought to be the dominent modes of transport. In classical physics (the 3 laws) heat ALWAYS is assumed to move to cold. Warmed air is less dense than colder air and rises. This applies to all gases include water vapour. Then along came Bohr et al and the quantum mechanics movement was born. Incidently, it was called quantum mechanics because the old guard, including Einstein, thought it was nonsense and wouldn’t have it in the physics arena. Now came the radiative ‘models’ and the infamous quantum jump. The jump, it was proposed, would only occur when an atom/molecule was excited by a photon of exactly the right energy (hµ nu not mu) to move an electron to the next available valency level. This was the model that solved the problem of aborption frequency and why, for example, CO² had 2 sharp aborption bands (2 degrees of freedom in the molecule). OK, so , an atom/molecule will only absorp a photon if said molecule is not already in the excited state ( to talk of heat is confusing). If the mole is already excited the photon will not be absorped and ALL excited mole want to return to ground state at the earliest possible moment. This can happen in µsecs. Bearing in mind the gas density of the atmosphere and that these things happen at the speed of light, the delay in the atmosphere is miniscule (probably). Quantum physics is a probability based science. IR will pass through any excited moles. Water vapour is a major IR aborber much more so than CO². Right, now for Classical physics. This is the kinetic branch. As air becomes less dense and rises it takes with it the excited molecules of gas (CO², H²O etc); This also happens at great speed just watch a thunder cloud develop in the tropics. So excited molecules are taken high into the troposphere to release their photons into the trop and stratophere. It is not good enough to talk about blankets or back radiation or layers of gas or any other ‘thought’ experiment. The pan of boiling water proposed by Dr Roy Spencer was a good one for the classical physics but I haven’t yet seen a good example for the quantum version. Read also Dr Roy’s IR experiment in a box and using an IR detector. Not ideal but not bad.

    00

  • #
    BLouis79

    Please Jo can we have a separate thread on Joseph Postma’s “Thermodynamic Atmosphere Effect”. It should be compulsory day 1 reading for anyone interested in global warming. His logic is compelling. But people have to read the whole paper…

    Paper via:
    http://climaterealists.com/index.php?id=7457
    or directly:
    http://www.tech-know.eu/uploads/Understanding_the_Atmosphere_Effect.pdf

    RIP greenhouse

    00

  • #
    Paul

    Concentrating attention on only one form of energy-transfer, radiation, leads to an unrealistic simplification of the weather system. What happens in an enclosed cylinder of gas, with radiation passing through first a vacuum, to eliminate conduction, then through still gas enclosed in a thermos-like cylinder and measuring the amount of radiation absorbed, is of interest in understanding our atmosphere but it is far from containing exhaustive information about it.

    My take on IR radiation, including the much vaunted back-radiation, is that you could compare the radiation fields to electrical potentials, so the net flows will always be in the direction of warmer to colder object. The rate of flow will be conditioned by the relative warmth of surrounding objects. Hence, we are warmer at night, with no external radiation from the sun hitting us directly, by reason of the fact, amongst others, that the air around us remains warmer than the background of dark outer space. Our homes are bathed in the potential energy radiated from the surrounding atmosphere and so remain warmer than the cold side of the moon.

    And it is not only the radiation from CO2 that keeps us warm at night, it is also the radiation from all the other molecules in the air around us.

    One fact that seems to be mostly ignored is that, once a molecule of CO2 [or H2O for that matter, 95% of the GHGs] receives a photon of long-wave radiation from the surface of the earth it does not just re-emit the photon, ad infinitum, as some seem to think. Rather the newly energised molecule continues to bump into adjoining molecules and its newly acquired energy is shared with those other molecules, Hence the temperature of the gas mixture increases and no photon is emitted. By the time you get a few meters above the surface there will be much less radiation from the earth’s surface, much of it already having been converted to heat.

    Since only a small portion of the energy that is incident on the surface of the earth is emitted as long-wave radiation this further reduces the impact of increasing the proportion of GHGs in the atmosphere. If, as has been suggested by some research, there is a compensating reduction in water vapour in the atmosphere as the other GHGs increase then the effect of increased atmospheric CO2 would be effectively neutralised, any way.

    By arbitrarily setting all other means of transporting heat from the surface of the earth to zero and concentrating on just one factor to explain all the heat transport, the hypothesis of CAGW has caricatured the real situation and blown things so far out of proportion that what may have been a fortuitous side-effect of human endeavour to improve living standards has become demonised as a threat to the very existence of all life on the earth.

    The question is not what happens to long-wave radiation leaving the surface of the earth, though that is interesting in itself, but how the earth moves towards an equilibrium temperature near the surface and what effect increasing the proportion of CO2 in the atmosphere has on that temperature.

    Since there is an equilibrium of incoming and outgoing radiation, occurring at about 5,000 metres about the surface, one would think that the processes of the weather, occurring in the oceans and the troposphere, with conduction at the surface warming the air and causing convection currents, latent heat of vapourisation at the surface of the oceans and lakes absorbing vast amounts of heat and transporting it to the clouds above, large ocean currents transporting even more heat from the equatorial regions to the polar regions, etc, etc, then the role of GHGs in mediating the near-surface air temperatures would seem to be limited to an extremely minor one at the best, and possibly none at all.

    Why not stick to the question that matters?

    Paul

    00

  • #
    Cole

    Hey Jo Anne,

    Ok, so the greenhouse gas effect you’re explaining would work the same as a blanket by adding insulation to the atmosphere. The problem being is dry air makes a better insulation layer than moist air (I work in building envelopes) and the air has to be confined in dead space…There is no such mechanism in our atmosphere.

    I would like to address your cold night vs inside explanation as well. For one, it’s dependent on an external heat source and no better than stating day is warmer than night. I’m from Canada and have lived in very cold areas so I understand a bit about staying warm.
    We dress in layers in dry clothes, the thicker the better. However, by your argument a wet layer closer to your skin should keep you more warm. Trust me it doesn’t.
    If you are in an area that is say -40c you can not be running, simply adding sweat to your clothing WILL KILL YOU because it increases the cooling effect between you and the atmosphere. Just as I strongly suspect that adding water to the atmosphere increases the cooling efficiency between earth and space.

    Here is a paper from Robert Ashworth, Nasif Nahle and Hans Schreuder, that explains it better in scientific terms, http://www.tech-know.eu/uploads/Greenhouse_Gases_Cool_Earth.pdf

    Cheers
    Cole

    00

  • #
    KR

    BLouis79 @ 233

    I took a quick look at Joseph Postma’s paper – it fails on multiple levels.

    Primary error:

    The blackbody will then be re-emitting just as much thermal infrared energy as the light energy it is absorbing.

    Followed, two sentences later, with:

    …imagine that you take a mirror which reflects infrared light, and you reflect some of the infrared light the blackbody is emitting back onto itself. What then happens to the temperature of the blackbody? One might think that, because the blackbody is now absorbing more light, even if it is its own infrared light, then it should warm up. But in fact it does not warm up; it‟s temperature remains exactly the same. (emphasis added)

    But the blackbody is receiving more energy than it did before – the incoming light from some hot source plus the reflected energy from the mirror. It will warm to a temperature where it’s radiating as much energy as it’s receiving, or it violates thermodynamics and the conservation of energy.

    The rest of the paper is more of the same. The radiative greenhouse theory does not violate thermodynamics! There’s definitely some uncertainty as to feedbacks, as to the scale of the effect – but insisting that the radiative greenhouse effect violates physics isn’t going to go anywhere.

    What I’m about to say is a bit on the harsh side, and will likely result in some insulting replies, but:

    These kinds of fundamental scientific errors make it very difficult for folks hearing these arguments to take the presenters seriously. If you want to be taken seriously by people who do thermodynamics for a living, then you need to recognize a bad argument at the start.

    I would really like to applaud Jo and Michael Hammer for putting up this post to clarify the subject.

    00

  • #
    John Watt

    Most of what is covered in the comments to date is covered in Dr John Nicol’s “Greenhouse Effect” analysis. Some of the contributors appear to have the necessary physics background to critically review Nicol’s analysis.Any takers? If he is right then all we need are some physics-savvy parliamentarians!

    00

  • #
    Roy Hogue

    Kr,

    I’ve read the paper and gone over your post several times and you appear to be saying that the blackbody should be heated additionally when its own emitted IR is reflected back to it. If so, you are certainly wrong. The photons that left the blackbody toward the mirror have removed energy from it and they obviously cannot be both within the blackbody and in transit between it and the mirror simultaneously. So when they arrive back at the black body they add back only the energy they removed when they left. QED: the blackbody cannot be heated by this means.

    If this could happen your face would catch fire every time you turn on your bathroom light while in front of the mirror.

    Thanks to Lionell Griffith for the good analogy.

    00

  • #

    Instead of arguing over this. Apparently Jo is a physicist or knows some physicists that could set up a laboratory experiment where a well isolated system could be checked to see if back radiation does in fact decrease the rate of heat loss when a lower temperature radiative surface is replaced with a slightly less lower temperature radiative surface. The added insulation argument does not seem to stand very well, as the atmosphere is many kilometers thick and increasing CO2 some small amount is not going to increase the thickness much.

    00

  • #
    John Brookes

    Roy Hague@238:

    So you have a black body in a vacuum (to avoid conduction & convection) which is illuminated by a light. At equilibrium the temperature of the body is such that it radiates away exactly the same amount of energy that it receives from the light. Any less and it warms up, any more and it cools down.

    Now you put a mirror which reflects infrared light so that some of the infrared light emitted by the body is reflected back onto the body. As KR says, the body now has more radiation incident on it than before. From the point of view of the body, it doesn’t matter where the extra radiation is coming from – there is simply more than before. The only way it can get rid of the extra energy is to warm up, and that is precisely what it does. This is not some kind of runaway feedback loop which will make your face catch fire when you look in the mirror.

    The thermos flask has a silvered inner surface to reduce energy losses from radiation by reflecting the outgoing radiation back into the hot liquid in the flask. If you put a little heater into a thermos, you will end up achieving a higher temperature than if you did so with a flask identical in all respects except that the inner surface was not silvered.

    00

  • #
    Paul

    Jo @161 Here’s another analogy: Energy flows through the system continuously like water through a river.

    If we add a dam, the dam provides not one new drop of water, but it raises the level of the water available in the river at that point. The blanket, the GHG gases are like dams on energy flow. The temperature builds up behind them because energy is continuously being pushed into the system.

    If you have a dam that is already overflowing, through a spill-way, and you put a small divider on the spill-way that diverts the overflow into two sections. Will that impede the flow? Will the level of water behind the dam have to increase to maintain the same flow as before the divider was put in place?

    I would contend that no measurable increase in water-level would occur. There would, however, be a theoretical increase in the flow-rate over the slightly reduced width of the spill-way. I suppose that there may be an infinitesimal increase in the height of the water behind the dam to account for the increased flow-rate over the slightly reduced width of the spill-way.

    In the same way, if some out-going long-wave radiation from the earth is absorbed by atmospheric CO2, surely that will only mean that there will be an increase in the rate of out-going radiation at other frequencies with, at worst, an infinitesimal increase in the temperature of the near-surface-level atmosphere.

    As far as I am concerned, the argument is not about infinitesimal increases in temperatures but about “Catastrophic Anthropogenic Global-Warming” with none of those terms being unnecessary. If the vanishingly-small increase of atmospheric CO2 added by human activities has a vanishingly small impact on near-surface atmospheric temperatures, then it is entirely benign and the whole CAGW scare is over.

    Paul

    00

  • #
    1DandyTroll

    A blanket is a physical insulation layer. That it is it traps air, and everything in it, to make it not move, that’s what is keeping you warm. If you shut down ventilation so the air don’t move it takes equally longer before you start to freeze, the time it takes depends on how fast your home leaks heat. Outside, it is the wind that makes you loose heat faster as well.

    The question is not so much if a colder object gives away heat but if the objects around it will absorbed that heat. So three objects with different heat content will, seemingly, automagically, equalize fairly quickly before they start loosing heat about equally fast. So the heat that colder objects radiate is pretty much moot because the warmer objects wont absorb the colder heat since it is already on that wave length. Of course different substances and all that . . . but natural substances isn’t the holy grail of thermo dynamics.

    As long as space is as cold as it is and as long as earth has a 360 degree evaporation area earth will undoubtedly always leak more heat the more heat it has.

    Try this for a thought: Earth’s speed around its own axle varies pending on which degree you’re at, between a couple of hundred miles an hour to a thousand miles an hour, which, apparently, mean the heat transfer to space varies as well all pending the wind shill so to speak. But the earth also travels around good old Sol at about 67 000 Km/h. And added to that the whole solar system is traveling around milky way at about 550 Km/s (which translates to 2 000 000 km/h.) All these speeds are depending upon all sorts of varying gravitational pulls from all sorts of stuff, and, apparently, all these pulls varies as in they are never really constant. So what happens with the “global” temperature of earth if the earth’s heat transfer to space varies with the speed when the speed constantly varies?

    Come to think of it, earth hasn’t been in this neck of the milky way wood for some 250 million years or so, but both the poles were most likely much colder than the equator, even back then. :p

    00

  • #

    Now this is what I would call an educational Post, and the best part of it all has been the comments.
    Sometimes you go to a site looking for information, and because you’re coming from a background of ‘wanting to know’, you tend to get ignored as the people who have a better idea of the Science concentrate on each other, whether they are sniping at each other, or trying to get their point across.
    However, this time, for some inexplicable reason, what has been said has given some of us, well me anyway, a better inkling of what it really is all about.
    Questions have been answered in ways that those of us ‘wanting to know’ can actually understand, and now, gaining that somewhat better understanding, we can perceive the more ‘in depth’ things a little better.
    THAT is what has made it so good.
    Way back at Comment number 177, michael hammer said:

    The $64 million (or is that $640 billion) question is how much.

    Therein lies the crux of the matter.
    Those who have a separate agenda have latched onto that part of it, and are using the Science discussed here (without understanding it fully or maybe even partially) as a source of endless money.
    I want to know about (the basics of) the Science, as that helps me when I attempt to mention what the consequences might be with respect to those emissions, not the consequences of what the CO2 may or may not do, but the consequences of moving away from one of the major sources of those emissions, that of electrical power generation, what Bob Brown refers to as ‘the big polluters.’
    While the concentration may be on the Science, (and the money) there is no real mention about the truth of what it might lead to.
    So, to Jo for posting this initial Post, and for all the Commenters, I owe many thanks for making something so complex just that little bit easier to understand.
    I hope a Post like this gets widespread notice, because something like this is what the average people who have little understanding need to see, because if it can help me, it can also help them.
    It hasn’t changed my opinion, but now I understand it all a little better, so thanks.
    Tony.

    00

  • #
    KR

    Roy Hogue @ 238

    I have to agree with John Brookes @ 240. At equilibrium what comes in must equal what goes out, or the system is not at equalibrium. Energy reflected to the object in question is, essentially, energy that doesn’t leave. Hence the rate of energy leaving (at any particular temperature) is less than it was without the mirror. Energy will then accumulate in the object (blackbody or graybody) until that object again radiates as much energy as it receives.

    That’s the core of the radiative greenhouse effect – modifying the amount of energy radiated by Earth at any particular temperature. Add a dollop of greenhouse gases that radiate part of their intercepted energy back to the surface, or raise the effective altitude where energy is radiated to space (and hence the temperature, due to the tropospheric lapse rate), and less energy leaves. Input then does not equal output, and the only thing that can happen is an energy change in the climate until output once again equals input – equilibrium at a different temperature.

    The climate is a throughput system – at every point, what enters should be what leaves, or the amount of energy at that point must change (basic conservation of energy). Energy comes in from the sun, the Earth holds a temperature that (with the current atmosphere) allows it to radiate the same amount of energy to space. It is not a steady state system – energy flows through, and the rates must match or temperatures change.

    The second law of thermodynamics is still satisfied – heat still flows from warmer to colder. But changing the greenhouse effect changes the rates, and hence the temperatures involved.

    Feedbacks (which drive the total temperature change), again, are another issue.

    00

  • #
    KR

    Roy Hogue @ 238

    An addendum – no, your face won’t catch fire when looking in the mirror.

    Why? Because the mirror reflects something less than 100% back to you. And the sum of a decreasing series like this is always finite, never a runaway.

    The formula for any feedback (like H2O response to warming) is:

    Result = Forcing/ (1 – gain)

    If the gain (percentage returned) is < 100%, this holds, and the result is a fixed, finite number. It may be big, it may be small (depending on the gain), but it’s limited.

    00

  • #
    cohenite

    KR, this is an ongoing issue, the interpretation of the 1st and 2nd laws of thermodynamics; I don’t think the people who are querying the greenhouse effect are quibbling with the laws but whether the prosposed GE is consistent with them.

    The GE relies on delay, usually illustrated by a blanket or coat, and return, via backradiation, to produce its warming. The delay is illusionary because of the radiative window and CO2 saturation [as shown in the recent Lu and Harde papers] which means upward LW in the window wavelengths leaves Earth at the speed of light. Kinetic transfer of energy from CO2 absorption to surrounding non-greenhouse gas molecules such as O2 and N2 is dwarfed by the heating of those molecules through contact with the surface. So on this basis the GE is a bit player at best.

    In terms of backradiation and the mirror image you refer to, there is no doubt with a constant solar heating replacing what the surface emits that any of that emitted energy which is returned must add to the total energy received at the ground. But does this backradiation, which is notoriously difficult to measure, heat?

    I have already referred to 2 pressing reasons to suspect the backradiation does not and they are mentioned at comment 175; and I have also referred to the ongoing issue of atmospheric pressure creating the atmospheric temperature profile without any need to refer to the GE.

    So, what are we left with; a GE which appears to be consistent with the 1st and 2nd laws but which has, possibly, no measureable effect.

    00

  • #

    Paul @ 241:

    In the same way, if some out-going long-wave radiation from the earth is absorbed by atmospheric CO2, surely that will only mean that there will be an increase in the rate of out-going radiation at other frequencies with, at worst, an infinitesimal increase in the temperature of the near-surface-level atmosphere.

    An increase in back radiation does not translate to an increase in temperature. Just an increase in the volume of IR flow between the surface and the atmosphere. To increase temperature you have to hold that radiation. Which is why there is no greenhouse effect. There is no science claiming either the surface, the oceans or the atmosphere are radiating at their maximum potential which would translate to an increase in temperature. Now I’m getting back to the point about entropy.

    00

  • #
    KR

    cohenite @ 246

    I’ve always hated the “delay” statement – that doesn’t match with what I know of the physics, and in fact is rather silly. I really wish folks would drop that. It’s just a bad/incorrect explanation.

    I think of the radiative greenhouse effect (RGE?) as a rate issue. The Earth, as a gray-body with a particular emission spectra, emits IR to space at a particular rate based upon temperature and effective emissivity (see this link, Power = emissivity * Area * SB constant * T^4). Greenhouse gases change the Earth’s effective emissivity to space, and hence change the temperature required to match the incoming power.

    Incidentally, the back-radiation is very easy to measure (point an infrared spectrometer up, or a pyrometer, and measure the energy coming down from the atmosphere) or to calculate (atmospheric composition, temperature, emissive spectra derived from that), and they match up. And that’s been true since real measurements started in the early 1950’s.

    00

  • #
    Kevin

    In reply to comment #148 (copied in part);

    “The point is that YES, obviously in the real world, blankets keep us warm. Pink batts “lift the temperature of your home in cold weather”. They don’t do it by supplying energy, they do it by blocking energy loss. The cooler item is not supplying a single new joule of energy, but there another mechanism of increasing an objects temperature. It’s called insulation. It’s a reality we all know and use every single day. Why deny it?”

    Well, yes and no, first off, the human body is not a good analogy for the Earth. The human body BURNS fuel (GASP) and creates heat, thus it is a true blackbody radiator. The Earth (in the context of the Sun/Earth/Universe system) is strictly a quasi-blackbody “RE-RADIATOR”. It is not a source of energy. Likewise the “GHG’s” in the atmosphere are narrow band “RE-RADIATORS” of infrared energy.

    A few specific points;

    “Pink batts “lift the temperature of your home in cold weather”.” Totally false, if you believe this I suggest you turn your furnace or heat stove off for a week and see how much the “pink batts” lift the temperature of your home.

    “They don’t do it by supplying energy, they do it by blocking energy loss.” Again totally false, they SLOW THE RATE OF ENERGY LOSS. Nobody knows how to “BLOCK ENERGY LOSS”. By slowing the rate of energy loss the “pink batts” keep your eyes from bugging out of your head when the fuel bill arrives.

    “but there (they are, sic, refering to Pink Batts)another mechanism of increasing an objects temperature.”, Sorry but again this is totally false, your furnace is increasing the temperature of the objects in your home. The pink batts just make the task easier for your furnace.

    The “GHG’s” in the atmosphere MAY slow the rate of cooling enough to cause a “higher equilibrum temperature” to exist, but all indicators are that this is not bloody likely (sorry I had to throw in some Aussie slang).

    Unfortunately for the GHG Theory there are adjacent wavelength windows in the spectrum of the blackbody radiation emitted from the surface of the Earth. When the surface warms by even a small amount (from whatever cause) the spectrum shifts to lower wavelengths and MORE IR ENERGY escapes to the cold vacuum of space. This causes the Earth to AUTOMATICALLY COOL BACK DOWN. Without a jolt of sunshine everyday we would all be damm cold……

    Cheers, Kevin.

    00

  • #
    BLouis79

    @KR #236

    Rather than having a quick look at Postma’s paper and querying the plain language analogy descriptions, try reading the physics and the formulae to see if there is a genuine error of logic. What he says makes perfect sense (and I presume he is smart enough not to make a silly mathematical calculation error).

    That his description pulls the rug from under climate science’s “greenhouse theory” will no doubt upset people wedded to that theory. True scientists will have no problems accepting the science and moving on.

    00

  • #
    KR

    BLouis79 @ 250

    I read through his paper, and saw that plain contradiction on the very first pass, contained in a single paragraph:

    Energy from input = energy out. Clear, physically supportable.

    Energy from input + plus some reflected energy != energy out. Bull****.

    Given such a clear error in basic physics, need I go further? That is such a clear error, and directly tied into all of his conclusions – the rest of the paper cannot stand on such assertions.

    00

  • #
    Kevin

    In reply to comment #25 from the original author (copied in part);

    “CO2 simply makes the atmosphere opaque to radiation at 15 microns and the net impact of that is to reduce the energy loss to space at that wavelength.”

    First, “opaque” is a somewhat vague term. Lots of specific terms are available in the field of optical engineering to be much more specific about this. “Optical Density” is one such term that would apply.

    Second, “reduce the energy loss” is a term that is sufficiently vague as to be almost meaningless. Is “reducing the energy loss” a compliment to “increasing the energy gain” ?

    Thirdly, energy flows continuously through all systems. It is not possible to “block”, “trap”, “stop”, etc. the flow of energy through a system. Various combinations of materials (i.e. GHG’s .vs. non-GHG’s) may slow or hasten the flow of energy through a system, but mankind has not yet observed any way to “reduce the energy loss” through a system, only the means to slow (or hasten) the rate at which energy loss occurs.

    If I might be so bold I would like to rephrase your summary statement as follows;

    “CO2 simply makes the atmosphere somewhat opaque to radiation at 15 microns and the net impact of that is to slow the rate of energy loss to space at that wavelength.” Fortunately adjacent shorter wavelengths allow ample energy to transit to the cold vacuum of space, thereby keeping the Earth from ever reaching a new “higher equilibrium” temperature.

    Cheers, Kevin.

    00

  • #
    Roy Hogue

    KR, John Brookes,

    I didn’t intend the face thing to be taken seriously. 😉

    I guess it’s the choice of words because what you posted said that the radiation returning from the mirror can heat the blackbody. It can’t for the reason I stated. If it could then the interaction with the mirror would have to create energy. The additional warmth comes from the fact that the blackbody isn’t cooling as fast as it would without the mirror and since it’s still getting radiation input from some nearby source it must warm some more to regain equilibrium. I understand all that.

    00

  • #
    cohenite

    KR@248: you say: “Incidentally, the back-radiation is very easy to measure”; I must disagree and pages 3 and 4 here will explain why:

    http://jennifermarohasy.com/blog/2011/02/more-on-the-stefan-boltzmann-equation/?cp=3

    00

  • #
    Willaim Gray

    There is No instrument that corectly measures background radiation. Radiation induction and then convection. Thats how it works.

    00

  • #
    John Brookes

    Damn you Roy Hogue! How am I supposed to know when you are joking?

    BTW, it turns out that Osram believe in the greenhouse effect – and they’d be engineers and all….

    00

  • #
    Alistair

    Tell me how the atmosphere can be dichroic?

    00

  • #
    Bryan

    KR

    Fig 32 from the Gerlich and Tscheuschner’s paper was shown to indicate the how impossible the Greenhouse Theory is!
    Its not supposed to be physically possible.

    Now KR something to really shock you!

    Could you give me a figure in backradiated downwelling long wavelength radiation in W/m2 for the effect on AGW due directly to human induced CO2.
    We will then find out if the pyrometer or even the pyrgeometer is capable of measuring this quantity.
    On receipt of your figure(be as generous as you like) I will furnish you with a recent official paper which makes some rather frank admissions about how accurate these instruments really are.

    00

  • #
    bananabender

    @John Brookes:
    May 16th, 2011 at 5:16 pm

    BTW, it turns out that Osram believe in the greenhouse effect – and they’d be engineers and all….

    It’s nothing but blatant marketing BS to baffle gullible buyers.

    These Osram bulbs are regular quartz halogen bulbs placed inside a standard incandescent bulb housing. QH bulbs have been used in car headlights for nearly 50 years. Halogen bulbs rely on the halide cycle to partially regenerate the filament allowing them to run hotter. They are brighter simply because the filament is hotter than a standard bulb. No back-radiation is involved.

    00

  • #
    bananabender

    When university qualified scientists and engineers actually believe that energy can freely flow from a lower to higher energy state [without a heat pump] all hope is lost IMHO. Poor old Niels Bohr must be spinning in his grave.

    With all due respect Jo and Michael you need to go back and repeat first year university physics.

    00

  • #
    BobC

    bananabender:
    May 16th, 2011 at 10:14 pm
    When university qualified scientists and engineers actually believe that energy can freely flow from a lower to higher energy state [without a heat pump] all hope is lost IMHO. Poor old Niels Bohr must be spinning in his grave.

    With all due respect Jo and Michael you need to go back and repeat first year university physics.

    I’m seeing some confusion in this debate that I think is due to making the assumption that most everything has blackbody characteristics. Indeed, if you have two blackbody absorbers/emitters in sight of each other, net energy flow will be from the hotter one to the cooler one.

    Gases under low pressure, however, do not behave like blackbodies. Instead, they absorb and emit radiation in narrow bands determined by their internal resonances (energy levels).

    Try this experiment: Use an ordinary laser pointer to put a spot of light on the surface of a running, frosted, 100W light bulb. It is easy to see the spot of red light on the surface of the bulb (after your irises narrow down quite a bit). This demonstrates that the laser is many orders of magnitude brighter than the light bulb in that narrow band. For the light bulb to have the illuminance of the laser within the laser bandwidth, it would have to have a temperature of 10^4 – 10^6 degrees (depending on the laser’s output power).

    Obviously, within that bandwidth, the energy flow via radiation is from the (colder) laser to the (hotter) light bulb.

    This is sometimes taken into account in the literature by assigning an “equivalent” temperature to the laser, which is typically very high — the actual value calculated as the temperature of a blackbody that would have the same irradiance in the laser’s bandwidth. This redefinition of “temperature”, however, is confusing also. Better to realize that the flow of energy (via radiation) between objects that are not blackbody emitters/absorbers may not be related to their actual temperatures.

    00

  • #
    BLouis79

    Part of the beauty of Postma’s explanation is that one can understand a real reason why a lot of earth-bound temperature phenomena are as they are.

    In the meantime, nobody has dared advance a hypothesis on the behavior of the IR laser in various gaseous environments in terms of absorption, scatter and backradiation.

    According to the CO2 backradiation theory, what temperature should a greenhouse full of CO2 be in the sun? …compared to a greenhouse full of any other gas? – surely an easy experiment to do and measure……which gaseous mix would make the greenhouse the hottest??

    00

  • #
    BLouis79

    Part of the beauty of Postma’s explanation is that one can understand a real reason why a lot of earth-bound temperature phenomena are as they are.

    In the meantime, nobody has dared advance a hypothesis on the behavior of the IR laser in various gaseous environments in terms of absorption, scatter and backradiation.

    According to the CO2 backradiation theory, what temperature should a greenhouse full of CO2 be in the sun? …compared to a greenhouse full of any other gas? – surely an easy experiment to do and measure……which gaseous mix would make the greenhouse the hottest??

    00

  • #
    KR

    There were multiple comments to the effect that “backradiation is not measurable”.

    Actually, it is. Pyrometers aren’t very accurate, but infrared spectrometers are. And these measurements are very available and up to date – see (for example) the Atmospheric Radiation Measurement (ARM) Program, especially the longwave and shortwave data sets publicly available.

    CO2, H2O, methane, ozone, etc., bands are easily identifiable in the downward spectra from the atmosphere.

    Bryan @ 259 – Figure 32 in G&T is incomplete, in that it doesn’t include energy flows up and down, or properly show the net energy flow to space. Stating (as they do) that it represents the greenhouse theory, and then arguing against it, is just a straw man argument.

    I really prefer seeing all sides of the debate at the table – a one sided discussion is usually misguided. But if the argument presented (such as “AGW violates thermodynamics”) is ridiculous and wrong from the outset, those presenting such an argument are going to have trouble being taken seriously. Again, kudos to Jo Nova and Michael Hammer for presenting this thread.

    00

  • #
    RJ

    @265

    But if the argument presented (such as “AGW violates thermodynamics”) is ridiculous and wrong from the outset,

    And you qualifications are?

    00

  • #
    RJ

    Ops your qualification are?

    00

  • #

    BobC @ 262,

    You are equivocating on the term “temperature” by blanking out the fact that the laser converts electrical energy into higher flux (ie brighter) than the lamp of “red” photons without using a thermal device. The laser is a quantum device but even it must obey the three laws of thermodynamics. You are also blanking out how the electrical energy came into existence in the first place.

    It is very easy to compute a positive output from a macro system if you fail to account for all the inputs into a system. It makes no difference if the system is a true black body or not.

    All macro systems MUST obey the three laws of thermodynamics.

    The first law: If you believe you have a macro system that has more output than the sum of all inputs, you have either made a mistake in your math, failed to account for all inputs, or are committing a fraud.

    The second law: If you believe you have a macro system that converts all of the input energy into another form of energy that can do useful work, you are mistaken. The only exception is a perfectly reversible (lossless, frictionless, etc) macro energy conversion process.

    The third law: a perfectly reversible macro energy conversion process is impossible.

    When you argue that quantum effects don’t have to obey the three laws you are failing to take into account that the three laws are time/space invariant laws and quantum effects are all about time/space dependent variations. Hence your argument does not take into account all the input factors. Especially since all macro systems are time/space integrations of the actions and properties of a very large number of micro quantum entities. Hence, the three laws are implicitly obeyed at the micro quantum level when viewed in FULL CONTEXT!

    In simpler language: it is always possible to say you can get something for nothing if you ignore significant portions of reality.

    There is no such thing as a free lunch. Someone or something somewhere will have to pay for it somehow sometime. For example, you find a ham sandwich on a plate in front of you, you eat it, and don’t pay for it. It is “free” in the sense you blank out how it came into existence and look only on the fact you ate it and did not pay for it. This is a time/space dependent (micro quantum) view of a “free” lunch. You dropped context and blanked out significant portions of reality.

    00

  • #
    Bryan

    KR
    What are we to make of you?

    You raised two substantive points but you dont appear to want to defend your position.

    Gerlich and Tscheuschner’s figure 32 is a little diagram showing that some people are silly enough to think that HEAT can move from a colder to a warmer surface.
    Their thermodynamics as usual are impeccable.
    However some of the labels seem to be “lost in translation” which for Germans is excusable.
    This was probably cleaned up before the final papers publication.
    You now tell us that Pyrometers aren’t very accurate, what about pyrgeometers which is usually the IPCC instrument of choice.
    Are you not concerned that the increase in downwelling LWIR due to the tiny level of human contribution to CO2 is being measured by inaccurate instruments?
    You are posting as an expert in Backradiation yet you cannot supply a figure in W/m2 that we should all be alarmed about.

    00

  • #
    BobC

    Lionell @ 268:

    You’re mixing up my posts — in #262 I was not addressing the laws of thermodynamics, just the facts about radiative energy transfer being dependent on the absorption/emission spectrums of the objects involved. The generalization that net radiative energy flow can only go from hot objects to cooler ones is true only if they have the same absorption/emission spectrum.

    Low pressure CO2, for example, only absorbs and emits in a few narrow bands within the Solar spectrum — outside those bands, it is essentially transparent.

    The ground surface, on the other hand, is usually much closer to a blackbody in its emission/absorption characteristics.

    I was simply pointing out that net radiative energy transfer between these two things is not only dependent on their temperatures, as their absortpion and emission spectra are very dissimilar.

    How you think that obvious fact violates the Laws of Thermodynamics is not evident to me. I certainly wasn’t making that claim.

    All macro systems MUST obey the three laws of thermodynamics.

    This statement is both circular reasoning and the fallacy of “mistaking the map for the territory”.

    We “know” that, say, the second law of thermodynamics (SLT) is a “Law of Nature”, because we observe that all known macro systems obey it. If we ever find a macro system that doesn’t obey it, then the only logical conclusion is that we were mistaken in elevating SLT to a “Law of Nature”. To do otherwise is equivalent to a man claiming the road he is standing on doesn’t exist, since it is not on the map he is looking at.

    The question of whether Solid-Oxide Electrolytic Cells violate the second law of thermodynamics seems obvious to me. However, it also seems obvious that Jo’s blog is not the right venue for discussing that. If you are interested, I authorize Jo to send you my email.

    00

  • #
    BobC

    Addendum to #270:
    Lionell, if you are arguing (as you seem to be) that I don’t think that the Laws of Thermodynamics are “by definition” true (handed down by God, as it were), you are right. I’m an empiricist — the world is what we discover it to be, not what some theory says it has to be.

    00

  • #
    BobC

    Addendum #2 to post #270:

    Lionell Griffith:
    May 17th, 2011 at 12:39 am

    BobC @ 262,

    You are equivocating on the term “temperature” by blanking out the fact that the laser converts electrical energy into higher flux (ie brighter) than the lamp of “red” photons without using a thermal device.

    There is no doubt that you can define “temperature” in such a way that the laser is a higher temperature than the light bulb. I even gave an example: Define the laser’s “temperature” as the temperature of a blackbody that would have the same irradiance in the laser’s bandwidth.

    Then, net radiative energy flow is from the object with the highest “temperature” to the one with the lower “temperature”. I’m not sure what this actually accomplishes, however, other than making you feel better about the observed facts.

    00

  • #
    Paul

    It now appears to me that one main cause of confusion in this discussion is conflating two entirely different concepts : —

    Quantum effects and statistical estimates.

    Referring back to the analogy of water released from a dam, one could say that, occasionally a drop of water may be splashed back over the spill-way. But one can never say that the current of water will flow backwards over the spill-way.

    Talking about photons and what they may or may not do has no bearing on the overall result in terms of how much energy is transmitted by radiation, nor in terms of the direction of that flow.

    This subject is difficult enough to comprehend without confusing such different concepts, IMHO.

    Paul

    00

  • #
    KR

    RJ @ 267

    No need for my qualifications – read the post from Michael Hammer, where we’re commenting now, which certainly covers the topic. The same information is available elsewhere – a quick Google search will take you to multiple discussions.

    Greenhouse gas warming doesn’t break the second law of thermodynamics – arguing that it does tells others that you don’t know the thermodynamics, and makes it very hard to be taken seriously in the debate.

    Bryan @ 269

    I don’t want to rehash G&T on this thread – that’s been done in detail here, here, and in the peer reviewed literature here, by people with much better thermodynamics backgrounds than mine.

    As to the anthropogenic CO2 forcing, that’s been estimated at 1.66 W/m^2 (pg. 136) as of the IPCC AR4 report.

    00

  • #

    Paul @ 273,

    That is exactly what is meant by the time/space dependency of quantum effects and the time invariant nature of the three laws of thermodynamics.

    Still, there is no such thing as a free lunch if you look at all factors in the energy transformation equation. Any “free” bit is always at the cost of something else and is therefor NOT free. What that something else might be is sometimes very easy to overlook but that is no excuse for overlooking it and thereby claiming the “free” bit is actually free.

    If you think you can get something for free without hidden or ignored cost, make such a device and become fabulously wealthy and, at the same time, make all of mankind fabulously wealth. Forgive me for thinking that the universe will expire long before you accomplish even the first part that task. For now, I make things that work by relying on the three laws of thermodynamics and managing the costs so they are affordable for the benefit received.

    00

  • #

    BobC @ 270,

    You seem to be arguing that macro things and processes are not made up of quantum micro things and processes because you are rejecting the notion that thermodynamics is not implicit in the workings of the quantum world. What then are macro things and processes made of if not an integration of quantum micro things and processes?

    The macro behavior is understood by time invariant laws. The quantum micro behavior is understood by time/space dependent laws. That the gap is bridged in the real world is evident. That the various hypothesis, models, and computational schemes yet developed by man cannot totally bridge the gap is irrelevant to the fact that reality bridges the gap as part of its basic nature.

    I refer you to my post at 275 for my further commentary on your notion that you can get more out of ANY system, micro or macro, than the integration of all of the inputs into the system.

    In other words, make the device, and then make yourself and all of mankind fabulously wealthy and I will become a believer. Until then, there is no such thing as a free lunch.

    00

  • #
    RJ

    @ 274

    No need for my qualifications – read the post from Michael Hammer, where we’re commenting now, which certainly covers the topic. The same information is available elsewhere – a quick Google search will take you to multiple discussions.

    I have read the post from MH.

    Have you read the Gerlich and Tscheuschner paper. Or the slayers book. Or the Postma post above at @97. Or any of many excellent posts above?

    00

  • #
    BobC

    I don’t know who you’re arguing with Lionell, but it’s not me. I don’t recognize anything I wrote in your post:

    You seem to be arguing that macro things and processes are not made up of quantum micro things and processes

    Well no, actually, I’m not. Where did I say that?

    … you are rejecting the notion that thermodynamics is not implicit in the workings of the quantum world.

    I seriously doubt I’m doing that, since I’m not sure even what that means. Are you implying that you fully understand the “workings of the quantum world”? Congratulations, then — you are one up on Richard Feynman.

    The macro behavior is understood by time invariant laws. The quantum micro behavior is understood by time/space dependent laws. That the gap is bridged in the real world is evident. That the various hypothesis, models, and computational schemes yet developed by man cannot totally bridge the gap is irrelevant to the fact that reality bridges the gap as part of its basic nature.

    Now here, you seem to be agreeing with me that the “Laws of Nature” (as determined by man) are provisional and likely (perhaps necessarily) incomplete, and that such laws do not determine reality. Great — but you seem to think that this statement in agreement with me is somehow correcting me.

    I refer you to my post at 275 for my further commentary on your notion that you can get more out of ANY system, micro or macro, than the integration of all of the inputs into the system.

    How can that be my “notion” when I have never claimed any such thing? I refer you to any post I’ve made anywhere. I do not know of, nor have I claimed to know of, any system that violates conservation of energy.

    In other words, make the device, and then make yourself and all of mankind fabulously wealthy and I will become a believer. Until then, there is no such thing as a free lunch.

    Snark ill becomes you. My posts have all been serious. If you have no serious answers to them (other than making up straw men and snarky comments) that is your problem, not mine.

    00

  • #
    KR

    RJ @ 277

    As per previous posts on this thread, yes, I’ve read G&T and Postma. Both contain critical physics errors that invalidate them. Michael Hammer (and Roy Spencer, in the “Yes, Virginia” link I posted in @18) both cover this ground quite well.

    I haven’t read the “Slaying” book, but I understand that Lindzen, Monckton, and Spencer all disagree with it.

    00

  • #
    BobC

    Paul:
    May 17th, 2011 at 2:03 am

    Talking about photons and what they may or may not do has no bearing on the overall result in terms of how much energy is transmitted by radiation, nor in terms of the direction of that flow.

    Since the energy transmitted by (EM) radiation is transferred by photon exchange, I’m not sure how you untangle them.

    Energy transfer from a hot object to a cooler one by EM radiation will proceed quit differently, for example, if the cool one is either a perfect blackbody, or 99% reflective.

    00

  • #
    co2isnotevil

    I actually agree that the GH effect does not violate the second Law. Radiative effects are different, the case in point being that the Earth is warmer than it would be without the Sun radiating energy our way. The Sun itself is warmer than the Earth, but the space between the Earth and the Sun is not. So, the Earth, being in contact with ‘cold’ space is warmed by virtue of radiation passing through that cold space.

    Relative to the GHG effect, the surface stays warm by delaying the release of surface energy. This is the subtle difference that most warmists fail to get.

    The law that’s broken by ‘consensus’ climate science is Conservation of Energy, not the second Law. This breaks because the 3.7 W/m^2 of incremental forcing is turned into 16 W/m^2 of incrementally emitted surface power (corresponding to a 3C rise). The ‘positive feedback’ explanation for this is what breaks Conservation. There are only 2 ways that extra energy (the 12.3 W/m^2 in excess of 3.7 W/m^2) can be emitted by the surface. First is if the Sun emits 12.3/.7 more power (.7 is 1 minus the average albedo), second is if the albedo decreases by enough to let 12.7 W/m^2 more power enter the system. On average, the planet reflects about 102 W/m^2. For it to reflect 12.7 W/m^2 less, the albedo must be reduced by 12.5%

    The 2 mechanisms warmists cite as the source of this energy is either decreased ice causing decreased reflection or water vapor ‘feedback’. First, even if all the ice on the planet melted, the reflectivity would not decrease by enough for this much effect and second, to achieve this much incremental H2O absorption would require increasing the average water content by more than a factor of 2. Neither of these is consistent with observations or theory.

    See here for more details on how the radiative balance of the planet behaves.

    00

  • #
    KR

    co2isnotevil @ 281

    While I really don’t want to revisit the discussions we’ve had on the subject here and on Skeptical Science, I’ll just point out two things. First:

    “There are only 2 ways that extra energy (the 12.3 W/m^2 in excess of 3.7 W/m^2) can be emitted by the surface. First is if the Sun emits 12.3/.7 more power (.7 is 1 minus the average albedo), second is if the albedo decreases by enough to let 12.7 W/m^2 more power enter the system. “

    These are both input statements. The radiative greenhouse effect is a constraint on output to space, namely incrementally insulating the Earth so that a higher surface temperature is required to pump the (unchanged amount of) input energy back out to space, through the radiatively insulating atmosphere, as infrared. So those lines in your post aren’t even discussing the greenhouse effect.

    Secondly, the 3.7 W/m^2 forcing from doubling CO2 (which has not happened yet) leads to feedbacks that are expected to add additional imbalances – 1.2 C or so for just CO2, not all 3 C predicted by the average climate scientist. So your “gosh, how can this be!?” approach isn’t justified by the numbers.

    00

  • #
    Bryan

    KR Says
    “I don’t want to rehash G&T on this thread – that’s been done in detail here, here, and in the peer reviewed literature here, by people with much better thermodynamics backgrounds than mine.”….

    Well you brought it up.
    Over at WUWT I managed to get one of the authors of your “peer reviewed reply” to admit that in that paper they had an incorrect use of the word Heat (that is all six of them).
    So your understanding of thermodynamics might well be up to theirs/

    Thanks for the reply

    …..”As to the anthropogenic CO2 forcing, that’s been estimated at 1.66 W/m^2 (pg. 136) as of the IPCC AR4 report.”…….

    What would you say if an official ARM publication admitted that some readings could be as much as 17W/m2 in error.
    Would you recommend that the world economy should be disrupted based on that kind of evidence?

    00

  • #
    KR

    Bryan @ 283

    The very word “Heat” has been a problem in this discussion, mainly because everyone seems to have a slightly different interpretation. I find it much clearer to speak of energy flows and accumulations – fewer chances for a misunderstanding.

    As to the “17W/m^2 in error” – well, that’s where I thought you were going to go.

    Some of the measurements, while extremely precise (very repeatable, shows any changes) are not terribly accurate (may be poorly calibrated to an absolute value). But the changes are actually pretty clear, and multiple measurements from multiple sources improve accuracy accordingly. Current radiative imbalance (even given the ENSO state and a strong solar minimum over the last few years – which I’ll point out has not kicked us into an ice age) is about 0.82 ± 0.12 W/m^2 (0.77 ± 0.11 W/m^2 in the oceans, where ~94% of the energy goes) average over the last five years or so (Schuckmann et al 2009).

    No doubt you’ll disagree with those numbers, I’ll just agree to disagree on that – but none of this changes the basic fact that, as Jo and Michael have pointed out, greenhouse gas warming doesn’t break the second law of thermodynamics. And folks (like G&T) who claim that are making a bad argument.

    00

  • #
    Ian Hill

    I can only smile at the irony of this quote from a news article about a planet found to be orbiting the star Gliese 581, about 20 light years away:

    With a dense carbon dioxide atmosphere — a likely scenario on such a large planet — the climate of Gliese 581d is not only stable against collapse but warm enough to have oceans, clouds and rainfall,” France’s National Centre for Scientific Research (CNRS) said in a press release.

    The scientist, having no vested interest in Earth’s climate future, says it how it is!

    00

  • #
    Bryan

    KR

    Yes you are correct that many people get the wrong definition of heat.
    However when they start to opine on thermodynamics after that, you can be pretty sure that it will be all downhill from there on.
    For Ordinary people its understandable.
    For Professional Physicists is inexcusable!

    You must be very easily pleased with regard to errors.
    For an effect of less than 2W/m2 you think at an error of 17W/m2 is satisfactory.

    00

  • #
    Lionell Griffith

    BobC,

    Me: All macro systems MUST obey the three laws of thermodynamics.

    You: This statement is both circular reasoning and the fallacy of “mistaking the map for the territory”.

    You: The question of whether Solid-Oxide Electrolytic Cells violate the second law of thermodynamics seems obvious to me.

    I infer from your responses that you do not view that the three laws of thermodynamics are a property of macro systems but are simply optional features for which we have found no counter examples.

    I further infer that it is more than likely that you think Solid-Oxide Electrolytic Cells need not obey the three laws because it is some how not a macro system.

    The net of it all is that you think a free lunch is possible. I say it is not and that any attempt to pretend that it is, is due to the accidental or willful mis-accounting of all inputs in to the system.

    Your contention that I must know everything about the quantum world to conclude that the three laws are implicit in it, indicates two things: that the three laws are optional for macro systems and/or that macro system are not made up of an integration of micro quantum things and processes.

    Lastly, you need not state something in so many words to say it. Words have meanings and the same concept can be expressed using different sets of words. As near as I can tell from your expressed words, you believe as I have identified.

    As I said before, produce the ability to make a truly free lunch and, as a consequence become rich, then I will become a believer. I am more than willing to bet that you will be working a very long time to come up with one in either the macro or micro world. Until that time, everything I design and build will be predicated on the fact that the three laws do in fact apply every time IN THE CONTEXT OF THE REAL UNIVERSE. Working in fantasy and science fiction does not count.

    00

  • #
    Bryan

    KR

    There are almost infinate versions of the “greenhouse theory”

    Some do break the second law while others do not.
    Had Michael said there is a two way exchange of radiation, then that would be fine.
    But to say there is a two way exchange of heat……
    Well as you know KR…..

    Heat travels spontaneously from a hot surface to a colder surface – never the reverse.
    Clausius second law.

    Kr, what is your personal version of the greenhouse theory?

    00

  • #
    BobC

    Apparently, Lionell, this is a religious thing with you — we are just going to have to disagree, as I’m not likely to be converted.

    Griffith:
    May 17th, 2011 at 6:45 am

    BobC,

    I infer from your responses that you do not view that the three laws of thermodynamics are a property of macro systems but are simply optional features for which we have found no counter examples.

    Despite your attempt to phrase this as negatively as possible, you have a kernel of truth here: I believe that “Physical Laws” are rules which seem (as far as our experience goes) to have universal application. These laws are based, ultimately, on empirical observation (which is why we aren’t going to see any physical laws come out of String Theory anytime soon).

    The curious thing for me is: How (for some people) do empirically based “Laws of Nature” become inviolate and sacred? It seems that there must be an act of faith involved here, since our experience isn’t universal. I do, in fact, consider physical law to be provisional and most scientific theory as well. I would claim that history is on my side here.

    When, for example, I see that solid-oxide electrolysis cells are well known to be able to produce useful work from ambient (single reservoir) heat, I see a potential violation of the Second Law of Thermodynamics. I am motivated to study the physics of such cells to see if I am right or have a misconception.

    You, on the other hand, have no need to study the physics of SOECs at all — you have revealed knowledge that the Second Law can never be violated, so actually understanding how such cells work is irrelevant. Though your faith may be different, you are a spiritual brother to the priests who refused to look through Galileo’s telescope.

    Like I said; This is a religious difference and we aren’t going to resolve it.

    The rest of your convoluted logic trying to determine “what I believe” is completely unnecessary — I’ve just told you what I believe.

    00

  • #
    Lionell Griffith

    BobC,

    You can believe in word salads all you like. I don’t happen to believe in them.

    It is clear to me that you don’t believe that proper empirical induction can lead to certainty. As a consequence you don’t believe you know anything for certain. How can you know what you believe without using induction and knowing something? Why then should I believe anything you say?

    Violation of the second law implies that a system violates the first law by getting more useful energy out than is input into the system. As a consequence, SOECs violates ALL the laws of thermodynamics. Extraordinary conclusions require extraordinary evidence. The truth of the matter is that either SOECs don’t exist or they don’t work the way you think they work.

    00

  • #
    Nullius in Verba

    BobC,

    I was trying to avoid discussing it, but I can see that isn’t going to work either.

    The electrolysis issue is a distraction. The details of electrolysis theory are calculated on the basis of the same laws of thermodynamics. The claim appears to be the combination of a confusion of different definitions of “efficiency”, a subtle misunderstanding of what is admittedly some pretty abstruse physics, and on at least some people’s part, the hyping of fairly routine by now “investment opportunities” in perpetual motion technologies. I’m not going to go into the details – you can accept or reject as you choose.

    (I did have a look out of curiosity – I like these sort of ideas as an opportunity to develop my understanding, and I’d like to thank you for introducing me to a new one. But I don’t think it’s appropriate to go into it further here.)

    However, just to satisfy you that we’re not being dogmatic about this, it is also a well-known issue in physics that the second law quite possibly does not apply to the beginning of the universe – the low entropy initial state is currently unexplained. Physicists don’t take the second law as axiomatic. But there are good reasons for thinking that it isn’t going to be violated in any ordinary or easily accessible circumstances.

    If you want to propose that it might not be universal, I’d suggest starting there rather than with over-unity devices. Even if you were right about the electrolysis cells, it’s not a persuasive starting point in a greenhouse effect debate. It’s poor tactics.

    I think most people here are in agreement that the greenhouse effect is poorly explained and poorly understood in the public arena. I agree that the conventional “trapping” or “backradiation” argument is at best misleading, and arguably completely wrong. But to make progress persuading the scientific community at large, you have to have a better understanding of physics than the climate physicists, and that means getting the explanation of the reason why right. It’s not a violation of the second law, it doesn’t involve net heat flowing spontaneously from cold to hot. Back-radiation does exist, and it does get absorbed with a heating effect, but it doesn’t matter because that’s not how the greenhouse effect works in a convective atmosphere anyway, and the much bigger issue is feedbacks and sensitivity. That’s where they’re vulnerable. Spending days and days arguing about the second law, while the warmists speed away into the distance – laughing – is not going to improve the situation. It’s a waste of time and effort.

    I don’t expect the argument to stop because I said this. But is it possible we might at least agree that the aim of this discussion ought to be to get a better understanding of the physics, and what all the different points of view actually say? It’s not to win arguments or defend positions. If you think someone has not understood, then do by all means explain further. If they have understood, but rejected it, then it would be best to leave it there.

    On which note, I think I’ll leave it there. It’s been an entertaining debate.

    00

  • #
    KR

    Bryan @ 288

    “Kr, what is your personal version of the greenhouse theory?”

    Quite frankly, my personal version is the textbook version. How I explain it to others, what math and analogies I use, varies with the audience and the interest, as shown here.

    We get 240 W/m^2 energy from the sun – this is affected by things like albedo (ice coverage, aerosols, clouds, parking lots). This warms the atmosphere and the surface of the Earth, heat (net energy) flowing from warmer to colder.

    The surface of the Earth sends heat to the atmosphere, a lesser amount comes back as “backradiation”, or thermal radiation from the atmosphere, again heat (net energy) flowing from warmer to colder.

    And the surface and the atmosphere radiate to space, with heat (net energy) flowing from warmer to colder.

    So far, no violations of any thermodynamics.

    Now in order to remain at equilibrium (a steady temperature average) the Earth needs to radiate that 240 W/m^2 to space. What governs this rate? Thermal radiation (emitted by any object over 0°K), the power of which is set by two variables – emissivity and temperature. This is the Stefan–Boltzmann law, where:

    Power = emissivity * SB * Area * T^4

    SB is the Stefan-Boltzmann constant of 5.670 400(40)×10−8 W·m−2·K−4.

    Rearranging for temperature as a function of emissivity,

    T°K = ( 240 W/m^2 / ( emissivity * SB ) ) ^ 0.25, or
    T°C = ( 240 W/m^2 / ( emissivity * SB ) ) ^ 0.25 – 273.15

    This works in Excel; set your emissivity as one cell, and refer to that in the next one over, and you can try what happens with different emissivities to space.

    Now, without the greenhouse effect, dirt, plants, water, and basically all non-metallic surfaces have an emissivity near 1.0 in the temperature range we’re interested in. The average emissivity of the surface of the Earth is ~0.98, and for that emissivity the Earth would radiate 240 W/m^2 at -16.8°C.

    Brrrrrr…

    So what happens with greenhouse gases? Some of the radiated energy is absorbed and reradiated back to the surface (effectively not leaving), when it reaches an altitude where the GHG thins enough to radiate to space, the lapse rate cooling with altitude (lower T) means less radiation. Clouds block a lot of IR as well, and clouds cover a significant portion of the globe. You can see that in the top of atmosphere notches, as shown here.

    The references I’ve looked at say the effective emissivity (judging from satellite spectra) is about 0.612, ~61% the emissivity of a blackbody at average surface temperature, which corresponds to an average surface temperature of 15.2°C. Which, oddly enough, is about what we measure with those thermometer things…

    So – it’s all about rates. If the rate of energy leaving the Earth doesn’t match the rate of energy coming in, we’re not at equilibrium, and the temperature will be changing. Add a dollop of GHG’s, effective emissivity to space changes (effective radiation altitude increases, bands widen), and a 0.1% change in emissivity means ~0.9°C in equilibrium temperature. At equilibrium, rate in = rate out. And GHG’s affect the emissivity – the only thing that can change to regain equilibrium is the temperature.

    After that, after a GHG change in forcing, come the feedbacks. The best estimates right now for a doubling of CO2 (direct is ~1.2°C) feedbacks result in 3°C total increase – I have only faint hopes that this is wrong. But maybe the majority of climate scientists are wrong, and we’ll be lucky.

    00

  • #
    KR

    Forgot to mention this in my last post:

    Inside the atmosphere we also have convection and latent heat (evaporation), which move energy around in the troposphere. This is fortunate – weather and this energy circulation move warm air towards the top of the troposphere, where they can radiate to space. We would be a lot warmer if the atmosphere was static and only radiative exchanges occurred.

    00

  • #
    co2isnotevil

    KR,

    Your mistake is that the reduction in the output to space flux is already accounted for by the 3.7 W/m^2 of incremental absorption (actually, only half of the incremental absorption is blocked). In order for 12.3 W/m^2 of extra emitted surface power to arise, either more power must enter the system (increased solar output/decreased reflection) or more power must be blocked (extra H2O absorption). Is it your contention that the feedback further increases CO2 and H2O such that incremental GHG’s block 16 W/m^2 of incremental power (actually 32 W/m^2 to account for the half it blocks and which never finds it’s way back to the surface)? Please do a little due diligence and tell us where all this extra power is coming from and/or what’s blocking it from escaping. Your hand waving argument of “it’s the feedback” is insufficient and tells me that you haven’t thought this through.

    BTW, I wouldn’t rely on skepticalscience for an objective perspective on physics applies to the climate system. Few of the authors and even fewer of the readers have a clue about actual physics. It seems that physics isn’t a prerequisite for most environmental sciences degree programs.

    George

    00

  • #
    John Brookes

    Tell you what, lets just assume that the greenhouse effect isn’t real. That it does contradict well known, thoroughly empirically tested science.

    OK. Now explain why the earth’s surface is the temperature that it is (and, no, you can’t put it all down to the urban heat island effect).

    Any takers? BGV, Bryan, RJ, BLouis79? Go on, have a crack. Just why is the surface of the earth as hot as it is?

    00

  • #
    Roy Hogue

    Damn you Roy Hogue!

    I guess I’ve graduated into the big league now, what with John Brookes cursing me and all.

    Thank you John! 🙂

    00

  • #

    So – it’s all about rates. If the rate of energy leaving the Earth doesn’t match the rate of energy coming in, we’re not at equilibrium, and the temperature will be changing. Add a dollop of GHG’s, effective emissivity to space changes (effective radiation altitude increases, bands widen), and a 0.1% change in emissivity means ~0.9°C in equilibrium temperature. At equilibrium, rate in = rate out. And GHG’s affect the emissivity – the only thing that can change to regain equilibrium is the temperature.

    No, it’s all about retention of energy in the form of temperature increase. At no time is more energy being held as temperature within the system. To raise the temperature more input energy is required.

    At no time has anyone been able to demonstrate that the earth is conveying heat energy away from the earth at it’s maximum(or, a ta fixed) rate. It is simply conveying energy away from the earth at the rate in which it comes in.

    00

  • #

    OK. Now explain why the earth’s surface is the temperature that it is (and, no, you can’t put it all down to the urban heat island effect).

    I’ve already outlined in some of my posts, and others have done it also, the temperature is a factor of the planet’s gravity, quantity and composition of the atmosphere. It is the weight of the gases which determines the temperature(average) of a planet’s atmosphere.

    This is already clearly verifiable with empirical evidence. The greenhouse gas theory has already been disproven by those little inconvenient things called measurements on other planets.

    Any IR absorption properties of the gases contained within an atmosphere actually do, is to increase the number of steps which radiation goes through from input to output. There is no temperature increase due to the properties of gases, which seem to be mistaken on this thread as being fluids or solids.

    00

  • #

    If you want the analogy with a blanket to be more accurate consider the survival blankets which are simply a silvered sheet of thin plastic.

    Ok, this has caused me to change my view, and is so obvious I dunno why I did not think of it. We use thermal silvered blankets to keep radiation off objects in space.

    So, now onto the real questions, at what point does the reflective property no longer add any value. All other things remain identical, if you have multiple layers of silver film covering an item that has the same overall conductivity and connectivity how many layers before the last layer has absolutely no effect. If the silver is thick enough to have 100% of the radiation either blocked or reflected back, would one more layer of silver do any more good? I think we all know the answer to that.

    The same thing should basically apply to the atmosphere and the CO2 in it. Once there is enough CO2 to capture every single photon, then no additional warming can come from it. It does not matter how many steps it takes to get to space, it is the same total energy, the only difference is if you double the CO2 the level at which it can finally escape into space will be a couple feet out of several miles higher in the atmosphere with .0035 degree F per foot lapse rate. Only the first saturation layer of CO2 can eefect the temperature of the ground after that there is not enough back radiation to penetrate the CO2 that is between the ground and the current photons available.

    00

  • #
    Kevin

    Re: comment #295 (copied in part);

    “OK. Now explain why the earth’s surface is the temperature that it is”

    Simple really, before time = zero the Sun is switched OFF and the temperature of the Earth’s surface is near zero degrees Kelvin (ignoring for a moment the heat from internal processes like volcano’s etc.). At time = zero the Sun is switched ON, thereafter the Earth’s surface is heated by a small amount every day. Due to the HUGE thermal capacity of the oceans and rocks this process takes quite a while, perhaps several thousand years. At some point the energy radiated from the surface of the Earth approximately equals the daily energy input from the Sun. This is the “equilibrium temperature”; although it is really most appropriately called a “quasi-equilibrium temperature” since it varies slightly forever after. These variations are of course caused by the cloud cover, albedo, orbital changes, etc. etc. So for all time after the Earth has reached a “quasi-equilibrium temperature” the temperature is always “racing” to catch up with the energy flowing into the system from the Sun. At the same time the radiation outwards from the surface is “racing” to cool off the Earth’s surface.

    Sometimes the incoming energy from the Sun is winning the race and the weather is warmer than we expect. Other times the radiation leaving the Earth is winning the race and the weather is colder than we expect.

    Crazy isn’t it ?

    Ironically enough increases in “GHG’s” actually work to speed up the cooling “race team”, let me explain further. All three known methods of heat transfer (conduction, convection and radiation) work in parallel. The ratio of each to the others varies in response to many variables which I won’t describe here. The important thing to consider is that “non-GHGs” transfer heat away from the Earth’s surface via conduction and convection. These two heat transfer mechanisms are rather slow compared to heat transfer via radiation. These mechanisms transfer heat at the ”speed of heat”, aka thermal diffusivity. Radiation on the other hand transfers energy at the “speed of light” which is well known to be quite speedy. So when you displace “non-GHGs” with “GHGs” slightly more energy flows through the atmosphere of the Earth at the speed of light versus the speed of heat. This causes the gases in the atmosphere to warm up slightly faster after sunrise and to also cool down slightly faster after sunset. I believe this effect is so small that we probably could never afford to attempt to measure it.

    (Disclaimer, this discussion of the “beginning” of the Earth and its origins it not in any way a discussion of the origins of life, religion, evolution, or associated topics. Just a thought experiment for your entertainment, and perhaps something to think about)

    Cheers, Kevin.

    00

  • #
    Mark D.

    All the discussion surprises me. If we are in agreement that photons radiate in all directions from a body black (or otherwise gray). Then assuming that body is warmer than absolute zero, there is no other possibility other than photons from a colder body will indeed find their way to a warmer body and their energy will be absorbed by the warmer body. This should be intuitive.

    I find it totally fascinating that this is part of the “well understood” climate science.

    PS freely substitute “frustrating” with “fascinating” in the above paragraph.

    00

  • #
    Matt b

    Can you guys humour me for a second, and my desires to seek comfort in the words of authority, but are there any esteemed physicists who agree that the theory of AGW contravenes the 2nd law of thermodynamics?

    I just note George’s dismissal above that “It seems that physics isn’t a prerequisite for most environmental sciences degree programs.”

    But that Hawking, for example, seems to know his physics and seems pretty concerned about AGW. Or say The Institute of Physics: “The institute’s position on climate change is clear: the basic science is well enough understood to be sure that our climate is changing, and that we need to take action now to mitigate that change.”

    So lets just get it straight, the world’s leading physicists do not seem to agree that AGW contravenes the 2nd Law, and I reckon they know a tad more about the 2nd Law as your average punter.

    00

  • #
    Kevin

    Mark wrote;

    “there is no other possibility other than photons from a colder body will indeed find their way to a warmer body and their energy will be absorbed by the warmer body. This should be intuitive.”

    I totally agree, however the main point of contention seems to be if a colder body can cause the temerature of a warmer body to rise. Yes indeed a colder body can transfer energy to a warmer body, however it CANNOT transfer a NET ENERGY GAIN to the warmer body since the warmer body is transferring EVEN MORE energy to the colder body. Any calculations that purport to demonstrate “Net Energy Gains” violate the first law.

    Cheers, Kevin.

    00

  • #
    Mark D.

    Matt B, you get no humor here. The bigger issue of AGW theory will prove to be a farce because of all the other dynamic energy flows not related to Co2. Sooner or later you’ll figure that out.

    Brilliant minds are nearly as easy to fool as the not so brilliant.

    00

  • #
    KR

    co2isnotevil @ 294

    I’ve discussed these matters with you before, and have read through a great deal of your website – which is a Gish Gallop of errors. Your “gain factor” is incorrect (not a constant, for one thing), you are still invalidly “halving” the TOA imbalance (which you have acknowledged in your own runs of climate models), and are using CO2+feedback to argue against CO2 forcing alone. Each alone is a major physics error – all together, it’s a bit astounding.

    “BTW, I wouldn’t rely on skepticalscience for an objective perspective on physics applies to the climate system. Few of the authors and even fewer of the readers have a clue about actual physics. It seems that physics isn’t a prerequisite for most environmental sciences degree programs.”

    Well, I’ve seen your version of physics. I have to say I am not impressed.

    00

  • #
    Mark D.

    Kevin @ 303, I agree fully there can be no “net” gain.

    00

  • #
    Mark D.

    John Brookes @ 295:….

    OK. Now explain why the earth’s surface is the temperature that it is

    John, I have the answer but you won’t like it: Because the Creator deemed it so.

    Go ahead prove me wrong

    00

  • #
    KR

    Waffle @ 297

    So – in your version of physics, at equilibrium (temperature not changing) the amount of energy coming into the climate does not have to equal the amount of energy leaving the climate?

    I always thought that was basic conservation of energy…

    The rate of energy going out has to equal the rate of energy coming in. Greenhouse gases don’t have an effect on incoming energy, but they do affect emissivity to space (output). The only other variable that can change to regain equilibrium is temperature.

    If there’s an imbalance in the climate (input output) then the only thing that can happen is energy accumulates or drops until the temperature changes and input power = output power.

    00

  • #
    Mark D.

    KR @ 293

    Inside the atmosphere we also have convection and latent heat (evaporation), which move energy around in the troposphere. This is fortunate – weather and this energy circulation move warm air towards the top of the troposphere, where they can radiate to space. We would be a lot warmer if the atmosphere was static and only radiative exchanges occurred.

    Nice to see that you have an open mind. Now if you would please quantify how much energy is moved around in these ways?

    00

  • #
    BobC

    291Nullius in Verba:
    May 17th, 2011 at 9:11 am
    BobC,

    I was trying to avoid discussing it, but I can see that isn’t going to work either.

    The electrolysis issue is a distraction.

    Agreed, absolutely. I hereby vow to drop it, after this post 😉

    If you want to propose that it might not be universal, I’d suggest starting there [the Big Bang] rather than with over-unity devices.

    Apparently, the “confusion [over] different definitions of ‘efficiency'” is yours — I never described an “over-unity” device, nor does anyone else claim that electrolysis cells are such, to my knowledge — certainly not the industrial engineers who developed the data I reported on.

    (I get the impression people are responding emotionally to my posts, rather than taking the time to see what I am actually claiming. Another good reason to drop it, as it doesn’t matter to AGW.)

    …the hyping of fairly routine by now “investment opportunities” in perpetual motion technologies.

    Where do you see that stuff? I’ve never encountered anything like that. I must expand my reading list beyond boring engineering journals! 🙂

    But I don’t think it’s appropriate to go into it further here.

    I heartily concur, and I’ll let that be the last word.

    00

  • #
    BobC

    Hallelujah Lionell! We are in substantial agreement! Thanks for making the effort to understand my post.

    Lionell Griffith:
    May 17th, 2011 at 9:10 am
    It is clear to me that you don’t believe that proper empirical induction can lead to certainty.

    You are quite correct — that is what I believe: In mathematics, where the “universe” is strictly defined by postulates, inductive certainty is possible. In the real world, induction cannot lead to certainty because we don’t know the boundaries of the universe.

    As a consequence you don’t believe you know anything for certain.

    You’ve nailed me again! I definitely lack the certainty of the True Believer.

    How can you know what you believe without using induction and knowing something?

    Because, my dear Lionell, there are degrees of knowing and of certainty. I lack the constitutional ability to achieve the absolute certainty of the True Believer, because I think it likely that True Believers are deluding themselves. (But, I don’t know that for certain! 😉 )

    Why then should I believe anything you say?

    Why should you believe anything a True Believer says? Is a person’s strength of belief related to their correctness? There are many reasons to believe or disbelieve someone — generally, you should do as much due diligence as is prudent, given the consequences of believing.

    [SNIP] I vowed: no more thermo.

    00

  • #
    cementafriend

    There have been comments about the accuracy/inaccuracy of the Gobal Heat Balance
    See slide 26 of the following http://climategate.nl/wp-content/uploads/2010/09/KNMI_voordracht_VanAndel.pdf If Trenberth knows that information in atmospheric radiation “window” is 66 instead of 40 w/m2 and has not moved to correct his balance or withdraw his papers he is committing scientific fraud. The higher radiation “window” going directly to space falsifies all the calculations of GHG.
    I repeat again the possible absorption of CO2 in the atmosphere of radiation from the earths surface calculated from engineering equations based on real measurements is insignificant. CO2 is heated, as are other gases, in the atmosphere by convection at the surface and mixing of gases. It then radiates to space at the top of the atmosphere. Even there the radiation of CO2 at the top of the atmosphere is very small compared with the “window”, radiation from water vapor and radiation from the top of clouds.
    The post by Michael Hammer is a diversion and “gives some air” to the AGW believers.

    00

  • #
    cementafriend

    Just found this which has relevence to my comment above http://motls.blogspot.com/2011/05/new-danish-experiment-confirms.html#more Click Lubos’s link to get to the Danish experiment which has been published in Geophysical Research Letters.

    00

  • #

    BobC,

    At least we agree on one thing. We each believe the other is a crackpot. Although you can’t be sure of that because that would be based upon empirical evidence of only a very limited sample of eternity. Clearly, from both of our perspectives, further conversation between us would be pointless. Though, again, you can’t really be sure of that either. It will have to be sufficient that I am certain of it. Sometimes good enough will just have to be good enough.

    00

  • #
    co2isnotevil

    KR,

    The emissivity of 0.61 is for the planet, not the surface which is still arbitrarily close to 1. The planet is surface + atmosphere where more than half of the radiated emissions originate from clouds. The average emissivity is calculated as the power emitted by the planet (240 W/m^2 @ 255K) divided by the power emitted by the surface (385 W/m^2 @ 287K), which is 240/385 = 0.62.

    You’re neglecting that the emissivity of the planet is modulated by the atmosphere in a dynamic manner as part of a control path varying cloud coverage and atmospheric absorption in order to maintain the planets energy balance. Relative to atmospheric absorption, CO2 provides 1/3 of the total as a static value, while the remaining 2/3 is the average of a temporally and spatially dynamic component varying over a several hundred percent range.

    The consequence is that the flux in and out of the planet varies seasonally, which is asymmetrically different per hemisphere and whose sum shows the planet loosing more energy than it’s receiving for 6 months of the year and gaining more energy than it’s losing for the other 6. This manifests itself as the 3C change in the global average temperature seen between it’s maximum in June and it’s minimum in January. There are 2 points illustrated here.

    First, is that the planet is not in a constant state of disequilibrium owing to incremental CO2. The net balance over a year averages to about zero, but during the year, it increases for 6 months and then decreases for the other. See this plot based on measured ISCCP data, which shows the net imbalance, Flux, as the difference between the albedo adjusted incoming power and the satellite measured outgoing power. The individual hemispheres, north and south clearly show the linearity of the systems response, which is expected considering that the system is easily modeled as a first order LTI. Notice that the individual hemisphere p-p energy imbalance is about 180 W/m^2! The tiny fraction of a W/m^2 represented by the year to year CO2 change will be washed out by the yearly variability and will not accumulate into any sort of ‘pending’ effect.

    Second, is that the planet responds relatively quickly to changes in the system or stimulus, whether due to seasonal solar variability, albedo changes, CO2 changes or anything else. If this wasn’t the case, we wouldn’t see a global average change of 3C p-p, but more importantly, we wouldn’t see the much larger seasonal changes in each hemisphere. This invalidates claims that the system responds too slowly to see the ‘predicted’ effects of incremental CO2.

    While you claim to have read some of what I’ve written, you still don’t understand. The Flux in the earlier plots is the flux in and out of the planets thermal mass or the sensible heat. When the planet receives more energy than it emits, it warms and visa versa. This can be written in equation form as,

    Pi*(1-a) = Po + Ps

    where Pi is the power arriving from the Sun, a is the albedo, Po is the post absorption power emitted by the surface and clouds and Ps is the sensible heat. If E is the total energy stored in the planets thermal mass, Ps can be recast as dE/dt. E is linearly proportional to the temperature of the thermal mass T, as in 1 calorie raises the temperature of 1 gram of water 1 C, where most of the planets thermal mass is ocean. Po is dependent on the emitted power which is proportional to T^4. The connection between Ps, E, dE/dt, T and Po defines the solvable first order differential equation describing the climate system and the response extracted from measured satellite data conforms exactly to the strict requirements for the solutions to this equation. I should point out that an almost exact numerical correspondence arises when the model considers that half of the power absorbed by the atmosphere escapes, never to affect the surface, moreover; the HITRAN based absorption model confirms that the 3.7 W/m^2 of claimed ‘forcing’ caused by doubling CO2 is indeed the incremental power absorbed by doubled atmospheric CO2.

    George

    00

  • #
    BobC

    Lionell Griffith:
    May 17th, 2011 at 2:51 pm
    BobC,

    At least we agree on one thing. We each believe the other is a crackpot.

    I must protest! Although you undoubtedly think I’m a crackpot, I merely believe that you suffer from an insufficiently wide range of experience.

    (I could be wrong, though 🙂 )

    00

  • #

    Just tumbling into this wide discussion, and it cost me 10 seconds to see that Joanna is right in this matter.The colder troposphere can increase the surface temperature because the earth is not a closed system, it is constantly fed with energy from the sun. This is as far as I know not contrary to the second law of thermodynamics. I will see if I can organise a simple lab test on this matter in the near future. Measuring=knowing.

    00

  • #

    Jo Nova wrote, in “This is generating many comments, see below for an update!”

    People are being caught by semantics. Technically, strictly, greenhouse gases don’t “warm” the planet (as in, they don’t supply additional heat energy), but they slow the cooling, which for all pragmatic purposes leaves the planet warmer that it would have been without them. It’s a bit like saying a blanket doesn’t warm you in bed. Sure, it’s got no internal heat source, and it won’t add any heat energy that you didn’t already have, but you sure feel cold without one. – Jo

    OK, let us look at this in “bits”, but the complete quote above can be easily used to quickly check if I have taken anything “out of context”,
    which, I have not done. If any reader thinks I have done so, please show us all how I have, because I have not done so intentionally.

    People are being caught by semantics.

    Agreed, yourself included.

    Technically, strictly, greenhouse gases don’t “warm” the planet (as in, they don’t supply additional heat energy), but they slow the cooling,

    Agreed, they reduce some forms of heat transfer dramatically, namely 2) and 3) as described in my Post 228 on this thread.
    ie,
    1) Thermal radiation losses.
    2) Conduction and convection of sensible heat.
    AND,
    3) Losses due to the latent heat of water vapourisation, which most forget, as water vapour makes air lighter ALSO causes / further enhances convection. Very effective it is too at cooling, hence “we” sweat”. OK, OK, so Jo “perspires”.

    Please also see my Post 2 in this thread for a useful way to think about “greenhouses” and what they actually do..
    http://www.globalwarmingskeptics.info/forums/thread-1212.html
    AND Post 6 – The (naked) cooling cannonball “thought experiment”.

    which for all pragmatic purposes leaves the planet warmer that it would have been without them.

    WRONG. WRONG. WRONG. Put simply, take just about any one location on the lite side of the planet {let’s say excluding the Polar regions).
    Now, what is the solar insulation received at the chosen by you point?
    OK, now, using the correct formula, which I think is, temperature of surface in Kelvin =(P in W/m2/5.67)^0.25*100, then add 273.15.
    What is the resulting temperature for the surface at that location in degrees C.?

    In the tropics, for example, this resulting surface temperature is usually up to and over 100 degrees Celsius.
    Hence you can fry an egg on a car bonnet at id day in many, many places.
    How in heavens name can that be “leaves the planet warmer that it would have been without” supposed “greenhouse gases…….

    Jo and many, many others have this the wrong way round completely, as Joe Postma has shown.
    The question is actually, on the lite side of the planet, WHY ARE THE SURFACE TEMPERATURES SO MUCH LOWER THAN THEY SHOULD BE,
    for the received solar insolation.
    The answer is simple, because the atmosphere cools the surface by 2) and 3) as already mentioned, AS THE THERMAL IMAGE OF A GREENHOUSE undeniably demonstrates.

    It’s a bit like saying a blanket doesn’t warm you in bed.

    Jo, and many, many others have got this the wrong way round too.
    You are cold, because of 2) and 3) before you get into bed,
    the bed is cold when you get in it, because it is at room temperature – it has no heat source of it’s own.
    Once in bed, the bedding around you warms up to (a maximum of) YOUR CORE BODY TEMPERATURE because of 2) and 3) from YOU to the bedding.
    Just like the greenhouse BECAUSE of reduced 2) and 3) to the atmosphere, you feel warmer, but you are only “looking”, or rather feeling this from your body temperature’s relative point of view.

    You feel warmer, because of LESS 2) AND 3)to the atmosphere, because of the bedding around you.

    Sure, it’s got no internal heat source

    Yes, the blanket DOES have a heat source, YOU.
    And the blanket also has a hot (and damp) air source too, YOU.
    Which aids cooling actually if you breath outside of the blanket.

    Have you not noticed if the blanket becomes wet (perspiring too much for instance due to more than one under the blanket “activity”..),
    you feel very cold, especially if there is a draft, or you stick your head out from underneath the blanket for some fresh air.
    This increases 3), which is yet, still so commonly, and utterly overlooked by most.

    I suppose Jo and many others will have to realise what insulation actually means first though.
    I have not tackled the view Jo clearly expresses that all radiation is positively absorbed yet,
    but let us return to that misconception later.
    There are obviously enough problems understanding insulation at present.

    00

  • #
    Phillip Bratby

    KR @ 252

    Why do you assume that the radiation of 240 W/m^2 at -16.8°C is all occurring from the Earth’s surface. Only some radiation (throught the so-called atmospheric window) is from the surface. Most radiation is from high up in the atmosphere. So why is -16.8°C applicable to the surface?

    00

  • #
    Matt b

    Bob C and Lionell, for what it is work I think you are both crackpots!

    00

  • #
    BLouis79

    @JohnBrooks #295

    Actually I think I have said earlier #233 that Postma’s basic scientific description of the “thermodynamic atmosphere effect” makes quite good sense to me, following Gerlich and Tscheuschner’s falsification of the “greenhouse effect”. Those presently in favor of the “insulation” and “backradiation” theories seem unable to describe a testable hypothesis of how those theories work – in particular:

    1. How an IR laser might behave when passing through a gas filled chamber of any particular gas or mix of gases in terms of energy absorbed/reflected/backradiated/etc.
    2. How a greenhouse filled with a particular gas or any specified mixture of gases might behave in terms of temperature

    So do please read the full Joseph Postma paper which explains why temperatures on earth are as they are – in the sun, in the shade, on the beach sand, on tarmac, etc.

    00

  • #
    Bryan

    BLouis79

    As far as I read into Postma he doesnt rule out radiative insulation altogether.
    The extent of the IR properties are very certainly much less than the IPCC postulate.
    But certain things are clear, CO2 does not “trap” heat.
    Its effect may even be to speed up heat transmission.
    However if CO2 and H2O make for a slightly warmer night-time the same radiative mechanism will absorb Solar originating IR during daytime leading to slightly cooler surface temperatures.

    00

  • #

    Bryan Post 322

    ” However if CO2 and H2O make for a slightly warmer night-time the same radiative mechanism ”

    I think you are confusing mechanisms, and their relative importances.
    H2O makes night time “warmer than it would otherwise be” by reducing heat escape,
    not by “back radiating” from aloft as you, correctly in my opinion, hint at.
    ie, clouds, particularly at night, act as a “blanket” reducing 2) and 3) heat losses from the earth’s surface,
    the same process that makes cloudy days more muggy. Untill, if warm enough, the eventual thunderstorm, that removes the “cloud blockage”.

    00

  • #
    RJ

    Kevin @ 300

    Radiation on the other hand transfers energy at the “speed of light” which is well known to be quite speedy. So when you displace “non-GHGs” with “GHGs” slightly more energy flows through the atmosphere of the Earth at the speed of light versus the speed of heat

    I’m unsure if your logic is right here.

    The key is how energy leaves the surface. By radiation, conduction or advection. I can not see how more CO2 will change this. So more CO2 will very slightly slow down the speed at which the same amount of radiation leaves the surface to the outer atmosphere.

    00

  • #
    thRealUniverse

    Well said Harry Huffman great site! There’s NO argument unless you are living in another universe ..hence my nickname on here! I feel sorry for all the warmists and skeptics alike who believe in strong and or weak “greenhouse effect” there are simply wrong. Ive heard Chris Monckton talk about some warming but what i find disturbing is that some of the major skeptics and i really appreciate what they are trying to do give so much ammunition to the AGW warmist alarmists they shouldn’t be, by saying there some so called warming that isn’t there at all.
    Just stick to the real cause of the climate, the Sun, and the heat capacity of the oceans, the earths rotation, the axial tilt and so on.

    00

  • #
    Allan M

    Bob @ 262

    Try this experiment: Use an ordinary laser pointer to put a spot of light on the surface of a running, frosted, 100W light bulb. It is easy to see the spot of red light on the surface of the bulb (after your irises narrow down quite a bit). This demonstrates that the laser is many orders of magnitude brighter than the light bulb in that narrow band.

    The fact that you can see the red dot on the surface of the light bulb indicates that the photons are being reflected (and scattered). The surface of the glass globe is not at the same temperature as the filament. If you shine the laser directly onto the filament you would not see the red dot.

    This is the flaw in the greenhouse theory: failure to consider reflection. A body can only absorb a photon which is at a higher energy level. A photon at the same or lower energy level will be reflected, so the back-radiation does not warm.

    Absorption means heating.
    Emission means cooling.
    Reflection means neither.
    Absorption + re-emission means neither.

    Watch for the sleight of hand as the cooling and reflection are not mentioned.

    Jo, if I said to a zoologist “well, a duck’s just a duck,” or to a geologist “well, a rock’s just a rock,” or to a chemist “well, a salt’s just a salt,” I think they would be justified in giving me a long lecture. (Sorry about the choice of all four-letter words, but it’s just what came to mind. Must get a new one.) “If it hits, it warms.” Not necessarily. What about reflection? It enables you to see the world out there.

    00

  • #
    Bryan

    Derek

    I said

    …..”However if CO2 and H2O make for a slightly warmer night-time the same radiative mechanism will absorb Solar originating IR during daytime leading to slightly cooler surface temperatures.”….

    My comment was to BLouis79 regarding Postma.

    If you read page 2 of Hans Schreuders new paper

    ….”more radiant energy will be blocked during the day than leaving the Earth at night(insulating effect)”.

    The same as my comment

    Hans Schreuders and Postma see are pretty much in agreement.

    http://www.tech-know.eu/uploads/Greenhouse_Gases_Cool_Earth.pdf

    00

  • #

    KR @ 308:

    So – in your version of physics, at equilibrium (temperature not changing) the amount of energy coming into the climate does not have to equal the amount of energy leaving the climate?

    I always thought that was basic conservation of energy…

    Please re-read my posts. I am stating that there is no fixed rate at which the atmosphere carries heat away from the surface via convection. I’m not arguing that the 2nd law of thermal dynamics in broken by the idea of ‘back radiation’. Unless you can demonstrate that the process of convection in our atmosphere is saturated then I can’t see how any absorption of IR energy from C02 is going to increase the temperature of the atmosphere. You need more input energy to do that.

    From what I understand of physics, C02 captures IR as heat and rises as it is displaced by cooler molecules. C02 cannot capture that heat twice which seems to be what the greenhouse gas theory suggests. At the point of back radiation the C02 cools. That the average surface temperature of the planet is increased by an increase in C02 gas is related to the very fact that more C02 is there. It is not because somehow C02 inhibits the IR which would otherwise fly off into space.

    The best C02 can do is either pass the ball, metaphorically speaking, or run with it. If C02 runs with the ball then it can’t take more than a capacity level of heat(which makes him run faster). And no-one in the pro greenhouse gas camp has shown that:

    A) C02 can’t run faster
    B) C02’s teammates can’t run faster should he choose to pass the ball(energy)

    C02 cannot magically capture and hold more energy than the input supplies, twice no less!

    00

  • #

    The only other variable that can change to regain equilibrium is temperature.

    This is where you are currently stuck with your insulation argument. The other variable height. Gases expand when they get hot, a woolen blanket doesn’t. To agree with the IPCC on the greenhouse gas theory is to concede that the volume of the atmosphere and elevation of the troposphere are constant.

    Ouch.

    00

  • #

    *tropopause(I hate having to correct myself :-/)

    00

  • #
    Paul

    I’ve just completed reading : —

    Understanding the Thermodynamic Atmosphere Effect
    Joseph E. Postma
    (M.Sc. Astrophysics, Honours B.Sc. Astronomy)
    March, 2011

    His conclusion, which is fully deserved after a full and closely reasoned account of planetary atmospheric temperatures, says it all, as far as I am concerned …

    The conclusion of this article is very simple: there is no such thing as a radiative Theory of the Greenhouse Effect, not in real greenhouses, and certainly not in any planetary atmosphere known to man. The true role of the atmosphere, on Earth, is that it cools the ground, not warms it. Therefore, there is no such thing as Anthropogenic Global Warming or anthropogenic-CO2 induced climate change, because that supposition is based on the false Theory of the Greenhouse Effect. Any monetary expenditure or political debate on this issue can therefore stop. Now. Or, those can exist only in so far as they are directed to eradicate the false science.

    The only thing I would change about that conclusion is to omit the word ‘Theory’ and say, “there is no such thing as a radiative Greenhouse Effect”. because, as we all know, there is such a theory. But there is no effect and he has just fully demolished the theory. However, as we also should know, there is such a thing as zombie science. The definition of zombie science is “when there is sufficient money being pumped into the theory it moves as if it were still alive.” CAGW and the Theory of the Greenhouse Effect on earth’s climate is just such.

    Paul

    00

  • #
    RJ

    @291

    That’s where they’re vulnerable. Spending days and days arguing about the second law, while the warmists speed away into the distance – laughing – is not going to improve the situation. It’s a waste of time and effort.

    The warmist are not speeding away in the distance. And a few days in this battle is no time at all

    And why not debate the validity or otherwise of the AGW backradiation theory. It is a big part of this debate

    00

  • #
    BobC

    Allan M:
    May 17th, 2011 at 7:37 pm

    Bob @ 262

    The fact that you can see the red dot on the surface of the light bulb indicates that the photons are being reflected (and scattered). The surface of the glass globe is not at the same temperature as the filament. If you shine the laser directly onto the filament you would not see the red dot.

    Allan, my point was that the laser is brighter than the light bulb, within its bandwidth. (In fact, only about 8% of the light is reflected from the surface of the bulb — most of the scattering comes from the frosting inside, where the light from the filament is also being scattered.)

    Looking at the bare filament would not be feasible (or wise) without a strong filter — but if you did that, you would find that it would take only a moderately powerful laser (10-20mW) to put a visible spot on the filament.

    00

  • #
    BobC

    Matt b:
    May 17th, 2011 at 4:47 pm

    Bob C and Lionell, for what it is work I think you are both crackpots!

    Well, given your experience and stature as a member of that class, that carries some weight!

    00

  • #

    I know I’m taking my life in my hands by posting here, but…

    Let me try to explain it another way. Look at it on the molecular level.

    The surface of the Earth is warm, and emits radiation… a single photon. The surface has lost energy. A single CO2 molecule in the atmosphere, much colder than the surface of the planet, absorbs that photon. It is now warmer. It is not as warm as the surface, but it is warmer than it was, and warmer than the surrounding O2 and N2 and other CO2 molecules in the atmosphere.

    Two things can happen. The first, and most likely in the lower troposphere where the atmosphere is dense, is that it transfers that energy to an O2 or N2 molecule through a collision. In this way the surface efficiently heats the atmosphere, even though the O2 and N2 have a very poor ability to conduct heat (i.e. they are good insulators).

    The second thing that can happen instead is that that warm CO2 molecule can cool through emission of a photon (this can happen anywhere, but is more likely to happen in the higher, less dense top of the atmosphere where collisions with other molecules are less frequent). That photon goes off in any random direction, and may be intercepted again along the way by another (cool) CO2 molecule. Or it may make it all the way out into space. Or it may make it all the way back down to the surface of the planet, where it will be absorbed by a cooler molecule (they aren’t all cooler, but many are… if you don’t believe me, dig down into the earth and tell me how warm the dirt is a foot down).

    Consider each of these molecular cases; (1) warmer surface heats a cooler CO2 molecule, (2) warm CO2 molecule heats a cooler O2/N2 molecule, (3) warmer CO2 heats another, cooler CO2 molecule, or (5) warmer CO2 molecule heats another (cooler) molecule at the surface.

    None of these individual events violates the second law of thermodynamics. Each is not only physically possible, but also probable. Each happens billions upon billions of times every pico-second in the Earth’s atmosphere.

    They could never, ever happen in such quantity that the cooler atmosphere actually warms the surface.

    But in aggregate, happening those billions of times over and over again, the net effect is to slow the cooling of the surface of the planet. Yet at the same time, that surface is being bombarded by constant solar radiation. Unable to cool as quickly as it would without CO2, the surface and atmosphere both heat further than they would in the absence of CO2.

    The sun heats the earth. The earth heats a cooler CO2 molecule. That warmed CO2 molecule heats cooler O2 or N2 molecules, or another cooler CO2 molecule, or a cooler molecule on the surface of the earth (“cooler molecule” here refers to any molecule that is capable of absorbing that photon… a molecule which is not capable of absorbing the photon is transparent to it and will simply “ignore” the energy, lest it be arrested for violating the Second Law of Thermodynamics and be sentenced to five years hard labor in a state pennitentary).

    The second law of thermodynamics, on a molecular level, is never, ever violated.

    These transactions can never happen in such quantity that the atmosphere actually loses energy and transfers it to a warmer surface. That would violate the second law of thermodynamics. The colder atmosphere cannot make the warmer surface even warmer. But it is possible for the sun to warm the surface, and the surface to warm the atmosphere, and for the process of warming the atmosphere to include some microscopic (molecular) interactions which do not in and of themselves violate the second law of thermodynamics but do slow the rate of heat loss from the surface in such a way that the surface and atmosphere both achieve higher temperatures than they would in the absence of the greenhouse effect.

    00

  • #
    Bryan

    Sphaerica (Bob)
    A single CO2 molecule in the atmosphere, much colder than the surface of the planet, absorbs that photon. It is now warmer. It is not as warm as the surface, but it is warmer than it was, and warmer than the surrounding O2 and N2 and other CO2 molecules in the atmosphere.

    When the CO2 absorbs the photon(15um) its temperature stays the same.
    The vibrational mode is activated.
    The CO2 most likely will lose the photons energy because its relaxation time is longer than the collision probability.
    The photon energy is rapidly shared out between N2 and O2 molecules causing local heating.
    The reverse process – a collision induced photon emission is much less likely and if it happens more likely it will be a longer wavelength photon that the 15um absorbed.

    00

  • #
    RJ

    @336

    Or it may make it all the way back down to the surface of the planet, where it will be absorbed by a cooler molecule (they aren’t all cooler, but many are… if you don’t believe me, dig down into the earth and tell me how warm the dirt is a foot down).

    If energy leaves earth and return a fraction of a second later. The CO2 would be cooler than the planet. Why. Because the cO2 can not be warmer than the object (earth) it receives the heat from

    I guess if the photon travel from the equator outwards to quite a height and arrives in the UK for example it might have more heat than the earth but this would usually not occur.

    00

  • #
    BLouis79

    @Sphaerica (Bob) et al – who support the radiative “insulation”/delay/etc theory

    This theory still sounds unphysical to me. But the beauty of science is that one can test hypotheses by experiment and scientists will accept the results and reproduce the results independently to verify.

    So think of an experiment that someone can do – for example with various gases of varying composition/pressure/etc in a greenhouse or of IR light as generated by a laser so it is unidirectional to test your hypothesis. Then all we need is a friendly physics lab somewhere on earth to run the experiment. No doubt there would be physicists happy to comply especially if we can find them a pittance of the funding spend on climate research.

    00

  • #

    Bryan,

    All true… although the first two statements get into a fuzzy area which I tried to avoid (to avoid confusion), because “temperature” is really a measure of average kinetic energy in a volume of molecules, and therefore by proxy energy in that volume. Absorption of the photon increases the energy of the CO2, and if the system as a whole were left to reach equilibrium, that energy would be transferred from the CO2 (as vibrational energy) to the surrounding molecules (as kinetic energy), raising the temperature of the body of gas as a whole.

    But, as I said, all of your statements are true, although I don’t see how they change my description in any way.

    00

  • #

    RJ,

    You are getting a bit lost in the idea of temperature as an absolute. Temperature is primarily (though not entirely) a measure of the average vibrational energy of the molecules in a solid, or the average kinetic energy of the molecules in a gas.

    You’re also getting a bit confused by thinking of the “surface” as if it is one large, solid, homogeneous object. It is not. There are leaves and dirt and rocks and water and on and on. Some are hot, cold, warm, luke warm, or hot at the top and cool underneath (or deeper down, in the case of water or soil). The temperature of the surface is also not constant… the sun gets low, and sets, and things cool.

    To make it clearer… take a single complex molecule on the surface of the earth… say a molecule or atom within a metal alloy of some sort. This can get very hot in the sun, and will emit infra red radiation in a variety of wavelengths, and quite a lot of it. If one of those emitted photons is in the proper wavelength, it may be absorbed by a CO2 molecule.

    The CO2 molecule is by no means hotter than the iron “that warmed it.” It is, however, warmer than it was before, and all of the individual events in my description still apply, including the ability of that CO2 molecule to radiate in the infrared, towards the surface, and for that photon to be absorbed by anything which is able to absorb that photon… perhaps a puddle or dirt beside the slab of iron, or thousands of miles away.

    Of course, I chose something that you can easily visualize as “hot in the sun,” but the same logic applies to anything. The point is that the whole is not homogeneous, and the individual components are at different temperatures and possess different properties in relation to heat and the absorption and emission of radiation.

    Really, this sort of sums up the problem. If you think of the system at too high a level — as if the atmosphere and the earth are large, homogeneous bodies — then your model fails you and you fail to understand the system properly.

    00

  • #

    As Jo suggests, a semantic problem muddies the waters. The problem is that the word “heat” makes ambiguous reference to the associated ideas. Some of us reference the word to one concept. Others of us reference the word to another. In this way, we misunderstand each other.

    In the language of thermodynamics, the word “heat” usually references the energy that crosses a boundary. However, there is an ambiguity (possibly attributable to Clausius) in which the energy that crosses a boundary is referenced by the “net heat” when this energy is radiative. Under the latter usage, the “net heat flux” is prohibited from pointing up a temperature gradient without a heat pump but a “heat flux” operates under no such prohibition.”

    As it muddies the waters, this ambiguity should be eliminated! I propose that we do the job by enhancing the vocabulary of climatology by a pair of terms that are in common use in the field of radiative heat transfer. At an (x, y, z) space point in a radiation field, the “vector irradiance” is the vector sum of those Poynting vectors that are incident on the referenced surface. The “vector radiosity” is the vector sum of those Poynting vectors that are transmitted through the referenced surface or that are reflected from this surface. The radiative heat flux is the vector difference of the vector radiosity and the vector irradiance.

    The “back radiation” is a kind of of vector irradiation. As it is not a heat flux (as a “heat flux” is defined in my disambiguated terminology) the back radiation is not bound by the second law of thermodynamics. However, as the back radiation vectorially subtracts from the vector irradiance in the determination of the radiative heat flux, the magnitude of the back radiation has an effect upon the magnitude of the radiative heat flux. In the circumstance that the back radiation points downward and the vector radiosity points upward, an increase in the magnitude of the back radiation reduces the magnitude of the radiative heat flux by the same amount.

    00

  • #
    L.J. Ryan

    Sphaerica (Bob):

    For argument sake, lets accept your slowly cooling Earth surface explanation. How then, does the Earths surface radiance increase beyond solar input? Said otherwise, if solar input is the sole factor for positive radiative energy input, then 240 W/m^2 is the maximum energy, 255 K is the maximum temperature via radiative input. Given your explanation and the logical extensions thereof, how does GHG physics reconcile the 150 W/m^2, the 33K delta?

    00

  • #
    RJ

    @ 341

    I still think it’s very unlikely that energy will leave the surface and return and further heat the earth.

    If the sun has not why would photons that will have less energy than the sun do this. In some very unusual circumstances it might happen but not often.

    00

  • #

    RJ,

    If the sun has not why would photons that will have less energy than the sun do this.

    This confuses me. The sun heats the earth by emitting a range of photons. The earth absorbs them. The earth re-emits them. If the earth re-emits less than it receives, it heats. As it heats, it emits more and more photons until it is emitting as many as it receives.

    This is how any object is heated. The earth is always receiving a lot of energy from the sun, and re-emitting that same amount of energy. What ultimately matters is how much the surface of the earth must emit in order to force the same amount of energy out of the system as it is taking in.

    All of the photons have very little energy in and of themselves. It’s the volume that matters, and certainly the energy is provided by the sun, and the volume provided by the sun is very, very large. That does not, however, mean that all other aspects of the system are inconsequential.

    To put it another way, greenhouse gases make it harder for the earth to get that radiation out into space. Not a lot harder. It’s not an overwhelming effect. But it’s enough to raise the temperature of the earth from 18˚C below zero, which is what it should be, to 15˚C above zero, which it is.

    Beware of phrases like “I still think…” What you are really saying is that you still feel that it can’t happen. That’s emotion, not logic.

    Break the system into components — the right components — and understand the individual interactions, and then you will understand the system as a whole.

    P.S. This has all been measured through instruments, and if you had the money, you could measure it yourself. The earth receives 241 W/m2 from the sun. The top of the atmosphere radiates 241 W/m2 away. Because they are in balance, the temperature of the earth does not change.

    The surface of the earth, however, receives and releases 517 W/m2 of energy. The difference is a result of many factors, but the main one is the greenhouse gas effect.

    This is not theoretical. This has all been measured. Anyone who refutes greenhouse gas theory must come up with another mechanism that explains how the surface of the earth is receiving 517 W/m2 of energy when the sun is only supplying 241 W/m2.

    00

  • #

    LJ Ryan,

    …how does GHG physics reconcile the 150 W/m^2, the 33K delta?

    First I’d like you to turn that question back on yourself. How do you explain the delta, given that both have been measured, without greenhouse effect? Why is the planet not 18˚C below zero? Why does the surface of the earth receive on average 517 W/m2 instead of a meager 241 W/m2 (when all of these numbers have been measured with instruments)?

    But, to answer your question directly:

    255 K is the maximum temperature via radiative input.

    This is a non-sensical statement. Temperature is a measure of heat/energy in a body. One doesn’t trade temperature, one trades heat/energy.

    …solar input is the sole factor for positive radiative energy input, then 240 W/m^2 is the maximum energy…

    This is true.

    So the system receives 240 W/m2, and emits 240 W/m2. Yet the surface of the earth, at 288˚K, emits 517 W/m2. The different is supplied by the greenhouse effect. Quite simply, some of the radiation fails to make it into space, and is redirected back. To overcome this, the planet heats until more than 240 W/m2 are emitted from the surface… enough that of the 517 W/m2, 240 W/m2 do make it out into space.

    00

  • #

    Sphaerica (Bob),

    What do you make of the units W/m^2?

    What impact does the fact that m^2 is a unit of area have on the numbers you are saying that have been measured?

    Does calling a number the consequence of a greenhouse effect actually make it the result of a greenhouse effect? If so, how?

    00

  • #
    KR

    L.J. Ryan @ 343

    “…if solar input is the sole factor for positive radiative energy input, then 240 W/m^2 is the maximum energy, 255 K is the maximum temperature via radiative input.”

    I’ve always been puzzled by this statement, as it is exactly backwards. 255 K is the minimum temperature required to radiate 240 W/m^2 of thermal energy, and can only be reached by a perfect blackbody with an emissivity of 1.0.

    Energy radiated scales by the emissivity and temperature. Lower emissivity objects (like, for example, the Earth) will have to be at a higher temperature to radiate 240 W/m^2.

    For example (sorry about the formatting/underscores, I was trying to build a table with proper columns), in order to radiate 240 W/m^2 as determined by the Stefan-Boltzmann equation:

    Emissivity __ T°K __ T°C
    1.0________ 255 ___ -18 (Perfect blackbody)
    0.98_______ 256 ___ -16.8 (With no greenhouse gases, Earth surface emissivity)
    0.8________ 270 ___ -3.45
    0.7________ 279 ___ 5.7
    0.614______ 288 ___ 15 (Current measured effective Earth emissivity)
    0.6________ 290 ___ 16.6
    0.5________ 303 ___ 30 (Much sweating!)
    0.1________ 454 ___ 180 (Average goes over 100 at 0.2 emissivity)

    00

  • #
    RJ

    Bob @345

    To put it another way, greenhouse gases make it harder for the earth to get that radiation out into space. Not a lot harder. It’s not an overwhelming effect. But it’s enough to raise the temperature of the earth from 18˚C below zero, which is what it should be, to 15˚C above zero, which it is.

    Have you read any of the posts above. Or the Postma paper. Or read the slayers book for another viewpoint

    00

  • #
    RJ

    @ 345 (not 342)

    This is not theoretical. This has all been measured. Anyone who refutes greenhouse gas theory must come up with another mechanism that explains how the surface of the earth is receiving 517 W/m2 of energy when the sun is only supplying 241 W/m2.

    So CO2 and water vapour magically create more energy in some way do they. The energy leaves earth and somehow more comes back

    Sorry but this is just nonsense that even counters common sense. And read the Postma paper if you really want an answer to this question.

    00

  • #
    L.J. Ryan

    Sphaerica (Bob):

    You can’t have it both ways either;

    1) The cooler atmosphere does not add energy, confers no additional surface heat, only slows cooling…therefore solar input represents the maximum.

    or

    2) The cooler atmosphere does add energy, confers heat to the surface…violating the 2nd law while explaining the delta T.

    00

  • #

    RJ,

    So CO2 and water vapour magically create more energy in some way do they. The energy leaves earth and somehow more comes back.

    No. Go back and re-read. The energy leaves the surface, but not all of it escapes into space until the planet achieves a temperature that balances out with in.

    …this is just nonsense that even counters common sense.

    As you are misinterpreting it, yes. You need to go back and try to figure out what part you misconstrued.

    If you can phrase a question properly, I can help you.

    00

  • #
    KR

    L.J. Ryan @ 351 – Have you read @348 yet?

    00

  • #

    L.J. Ryan,

    1)…solar input represents the maximum.

    Maximum input into and output from the system, yes. Maximum internal temperature, no.

    2)The cooler atmosphere does add energy, confers heat to the surface…

    Again, no. It does not “add” energy, it contains it. Yes, it does confer heat back to the surface, on a microscopic scale, without violating the second law of thermodynamics.

    Again, by viewing the atmosphere as one single, homogeneous blob and the surface as another single, homogeneous blob, you are oversimplifying the system and applying the second law of thermodynamics where it does not belong.

    By your logic, a flame thrower cannot burn a recently baked cake. The cake just came out of the oven and is hot to the touch. The flame thrower is cool enough for me to hold. Yet I can pull the trigger and burn the cake. Why? Because the flamethrower is a complex system, not a single homogeneous object which is magically defying the laws of physics.

    Please go back and re-read what I wrote. You do not currently understand the system.

    00

  • #
    L.J. Ryan

    Sphaerica (Bob)354:

    Containing energy increases temperature beyond that of input? Is this only true for radiative transfer or can similar fantastic performance found with conductive insulation?

    Can a electrically powered IR heater at 255K heat itself to 288K with insulation, reflection or re-radiation…microscopically or otherwise?

    By your logic Sphaerica (Bob), your recently baked cake can increase the flamethrowers temperature.

    00

  • #
    RJ

    @ 355

    Can a electrically powered IR heater at 255K heat itself to 288K with insulation, reflection or re-radiation…microscopically or otherwise?

    Based on his example below I assume it could do. Or a person could cook themselves in a container of CO2

    This is not theoretical. This has all been measured. Anyone who refutes greenhouse gas theory must come up with another mechanism that explains how the surface of the earth is receiving 517 W/m2 of energy when the sun is only supplying 241 W/m2.

    00

  • #
    KR

    RJ @ 356

    “Can a electrically powered IR heater at 255K heat itself to 288K with insulation, reflection or re-radiation…microscopically or otherwise?”

    Quite easily. See post @348.

    255 K is the minimum temperature required to radiate 240 W/m^2 to space, not the maximum.

    00

  • #

    L.J. Ryan,

    You’re being silly and translating my statements into insanity. It may be entertaining for you, but it is hardly helping you to understand matters that you clearly do not.

    The temperature of a body is proportional to its energy.

    The energy emitted by a body is proportional to the fourth power of the temperature.

    Greenhouse gases prevent energy from escaping, increasing temperature.

    When the temperature of the earth reaches the point where it is able radiate away as much as it is absorbing, it has reached equilibrium at the new temperature.

    Take all the crazy pot shots you want. Being ignorant of the science while shouting your incredulity does not help you, or anyone, to understand the science better.

    00

  • #
    BobC

    Sphaerica (Bob):
    May 17th, 2011 at 11:08 pm

    Nice analysis @ 336.

    My only nits to pick have to do with the fuzzy definition of temperature that you use. To absorb an IR photon, a CO2 molecule doesn’t have to be cooler than the molecule that emitted the photon. Maintaining your excellent strategy of analyzing at the individual molecular level:

    When a CO2 molecule absorbs an IR photon what happens is:

    1) A vibrational mode of the CO2 molecule is excited. This increases the molecule’s internal energy, but doesn’t increase its temperature, as it is normally defined – e.g., the kinetic energy of the molecule. It is only the kinetic energy of the molecule that affects pressure, volume, etc.

    2) The photon’s momentum is added to the CO2 molecule’s momentum (momentum is conserved) and hence changes the molecules velocity slightly (IR photons have very little momentum). This change in velocity will change the CO2 molecule’s temperature slightly.

    Note that there isn’t anything that will prevent a CO2 molecule from absorbing (or emitting) a photon that depends on its velocity and kinetic energy. e.g., it doesn’t matter how hot or cool it is w.r.t. the source of IR.

    (We’re assuming that the time to absorb or emit a photon is small w.r.t. the mean time between collisions, conditions that obtain high in the atmosphere. Things get much more complicated if collisions and photon absorption/emission start affecting each other. For one thing, many of the vibrational modes get excited by inelastic collisions between molecules.)

    Note also that the molecule can either gain velocity (and hence K.E.) if it is moving away from the photon; or lose velocity and K.E. if it is moving towards the absorbed photon.

    If the IR source is broadband (like a thermal source), then the net effect is to heat the molecules. This can be seen by considering what the Doppler Effect does:

    a) If a CO2 molecule is moving toward the source, the Doppler shift increases the photon’s frequency (and hence its energy) — thus a slightly less energetic photon will suffice to excite the internal resonance.

    b) If the CO2 molecule is moving away from the photon, the Doppler shift reduces its effective energy, so a more energetic photon is required to excite the molecule and the momentum added (and hence velocity change)is greater.

    Since the absorptions that increase the K.E. are slightly larger than the ones that decrease it, the net effect is that the CO2 molecules gain K.E. and become hotter. This also does not depend on the initial K.E. of the CO2.

    (BTY, If one uses a narrow band source, like a laser, and positions its energy just below the molecular resonance, this effect can be reversed and the molecules cooled down. (Essentially, you only supply photons that can be captured head on and hence reduce the molecule’s velocity.) This is how laser cooling works.)

    Doppler shift has the opposite effect during photon emission and works, on average, to decrease the K.E. of the gas molecules.

    None of this, so far, has anything to do with how hot the CO2 gas is, or how hot the thermal source (the ground) is. IR radiation will be traded back and forth between the two regardless.

    One very simple way of analyzing the effect of this on the resultant radiation flux is to consider the ground to be a 100% reflecting mirror, and the CO2 a 50% reflecting mirror, for the wavelengths that are resonant with the CO2.

    To justify this:

    1) If we assume the ground to be a blackbody at thermal equilibrium, then it absorbs all the IR photons that impinge upon it, and re-emits the same amount back toward the sky. Hence, we can model it roughly as a 100% reflecting mirror for the purposes of calculating radiation flux.

    2) The CO2 can be considered a 50% reflecting mirror (for radiation flux purposes), if we assume that all IR photons in the absorption band that leave the ground are absorbed, but only half (at most) of them are re-emitted back towards the ground.

    Optically, a partially reflecting mirror spaced a short distance away from a totally reflecting mirror constitutes a Gires-Tournois cavity.

    The interesting thing about a Gires-Tournois cavity is that, from the outside, it looks exactly like a mirror, reflecting all the light that impinges upon it. (This is analogous to the Earth’s energy balance — as much energy radiated away as absorbed, as seen from above the atmosphere.) Inside the cavity, however, the situation is much different — the radiation flux is multiple times higher than it is outside, the multiplier being roughly 1/(1-R), where R is the partial mirror’s reflectivity. Heuristically, this can be thought of as the result of “recycling” the light between the two mirrors.

    Another way to think of it (for the case of 50% reflective top mirror): For the reflected flux outside to equal the incident flux, the inside flux must be twice as high, since only 1/2 of it is transmitted to the outside.

    (This is, of course, ignoring all wave interference effects, which have no analog in the Earth-atmosphere system.)

    Given this model, what effects can we expect to see in the Earth-atmosphere system? We would expect the radiation flux (in the bands where the CO2 molecule is resonant) to be ~ twice as high as they would be without the CO2.

    Consider a thought experiment where we start with no CO2 in the atmosphere, and the Earth is a blackbody in radiative equilibrium with the sky – then we add the CO2. The flux of IR photons will double (in a narrow band, resonant with CO2) and the surface of the Earth will now be receiving more energy than it’s radiating. Hence the temperature of the Earth will rise slightly to re-establish equilibrium.

    Note that the Gires-Tournois model explains how the flux within the atmosphere can be higher that the flux above the atmosphere, even though the energy balance (as seen from above the atmosphere) hasn’t changed. For those who have a hard time believing that a Gires-Tournois cavity acts this way, check out the analysis in the Wiki link (of course, ignoring wave interference effects).

    It also explains why this effect is independent of the actual temperature of the CO2 layer — it acts as a 50% mirror regardless of its temperature.

    This sounds a lot like the “radiation blanket” effect that Jo and others have been talking about, qualitatively derived from well-known physical effects. (And I didn’t even mention the 2nd law of thermodynamics, which appears to be largely irrelevant here. I’m not saying that it is violated, just that considering it does not aid in the analysis.)

    00

  • #
    BLouis79

    @Terry Oldberg #342

    While semantic problems may be muddying the waters a little. Much more muddying us caused by unscientific, illogical thought processes.

    1. Incorrect analogies will get science nowhere.
    2. Untestable hypotheses will get science nowhere.
    3. Misinterpretations of basic science will get the discussion nowhere, but can be conclusively demonstrated to be misinterpretations by experiment.

    Climate science is going nowhere until climate scientists start talking to physicists about experimental verification of the precise phenomena they claim to be causing “warming”. No end of observation of confounding variables will help and it has not to date except to bury the credibility of climate science.

    00

  • #
    co2isnotevil

    RJ,

    You have been deluded by Trenberth et all into believing that the surface is receiving 517 W/m^2 of incident power. In fact, it’s only receiving 240 W/m^2 of ‘new’ power from the Sun. There is no other substantial source of energy in the system! The whole concept of ‘back radiation’ as ‘forcing’ is an absurdity. It’s really just an accounting trick to make the silly CAGW hypothesis seem plausible to the ignorant. Any joule of energy returned to the surface by the atmosphere originated from the surface and must be subtracted out from the surface flux before it can be added back in, moreover; that joule of energy originated from the Sun sometime in the past. The key piece you seem to be missing is the temporal aspect which is fooling you into counting the same joules more than once.

    George

    00

  • #
    L.J. Ryan

    KR 348 & 358

    Ah yes, the silliness of your “effective emissivity”.

    This CALCULATE “property” “effective emissivity” is purely unphysical. It’s a trick, a falsehood an attempt to obfuscate GHG physics.

    Being a quite easy task KR, what can you place in front of an electrically driven IR heater @ 255 K to increase it’s temperature to 288K.

    00

  • #
    Paul

    Climate science is going nowhere until climate scientists start talking to physicists about experimental verification of the precise phenomena they claim to be causing “warming”. No end of observation of confounding variables will help and it has not to date except to bury the credibility of climate science.

    I think that the comments on this thread illustrate very clearly the unscientific nature of the claims of CAGW.

    The normal route for a scientific theory is for a testable hypothesis to be formulated, followed by experimentation and recording of the results. The next step is for others to see if they can replicate the experiments and get the same results. Finally, when it is clear to everyone that the hypothesis does a reasonable job of explaining the observed physical behaviour the hypothesis is elevated to the status of theory. After much more testing and confirmation of the theory, when it has been found to apply in a more or less universal manner, such as the operation of gravity throughout the universe, it is elevated to a law of physics. At no point in this process is it no longer allowable to question the hypothesis or to propose changes to it that may bring the theoretical construct nearer to the real situation as observed in the real universe. There are no ‘believers’ and no ‘heretics’ in this scientific endeavour. All are aware that our knowledge is, at best incomplete and at worst bordering on ignorance.

    In the case of CAGW this route has not been taken. Instead, a few influential people having taken hold of a conjecture, established a centralised bureaucracy in the United Nations to look for a political solution to the imagined problem. The rest of the history is somewhat murky, with lots of data-fudging, refusals to share data, use of shoddy statistical tools, cherry-picking of proxies, etc, etc, ad nauseam, in order to heighten the alarmism on a global scale to where even the man in the street cannot remain uninterested in the physics of the atmosphere, supposing that the very future of the human race is at stake unless we are taxed out of our current prosperity.

    In this present discussion I see a mixing of things that are, by nature, remote from the experience of the vast majority of people, such as the behaviour of photons at differing frequencies, with things that are well-known every day experiences, such as watching the water flow over the top of a dam. These diverse matters are then strung together in narratives that are entirely free from any constraints other than that supplied by the author’s credulity and we are then invited to believe the resultant narrative because it is supposedly ‘scientific’!

    Where are the testable hypotheses?
    Where are the experiments?
    Where are the numerical results of the experiments?
    Where are the mathematical formulas that describe the processes and allow the parameters to be accurately measured?
    Where are the confirmatory replications of these experiments?
    Where are the mechanisms by which all this is taking place?
    How do these new results mesh with what has been painstakingly derived by the scientific method by researchers over the past hundreds of years?

    Maybe I have missed something, but to my knowledge the answer to each and every question above, excepting the last, is the same – missing in action!

    The answer to the last question seems to be, to some here, to question the validity of the scientific knowledge acquired in the past.

    The end result, strangely enough, is that we are witnessing the demise of the scientific method and seeing it replaced by myths. Myths that are to be enforced by Governmental propaganda and the force of the state.

    Paul

    00

  • #

    Paul:

    You’re right. The IPCC’s claims are not testable. Thus, they are not scientific claims.

    00

  • #
    KR

    L.J. Ryan @ 362

    Well, that’s a rather badly worded challenge.

    * Take a plain electric heater with 240 watts coming into it, in a cool room so that it is at equilibrium at 255K. At that point it’s radiating 240 watts into the room.

    * Wrap it in insulation so that it loses energy less efficiently. It’s now radiating something less than 240 watts. What happens? By conservation of energy, that extra (not lost) heat will accumulate in the heater.

    * The heater warms up until 240 watts again exits the insulation, so that input = output. And the heater is now warmer – 288K if we’ve insulated it sufficiently, even hotter if we’ve insulated it even more.

    That’s the core of the radiative greenhouse effect, L.J. The greenhouse gases make the Earth cool less effectively than it would without them. As per my post in @348, with no GHG’s present, the emissivity of the Earth to space (radiation, the only way to lose energy to space) is 0.98, and the temperature would reach equilibrium at -16C.

    With GHG’s we’re at 15C. And rising…

    That’s the physics, L.J., and the temperature. Not a trick, but the reality.

    Indulge me a bit, L.J. Take a look at the top figure in these spectra. A blackbody (emissivity 1.0) will radiate a spectra that looks like one of the smooth lines, such at 240K and 260K. That is the most effective emission theoretically possible at a particular temperature! But the notches in the emission spectra at the top of the atmosphere indicate that the Earth is not radiating like a blackbody – it has a lower emissivity. And hence the temperature required to emit 240 W/m^2 to space has to be higher than 255K.

    If you cannot see the importance of this, if you don’t recognize that 255K is a minimum temperature to radiate 240 W/m^2, then you are simply wrong, and I cannot help you. As Jo and Michael have stated, the greenhouse effect is like insulation, slows the loss of energy to space, and does not violate the 2nd law of thermodynamics.

    00

  • #
    Paul

    You’re right. The IPCC’s claims are not testable. Thus, they are not scientific claims.

    And yet they are being given the force of law, already here in New Zealand, and in proposal in Australia!

    Paul

    00

  • #

    BobC,

    I was trying to avoid all of the many complexities you brought up, as well as others, by simply treating (terming) “energy” as “heat,” even though that’s a gross oversimplification. Again, determining the temperature of an individual molecule is almost a pointless exercise, when temperature was devised as a measure of a bulk of a substance, and in fact was devised before we understood what temperature and heat really were (i.e. manifestations of kinetic and/or vibrational energy).

    But the point of the explanation was to bring it to the molecular physics level, so that meant taking some liberties with the terminology.

    In reality, when considering a quantity of a substance rather than a single molecule, the vibrational energy picked up by CO2 molecules will be transferred somewhat randomly to surrounding molecules of any sort as vibrational, rotational or translational kinetic energy. In the end, due to equipartition of energy and a large enough number of molecules, this will wind up being evenly distributed throughout the various kinetic modes (degrees of freedom) of the molecules in the volume so that the temperature reflects the mean energy of the molecules, and so is a proxy for the total energy.

    Given all of this… I simplified and used the terms “warmer” and “cooler” when what I really meant was “gained energy” or “lost energy,” but all the while knowing that trying to explain all of the distinctions and complications and distinguishing between “energy” and “heat” would either (1) confuse some readers or (2) give some readers tangents along which to diverge and lose sight of the point.

    00

  • #

    Paul,

    Where are the testable hypotheses?
    Where are the experiments?
    Where are the numerical results of the experiments?
    Where are the mathematical formulas that describe the processes and allow the parameters to be accurately measured?
    Where are the confirmatory replications of these experiments?
    Where are the mechanisms by which all this is taking place?
    How do these new results mesh with what has been painstakingly derived by the scientific method by researchers over the past hundreds of years?

    Maybe I have missed something, but to my knowledge the answer to each and every question above, excepting the last, is the same – missing in action!

    You have missed something.

    All of this is there to be had… even more so since the development of the Internet, so it is almost all at the fingertips of every man, woman or child with an Internet connection. The biggest roadblock to complete access is the fact that the publishing journals are also private businesses, and so want to make money… which means expensive purchases or journal subscriptions to get at the data.

    Especially if you don’t trust the IPCC, I would strongly recommend going straight to the source and actually reading the papers. Many of them are very approachable, and many are readily available somewhere on the Internet (as PDFs). I almost always read an actual paper, rather than some blogger’s twisted take on the paper itself. Then I read related or referenced papers.

    There is a wealth of information out there. You just need the time and energy to look at it.

    I would also very highly recommend reading Spencer’s Weart’s “The Discovery of Global Warming.” It is, in effect, a step by step history of climate science, beginning with Tyndall’s work in 1859.

    00

  • #
    Paul

    While I would not dispute that adding an insulation layer has the effect of slowing the release of heat from a heated object, what has not been established is that the only form of insulation possessed by the earth-atmosphere system are the so-called GHGs.

    Surely any atmosphere will have the property of an insulator because heat is lost from the surface by more ways than by radiation?

    What about the effect of the oceans on the conservation of heat in the atmosphere?

    Some relevant questions : —

    1) How much heat leaves the surface of the earth by means of a) Conduction b) Latent heat of vaporisation c) Radiation?

    2) If the temperature of the air in contact with the surface approaches the temperature of the surface will not the rate of heat-loss by radiation be reduced towards zero?

    3) Is it legitimate to treat the earth-atmosphere system as a two-dimensional system for the mathematical solution to any equation?

    4) Is it legitimate to derive a theoretical black-body temperature for the earth-atmosphere system and then equate that to the global-average near-surface air temperature? Is it any more legitimate to base one’s argument upon this illegitimate comparison?

    As far as I can discern, most of the case for CAGW is no better than the smoke and mirrors of a magician by which he creates his illusions that entertain the crowd!

    Paul

    00

  • #
    Paul

    You have missed something.

    All of this is there to be had… even more so since the development of the Internet, so it is almost all at the fingertips of every man, woman or child with an Internet connection. The biggest roadblock to complete access is the fact that the publishing journals are also private businesses, and so want to make money… which means expensive purchases or journal subscriptions to get at the data.

    OK, I was a bit too loose in my wording. I aught to have included words to the effect “in the public discussion of” …

    That is where there is a real problem, there is a huge propaganda effort going on. We are bombarded with misinformation by the hour. And if a scientist dares to raise his head and question any of this ‘official’ information he is subject to so much repression that only those whose livelihood is secure are able to take the risk.

    Meanwhile, how many people have the time, resources, education and resolve to dig for the actual scientific knowledge that exists, most of which totally refutes the current propaganda and is therefore ignored by it?

    Paul

    00

  • #

    Sphaerica (#368):

    I don’t believe Paul has missed anything. The IPCC’s conclusions are based upon studies that have had an illogical and non-scientific methodology (see http://judithcurry.com/2011/02/15/the-principles-of-reasoning-part-iii-logic-and-climatology/ ).

    00

  • #
    Richard S Courtney

    Friends:

    This debate about the 2nd Law of TD is a distraction from the real point which is that if one agrees the ‘conventional’ theory of how GHGs operate then additional atmospheric CO2 concentration cannot provide discernible increase to global temperature.

    Before explaining this point I wish to say that the warming effect of GHGs does not contravene the 2nd Law of TD.

    Heat goes from warmer to cooler and space is colder than the surface of the Earth. Hence, the 2nd Law says that heat must flow from the surface to space, and it does.

    But heat is transferred by conduction, convection and radiation. So, there is no disagreement with 2nd Law if one of those mechanisms seems to transfer net energy in the ‘wrong’ direction so long as the other mechanisms move more heat in the other direction.

    Indeed, the GHGs only emit IR but most of the solar energy obtained by the Earth’s surface from the Sun is not IR.

    So, the 2nd Law decrees that for GHGs to provide net downward IR flux then much more energy must be transported upwards by other mechanisms than IR flux. And when that is recognised then one can consider how those other mechanisms must respond – according to 2nd Law – as a result of the large downward IR flux from GHGs.

    Such a consideration leads to an important conclusion.

    Firstly, it is important to recognise that the effect of increasing CO2 above its present atmospheric concentration can only be very small because almost all the IR from the surface that the CO2 in the air can absorb is being absorbed (and it is not possible to absorb more than all).

    Then, and very importantly, the very small effect of increased atmospheric CO2 will be negated by increase to evapouration. All the IR is absorbed within a few microns of the surface of any moisture (i.e. the oceans, lakes, rivers, streams, damp soil). This absorbed energy in the very thin surface layer increases evaporation so does not result in significant additional IR from the surface: instead, it increases latent heat loss.

    This increase to latent heat loss increases with increased temperature and, therefore, the oceans have a maximum surface temperature of 305 K (i.e. 32 deg.C).
    (ref. Ramanathan & Collins, Nature, v351, 27-32 (1991))

    Those who support the AGW hypothesis know this conclusion is inescapeable so they suggest that the increased evaporation will increase the water in the air. Water absorbs over almost all the IR spectrum and it is suggested that, therefore, the increased water vapour will induce much warming (this is called the Water Vapour Feedback: WVF). But that ignores the fact that ‘what goes up comes down’ as rain. There is no WVF but there is a faster hydrological cycle. This is proven by the fact that global temperature has risen over the past 50 years but atmospheric water vapour concentration has fallen.

    Simply, the AGW hypothesis is false because the so-called Greenhouse Effect (GE) of GHGs does not contravene the 2nd Law of TD and the way the GE obeys the 2nd Law is by increasing the rate of the hydrological cycle so additional atmospheric CO2 has no discernible effect on global temperature.

    Richard

    00

  • #
    L.J. Ryan

    Sphaerica (Bob):

    “Greenhouse gases prevent energy from escaping, increasing temperature.

    1) What is the source and magnitude of this energy…is it 240 W/m^2?

    When the temperature of the earth reaches the point where it is able radiate away as much as it is absorbing it has reached equilibrium at the new temperature.

    2)New equilibrium ok: Earth @ 255 K radiating 240 W/m^2 heats atmosphere (ε=1) to 255K. Atmosphere at 255 K radiates 240 W/m^2 toward the surface and 240 W/m^2 space-ward (OLR) and we have new equilibrium.

    2a)New equilibrium ok: Earth @ 255 K radiating 240 W/m^2 heats atmosphere (ε= < 1) to temperature < 255K, but also transmits flux (1-ε)space-ward. So although the atmosphere temp is NOT 255K system equilibrium is reached. That is, ε240 W/m^2 via atmosphere plus (1-ε)240 W/m^2 = 240 W/m^2 OLR we have new equilibrium.

    So in both cases equilibrium is reached with Earth at 255K…no need to increase temp.

    00

  • #
    KR

    Richard S Courtney@ 372

    “So, the 2nd Law decrees that for GHGs to provide net downward IR flux then much more energy must be transported upwards by other mechanisms than IR flux.”

    A fascinating statement, given that actual measurements show that IR from the surface is 396 W/m^2, latent heat is 80 W/m^2, and convection is 17 W/m^2. Measured backradiation is on the order of 333 W/m^2.

    More energy (net energy, heat) goes up from the surface than down – but IR represents a major portion of upward energy, and all of the downward energy. You’re entitled to your own opinion, Richard, but not your own facts.

    As regards to water vapor – roughly 4% more water vapor (another actual measurement) is in the atmosphere than in the 1970’s, due to warming. That’s one Lake Erie, or about 500+ Sydney Harbors, as I understand it. The relative humidity has remained stable or even decreased, but the specific humidity (the total number of water molecules) in the atmosphere has increased – warmer air holds more water vapor for the same relative humidity. And hence, more greenhouse effect.

    00

  • #
    Paul

    Well, well, well. So this is the best that can be said in defense of the conjecture of CAGW!!!

    Here’s what all scientists agreed they knew by 1988

    To stay at a constant temperature, the Earth must radiate as much energy as it receives from the Sun. We receive this energy mostly as visible light which warms the surface. Being much cooler than the Sun, the Earth radiates most of its energy as infrared rays. A calculation using basic laws of physics shows that a planet at our distance from the Sun, emitting the same total amount of energy as it receives, will have a temperature well below freezing. Then why is the actual average surface temperature higher, about 14°C? Infrared radiation beaming up from the surface is intercepted by “greenouse” gas molecules in the lower atmosphere, and that keeps the lower atmosphere and the surface warm. The radiation that finally escapes is mostly emitted from higher levels of the atmosphere, levels that are indeed well below freezing (-18°C, for details see the essay on simple models).

    The nitrogen and oxygen gases that make up most of the atmosphere don’t intercept infrared radiation. The most important greenhouse gases are water vapor and carbon dioxide (CO2). The level of carbon dioxide in the atmosphere was observed to be rising rapidly, and the only reasonable explanation was that this was due to the enormous emissions from human activities.

    A rather straightforward calculation showed that doubling the level of carbon dioxide in the atmosphere… which would arrive in the late 21st century if no steps were taken to curb emissions… should raise the temperature of the surface roughly one degree C. However, a warmer atmosphere would hold more water vapor, which ought to cause another degree or so of warming. Beyond that the calculations got problematic. Cloudiness was likely to change in ways that could either enhance or diminish the warming, and scientists did not understand the complex processes well. Moreover, humanity was emitting ever increasing amounts of smoke and other pollution; again scientists were not sure how this might affect climate. Only better observations and computer models could attempt to project the outcome.

    The gratuitous assumptions and gregarious reasoning in this summary is just what one has come to expect from the ‘Global Carbon Hysteria Machine’! You’re teaching your grandmother how to suck eggs!

    Let me take that fine summary apart a bit to see how much water it holds.

    Here’s what all scientists agreed they knew by 1988

    Argument from authority. No admission of the serious disagreement amongst scientists, especially the old guard who actually knew something about the atmosphere.

    To stay at a constant temperature, the Earth must radiate as much energy as it receives from the Sun.

    No mention that it is the earth-atmosphere system that must approximate the behaviour of a theoretical black body. They have already lost the plot.

    The radiation that finally escapes is mostly emitted from higher levels of the atmosphere, levels that are indeed well below freezing (-18°C, for details see the essay on simple models).

    A frank admission that the earth-atmopshere system functions, not at ground level, but several miles above the surface. What causes the temperature variation between that level and the surface is completely explained in terms of the adiabatic lapse rate. I have yet to see anything that would indicate that more CO2 in the atmosphere would have any effect on the lapse rate other than to enhance it.

    The nitrogen and oxygen gases that make up most of the atmosphere don’t intercept infrared radiation. The most important greenhouse gases are water vapor and carbon dioxide (CO2).

    One would suppose, from this, that nitrogen and oxygen, in the absence of CO2 would not warm at all, but would remain the same temperature as outer space which borders the atmosphere on the outside, away from the earth’s surface.

    The level of carbon dioxide in the atmosphere was observed to be rising rapidly, and the only reasonable explanation was that this was due to the enormous emissions from human activities.

    This is argument from ignorance. In fact much evidence points to atmospheric CO2 being a consequence of rising temperatures, especially in the oceans, rather than the driver. Hence, since it is not disputed that the earth has been warming over the past two or three centuries since the Little Ice Age, one would expect there to be an increase of atmospheric CO2 consequent on that warming. Anthropogenic contributions to atmospheric CO2, although non-negligible, are not the sole source of any observed increase. Moreover the actual evidence is not as monolithic as is claimed, much evidence of higher atmospheric CO2 in actual measurements in the recent past have to be totally ignored in order to maintain the myth that it is ‘us doing it’.

    A rather straightforward calculation showed that doubling the level of carbon dioxide in the atmosphere… which would arrive in the late 21st century if no steps were taken to curb emissions… should raise the temperature of the surface roughly one degree C. However, a warmer atmosphere would hold more water vapor, which ought to cause another degree or so of warming. Beyond that the calculations got problematic. Cloudiness was likely to change in ways that could either enhance or diminish the warming, and scientists did not understand the complex processes well. Moreover, humanity was emitting ever increasing amounts of smoke and other pollution; again scientists were not sure how this might affect climate. Only better observations and computer models could attempt to project the outcome.

    This is a non-sequitur based on the argument from ignorance and can only be maintained by fear of the unknown.

    Now that is not science.

    QED

    Paul

    00

  • #
    BobC

    Paul,

    A lot of the actual calculations involving the “Greenhouse Effect” are done with the US Air Force’s numerical program, MODTRAN4 (LINK). This program is available to the public for a fee of $300 (includes user support) — several past commentators have copies and have reported on its output.

    The calculations have to be done numerically, by spectral bands and are too tedious to do by hand. That said, the results are pretty much what you would expect — doubling CO2 will result in ~1deg temp rise. Since CO2 has been growing at ~0.5%/yr for the last 50 years, we can expect it to double in 140 years. Not a very big deal.

    Also, just modeling radiative transfer leaves a lot of stuff out, some of which may be even more important to the climate system’s response to GHG, such as clouds, precipitation, and general convection strength. (A thunderstorm, for example, dumps an enormous amount of heat to space via lifting warm air from the ground to altitudes up to 100,000 feet, where it cools by radiation directly to space. A friend of mine who worked at NCAR for 10 years modeling thunderstorms said that a 3% increase in thunderstorm activity would cancel out any heat increase caused by doubling CO2.)

    The junk science comes in when the modelers start making unsupported assumptions about how the rest of the climate system will respond to such a mild warming (for example, water vapor, clouds, and convection strength). That their assumed positive feedbacks have never been observed after natural warming events doesn’t seem to bother them (at least as long as lots of grant money is on the table.)

    As for testable assertions, there are plenty of failed predictions from the AGW community over the last 30 years. So far, the AGW hypothesis has yet to demonstrate any predictive skill distinguishable from chance.

    My personal opinion: The Radiative Greenhouse Effect is real but doubling CO2 will make a difference that may (or may not) be detectable. I also believe that trying to control the concentration of CO2 is probably the least effective method of climate control conceivable.

    When I’m not so busy I can try to find you some links to this stuff. You can find it yourself by typing
    “BobC site:http://joannenova.com.au
    (without the quotes) into google and looking at the links in my old posts.

    00

  • #
    Mark D.

    This thread has attracted quite the swarm of flies. Of course we know what flies are attracted to and that comes from the posterior of bulls (for example).

    KR @ 365:

    * The heater warms up until 240 watts again exits the insulation, so that input = output. And the heater is now warmer – 288K if we’ve insulated it sufficiently, even hotter if we’ve insulated it even more.

    That’s the core of the radiative greenhouse effect, L.J. The greenhouse gases make the Earth cool less effectively than it would without them. As per my post in @348, with no GHG’s present, the emissivity of the Earth to space (radiation, the only way to lose energy to space) is 0.98, and the temperature would reach equilibrium at -16C.

    With GHG’s we’re at 15C. And rising…

    That’s the physics, L.J., and the temperature. Not a trick, but the reality.

    Wrong! Where is the convection, the moisture, the clouds THE REALITY!

    KR @ 374:

    A fascinating statement, given that actual measurements show that IR from the surface is 396 W/m^2, latent heat is 80 W/m^2, and convection is 17 W/m^2. Measured backradiation is on the order of 333 W/m^2.

    More energy (net energy, heat) goes up from the surface than down – but IR represents a major portion of upward energy, and all of the downward energy. You’re entitled to your own opinion, Richard, but not your own facts.

    Nor are you entitled to your own “facts” for example the global temperature for the last ten years, the contrary trends in temperature compared to co2 trends, or the failing sea level predictions just for a few?

    KR you can spin webs or throw manure about the effects of co2 in the atmosphere but you absolutely cannot accurately calculate the ability of the TOTAL atmosphere to move heat from anywhere to anywhere.

    The flies can always find the stink.

    00

  • #
    Willaim Gray

    Send in the clowns, um CLOUDS I mean.

    http://wattsupwiththat.com/2010/07/24/willis-publishes-his-thermostat-hypothesis-paper/

    And over 2 watts an article proving sun/cloud causation.

    00

  • #
    L.J. Ryan

    KR 365

    * Wrap it in insulation so that it loses energy less efficiently. It’s now radiating something less than 240 watts. What happens? By conservation of energy, that extra (not lost) heat will accumulate in the heater.

    * The heater warms up until 240 watts again exits the insulation, so that input = output. And the heater is now warmer – 288K if we’ve insulated it sufficiently, even hotter if we’ve insulated it even more.

    A self powered heater, fantastic! KR why not insulate the room and warm it to 288K with 255K radiative heater. Though insulating a room with a 255K heat source implies a walk-in freezer, not a warm room. Details, details…I’m sure GHG physics has a well reasoned canard to side step this reality.

    00

  • #
    Mark D.

    Sphaerica (Bob): @ 345:

    This is not theoretical. This has all been measured. Anyone who refutes greenhouse gas theory must come up with another mechanism that explains how the surface of the earth is receiving 517 W/m2 of energy when the sun is only supplying 241 W/m2.

    Holy crap that is a runaway.

    Can YOU explain why it has not run away?

    What you are describing is feedback with no source of power. Please elaborate.

    00

  • #

    BobC (#376):

    You assert that “there are plenty of failed predictions from the AGW community over the last 30 years” but as I understand it, the IPCC climate models make “projections” rather than predictions. The former differ from the latter in the respect that they are not falsifiable. Can you provide citations to some predictions (not projections) of the IPCC’s models?

    00

  • #
    Mark D.

    Terry Oldberg @ 381
    That is a semantically challenged post. The technical difference meets with the pedantic. You can use logic as a ploy or as a tool. The difference to me is whether or not truth is your goal.

    00

  • #
    KR

    L.J. Ryan @ 379

    Well, you’ve just proven that you did not read my post @365.

    * The climate is a dynamic system, with energy flowing into and out of it.

    * You appear to have built a static thought experiment with a 255 K “heater” that does not have any input energy (hardly a heater, rather more of a brick). That’s not a dynamic system, but a fixed system, with no energy flow.

    * In a fixed system, there’s nothing that will raise the temperature except a hotter object. The total joules of energy are fixed.

    * But in a dynamic system, the numbers that must balance are the total energy coming in and the total energy going out. That’s just conservation of energy, the first law of thermodynamics.

    * The output energy (in a radiative system like the Earth in space) is set by temperature and the radiative efficiency of the object.

    I suggest you reread my post @365, and also @348. If you don’t get it then, well, shrug, I’ll leave you be with your opinions. Some people (like Roy @254) get it, great – it’s not worth my while to attempt to pound it into an unwilling head.

    00

  • #
    jfat

    Lionell at comment #3:

    BS!
    First argument:
    1. ANY heat absorbed by the cold object that is coming from the warm object is no longer in the warm object.
    2. The warm object is colder by that amount.
    3. Any of the head from 1 that radiates back to the warm object simply REPLACES the prior lost heat.
    4. The warm object simply returns to its original temperature.
    Conclusion: back radiation does not and cannot warm a warm object beyond its original temperature. In fact it is always less because the “back” radiation is re-emitted spherically rather than totally reflected back to the warm body.
    Every other incident source of heat on the warm object is sourced from an EXTERNAL warmer object even if it is via the so called colder body.

    Lionell’s points above show the following: without the cold object the heat absorbed by it in point (1) would be lost from the warm object for good, leaving it at the temperature of point (2). But in point (3) some heat does get back to the warm object giving it the higher temperature of point(4).

    Lionell’s conclusion, while correct, totally misses the point. He in fact PROVES that the presence of a cold object can cause a warm object to be warmer than it would otherwise have been, with no violation of thermodynamics, yet curiously he sees no significance in this.

    00

  • #
    Kevin

    To RJ: (re comment # 324);

    Yes indeed at first look it seems that “GHG’s” should slow the flow of heat through the atmosphere since the energy has to be emitted by the surface, then travel to the “GHG’s”, be absorbed and remitted, then travel back to the surface. Surely this would slow the transfer of heat/energy through the Sun/Earth/Atmosphere system RIGHT?

    Well I posit that increases in “GHG’s” MUST be countered by decreases in “non-GHG’s”. After all there are only 1 million PPMs of molecules in any volume of gas/solid/liquid. So if the “GHG’s” increase by 100 ppm the “non-GHG’s” MUST decrease by 100 ppm.

    So since the “non-GHG’s” transfer heat primarily via conduction and convection/advection which are much slower than heat transfer by radiation the increase of “GHG’s” actually speeds the flow of heat through the Sun/Earth/Atmosphere System.

    I know this seems a bit illogical, and I admit I had to think on it for a while before concluding that “GHG’s” “hasten the flow of heat through the atmosphere”, but I am still of the opinion that this is in fact the case.

    I do of course remain open to alternative explanations.

    Cheers, Kevin.

    00

  • #
    Kevin

    To Lionell (re: comment #347)

    “What do you make of the units W/m^2?”

    I for one take these as units of POWER per UNIT AREA. These are NOT units of ENERGY. Power and Energy are related units, but they ARE NOT INTERCHANGABLE!!!!!

    One of the most useful things I learned early in my engineering career was to “Always Reconcile Your Units”. This knowledge has served me very well for many decades. In fact when something I build does not perform as I expect (which is the majority of the time) the FIRST thing I do is check and then double check my units.

    I must be in the same “crackpot” camp as you since I agree with everything you say, unless you have predicted
    the next Super Bowl winner, I did not notice your prediction, but I probably disagree.

    Cheers, Kevin.

    00

  • #

    Mark D.

    Can YOU explain why it has not run away?

    How do you equate what I said to a runaway? You can’t just make stuff up, and then declare that I must defend whatever you’ve conjured up.

    00

  • #
    Mark D.

    My dear Bob, it is you that are conjuring methinks. Look at your own numbers: 517 W/m2 of energy when the sun is only supplying 241 W/m2.

    What is the miracle? Can we patent it?

    00

  • #

    jfat,

    You misrepresent the issue.

    The CAGW argument is that GHG back radiation will cause catastrophic global warming. What I have shown is that the surface cannot have more energy than it originally had as a consequence of back radiation sourced from the surface. So either the surface was already at a catastrophic level or cannot possibly reach such a level by GHG back radiation.

    However since GHG thermalizes absorbed IR almost completely and starts a chain of events such that very little energy is returned to the surface. The chain of events will be the extra thermal energy in the GHG gets transferred to adjacent gas molecules which, in turn, causes the gas to expand which, in a gravitational field, convects away from the surface. So not only cannot the so called back radiation CAUSE catastrophic global warming, the warming it does cause is not significant enough to do much of anything except be used as a fictional cause of alarm in order to commit a monumental scam on a global scale.

    00

  • #

    Kevin,

    Welcome to the “crackpot” camp.

    Yes. One must always reconcile units with what you are trying to compare measurements. Simply having the same units is not enough. They must be measures of the same kind of thing.

    Since W/m^2 is a measure of flux, one must know the spatial distribution of the flux to know the significance of a comparison of two flux values from two different places in a system. That they are different is by itself an uninterpretable distinction beyond the fact they are different.

    As for the next Super Bowl winner, I predict that it will be the team who has the most points when the game is over. Also, I predict that both teams will be football teams and that there will be a lot of TV ads broadcast during the game. I leave the rest of the predictions to you. Even though I watched most of the last Super Bowl game and rather enjoyed it, I don’t remember who won.

    00

  • #
    L.J. Ryan

    KR 383

    Fixed system…you’re sidestepping the issue.

    KR you built a “static” thought experiment with a 255 K “heater”. I simply expanded upon your answer (365). I suspect you don’t like the conditions because they exposes your nonsense.

    Don’t run away KR, answer the question:

    Why not insulate the room and warm it to 288K with 255K radiative heater?

    00

  • #

    Mark D (#382):

    I am unable to decode your message hence unable to reply. Are you saying that the IPCC models make predictions? Are you saying that the models make projections? Are you saying that there is not a meaningful distinction between a projection and a prediction? Please respond.

    00

  • #
    L.J. Ryan

    Sphaerica (Bob):

    Are you ducking my question on 373?

    00

  • #
    BLouis79

    @KR #383
    Postma’s atmosphere paper describes a lot of essential basic physics. The earth as a system can only gain or lose heat by radiation. Conduction and convection are unable to transfer heat energy in a vacuum. Therefore, the energy of earth flows in and out by radiation (excepting energy contibutions of spacecraft and asteroids) and is described by the climate scientist’s much quoted Stefan-Boltzmann law.

    So when the earth is in thermal radiative equilibrium with other radiation sources = sun (ignoring small contributions from all the other stars in the universe), then the temperature (average) of earth is pretty much set by solar output and earth albedo. Unless CO2 changes albedo, CO2levels will be irrelevant to earth surface temperatures.

    00

  • #
    Bryan

    Richard S Courtney:

    I agreed with most of your post but was puzzled by this part;

    …”But heat is transferred by conduction, convection and radiation. So, there is no disagreement with 2nd Law if one of those mechanisms seems to transfer net energy in the ‘wrong’ direction so long as the other mechanisms move more heat in the other direction.”…..

    In the Earth system for radiation there is always more radiative flux leaving the hotter body than the colder.
    This means that heat travels in the direction of net flux as usual from higher to lower temperature.
    Perhaps you have other examples where one component of heat transfer goes “the wrong way”.
    I certainly cannot think of any!

    00

  • #
    Richard S Courtney

    KR:

    I had to laugh at your post at #374 that quotes my having said at #372:

    “So, the 2nd Law decrees that for GHGs to provide net downward IR flux then much more energy must be transported upwards by other mechanisms than IR flux.”

    And responds saying;

    A fascinating statement, given that actual measurements show that IR from the surface is 396 W/m^2, latent heat is 80 W/m^2, and convection is 17 W/m^2. Measured backradiation is on the order of 333 W/m^2.

    More energy (net energy, heat) goes up from the surface than down – but IR represents a major portion of upward energy, and all of the downward energy. You’re entitled to your own opinion, Richard, but not your own facts.

    Firstly,I will not take comments about “my own facts” from you when you are a proven liar.

    Secondly, if more energy enters the Earth than leaves it then how do you explain that the Earth’s temperature does not rise until the Earth melts?
    My statement that you dispute is a truism.

    Thirdly, the Kiehl-Trenberth energy budget is wrong (but you did know that really).

    Richard

    00

  • #
    co2isnotevil

    The reason the surface is warmer than 255K is simple. The Earth radiates 240 W/m^2 into space because that’s how much it receives from the Sun (after reflection). The surface must emit more, i.e. be warmer, because atmospheric absorption and clouds blocks on average, about 38% of the power emitted by the surface. Keep in mind, that radiation from clouds is just delayed surface radiation and that only radiant energy can leave the planet. If 38% of the emitted power is blocked, the surface must emit 62% more so that the required 240 W/m^2 escapes. This 62% corresponds to the aforementioned planetary emissivity of 0.62. If you want a fluid flow analogy, consider a water filter. The pressure on the inlet will be a little higher than the pressure on the outlet and the finer and denser the filter, the greater the pressure differential.

    Another point is that radiation from the upper atmosphere is relevant only for frequencies absorbed by the atmosphere. Frequencies in the transparent region, comprising about 24% of the power emitted by the surface, passes directly from the surface, and indirectly clouds, out into space as if the atmosphere wasn’t even there. In bands where absorption occurs, half of the asorbed surface power ultimately leaves the top of the atmosphere. This can be confirmed by examining the emitted energy spectrum of the Earth from space. In the 15u band, which at 100% saturation means that 100% of the power emitted by the surface will be absorbed, the emitted power density at the top of the atmosphere is not 0, but about half of the surface emitted power. This applies not only to bands subject to atmospheric absorption, but also to radiant energy from the surface and intercepted by clouds.

    Atmospheric simulations show that after accounting for clouds, about 24% of the surface power passes through the transparent region. This means that 76% must be absorbed by the atmosphere and clouds. If half of this escapes into space, 38% ends up leaving and 38% is returned to the surface. Adding 24% to 38% results in 62%, corresponding to the emissivity. Adding 38% of the 385 emitted by the surface (146 W/m^2) to the 240 W/m^2 reveived by the Sun gives us 386 W/m^2, which not coincidentally, is the average surface power emitted by the planet.

    George

    00

  • #
    Richard S Courtney

    Terry Oldberg:

    At #381 you say to BobC:

    You assert that “there are plenty of failed predictions from the AGW community over the last 30 years” but as I understand it, the IPCC climate models make “projections” rather than predictions. The former differ from the latter in the respect that they are not falsifiable. Can you provide citations to some predictions (not projections) of the IPCC’s models?

    In 2000 I was one of several scientists from around the world who was invited to give a briefing at the US Congress in Washiington, DC. And during the Q&A Session somebody stood and made the assertion that

    The IPCC does not make ‘predictions’ but only makes ‘projections’.

    I replied;

    Sir,
    You say the IPCC does not make predictions.
    The IPCC says the world is going to warm.
    I call that a prediction.

    The politicians applauded but the questioner did not respond.

    Since then the world has not had statistically discernible warming.

    Richard

    00

  • #
    Richard S Courtney

    Bryan:

    Thankyou for your post at #395. It makes a rational dispute of my comment that is a stark contrast to the several troll comments in this thread. It says to me:

    I agreed with most of your post but was puzzled by this part;

    …”But heat is transferred by conduction, convection and radiation. So, there is no disagreement with 2nd Law if one of those mechanisms seems to transfer net energy in the ‘wrong’ direction so long as the other mechanisms move more heat in the other direction.”…..

    In the Earth system for radiation there is always more radiative flux leaving the hotter body than the colder.
    This means that heat travels in the direction of net flux as usual from higher to lower temperature.
    Perhaps you have other examples where one component of heat transfer goes “the wrong way”.

    I certainly cannot think of any!

    But there are many, and collectively they are called ‘heat pumps’. Examples include refrigerators and heaters that use ‘cold’ ground to warm dwellings.

    The important facts of heat pumps are that
    (a) the energy deposited on the hot side is greater than taken from the cold side
    and
    (b) the increase to deposited energy increases the temperature at the hot side.

    The so-called Greenhouse Effect (GE) is a radiative heat pump that is powered by the Sun.

    Importantly, as my post that you query explained, the temperature of the GE ‘hot side’ (i.e. the Earth’s surface) is very, very close to its maximum limit.

    Richard

    00

  • #
    Paul

    BobC:
    May 18th, 2011 at 12:03 pm

    My personal opinion: The Radiative Greenhouse Effect is real but doubling CO2 will make a difference that may (or may not) be detectable. I also believe that trying to control the concentration of CO2 is probably the least effective method of climate control conceivable.

    When I’m not so busy I can try to find you some links to this stuff. You can find it yourself by typing
    “BobC site:http://joannenova.com.au”
    (without the quotes) into google and looking at the links in my old posts.

    Thanks for the offer, Bob, but I’m not really interested in doing such calculations because I do not see the relevance of looking at parts of the climate system independently from the entire system when they are obviously not independent. Besides, what are the assumptions behind those calculations? Do they pertain to the real atmosphere or just to laboratory conditions that will never be found in the real atmosphere?

    If the temperature at any altitude is set by the adiabatic lapse rate then any change in the radiative properties of the atmosphere by the addition of more CO2 [or H2O] will be compensated exactly by one of the other mechanisms of heat transfer. The net result will be approaching a zero change in temperature. Hence calculating an exact figure for heat transfer from the surface to outer space by some algorithm is at best academic and at worst misleading since it does not apply to the real world.

    For what it is worth, my opinion is that an increase in atmospheric carbon dioxide will assist the transfer of heat through the atmosphere, not slow it down. But, again, this is academic. If the other forms of heat transfer adjust to compensate, the end result will be identical temperatures at any given altitude.

    I did have the same attitude towards this ‘negligible’ effect as some others who have been mentioned, but have changed my view following further study due to this thread and now consider it to be a zero effect.

    If you compare the flow of heat, from the hotter to the colder regions, with the flow of water, which always flows to the lowest level, then the idea that radiation can cause heat to flow from the cooler air to the warmer surface of the earth looks about as impossible as water flowing up-hill.

    The issue, to me, seems to hinge on the answer one gives to the question : “How does a photon know when it comes from a hotter source than the object it now impinges on?” Someone earlier on suggested that this is because of the energy levels that pertain to both the objects and their emitted photons. A photon emitted from a warmer object has more energy than a photon emitted from a cooler object. Hence one would think that a photon impinging on a warmer object than that from which it was emitted would simply be reflected away, not having enough energy to overcome the electromagnetic fields surrounding the warmer object. In which case, “If it hits it warms” is a non-sequitur because the condition of it ‘hitting’ would imply that the second object ‘hit’ is at a cooler temperature than the object from which the photon was emitted.

    Unfortunately I am not sufficiently up on electromagnetism and heat-flows to state that dogmatically as being the case but I somehow doubt that it is not so. There has to be a reason why, statistically, the flow of heat always ends up going from the warmer to the cooler regions. It is not as if photons are intelligent and have a choice about the matter, as Jo herself has said.

    As to doubling the proportion of atmospheric carbon dioxide, I do not think that there is enough carbon in fossil fuels available, no matter how long it is burned, to achieve that concentration. Someone may like to prove me wrong on that, but since the land, ocean and biotic sinks for carbon are so much greater than the capacity of the atmosphere to hold carbon dioxide it is not just a linear relationship that is in question. Has anyone seen a mathematical function that describes this? To assume that all the currently measured increase in atmospheric CO2 is from human origin and then to extrapolate that linearly, as is being done, seems to me to be the antithesis of being scientific.

    Finally, I would like to know why it is suggested that the only radiation from the atmosphere is by CO2, H2O and other molecules that have the ability to absorb and emit photons. Is it being said that an atmosphere of nitrogen and oxygen, at ambient temperatures, is not able to emit heat other than by contact with another molecule? We know that those molecules are heated to some degree by the incoming solar radiation. If they cannot impart that heat other than by impinging on some other molecule then one would expect the atmosphere to keep on warming indefinitely in the absence of the so-called GHGs. That also seems to me to be an unsustainable position to take.

    Paul

    00

  • #
    RJ

    co2isnotevil: @ 361

    “RJ

    You have been deluded by Trenberth et all into believing that the surface is receiving 517 W/m^2 of incident power. In fact, it’s only receiving 240 W/m^2 of ‘new’ power from the Sun. There is no other substantial source of energy in the system! The whole concept of ‘back radiation’ as ‘forcing’ is an absurdity. It’s really just an accounting trick to make the silly CAGW hypothesis seem plausible to the ignorant.”

    Not me. I agree with you.

    00

  • #
    cohenite

    Richard@396; indeed KR@ 374 is peddling mischief; the KT diagram/cartoon is wrong because the window is not 40w/m2 but 60w/m2 which disproves AGW in itself; but it is also misleading in featuring gross radiative flux and not net flux: looking at KT 1997:

    http://www.atmos.uiuc.edu/colloquia/080430.htm

    Now 323W/m2 of backradiation or DLW sounds impressive but when you reduce it to a net surface flux the figures become interesting; the NET fluxes from the Surface are:
    a. Direct Radiation “through the window” to Space is 40W/m2 [which as noted is wrong].
    b. Fluxes into the Atmosphere:
    (1) Evaporated Water Vapour, 78W/m^2
    (2) NET Radiation, 26W/m^2
    (3) Conduction, 24W/m^2

    By comparison the 2008 K&T cartoon gives a NET upward radiation flux from the surface of 33w/m2 with a downward adjustment to water vapour to 76w/m2 and conduction to 16w/m2 but the point holds; that point is more net heat is leaving the surface through methods other than radiation, particularly water; that to me means 2 things; water is a dominant mover of heat compared to CO2 and the sun’s 168/166 w/m2 is a far more dominant heater than CO2 backradiation.

    00

  • #

    Hi all, long time no chat.
    Here I am sitting in my retreat on a cool cool Queensland night with one of those 1000w strip heaters trying to break the chill, enjoying reading almost all of the 400+ comments on this excellent thread. (thnx Jo)

    I was struck by an idea whilst reading some of those comments, tell me if I’m crazy and wasting my time.

    I’m going to rig-up a suspender with some coat-hanger wire and attach a 10″ length of iron bar to it. (picture a guitar player with a mouth harmonica lined up to his lips, suspended from his shoulders). I will then attach this unit to the front of my 1000w bar heater with the “harmonica” iron bar about 6″ in front of the strip heater.

    If all I’ve read from some of the obviously intelligent people in the know about back-radiation and its effects on temperature, my 1000w bar heater should very very soon be giving off close to double the heat.

    It’s amazing, almost twice the heat with a very simple “back-radiation” unit rigged up from household items readily available to everyone. Superrr.

    p.s. Does anyone know a good patent lawyer?

    00

  • #

    On a serious note, some of you contend that an Earth without GHGs (including no H2O I presume) will be much colder.
    I would have thought it would be much much warmer, in fact as warm as the warmest part of the day.

    If you believe the planet would be cooler without GHGs i.e. an atmosphere which consists ONLY of non-GHG gasses such as Nitrogen, Oxygen etc then please explain to me what happens to the warmth displaced to the atmosphere via conduction if this atmosphere can’t shed that warmth via radiation.

    00

  • #
    Bryan

    Richard S Courtney

    Thank you for your post reply.

    The second law refers to spontaneous heat flow so we can rule out refrigerators and heat pumps.

    However you have expanded your post to say that the atmosphere is a heat pump powered by the Sun.
    So presumably in your model the heat is extracted at the troposphere or above and deposited at the Earth surface.
    This would mean that during daylight the temperature in the troposphere would drop compared to that of nighttime.
    I’m not sure that there is any evidence of that happening.

    …”But heat is transferred by conduction, convection and radiation. So, there is no disagreement with 2nd Law if one of those mechanisms seems to transfer net energy in the ‘wrong’ direction so long as the other mechanisms move more heat in the other direction.”…..

    In the Earth system for radiation there is always more radiative flux leaving the hotter body than the colder.
    This means that heat travels in the direction of net flux as usual from higher to lower temperature.
    Perhaps you have other examples where one component of heat transfer goes “the wrong way”.

    I certainly cannot think of any!

    But there are many, and collectively they are called ‘heat pumps’. Examples include refrigerators and heaters that use ‘cold’ ground to warm dwellings.

    The important facts of heat pumps are that
    (a) the energy deposited on the hot side is greater than taken from the cold side
    and
    (b) the increase to deposited energy increases the temperature at the hot side.

    The so-called Greenhouse Effect (GE) is a radiative heat pump that is powered by the Sun.

    Importantly, as my post that you query explained, the temperature of the GE ‘hot side’ (i.e. the Earth’s surface) is very, very close to its maximum limit.

    Richard

    00

  • #
    Bryan

    oophs!!
    Left large part of previous post -disregard
    No excuse as Jo has a preview feature

    00

  • #
    RJ

    Question (on post 137

    What happen to a photon released from a colder object when it hits a warmer object.

    Does it simple bounce off and continue on it’s journey. Or is it absorbed but does not warm the object just because the warm object releases more energy than it receives from the photon.

    And if it is absorbed what happen to the energy in the photon.

    Because I can not see how a colder object could ever warm a warmer one. But doesn’t net energy mean it could if the outward flow could be stopped from the warmer object

    And not all protons are equal. So how does this impact when a photon hits an object. I assume colder object photons would warm less.

    00

  • #
    rpeo

    Somewhere someone on this blog refers to this url – it worried me overnight. The reference source on this blog is no longer apparent to me.

    http://www.drroyspencer.com/2010/07/yes-virginia-cooler-objects-can-make-warmer-objects-even-warmer-still/

    I find disturbing the assumption so graphically illustrated in this thought experiment article. I do not think the hotter steel object will be heated by the cooler one – the reverse should be the case with the hotter one maintaining its measured temperature and the cooler one heating up to a higher temperature.
    Roy Spencer who wrote this makes a big issue of his credentials – PhD and NASA worker.
    I bought his book, am a simple medical practitioner who took a law degree out of mid life boredom and studied first year university physics and chemistry very many years ago – I think his thought idea is bunk.

    00

  • #
    RJ

    Just to expand on this (@407)

    If two oil filled heaters with different temperatures were in a vacuum. Would the photons from the colder heater not heat the warmer heater only because the warmer heater releases more photons than it receives. Or are the photons different. And because of this difference do not heat the warmer heater.

    Hope this makes sense

    00

  • #
    cohenite

    Rancourt describes the ‘greenhouse’ effect here, but concludes it is less than conventional AGW theory supposes:

    http://climateguy.blogspot.com/2011/05/radiation-physics-constraints-on-global_12.html

    00

  • #
    Mark D.

    Bryan @ 405:

    The important facts of heat pumps are that
    (a) the energy deposited on the hot side is greater than taken from the cold side

    Sorry No. The energy is equal plus whatever energy was added (the electricity for the compressor for example) A heat pump does not “create” heat it consolidates heat.

    00

  • #

    RJ,

    Whether or not the photon is absorbed depends on its energy/frequency, and the mechanical properties of the recipient molecule. It has nothing whatsoever to do with heat or temperature. Of course, in aggregate, a warm mass of molecules are going to tend to emit more than they absorb, relative to a cooler mass of molecules. But on an individual basis, nothing is keeping some energy from flowing from the cooler mass to the warmer mass (i.e. a photon from being emitted from within the cooler mass and being absorbed by the warmer mass). On average, energy flows from the warmer to the cooler, but it’s a give-and-take, back-and-forth thing. The net flow is warm to cool because the warmer object emits more than the cooler object, and so has more chance of losing heat than gaining it.

    Put another way, heat, temperature, and the laws of thermodynamics are concepts which apply at the macroscopic level, not the microscopic level. They apply when one is talking about billions and billions of atoms or molecules, and there is a net average change.

    In the specific case of an individual photon (of the right frequency, in the infrared), and a CO2 molecule, if that CO2 molecule is not yet vibrating in that wavelength, then it may absorb the photon. An O2 or N2 molecule, regardless of its “temperature” (kinetic energy) will be transparent to that photon, no matter what, because it has no physical ability to vibrate at that frequency (i.e. there’s nothing that it can do with radiation of that frequency).

    This is a large part of understanding the greenhouse gas effect. Heat is not the only form that energy takes, and not all energy is created equal. Some molecules are transparent to some frequencies of radiation, and absorb or emit others.

    As a result the operation of the system is not anywhere near as simple as saying “the surface is hot, the atmosphere is cold, so the atmosphere cannot warm the surface.” The system is more complex than that, it has more “moving parts” than just a slab of ground and a slab of air, and if you try to think of it in those over simplified terms then you’ll be fooled into thinking that the greenhouse gas effect can’t exist.

    The atmosphere is not a single object, nor is it a homogeneous mass consisting of only one type of molecule and only one mechanism for transferring energy. There are many moving parts, and you must consider them all, and the net result of all of the possible interactions, in order to understand the system.

    00

  • #
    RJ

    OK I think I have found the answer to my question above in the slayers book ch 18 Claus Johnson

    An ideal blackbody absorbs all incoming radiation and remits all absorbed energy below cut off.

    Conservation of energy requires absorbed frequencies above cut-off to be stored in some form, more precisely as heat energy thus increasing the temperature of the blackbody.

    So the radiation would initially be absorbed but this would not raise the temperature of the warmer body.

    00

  • #
    Bryan

    Mark D

    The important facts of heat pumps are that
    (a) the energy deposited on the hot side is greater than taken from the cold side

    Sorry No. The energy is equal plus whatever energy was added (the electricity for the compressor for example) A heat pump does not “create” heat it consolidates heat.

    Yes I agree with you only the top quarter is my post.
    I copied the previous post and did not delete the redundant bit before posting.
    See the OOPHs post underneeth

    00

  • #
    RJ

    Bob @412

    The net flow is warm to cool because the warmer object emits more than the cooler object, and so has more chance of losing heat than gaining it.

    Thanks for the reply.

    Emits more? More photons? Or the same number of photons but with a higher frequency from the warmer object.

    00

  • #
    DavidA

    Taking the adjacent bar experiment to the extreme, lets hypothosize that we have one bar and it resides inside a hollowed sphere made out of the same conductive material. An electrode heat source is used to heat the centre bar. The whole experiment in constained within a vacuum, within the sphere and beyond the sphere.

    Does anyone seriously think the equilibrium temperature of the bar in this example is the same as when it is not surrounded by the sphere?

    00

  • #
    KR

    This has been an interesting and very active thread.

    Lots of posts and replies to what I’ve written, but most of it (IMO) is now in the category of “But yes!” “But no!” exchanges – the point where nobody is convinced. That, and insinuations on my character and honest, but I rather expect that in blog discussions.

    I’ll just point to my posts @292 (greenhouse theory, no violations of thermodynamics) and @348 (temperatures required to emit 240 W/m^2 to space for various emissivities), as I believe they cover some important information. The most efficient possible thermal radiator is a blackbody with it’s characteristic emission curve – anything with notches or dips below that curve is less efficient, and has to have a higher temperature to radiate the same power as that blackbody. 255K is the absolute maximum minimum temperature required to radiate that power, and that requires a perfect blackbody.

    I believe this link (language warning) shows the appropriate blackbody curve shape, and also expresses my feelings on this subject.

    The critical bit of thermodynamics here is the law of conservation of energy – what comes into the climate must equal what comes out (flow rates), or the amount of energy and hence temperature in the climate will change. Net energy moves from warmer to colder, and the second law of thermodynamics is not violated.

    Once again, major kudos to Jo Nova and to Michael Hammer for posting this thread in the face of it’s unpopularity with some skeptics.

    Adieu.

    00

  • #

    RJ,

    Both, and neither. That is, a warmer object is going to have more energy, so it’s going to emit more photons than a cooler object of the same substance. Rates of emission are partly dependent on molecular physics/mechanics, however, so for example a CO2 molecule radiates far more frequently than an O2 or N2 molecule. They also radiate in completely different frequencies, each determined by the energy required for the particular rotational or vibrational energy involved.

    Consider a simple O2 molecule. It looks like a tightly bound dumbbell, with two oxygen molecules joined by a very strong bond… O=O. This molecule can move through space in 3 directions (left/right, up/down, forward/backward). It can rotate meaningfully (meaning with a noticeable amount of energy) in two directions, because it’s symmetry means that when you look down it’s length, and spin it, there’s very, very little angular momentum. The energy involved in spinning it when looking straight through it is so small that we ignore it.

    It can also vibrate. Imagine the oxygen bond behaving like a spring, and the oxygen molecules alternately bouncing farther apart, then closer together. But because the oxygen bond is so strong, this vibration is very limited in range, and the molecules move very quickly.

    Now consider an H2O molecule. It is shaped like a V or a boomerang, with the oxygen in the middle and the hydrogen attached to the oxygen… H/O\H. Like an O2 molecule, it can move through space in three dimensions. Unlike an O2 molecule, it can have meaningful rotational energy in all three dimensions, because it’s not symmetrical. Spinning it in any of the three directions takes energy.

    The H2O molecule can also vibrate in a surprising number of ways. See this animated graphic of H2O vibrating.

    Now consider a CO2 molecule. It is normally organized in a straight line, like O2, so it looks like this… O=C=O. This implies the same degrees of freedom as O2. It can move in all three dimensions, and it can rotate in two.

    But CO2 has another capability. The bonds are not nearly as strong and tight as an O2 molecule’s bonds, so it is capable of a vibrational state where it alternately bends. See this animation to compare vibrations for O2, N2 and CO2. Because it’s no longer symmetrical when it’s vibrating, it can also rotate in another dimension when it’s vibrating, giving it another sort of motion.

    And, of course, molecules can mix and match some of these motions, producing combinations of vibration/rotation and their accompanying energy.

    So… we have different molecules, which because of their geometry and organization and strength of bonds the different types of molecules can move, rotate and vibrate in very different ways. Each of these rotational and vibrational states needs a different amount of energy. Spinning a larger H2O molecule with more angular momentum takes more energy than spinning an O2 molecule.

    There are two basic ways to cause these rotations and vibrations. One is through ordinary collisions. Two molecules strike, and depending on how they hit, they can make each other change direction, or start or stop rotating and vibrating in various ways. In this way, the molecules trade energy, and change “state” (vibrating, rotating, whatever).

    The other way for them to change their rotation or vibration, and to gain or lose energy, is through the emission or absorption of a photon. This happens pretty much at random, although certain states and molecules are more stable than others, so the chance of emission varies from case to case.

    That photon (this is quantum mechanics) is going to have the precise amount of energy (frequency) corresponding to the motion involved. This means that when a CO2 molecule vibrating “this” way emits a photon, it will have an energy X. If the CO2 molecule is vibrating a different way, it will have energy Y. This in turn is completely different from the energy emitted when an O2 molecule stops rotating.

    The reverse is also true. If a CO2 molecule is struck by a photon, that photon must have the correct energy (with some small leeway) to impart rotation, vibration or both in the molecule.

    This is very important. If the photon has the wrong energy to affect that particular molecule, the molecule will be transparent to that photon. The molecule can only absorb, and emit, photons of frequencies (energy) related to the motions allowed for that molecule.

    The upshot of this is that almost all molecules are transparent to very high energy, high frequency radiation like gamma rays. Molecules in gaseous form in the atmosphere are transparent to visible light, and so most light reaches the surface (H2O in liquid form, i.e. condensed in clouds, is another story). Molecules in gaseous form are each opaque to different frequencies of infrared radiation, depending on their nature.

    H2O absorbs/emits a lot of different frequencies of infrared radiation, because it has so many different rotational and vibrational states.

    CO2 absorbs/emits certain frequencies of infrared radiation, much of which can overlap with those of H2O.

    O2 and N2 absorb/emit very little infrared, and usually in very, very low frequencies (i.e. very low energy), and at a very slow rate. For this reason, O2 is a good insulator. O2 is basically transparent to most heat radiation (i.e. the frequencies normally emitted by warm objects, in the infrared), and is unlikely to either absorb or emit such radiation.

    Thus, the different molecules have very, very different properties when it comes to absorbing and exchanging energy in the atmosphere, and to understand what is going on, this all needs to be considered.

    00

  • #
    Mark D.

    Bryan @ 414 I see now, and thank you for sucking me into an argument with Richard Courtney! 🙂 I believe Richard understands heat pumps very well and that the comment he made @ 399

    the energy deposited on the hot side is greater than taken from the cold side

    is either the result of expedited typing or semantics. I’ll wait for his reply.

    00

  • #
    co2isnotevil

    Without GHG’s, the global albedo would about 0.1 (no GHG, no clouds) and the surface temperature would be about 278K, or about 5-6C. If you count the negative effect of GHG’s as clouds and surface snow/ice reflect energy and the positive (clouds and GHG’s recycle power back to the surface) the net effect is to warm the surface by only about 10C. It’s just not possible for any amount of CO2 to increase the net warming by 3-5C.

    What I find absolutely incredible about the warmist vs. scientist debate is that warmists refuse to apply simple sanity checking to their conclusions.

    00

  • #
    BobC

    If the IPCC claims no predictive skill for their “projections” (the only way they could be “non falsifiable”), then the proper policy response is to ignore them.

    I’ve not gotten the impression that that is the IPCC’s official position.

    Terry Oldberg @381:
    May 18th, 2011 at 1:17 pm
    BobC (#376):

    You assert that “there are plenty of failed predictions from the AGW community over the last 30 years” but as I understand it, the IPCC climate models make “projections” rather than predictions. The former differ from the latter in the respect that they are not falsifiable. Can you provide citations to some predictions (not projections) of the IPCC’s models?

    00

  • #
    Richard S Courtney

    Bryan:

    I am replying to your post at #405.

    No problem with the ‘cut and paste’ problem: what you are saying is clear.

    Your main point is your statement saying;

    The second law refers to spontaneous heat flow so we can rule out refrigerators and heat pumps.

    Sorry, but all thermodynamical systems – including all heat pumps of all kinds – obey the 2nd Law of Thermodynamics.

    The important point is that the GE is a heat pump powered by the Sun.

    The atmosphere is a dynamic system which is powered by the Sun and this is why, for example, we experience weather. Indeed, ‘warmists’ forget this basic fact that almost everything happening in the atmosphere is powered by the Sun (‘almost all’ and not ‘all’ because geothermal effects contribute some input). So, warmists are fond of saying that radiative physics is all that matters, but it is not; e.g. there is no radiative physics involved in the transitions between water, water vapour and ice but those transitions move a lot of heat.

    Richard

    00

  • #
    Richard S Courtney

    RJ:

    At #407 you say;

    Because I can not see how a colder object could ever warm a warmer one. But doesn’t net energy mean it could if the outward flow could be stopped from the warmer object

    Sorry, but if you own a microwave oven then you are familiar with “a colder object” that warms “a warmer one”.

    What you say is true of conduction and convection but is not true of radiative excitation. Electricity provides the power to heat the food in a microwave oven, and the Sun provides the power to induce the radiative excitation of GHG molecules in the atmosphere. However, the effect of CO2 in the atmosphere is almost ‘maxed out’ so increased CO2 in the air cannot induce discernible additional warming.

    Richard

    00

  • #
    Richard S Courtney

    MarkD:

    Thankyou for your clarification at #411.

    Yes, that is what I meant. Perhaps I simplified too much.

    Richard

    00

  • #

    Richard Courtney (#397):

    That the Earth warms can be viewed as an incompletely specified prediction. By definition, a “prediction” is a proposition that states the outcome of a statistical event or the value that is assigned to the probability of this outcome. For statistical significance of the claims that are made by the associated model, this event must be an element in a sequence of statistically independent events, a number of which are observed but not used in the construction of the model. So far as I’ve been able to determine, the IPCC has not yet identified this sequence, the complete set of outcomes for the events in this sequence, the protocol by which events in this sequence are selected for observation or the manner in which these outcomes are measured. Thus, I side with Kevin Trenberth when he says ( URL = http://blogs.nature.com/climatefeedback/recent_contributors/kevin_trenberth ) that the IPCC does not make predictions and never has.

    Data from the study of Green and Armstrong suggest IPCC climatologists confuse non-falsifiable “projections” with falsifiable predictions. This confusion may be associated with the belief among IPCC climatologists that their studies are scientific ones. I discuss this issue in detail in the article at http://judithcurry.com/2011/02/15/the-principles-of-reasoning-part-iii-logic-and-climatology/ . In the article I find that the methodology of IPCC research is illogical and unscientific but the ambiguity of reference by terms in the language of climatology to the associated ideas can make it sound logical and scientific.

    00

  • #
    Richard S Courtney

    Terry Oldberg:

    I am sorry to say that I think your post at #426 is sophistry.

    The IPCC predicts that the Earth will warm and “projects” a range of probable trajectories for that warming. None of those trajectories says the Earth will cool.

    One can asign probabilities to the trajectories (i.e. the “projections”) but the prediction of warming and not cooling is either right or wrong although there is a small possibility that neither warming or cooling will occur.

    This is similar to predicting that a coin will fall heads and not tails. The prediction is either right or wrong although there is a small possibility that the coin will land on its edge.

    One can project the trajectory of a coin before it lands heads or tails. Similarly, one can assert that the global temperature has yet to reach the state where one can determine if the prediction of warming or cooling is fulfilled. But that assertion does NOT mean the IPCC has not predicted warming and not cooling. And the fact is that as yet there is no statistically discernible warming or cooling over the last decade.

    00

  • #
    Richard S Courtney

    Terry Oldberg:

    I apologise that my post at #426 was sent before I intended (as can be seen by my failure to put my name to the end of it) so it was not checked.

    Of course, that post was answering your post at #425 (not #426 as it erroneously says).

    Richard

    00

  • #

    BobC (#421):

    The word “prediction” references a different concept than the word “projection.” Thus, it would be logically impossible for the IPCC to claim predictive skill for its projections.

    The data of Green and Armstrong demonstrate that climatologists confuse the two concepts. Thus, perhaps, it can seem to a climatologist that a projection can exhibit predictive skill.

    00

  • #

    May I just point out that there seems to be a radiation obsession at present, that is clouding many people’s judgment.

    Emitted IR is the result of the temperature of the object in question,
    due to the main / most powerful cooling or warming processes involved.

    Which does not mean necessarily IR is the cause,
    or even particularly important in explaining why the object is the temperature it is.

    Put another way.
    Emitted IR by an object is a temperature signal of the object, but,
    that tells us nothing of what the main processes are that are either warming or cooling the object in question.
    Assuming radiation is the main warming or cooling process appears to be a widespread error at present,
    when evidence, and every day observations, show it is but a minor player within the planet’s climate system.

    00

  • #
    BLouis79

    @Derek #429 “there seems to be a radiation obsession”
    I understand that for the earth as a whole planet in a vacuum, conduction and convection are effectively excluded. See also #394.

    Further, it is self evident that any effort to convert potential energy (chemical, nuclear) to heat on earth will add heat energy to the earth’s surface. This has been quantified by Bo Nordell and yet summarily dismissed by climate scientists and nuclear energy proponents.

    When the earth is in radiative thermal equilibrium with the sun, any surface and atmospheric composition or temperature changes become irrelevant unless they result in a change in albedo or emissivity, because the warmer earth can only absorb less energy from the sun (assuming a constant solar output, albedo, and emissivity). I understand emissivity is generally assumed to be 1 and unchanging.

    Analysis of earth temperature is heavily biased by proximity of humans to surface temperatures, as opposed to astronomers/astrophysicists who are more akin to looking at earth from space.

    It appears from recent research that cosmic rays have a significant impact on cloud formation and hence albedo. So we know that changing solar radiance and probably cosmic rays can change earth’s temperature. It remains to be demonstrated if any earthbound activity can change the earth’s albedo and emissivity and result in a change in earth’s temperature.

    It is probably an expression of fundamental egocentrism that humans think they control the planet in the universe.

    00

  • #
    CO2ISLIFE

    Have a looke here folks if you want some great stuff, properly worked out and well referenced:

    http://www.biocab.org/Index.html

    Here is a site that actually has someone doing calculations based on proper physics. One of their discussions actually goes right back Into the theory of electromagnetic radiation pressure which underpins the laws of thermodynamics and shows how radiation pressure rules the quantum world and how the second law actually plays out in practice with high pressure overcoming low pressure.

    A particularly good paper is the PDF on:

    “Disagreement between the Warming of the Surface by the Downwelling Radiation Hypothesis and the Theory of the Electromagnetic Radiation Pressure”.

    The approach here is incredibly revealing and supported by some basic calculations and a good suite of references. … Enjoy

    00

  • #

    Richard Courtney (#426):

    You’ve muddied the waters by making my character the issue. For the sake of continuity, I’ll ignore this diversion from the issue at hand.

    Your implied description of a “prediction” is too narrow for it ignores the relationship between predictions and the associated set of independent statistical events. A coin flip is an example of an independent statistical event. Thus, it is legitimate to speak of “predicting” the outcomes in a sequence of coin flips.

    In the case of the IPCC’s “prediction” of warming no such events are evident in AR4. Such an event would have been fully specified, for example, if it had extended over the 30 year period starting on January 1, 1987 and ending on January 1, 2007, if the prediction had been made on January 1, 1987, if the outcome had been observable on January 1, 2007, if one of the two possible outcomes were that the average increase in the HADCRUT3 global temperature time series exceeded the 150 year median and that the other of the two possible outcomes were the complement of the first outcome.

    00

  • #
    thRealUniverse

    “How does a photon know when it comes from a hotter source than the object it now impinges on?”
    A photon moves along the light cone from spacetime point A to point B , thermodynamic state of particle at B is unknown to A unless exchange of photons between them but it all occurs along the light cone.
    A property of the universe called the thermodynamic arrow of time, which is a subject of cosmology!.
    Part of the universal structure and the nature of the particles.

    00

  • #
    Paul

    “Kr, what is your personal version of the greenhouse theory?”

    Quite frankly, my personal version is the textbook version. How I explain it to others, what math and analogies I use, varies with the audience and the interest, as shown here.

    We get 240 W/m^2 energy from the sun – this is affected by things like albedo (ice coverage, aerosols, clouds, parking lots). This warms the atmosphere and the surface of the Earth, heat (net energy) flowing from warmer to colder.

    That is a mighty big assumption, right at the start. You have assumed gratuitously that the earth is equivalent to a disk with the same surface area as the sphere that it actually is. Then, without the slightest attempt to consider what effect that might have on your calculations you jump right into calculating the blackbody temperature of the earth’s surface. Such over-simplification is not justified, especially when so much hinges on the question.

    The rotation of the earth carries much energy from the exposed side of the earth to the unexposed side of the earth. This energy needs to be quantified and allowed for in arriving at the ‘theoretical’ near-surface air temperatures.

    Also, the equatorial/polar heat flows, mainly in the oceans but also in the atmosphere, need to be quantified and allowed for in arriving at the ‘theoretical’ near-surface air temperatures.

    One has to be rather sceptical of any global average near-surface air temperature as having any scientific significance given that all temperatures are applicable only at their given latitude, the effect of ocean heat-sinks, the daily rotation of the earth and many other confounding factors.

    The surface of the Earth sends heat to the atmosphere, a lesser amount comes back as “backradiation”, or thermal radiation from the atmosphere, again heat (net energy) flowing from warmer to colder.

    And the surface and the atmosphere radiate to space, with heat (net energy) flowing from warmer to colder.

    So far, no violations of any thermodynamics.

    Once again you have assumed, without any justification, that all energy leaves the surface of the earth by means of radiation. This is clearly not so, and hence your calculations that follow must be in error to the extent that the real earth behaves in a different manor from your assumption.

    Now in order to remain at equilibrium (a steady temperature average) the Earth needs to radiate that 240 W/m^2 to space. What governs this rate? Thermal radiation (emitted by any object over 0°K), the power of which is set by two variables – emissivity and temperature. This is the Stefan–Boltzmann law, where:

    Power = emissivity * SB * Area * T^4

    SB is the Stefan-Boltzmann constant of 5.670 400(40)×10−8 W·m−2·K−4.

    Rearranging for temperature as a function of emissivity,

    T°K = ( 240 W/m^2 / ( emissivity * SB ) ) ^ 0.25, or
    T°C = ( 240 W/m^2 / ( emissivity * SB ) ) ^ 0.25 – 273.15

    This works in Excel; set your emissivity as one cell, and refer to that in the next one over, and you can try what happens with different emissivities to space.

    Now, without the greenhouse effect, dirt, plants, water, and basically all non-metallic surfaces have an emissivity near 1.0 in the temperature range we’re interested in. The average emissivity of the surface of the Earth is ~0.98, and for that emissivity the Earth would radiate 240 W/m^2 at -16.8°C.

    Brrrrrr…

    As stated above, this fictitious figure of -16.8°C is purely an artefact of your simplistic assumptions which in no way correspond to the real world.

    So what happens with greenhouse gases? Some of the radiated energy is absorbed and reradiated back to the surface (effectively not leaving), when it reaches an altitude where the GHG thins enough to radiate to space, the lapse rate cooling with altitude (lower T) means less radiation. Clouds block a lot of IR as well, and clouds cover a significant portion of the globe. You can see that in the top of atmosphere notches, as shown here.

    So you say ‘some’ of the radiated energy is absorbed and re-radiated back to the surface. How much? How have you quantified this supremely important aspect of the entire debate? What known laws of gasses would apply?

    Right here is the crux of the question and it looks more like a ‘black box’ [i.e. unknown] than a scientific explanation to me. Am I to believe you just because you [and the IPCC] say there is this effect?

    The references I’ve looked at say the effective emissivity (judging from satellite spectra) is about 0.612, ~61% the emissivity of a blackbody at average surface temperature, which corresponds to an average surface temperature of 15.2°C. Which, oddly enough, is about what we measure with those thermometer things…

    So are you telling me that the signature temperature of the earth-atmosphere system, when viewed from a satellite, looks like a blackbody with a temperature of 15.2°C? Or are you saying that this is the signature of the surface of the earth when so viewed? If the latter it would be hardly surprising, would it now?

    So – it’s all about rates. If the rate of energy leaving the Earth doesn’t match the rate of energy coming in, we’re not at equilibrium, and the temperature will be changing. Add a dollop of GHG’s, effective emissivity to space changes (effective radiation altitude increases, bands widen), and a 0.1% change in emissivity means ~0.9°C in equilibrium temperature. At equilibrium, rate in = rate out. And GHG’s affect the emissivity – the only thing that can change to regain equilibrium is the temperature.

    After that, after a GHG change in forcing, come the feedbacks. The best estimates right now for a doubling of CO2 (direct is ~1.2°C) feedbacks result in 3°C total increase – I have only faint hopes that this is wrong. But maybe the majority of climate scientists are wrong, and we’ll be lucky.

    Well I think that your argument has been shown to be inaccurate at best and completely erroneous at worst. I do thank you, however, for putting your explanation forward.

    This leads me to suggesting that there are some very obvious experiments that need to be carried out to test the ‘accepted science’ of which this is an example.

    1) Measure the heat of both a sphere and a disk of the same surface area exposed in a vacuum to an equivalent incident radiation and compare their equilibrium temperatures. [Make the distance from the heat source such that the radiation received per square unit corresponds to the division by 4 when using the disk as a substitute for the sphere.]

    2) Repeat the above experiment while allowing a measured flow of air over the surfaces.

    3) Repeat the above and add a measured amount of water to evaporate on the surfaces.

    4) Introduce rotation to the sphere so that it has

    Maybe someone with that program that does these heat-loss and temperature calculations can simulate the above experiments virtually.

    At any rate, until these conjectures are quantified and verified by real-world experiments, they do not qualify for being called scientific. They are no better than philosophical day-dreams.

    Paul

    00

  • #
    RW

    Over 400 posts now (I’ve not read them all) The one at skeptical science is over 1000 posts!

    I do not get how the greenhouse effect violates the second law, because it not about energy traveling from cold to warm through a conduction process. Does anyone actually think that a photon cannot travel from the colder atmosphere toward the warmer surface?

    00

  • #
    L.J. Ryan

    Richard S Courtney: 423

    I applaud your efforts taking on the AGW, crowd congressional testimony and otherwise. With the political correct nature of climate science, (as ridiculous as that sounds) the bureaucratic spotlight must be burning hot, so again kudos.

    I do however, question your microwave oven analogy. Specifically, microwave ovens temp is irrelevant as IR is not it’s mechanism. Microwave energy level and the radiated body’s energy are the sole determining factors of energy exchange. If energy of microwaves are greater then that of the body in it’s path, the body’s energy will increase. If the body’s energy is equivalent or greater, the microwaves will add nothing.

    If you were to microwave a blackbody, the radiated IR would represent maximum temp for that oven. So if, for example, the found maximum temperature of the oven was 500 C, then an body preheated to 500 C would not increase in temperature when in the microwaves path…no increase in energy.

    Likewise, GHG which absorbs terrestrial radiation then re-radiate downward can not add any energy to the hotter/higher energy source…thereby no additional heating. If the atmosphere is warmer/higher energy then the surface it will add energy…thereby heating.

    One more, more obvious example. A red hot heating element (~730 C) can not heat a body beyond 730 C. Radiative quantities follow the same thermodynamic laws as other transfer methods. Therefore, radiative transfer equations notwithstanding, are limited by both radiating and absorbing bodies.

    00

  • #
    L.J. Ryan

    RW 435

    GHG absorbs IR from the Earth’s surface—no violation

    GHG radiates isotropically—no violation

    Terrestrial bound GHG emission are absorbed by the higher temperature—no violation

    Absorbed back radiation raises energy of higher energy surface energy—VIOLATION

    Proponents and some skeptics, try to sidestep this clear violation by introducing qualifiers such as NET energy transfer and/or claiming backradiation is the radiative equivalent of conductive insulation. Yet insulation does not “force” warming beyond input. Solar input, 240 W/m^2 SW is the only radiative input to the Earth. Therefore RW, 240 W/m^2 (255K) is the maximum energy via solar radiance…forcing, or insulating can not increase this constant.

    Radiative inputs are not additive. Said otherwise two 240 W/m^2 radiative sources incident on a single surface is not equivalent to one 480 W/m^2 radiative source incident on that same single surface.

    I realize this shortest of primers, does not provide insight to the actual experiences (288K surface); but after all, this blog is to hash our the falsehood of forcing.

    00

  • #
    RW

    L.J. Ryan (RE: 436),

    “Absorbed back radiation raises energy of higher energy surface energy—VIOLATION”

    I don’t see how. Are you saying that the surface cannot absorb the energy of a back radiated photon from the atmosphere? Where then does the photon go? How does the surface absorb the photons coming from the Sun?

    00

  • #
    L.J. Ryan

    Sphaerica (Bob):

    Are you avoiding my questions (373)? Since 373 you’ve posted 3 times while avoiding my retort to your 358. I’ll save you the trouble of panning back:

    Your comments 358—replying to 355

    You’re being silly and translating my statements into insanity. It may be entertaining for you, but it is hardly helping you to understand matters that you clearly do not.

    The temperature of a body is proportional to its energy.

    The energy emitted by a body is proportional to the fourth power of the temperature.

    Greenhouse gases prevent energy from escaping, increasing temperature.

    When the temperature of the earth reaches the point where it is able radiate away as much as it is absorbing, it has reached equilibrium at the new temperature.

    Take all the crazy pot shots you want. Being ignorant of the science while shouting your incredulity does not help you, or anyone, to understand the science better.

    My reply 373

    “Greenhouse gases prevent energy from escaping, increasing temperature.—Sphaerica (Bob):

    1) What is the source and magnitude of this energy…is it 240 W/m^2?

    “When the temperature of the earth reaches the point where it is able radiate away as much as it is absorbing it has reached equilibrium at the new temperature.—Sphaerica (Bob):

    2)New equilibrium ok: Earth @ 255 K radiating 240 W/m^2 heats atmosphere (ε=1) to 255K. Atmosphere at 255 K radiates 240 W/m^2 toward the surface and 240 W/m^2 space-ward (OLR) and we have new equilibrium.

    2a)New equilibrium ok: Earth @ 255 K radiating 240 W/m^2 heats atmosphere (ε= < 1) to temperature < 255K, but also transmits flux (1-ε)space-ward. So although the atmosphere temp is NOT 255K system equilibrium is reached. That is, ε240 W/m^2 via atmosphere plus (1-ε)240 W/m^2 = 240 W/m^2 OLR we have new equilibrium.

    So in both cases equilibrium is reached with Earth at 255K…no need to increase temp.

    Any interested readers should note your evasion, and realize you do not feel confident enough to defend your sophistry.

    00

  • #
    co2isnotevil

    As another example, if you take a 1000C object and put it in front of a 730C heater, the 730C heater will initially get warmer from the radiation of the hotter object, but in the steady state, both objects will converge to 730C. This is also true if the 2 objects are contained within an insulated box, the only difference being it takes longer to reach the steady state.

    This brings up an important point that warmist analogies consistently gloss over. The GHG’s in the atmosphere are quite different from an insulating blanket, or even a greenhouse for that matter. It’s more like a blanket with lots of holes in it, corresponding to the transparent window. You just can’t add enough CO2 to the atmosphere to make the transparent window go away. Even cloud cover won’t block the radiation, simply because clouds radiate themselves. How else can we detect cloud top temperatures from space?

    Cloud tops radiate approximately half the total flux absorbed by the cloud. This is because the surface area of a radiating cloud is approximately twice the surface area over which power from either the Sun or surface arrives and power fluxes have units of power per unit area. It’s considered that clouds are cold because of the lapse rate, but the power radiated by clouds is a function of the power being absorbed. The adiabatic lapse rate to do with setting the height of clouds based on their temperature, not establishing the temperature.

    00

  • #
    Paul

    RW:
    May 19th, 2011 at 11:22 am

    I do not get how the greenhouse effect violates the second law, because it not about energy traveling from cold to warm through a conduction process. Does anyone actually think that a photon cannot travel from the colder atmosphere toward the warmer surface?

    The answer to that final question is very simple : the second law of thermodynamics refers to the macro-behaviour of the system, not to the behaviour of a single element of that system. And it applies to radiation just as much as it applies to conduction.

    The second law, in effect says, ‘on average more energy will flow from the warmer to the cooler region’ so that even supposing some photons are emitted from the cooler atmosphere towards the surface of the earth, on average their number will be less than the number of photons emitted from the surface of the earth and absorbed within the cooler atmosphere. On average, the flow of heat will be only in the direction of hot to cold.

    The reason why this occurs is also easy to understand : if the initial conditions of the system is totally random, given sufficient time the behaviour of the many particles involved, solid, liquid and gaseous, become conditioned by the behaviour of the other particles. So the organisation of the system becomes more correlated.

    Entropy (arrow of time)
    An important difference between the past and the future is that in any system (such as a gas of particles) its initial conditions are usually such that its different parts are uncorrelated, but as the system evolves and its different parts interact with each other, they become correlated.[3] For example, whenever dealing with a gas of particles, it is always assumed that its initial conditions are such that there is no correlation between the states of different particles (i.e. the speeds and locations of the different particles are completely random, up to the need to conform with the macrostate of the system). This is closely related to the Second Law of Thermodynamics.

    This is an inviolable process and the more particles involved the closer it comes to the statistical average. It ‘converges’ on the one-way heat flow not ‘diverges’.

    Looked at in terms of the analogy of water flowing over a water-fall, one can easily see that some drops of water may be splashed upwards, against the force of gravity, but they will then be drawn back by gravity and so the net flow of the waterfall will not be even marginally affected.

    Looking at the statistical probability aspect of the issue, suppose you toss a coin and observe the results. On the first throw you will get either ‘heads’ or ‘tails’. So you may say that the odds are 0 : 1 either way.

    Increase the number of trials to ten and the likelihood of getting five ‘heads’ and five ‘tails’ is greater than any other result. Take that to 1000 trials and the proportion gets even closer to 0.5 : 0.5. As ‘N’, the number of trials tends towards infinity, the ratio of heads to tails tends towards the proportions 0.5 : 0.5 even closer. That is the basis of statistical predictions. That is why Casinos make money, on average, and the players lose money ‘on average’. No one can beat the odds in the long run.

    Getting back to topic, no amount of discussion back and forth about this crucial issue in the CAGW debate can resolve this, and the only way that it can be finally resolved is by setting up an experiment that can quantify the exchange of energy between a solid and an air-mass containing varying known amounts of Tindall gasses. This is such a simple experiment for the multi-billion dollar CAGW team to perform that, if it could be demonstrated, which I doubt, they would have been expected to have performed and published the results for all the world to see.

    The reason why this has not been done, I suspect, is that those behind this alarmism know only too well that once this has been done, published and critiqued, their game will be up!

    Paul

    00

  • #
    co2isnotevil

    Sphaerica,

    Your statement that ‘Greenhouse gases prevent energy from escaping, increasing temperature” is misleading at best.
    The correct phrase is ‘Greenhouse gases prevent energy from escaping, slowing down cooling.’
    Solar power causes warming and the lack of solar power causes cooling.
    Clouds and GHG’s slow down the cooling.

    The dynamic effect of GHG’s and clouds is an average of 2/3 of the total effect, while CO2 supplies a constant value for the remaining third. The total effect, as defined here, is also known as the operating point of the system, which is defined by a nominal cloud coverage and a nominal water vapor concentration. These nominal values are the average values of dynamic control knobs maintaining the planets energy balance by modulating the flux passing through the atmosphere. More precisely, the overriding requirement for the planet to be in steady state equilibrium with it’s source of energy requires these control knobs to have specific values for this requirement to be met. This can be modeled by a feedback control system, but not the IPCC model of a feedback system controlling surface temperatures, but of a feedback control system maintaining the planets energy balance.

    If the static component of absorption was a few percent higher, or lower, the climate system has enough dynamic range to adjust the total clouds and GHG’s such that a steady state equilibrium with the Sun can be maintained. Because the gain is low, the climate behavior is far from ideal, so there will be surface temperature changes, but they will be less than the intrinsic effect of CO2, not more. This is a consequence of the net negative feedback intrinsic to the properly modeled feedback control system.

    George

    00

  • #
    Paul

    As an addendum to my last post, I’ve thought of another analogy which may clarify the discussion.

    When kayaking down the Whanganui River, some years ago now, I learned that you can ‘work’ the rapids, so that instead of just going down the infrequent rapids once, one may get into an eddy at the side of the river that is sufficiently slow to allow you to paddle almost to the top of the rapid where the still water begins to break into white water. Then, by putting your paddle deep into the flow your kayak can turn into the central flow and you can have that excitement all over again – as long as you like to play.

    The questions I would ask, then, are :

    1) “Does the eddy current increase/decrease the flow-rate of the central current?”

    2) “Does the eddy current increase/decrease the flow-rate of the river below that point?”

    I think that the answer to the first question would be that the eddy increases the flow rate of the central current but that the answer to the second question is that the flow rate of the river is the same below the eddy as it was above the eddy.

    Applying the analogy to the flow of photons between the earth’s surface, through the atmosphere and out to space, if photons flow from the atmosphere to the surface then the only effect that will have will be to increase the flow of photons from the surface to the atmosphere. The flow of photons going from the atmosphere to space will not change.

    Hence, I would expect that the temperatures of the surface and of the atmosphere to be unaffected by this so-called back-radiation.

    But, then again, the answer is to devise an experiment and quantify the measurable parameters and put an end to the matter once and for all. Isn’t that the ‘scientific method’?

    Paul

    00

  • #
    L.J. Ryan

    RW: 438

    First off, all lower energy is unable to raise increase higher energy. Tell me KR, if two heat sources at -18 C are facing one another do expect the temperature of one or both of them to increase to 15C? Seriously RW whats the answer? Can you support your position?

    Regarding your question:

    For the surface to become warmer due to the absorption of the photon from a greenhouse gas, higher energy vibrational states must become occupied in the Earth’s surface materials. A photon from a lower temperature emitter cannot warm the surface to a higher temperature because that lower energy photon cannot excite the necessary higher energy vibrational modes. That photon can slow down the cooling of surface at night, since its emission at night will cool the surface and the returned photon will be at a higher energy than the surface is by the time the photon returns. This is the equivalent of the process when we put hot coffee in a thermos, thereby slowing down its cooling rate. But, the returning photons from the reflective wall in the thermos never heat the coffee to a higher temperature than it was at when it was poured into the thermos.

    00

  • #
    Paul

    But, the returning photons from the reflective wall in the thermos never heat the coffee to a higher temperature than it was at when it was poured into the thermos.

    So any ‘Greenhouse’ effect can only exist at night. During the day it cannot increase the temperature of the earth’s surface and at night it may slow the cooling. Right?

    Paul

    00

  • #
    Paul

    co2isnotevil:
    May 19th, 2011 at 1:38 pm

    This brings up an important point that warmist analogies consistently gloss over. The GHG’s in the atmosphere are quite different from an insulating blanket, or even a greenhouse for that matter. It’s more like a blanket with lots of holes in it, corresponding to the transparent window. You just can’t add enough CO2 to the atmosphere to make the transparent window go away. Even cloud cover won’t block the radiation, simply because clouds radiate themselves. How else can we detect cloud top temperatures from space?

    This brings another important question to my mind.

    When the instrument reads the temperature of the clouds is it not doing so because photons from that source are impinging on the instrument? If so, does that not prove that the photons are absorbed regardless of the energy state of the receiving material? But does that mean that the instrument is getting warmer by the exchange?

    The second question, that follows, is, when scientists measure the back-radiation of the atmosphere, are they not in fact just measuring the background temperature of the atmosphere? Does that then translate into a flow of energy from the cooler atmosphere to the warmer measuring instrument, or, more to the point, to the surface of the earth?

    Paul

    00

  • #
    co2isnotevil

    Paul,

    Yes, the instrument gets warmer, but not by enough to be measurable.

    What they call ‘back radiation’ includes only half of 2 cyclic energy fluxes. About 100 W/m^2 is included for latent heat, although latent heat is mostly returned to the surface as weather. Thermals are also included, which are only half of a vertical circulation, where rising hot air is replaced by falling cool air somewhere else. The only effect of thermals is to locally increase the thermal conductivity between the surface and space, which is the opposite of clouds. Trenberth’s accounting also includes other dubious components, like the heating of the atmosphere by the Sun. It’s all just part of a shell game to count energy more than once when it makes CO2 seem more powerful than it really is.

    If you want to measure the ‘back-radiation’, it must be measured at night, since during the day, all you see is solar heating. If it really was the 400-500 W/m^2 claimed, the night would be pretty warm.

    George

    00

  • #
    Richard S Courtney

    Terry Oldberg:

    Your post at #432 is more sophistry. And I have not raised the issue of your character.

    As I explained at #426, the IPCC predicts warming and projects a range of trajectories for that warming.

    Section
    10.7.1 Climate Change Commitment to Year 2300 Based on AOGCMs
    in the Report from WG1 (i.e. the “science” Working Group) of the most recent IPCC Report (AR4) can be read at
    http://www.ipcc.ch/publications_and_data/ar4/wg1/en/ch10s10-7.html

    It says

    The multi-model average warming for all radiative forcing agents held constant at year 2000 (reported earlier for several of the models by Meehl et al., 2005c), is about 0.6°C for the period 2090 to 2099 relative to the 1980 to 1999 reference period. This is roughly the magnitude of warming simulated in the 20th century. Applying the same uncertainty assessment as for the SRES scenarios in Fig. 10.29 (–40 to +60%), the likely uncertainty range is 0.3°C to 0.9°C. Hansen et al. (2005a) calculate the current energy imbalance of the Earth to be 0.85 W m–2, implying that the unrealised global warming is about 0.6°C without any further increase in radiative forcing. The committed warming trend values show a rate of warming averaged over the first two decades of the 21st century of about 0.1°C per decade, due mainly to the slow response of the oceans. About twice as much warming (0.2°C per decade) would be expected if emissions are within the range of the SRES scenarios.

    So, the IPCC says “The committed warming trend values show a rate of warming averaged over the first two decades of the 21st century of about 0.1°C per decade”.
    n.b. That is “committed warming” that will occur because of effects in the past.

    That is a prediction and no amount of sophistry can deny it is a prediction.

    But there has not been “0.1°C per decade” for the first of half of “the first two decades of the 21st century”.

    Richard

    00

  • #
    Bryan

    Paul at 446

    The instrument pointing up at the cold night sky will have more radiation leaving the instrument than entering it.
    Essentially at the heart of these instruments you will have a thermocouple and the output voltage will return a negative value.
    If pointed down to the surface.
    The instrument will have less radiation leaving the instrument than entering it.
    The instrument output voltage will return a positive value.
    These voltages are then scaled to (P/m2)(pyrgeometers) or to (K) temperature (IR thermometer) using a set of very “iffy” assumptions such as; perfect black body, Stephan Boltzmann equation and can an atmospheric line spectra emission be measured or the same scale as the continuous emission from Earth surface and usually have a fixed value for emissivity.
    The figures are very unreliable
    So you can see why the figures in say KT97 are constantly being revised for any new such poster.

    00

  • #
    cohenite

    George says:”If you want to measure the ‘back-radiation’, it must be measured at night, since during the day, all you see is solar heating. If it really was the 400-500 W/m^2 claimed, the night would be pretty warm.

    Exactly; and what has the much awaited Watts paper found; exactly the opposite; bad temperature siting has overestimated the heat trend at night:

    http://pielkeclimatesci.files.wordpress.com/2011/05/r-3671.pdf

    00

  • #
    Smoking Frog

    Pour a hot liquid into a thermos bottle. The outer part of the bottle will be cooler, and the liquid will be warmer, than if the bottle were an ordinary bottle. So far, so good?

    Now, instead, pour a hot liquid into an ordinary bottle, and somehow (magically) gradually transform the bottle into a thermos bottle. At any given time in this process, the liquid will be warmer, and the outer part of the bottle will be cooler than if the transformation up to that time had not occurred. So far, so good?

    But that doesn’t mean that the liquid is being warmed? Right, it doesn’t, but now supply the liquid with a heater (analogous to UV from the sun). In that case, the liquid will get warmer as the bottle is being transformed, and the outer part will get cooler. When the bottle is ordinary, the heater can’t keep the liquid at so high a temperature as it can keep it at any given point during the transformation.

    Adding CO2 to the atmosphere is like transforming the bottle.

    So I disagree with the idea that there’s a semantics problem. The warming is real, but this doesn’t mean that heat is flowing from a cooler object to a warmer object. The heat is coming from the heater.

    00

  • #
    L.J. Ryan

    Smoking Frog: 451

    If a 90 C probe is inserted to the thermos bottle, what is the maximum achievable temperature, via the probe, of your hot liquid?

    If the Earth’s surface is radiated by 240 W/m^2 (probe), what is the maximum achievable temperature, via this radiation, of Earth’s surface.

    00

  • #
    BLouis79

    @Paul #441″ the only way that it can be finally resolved is by setting up an experiment that can quantify the exchange of energy between a solid and an air-mass containing varying known amounts of Tyndall gases”

    Totally agreee we need experimental verification of whatever phenomenon the climate scientists are thinking of – wish they’d tell us exactly.

    I think the reason the experiment hasn’t been done is more because of the confidence in the present pseudo-scientific approach. It’s pretty hard to do an experiment if one can’t come up with a falsifiable hypothesis of the physical mechanism of the backradiation or radiative insulation theories.

    Found a book by Sharkov on microwave remote sensing of the earth, which describes in much detail the relevant radiative transfer physics. http://www.iki.rssi.ru/asp/pub_sha1/pub_sha1.htm

    Interesting that there is an equation 6.2 Ch6, p210 that describes how blackbodies emit more spectral radiation into a transparent medium than a vacuum by a factor of (refractive index squared). Haven’t managed to find out the refractive index of CO2 to infrared yet…. The refractive indices for visible light in gases are very close to 1.000.

    Ch11 on “selective radiation” is relevant to CO2 molecules in gases, but the physics is fairly dense. In the meantime, can anyone tell me where to find data on the longwave scattering transmittance (or cross section) of a greenhouse effect gas??

    00

  • #

    L.J. Ryan,

    So in both cases equilibrium is reached with Earth at 255K…no need to increase temp.

    So your position is that the temperature of the earth is 255K, as dictated by a solar input of 240 W/m2, and you’re okay with that?

    00

  • #
    KR

    Paul @ 434

    I have a few responses to what you wrote, concerning my post at @292 – I’ll start by thanking you for addressing the content.

    You have assumed gratuitously that the earth is equivalent to a disk with the same surface area as the sphere that it actually is.

    No, I have not. Peak insolation perpendicular to the sun at noon is about 1000 watts, after passage through the atmosphere. I have perhaps assumed to much familiarity with the subject (my apologies if so), but 240 W/m^2 is the average across the surface of the Earth taking into account insolation angles, sphericity of the Earth, and night/day cycles.

    …you have assumed, without any justification, that all energy leaves the surface of the earth by means of radiation.

    Please see @293, where I added what I forgot to explicitly state in @292 – that convection, latent heat, and IR are all involved in energy leaving the surface. The convection and latent heat actually help cool the Earth by overturning the troposphere (weather), allowing the atmosphere to radiate more energy to space than it could otherwise with purely radiative distributions.

    (KR) Now, without the greenhouse effect, dirt, plants, water, and basically all non-metallic surfaces have an emissivity near 1.0 in the temperature range we’re interested in. The average emissivity of the surface of the Earth is ~0.98, and for that emissivity the Earth would radiate 240 W/m^2 at -16.8°C.

    Brrrrrr…

    (Paul) As stated above, this fictitious figure of -16.8°C is purely an artefact of your simplistic assumptions which in no way correspond to the real world.

    I’m well aware that this state is “fictitious”, which is to say not what happens in reality. However, the statement (by some) that no greenhouse effect exists at all, no slowing of energy by GHG’s, would lead directly to this state. I noted this as a reductio ad absurdum – the greenhouse effect is real.

    So are you telling me that the signature temperature of the earth-atmosphere system, when viewed from a satellite, looks like a blackbody with a temperature of 15.2°C? Or are you saying that this is the signature of the surface of the earth when so viewed?

    Absolutely not. The spectra of the Earth-atmosphere system looks like this (top figure). A blackbody radiative curve, the most efficient thermal radiator possible, would look like one of the dotted lines in that figure. The notches in the emission curve show where the emissivity of the Earth/atmosphere is <1.0.

    Backradiation, not too incidentally, is shown in the bottom figure of that link. If the atmosphere did not radiate energy towards the surface, the only thermal spectra we would see would be the 3°K microwave background radiation of outer space. And that energy is received by the surface – a photon is a photon, no ID cards to say where it came from. It’s just that the amount received is considerably less than the total emitted/convected/evaporated, heat still goes from warmer to cooler.

    If the emissive spectra of the Earth looked like a blackbody, we would be a LOT colder.

    And those notches, those inefficiencies in radiation, have an effect on what temperature the Earth needs to be to radiate 240W/m^2 to be in balance. As per @348, in order to radiate 240 W/m^2 as determined by the Stefan-Boltzmann equation:

    Emissivity __ T°K __ T°C
    1.0________ 255 ___ -18 (Perfect blackbody)
    0.98_______ 256 ___ -16.8 (Theoretic no greenhouse effect planet, just Earth surface emissivity)
    0.8________ 270 ___ -3.45
    0.7________ 279 ___ 5.7
    0.614______ 288 ___ 15 (Current measured effective Earth emissivity)
    0.6________ 290 ___ 16.6
    0.5________ 303 ___ 30 (Much sweating!)
    0.1________ 454 ___ 180 (Average goes over 100 at 0.2 emissivity)

    So what happens with greenhouse gases? Some of the radiated energy is absorbed and reradiated back to the surface (effectively not leaving), when it reaches an altitude where the GHG thins enough to radiate to space, the lapse rate cooling with altitude (lower T) means less radiation. Clouds block a lot of IR as well, and clouds cover a significant portion of the globe. You can see that in the top of atmosphere notches, as shown here.

    So you say ‘some’ of the radiated energy is absorbed and re-radiated back to the surface. How much? How have you quantified this supremely important aspect of the entire debate? What known laws of gasses would apply?

    Um, 333 W/m^2 from direct measurements at various locations across the surface of the Earth for longwave radiation?

    If not by measuring, by prediction – in order to exactly predict these values you need to do the math – that’s what line-by-line calculations like MODTRAN and others. I recommend Googling “radiative atmospheric modeling” to see some of these. These are the equivalent of numerical integration under a curve for a problem that does not have a simple formulaic solution.

    The predictions match the measurements (a good support for the atmospheric models) – from those we can estimate what would happen as gas levels change.

    What known laws of gases would apply? Well, all of them, actually, including the absorptive/emissive spectra of all atmospheric gases, known density and distribution, etc.

    Your final remarks about spheres/disks, rotation, etc., are items I believe I addressed at the start of this post.

    Again, in a dynamic energy situation like the climate, with energy coming in and going out, conservation of energy requires that the rate of energy leaving equal the rate of energy arriving in the climate. Energy (net) is always traveling from warmer to colder, so the second law of thermodynamics is not violated by the greenhouse effect.

    Greenhouse gases affect the rate at which the Earth can radiate to space for any given temperature. Additional greenhouse gases reduce that rate and the temperature must rise to regain equilibrium (~3.7 W/m^2 for a doubling of CO2, or ~1.1°C of direct forcing). Any feedbacks (positive or negative, mind you, still some uncertainty there) will scale that number some by inducing additional changes in the rate of energy leaving to space.

    00

  • #
    L.J. Ryan

    Sphaerica (Bob): 454

    Your avoiding my questions. So again Sphaerica (Bob) you said:

    “Greenhouse gases prevent energy from escaping, increasing temperature.”

    My question

    1) What is the source and magnitude of this energy…is it 240 W/m^2?

    00

  • #
    L.J. Ryan

    KR

    Why bother posting if your not willing to defend your position. To the interested reader and so you KR have no excuses for not responding I re-posted our debate starting at 365. KR is block-quoted.

    L.J. Ryan @ 362
    Well, that’s a rather badly worded challenge.

    * Take a plain electric heater with 240 watts coming into it, in a cool room so that it is at equilibrium at 255K. At that point it’s radiating 240 watts into the room.

    * Wrap it in insulation so that it loses energy less efficiently. It’s now radiating something less than 240 watts. What happens? By conservation of energy, that extra (not lost) heat will accumulate in the heater.

    * The heater warms up until 240 watts again exits the insulation, so that input = output. And the heater is now warmer – 288K if we’ve insulated it sufficiently, even hotter if we’ve insulated it even more.

    That’s the core of the radiative greenhouse effect, L.J. The greenhouse gases make the Earth cool less effectively than it would without them. As per my post in @348, with no GHG’s present, the emissivity of the Earth to space (radiation, the only way to lose energy to space) is 0.98, and the temperature would reach equilibrium at -16C.

    With GHG’s we’re at 15C. And rising…

    That’s the physics, L.J., and the temperature. Not a trick, but the reality.

    Indulge me a bit, L.J. Take a look at the top figure in these spectra. A blackbody (emissivity 1.0) will radiate a spectra that looks like one of the smooth lines, such at 240K and 260K. That is the most effective emission theoretically possible at a particular temperature! But the notches in the emission spectra at the top of the atmosphere indicate that the Earth is not radiating like a blackbody – it has a lower emissivity. And hence the temperature required to emit 240 W/m^2 to space has to be higher than 255K.

    If you cannot see the importance of this, if you don’t recognize that 255K is a minimum temperature to radiate 240 W/m^2, then you are simply wrong, and I cannot help you. As Jo and Michael have stated, the greenhouse effect is like insulation, slows the loss of energy to space, and does not violate the 2nd law of thermodynamics.

    (Post 379)

    KR 365

    * Wrap it in insulation so that it loses energy less efficiently. It’s now radiating something less than 240 watts. What happens? By conservation of energy, that extra (not lost) heat will accumulate in the heater.—KR 365

    * The heater warms up until 240 watts again exits the insulation, so that input = output. And the heater is now warmer – 288K if we’ve insulated it sufficiently, even hotter if we’ve insulated it even more.—KR 365

    A self powered heater, fantastic! KR why not insulate the room and warm it to 288K with 255K radiative heater. Though insulating a room with a 255K heat source implies a walk-in freezer, not a warm room. Details, details…I’m sure GHG physics has a well reasoned canard to side step this reality.

    (Post 383)

    L.J. Ryan @ 379

    Well, you’ve just proven that you did not read my post @365.

    * The climate is a dynamic system, with energy flowing into and out of it.

    * You appear to have built a static thought experiment with a 255 K “heater” that does not have any input energy (hardly a heater, rather more of a brick). That’s not a dynamic system, but a fixed system, with no energy flow.

    * In a fixed system, there’s nothing that will raise the temperature except a hotter object. The total joules of energy are fixed.

    * But in a dynamic system, the numbers that must balance are the total energy coming in and the total energy going out. That’s just conservation of energy, the first law of thermodynamics.

    * The output energy (in a radiative system like the Earth in space) is set by temperature and the radiative efficiency of the object.

    I suggest you reread my post @365, and also @348. If you don’t get it then, well, shrug, I’ll leave you be with your opinions. Some people (like Roy @254) get it, great – it’s not worth my while to attempt to pound it into an unwilling head.

    (Post 391)

    KR 383

    Fixed system…you’re sidestepping the issue.

    KR you built a “static” thought experiment with a 255 K “heater”. I simply expanded upon your answer (365). I suspect you don’t like the conditions because they exposes your nonsense.

    Don’t run away KR, answer the question:

    Why not insulate the room and warm it to 288K with 255K radiative heater?

    Any interested readers should note your evasion, and realize you do not feel confident enough to defend your sophistry.

    00

  • #
    KR

    L.J. Ryan @ 457

    This has been a long and active thread, L.J. – I certainly may have misstated or misquoted something. If I have, my apologies.

    To start fresh on this analogy:

    * Start with an efficient heater in a cold room, receiving 240 watts of power. Assume that at equilibrium (heater temperature not changing) that it’s temperature is 255K, and that it’s radiating 240 watts into the room. It’s total internal energy (even though that energy is flowing) is unchanging.

    * Wrap that heater in insulation. The insulation slows the flow of energy to the room, so that it’s no longer radiating 240 watts.

    * The heater will accumulate energy (the difference between incoming and outgoing energy), and it will warm up until it is once again radiating 240 watts to the cold room.

    * With enough insulation the heater will reach 288K…

    * If you want to insulate the room, go ahead – the room will warm up, less energy (the difference between heater thermal radiation and room thermal radiation) will leave the heater, and the heater will warm up too. Of course, if the outside is still cold, the room will emit energy to the outside, the heater will emit to the room, and the heater will still be warmer than the room. So if the room is 288K, with cool walls, the heater must be > 288K. That’s the second law of thermodynamics in action.

    If you want to try this, just measure the temperature of a radiator in a cold room. Then wrap it in a blanket, wait a bit, and measure it’s temperature again.

    Back to the climate. At the top of the atmosphere 240 W/m^2 must leave as thermal radiation in order to equal the incoming 240 W/m^2 of visible light. The most efficient thermal radiator possible (emissivity 1.0) is a blackbody, which has a particular spectra. The Earth’s surface actually averages 0.97-0.98 emissivity; to radiate 240 W/m^2 without a greenhouse effect the Earth would only have to average about -17C!

    But greenhouse gases absorb/emit IR – a great deal of it back to the surface (not leaving), and when it does emit to space it’s high in the atmosphere, where it’s much colder (less radiation). That makes the Earth ~40% less efficient at radiating to space, an emissivity of ~.61 (measured!!!), and leads to the average temperature of ~15C.

    Any interested readers should note your evasion, and realize you do not feel confident enough to defend your sophistry.

    No evasions, no specious arguments, L.J. – just honestly stating what I understand to be the case, based on fundamental physics, repeated measurements, and observations.

    00

  • #

    KR (#455):

    There is a question of the mechanism of “the greenhouse effect.” In post #15, Nullius in Verba argues that the proposed mechanism in which the back radiation retards heat transfer at Earth’s surface: “doesn’t conflict with the second law, but it’s still not quite correct because it ignores convection.” He argues that if the back radiation were to cause the rate of decline of temperature with altitude to exceed the adiabatic lapse rate an instability would arise that would carry away the excess heat from the back radiation by free convection. Thus, he says, the net effect from an increase in the back radiation is “completely cancelled out.” He goes on to suggest that increases in greenhouse gas concentrations do increase Earth’s surface temperature but they do this by raising the altitude from which heat ultimately leaves the Earth thus lowering the effective emitting temperature. Thus, “the greenhouse effect” should not be associated with the back radiation at Earth’s surface but rather with a lowering of the effective emitting temperature in the upper atmosphere. Do you agree with Nullius?

    00

  • #
    KR

    Terry Oldberg @ 459

    That’s an excellent question, Terry. While I don’t have the MODTRAN codes (or more modern, more accurate versions) readily available, it’s my understanding that the rise in emission altitude and corresponding drop in emission temperature is the dominant effect, with band broadening of interception the secondary.

    Band broadening is often misunderstood, but if you think of a bell shaped curve (a combination of Gaussian and Lorentzian distributions) for each emission band, with a saturation clipping at some level, increasing the band with increased GHG’s broadens the band at the clipping point. This widening is a big part of the logarithmic response to CO2 change.

    The lapse rate of the atmosphere is determined by total energy movement, including convection, evaporation, and IR – the tropopause (where weather essentially stops) has been measured to have risen somewhat over the last 40 years as the Earth has warmed.

    So yes, the rise in emission altitude drops emissions to space until the entire atmosphere warms, but that is driven from the surface, which must warm as well. Atmosphere gains energy, surface gains energy, backradiation increases, temperatures go up all around. Top of atmosphere effects are big (the elephant), but from there on it’s turtles all the way down…

    I think a lot of the confusion about the GHE is conveyed in this cartoon. Analogies are imperfect – but often the fastest way to convey an idea. Just don’t get caught on the aspects of the analogy that don’t correspond to the complex subject being explained.

    00

  • #

    KR(#460):
    Thanks for taking the time to respond.

    I think it would be well if professional climatologists were to modify their descriptions of the mechanism which they propose for anthropogenic global warming. This modification should, it seems to me, include retirement of the inappropriate and misleading terms “greenhouse effect” and “greenhouse gases.”

    00

  • #
    KR

    Terry, I tend to agree.

    00

  • #
    Steeptown

    KR@460

    The lapse rate of the atmosphere is determined by total energy movement, including convection, evaporation, and IR

    So you don’t think that the lapse rate has anything to do with gravitational compression of the atmosphere?

    00

  • #
    mkelly

    I see lots of folks talking about back radiation. I find no mention of this in any heat transfer equation. Please indicate where I would enter this “back radiation” in these.

    A simple equation: W/m^2 = e*(SB)*(T1^4-T2^4)

    No back radiation.

    The two minimum requirements for heat transfer are path and temperture gradient. Period.

    Please present your discussions within the confines of accepted heat transfer equations.

    00

    • #
      Jose_X

      You are ignoring radiative transfer equations.

      For a more complete picture, analyze the radiation that is occurring.

      00

  • #

    L.J. Ryan,

    I’m not avoiding your questions, so stop accusing me of doing so. I don’t answer most of them because they appear to be attempts to lead the conversation towards foolishness. If you don’t play games, I will answer you. If you play games, or if your level of discourse drops too low, then I will ignore you.

    I would also point out that you ignored my question, which I felt needed to be answered in order to understand where you were heading with yours (again, I’m not going to play games with you, and if I think you’re playing games, I’ll just stop conversing… not because you “won,” but simply because I’m a human being, and no matter how much time you have on your hands, I consider mine to be valuable to me).

    In answer to your question: the sun supplies 240 W/m2 on average to the earth as a closed system, and at the top of the atmosphere the earth radiates away 240 W/m2. Within that system, however, the surface of the earth experiences 517 W/m2 as a result of the various interactions in the system (reflection, absorption, greenhouse effect, etc.).

    But you already know this, because I already stated it several times in my own posts, and it is also readily available in the literature — which is why I didn’t really bother to answer you the first time.

    Now you can answer my question: Are you comfortable with the temperature of the earth’s surface being 255˚K (since the actual average temperature of the earth’s surface is 288˚K)?

    [wow indignant here on a skeptical web site! and “readily available in the literature” Hmmmmm] ED

    00

  • #
    BLouis79

    @TerryOldberg #459 “raising the altitude from which heat ultimately leaves the Earth thus lowering the effective emitting temperature”

    So I presume that makes a third theory after the “backradiation” and “insulation” theories. If the surface is warmed and convection raises temperature and the tropopause rises, then we have a slightly larger effective radiating sphere and a slightly larger effective absorbing sphere. I assume that albedo and emissivity are the same. If not please clarify. Then do the math according to the relevant formulae and see what you get. (Follow the original Postma calculations.)

    Now I’m wondering about the errors in the existing Postma calculations (and climate scientists) caused by errors in radiating surface area if the “effective blackbody (or greybody) surface is at 5km altitude”. Then we also have to check if refractive effects are significant in making the effective absorbing surface area bigger.

    00

  • #
    L.J. Ryan

    KR 458

    If you want to try this, just measure the temperature of a radiator in a cold room. Then wrap it in a blanket, wait a bit, and measure it’s temperature again.

    KR you’re not serious? I’m assuming you misstated or misquoted something.

    00

  • #
    BobC

    Terry Oldberg: (@428)
    May 19th, 2011 at 5:34 am
    BobC (#421):

    The word “prediction” references a different concept than the word “projection.” Thus, it would be logically impossible for the IPCC to claim predictive skill for its projections.

    What’s your point, Terry?

    The only crisis is a predicted crisis — so if nobody’s predicting a crisis, then what is all the fuss about? Why have a carbon-dioxide tax?

    00

  • #
    KR

    Steeptown @ 463

    So you don’t think that the lapse rate has anything to do with gravitational compression of the atmosphere?

    That has quite a bit to do with it, but given convection, ozone absorption of UV, and other things, the temperature profile looks like this. Simple adiabatic expansion/compression would give a monotonic temperature profile (continuously decreasing with altitude) to the atmosphere.

    mkelly @ 464

    Please indicate where I would enter this “back radiation” in these.

    A simple equation: W/m^2 = e*(SB)*(T1^4-T2^4)

    As the energy coming from T2 – however, since surface and atmosphere are of two different emissivities, the equation should be:

    W/m^2 = e(surface)*SB*T(surface)^4 – e(atmosphere)*SB*T(atmosphere)^4

    As measured, averaged over the surface of the earth, that’s W/m^2 = 396 – 333 = 63 W/m^2

    BLouis79 @ 466

    If the surface is warmed and convection raises temperature and the tropopause rises, then we have a slightly larger effective radiating sphere and a slightly larger effective absorbing sphere.

    No – as GHG concentration increases, the altitude where there are few enough molecules that IR emitted to space can reach it without interception (the “effective emission altitude”) will increase. Given the tropospheric lapse rate, higher altitudes are colder, and hence less energy is emitted from the GHG, emissivity goes down. Then the atmosphere warms up from the bottom (surface) until the IR at the effective emission altitude (altitudes, actually, given multiple GHG’s) increases to balance.

    L.J. Ryan @ 467

    If you want to try this, just measure the temperature of a radiator in a cold room. Then wrap it in a blanket, wait a bit, and measure it’s temperature again.

    KR you’re not serious? I’m assuming you misstated or misquoted something.

    I am quite serious. For the closest approximation to the radiative only transfer at the top of the atmosphere, do this with an IR heater. Measure the heater element with and without insulation and see what happens when heat loss from the element is reduced.

    In the case of the climate the heater (the Earth) is powered by sunlight, rather than electricity – but this is exactly the case I was talking about.

    00

  • #
    Paul

    Without replying specifically to anybody, I am now coming to the realisation that the so-called ‘settled science’ and the only ‘evidence’ for the Greenhouse conjecture can be confined entirely to computer simulations.

    This has been through the reading of a link that I was directed to concerning the history of this conjecture, which, although it ends in the 1980’s, shows only a pursuit of proving by any means possible an effect which was doubted by nearly all those who were conversant with the behaviour of the atmosphere and climate, including the old-guard of scientists involved in this area, most physicists in other fields and the general public.

    In other words, there are no physical experiments that have been performed by which to check the conjecture and instead of physical experiments we have had recourse to computer calculations and simulations to ‘prove’ the conjecture.

    Now there is good reason why physical experiments are needed as a foundation for the scientific method and that is because the real world does not make any assumptions and neither does it ignore anything, no matter how obscure or seemingly insignificant it might be. Not only that, but if others are not able to replicate the experiment then the conclusions drawn from the experiment are not accepted.

    And we all know, or should know, that any program, no matter how sophisticated it might be, embodies certain assumptions and omits matters of which humanity is ignorant. So there is no way that computer simulations can validly take the place of physical experiments. This seems to be the fulcrum on which this debate now turns. The belief or otherwise in the postulated Greenhouse effect hinges on whether or not one accepts the output of computer programs as evidence confirming this hypothesis.

    Hence, I am confirmed in my opinion that the Greenhouse conjecture on the recent warming of the earth, following the minimum of the Little Ice Age, is unverified.

    Paul

    00

  • #
    KR

    Paul @ 470

    Without replying specifically to anybody, I am now coming to the realisation that the so-called ‘settled science’ and the only ‘evidence’ for the Greenhouse conjecture can be confined entirely to computer simulations.

    I cannot disagree with you enough.

    Radiative greenhouse theory started with physics predictions made by Svante Arrhenius (1896) – it’s only become more detailed since then. And the predictions made from the physics have been measured. See page 265 of that link: more warming at the poles, greater warming in winter than summer, more on land than water, more in Northern than Southern hemisphere, less cooling at night – predicted in 1896, and all observed.

    In other words, there are no physical experiments that have been performed by which to check the conjecture…

    Cooling stratosphere (only GHG warming produces this signature) – checked. Increased backradiation – checked. Decreasing IR to space at GHG wavelengths – checked. Increasing global temperatures – checked (ice melt, movement of growth zones, and even a few thermometers). Basic CO2 spectroscopy – checked. Paleo evidence for forcings and sensitivities in the past – checked.

    I could go on, but I believe that’s sufficient to prove the point.

    Computer models are useful to make predictions – what will happen if GHG’s increase, or decrease, etc. But the theory is based on the physics, and the effects have indeed been measured in the “real world”.

    00

  • #
    Paul

    KR: @455
    May 20th, 2011 at 12:07 am

    Thanks for your reply. It is good when the issues are debated rather than the person. I am only interested in clarifying the issues and not in ‘scoring points’.

    However, I see from your response, that I did not word my post accurately enough. When I said, “you assumed …” I ought to have said, “your calculation is based on the assumption …”. Hence your responses, from my perspective, miss the points that I was making.

    1) Averaging the insolation received by the earth-atmosphere system from the sun, as though it were received, uniformly and uninterruptedly, by a disk of the same area as the surface of a globe, does have a significant effect on any subsequent calculations. Any figure calculated with this simplifying assumption is invalid as far as I am concerned.

    2) Again, your calculation took no account of any other modes of heat transport, so the result is fictitious and unreal and entirely worth only of being ignored.

    The much vaunted figure of 33 degrees Celsius that is attributed to the ‘Greenhouse effect’ simply leaves out so much of the real world that it is of no value, as far as I am concerned. How much heat is transferred from the light side of the earth to the dark side of the earth every 24 hours? How much of the overnight temperature of near-earth atmosphere is the result of heat given up by the ground, the ocean or latent heat release due to condensation during the hours of darkness?

    We already know that the surface temperatures of the moon are very different from those on earth and different also from those of a blackbody. So you need to go back to the drawing board and come up with a formula for the surface temperature difference, due to the Greenhouse effect, that takes everything into account and not just attribute any difference from a theoretical blackbody temperature of a disk and the observed temperature of the globe to the presence of Tindall gasses in the atmosphere.

    Your point that if the atmosphere did not radiate back-radiation the only thermal spectrum we would see would be the background the 3°K microwave background radiation of outer space, to my mind, ignores the fact that any temperature in the atmosphere would have that spectrum, not just the temperature of the Tindall gasses.

    So we have a warm atmosphere that shields us, like a blanket, from the cold of outer space when the part of the surface we are on is not exposed to the sun’s radiation. The atmosphere also shields us, like a protective sun-screen, from the warmth of the sun’s radiation during the day. Is the latter a ‘Greenhouse effect’?

    Maybe I am a little hard to convince, but it seems to me that this ‘Greenhouse effect’ has been more than a little oversimplified and is more than a little lacking in scientific rigour. But then again, not so as to slow the IPPC and the Governments of the world in solving the ‘problem’, has it!

    Paul

    00

    • #
      Jose_X

      1) Have you ever heard of energy conservation? This is real physics. You don’t have to know exactly how some energy got there in order to know it is there. That said, the simple model you described is not what the IPCC or academics use in the literature and to make predictions. In order to know whether a simplification goes too far or not you have to analyze it within some context and try to confirm with measurements. Many people (“d….rs”) have made up their minds and don’t care to compute or analyze very much in order to see if their intuition was on or off the money. They also may not run into a convincing argument given that the most convincing arguments require a fair amount of attention to detail/math and trial against accumulated evidence (taking into account that some evidence will always be bogus).

      2) See 1.

      The blackbody is not a theoretical item only. At IR temperatures, soil and water emit between .9 and 1 emissivity. Of course, there is latent heat and convection effects that are also taken into account in complex modelling.

      The atmosphere can have a temperature, but if the molecules are unlikely to radiate (as dictated by quantum mechanical analysis and verified by observation), then we don’t get any spectrum. Spectrum analysis is used all over physics. Gases don’t radiate as solids do. Isolated gas molecules have too few degrees of freedom (complexity) to fill out most of the spectrum.

      The night time “blanket” effect of the atmosphere is due to greenhouse gases. Without them, the “blanket” would not exist. Keep in mind a real blanket is a solid and radiates extensively and rather differently from a gas. N2, O2, and other gases don’t offer a “blanket” to us, but CO2, H2O, and some other ghg do (even though these ghg don’t radiate like solids either).

      Ozone “block” uv radiation through a similar mechanism as ghg absorb lower frequencies, but because this overall ozone effect doesn’t reflect (from our pov) the end game of a greenhouse, it is not called the greenhouse effect.

      The math and physics which are involved in analyzing the atmosphere are well grounded. Many people can have little clue of this math and physics, yet be able to understand or explain the essence of the greenhouse effect.. obviously oversimplified (eg, without any invoking of quantum mechanics or solving complex differential equantions).

      [we don’t permit the use of the “D” word here] ED

      00

  • #
    Smoking Frog

    L.J. Ryan said:

    If a 90 C probe is inserted to the thermos bottle, what is the maximum achievable temperature, via the probe, of your hot liquid?

    90 C. So what?

    If the Earth’s surface is radiated by 240 W/m^2 (probe), what is the maximum achievable temperature, via this radiation, of Earth’s surface.

    About -18 C., I think. So what?

    00

  • #
    cohenite

    KR:

    “mkelly @ 464

    Please indicate where I would enter this “back radiation” in these.

    A simple equation: W/m^2 = e*(SB)*(T1^4-T2^4)
    As the energy coming from T2 – however, since surface and atmosphere are of two different emissivities, the equation should be:

    W/m^2 = e(surface)*SB*T(surface)^4 – e(atmosphere)*SB*T(atmosphere)^4

    As measured, averaged over the surface of the earth, that’s W/m^2 = 396 – 333 = 63 W/m^2”

    1/2 right; because (A + B)^4 ≠ A^4 + B^4, with (A + B)^4 being the bulk e* from the surface and atmosphere, as rendered in your eqn, this will not be a correct representation of the total exchange calculated from location to location, A^4 + B^4, since there will many examples of where the atmosphere is warmer and therefore has a higher emissivity than the surface, such as over ice. In this respect the eqn depicting gross backradiation figures is as flawed as the measure of GAT.

    In respect of your dismissal of pressure being the determinant of the atmospheric temperature profile because that profile shows levels of increasing temperature; the relationship between pressure and temperature is shown by P1V1/T1 = P2V2/T2

    The reversions in temperature with height can be explained by variations in volume, V, and insolation in the UV spectrum from the sun which cause the pressure/temperature decline with height to be changed due to external temperature T1 and T2 effects.

    00

  • #
    Paul

    Radiative greenhouse theory started with physics predictions made by Svante Arrhenius (1896) – it’s only become more detailed since then. And the predictions made from the physics have been measured. See page 265 of that link: more warming at the poles, greater warming in winter than summer, more on land than water, more in Northern than Southern hemisphere, less cooling at night – predicted in 1896, and all observed.

    In other words, there are no physical experiments that have been performed by which to check the conjecture…

    Cooling stratosphere (only GHG warming produces this signature) – checked. Increased backradiation – checked. Decreasing IR to space at GHG wavelengths – checked. Increasing global temperatures – checked (ice melt, movement of growth zones, and even a few thermometers). Basic CO2 spectroscopy – checked. Paleo evidence for forcings and sensitivities in the past – checked.

    I could go on, but I believe that’s sufficient to prove the point.

    Computer models are useful to make predictions – what will happen if GHG’s increase, or decrease, etc. But the theory is based on the physics, and the effects have indeed been measured in the “real world”.

    Oh dear, I will have to learn to choose my words more carefully. I was trying to assert that the matter that this topic is specifically dealing with remains unproven by direct experiment. The things that you refer to are circumstantial evidence and could be produced by something other than the presence of Tindall gasses in the atmosphere.

    Proving things by circumstantial evidence is fraught with dangers as can be seen by the number of people imprisoned for crimes that they did not commit. We’ve had a couple of such cases here in New Zealand in recent weeks and in the latest case it was an over-zealous detective who extracted a ‘confession’ by one of the two men that enabled the jury to conclude, from other circumstantial evidence, that they were guilty beyond reasonable doubt. Fortunately for the two men their mothers both refused to give up on their boys, kept digging for the truth and eventually got them both a full exoneration and compensation for wrongful imprisonment.

    The correspondence of that case with the current Global Warming affair strikes me as being too close for comfort.

    Paul

    00

    • #
      Jose_X

      What direct evidence proves gravity exists?

      We have models (many useful ones of the past were eventually proven insufficient) that allow us to understand and make predictions of some sort.

      A major difference between climate science and other sciences is that the feedback loop of predicting, observing, analyzing, etc, runs slowly.

      Much of the modelling, analysis, etc, in climate science (I say this from my limited experience with it) is based on physics equations and results that have been used successfully in other areas of science.

      00

  • #
    L.J. Ryan

    Sphaerica (Bob): 465

    Within that system, however, the surface of the earth experiences 517 W/m2 as a result of the various interactions in the system (reflection, absorption, greenhouse effect, etc.).

    The colder atmosphere cannot make the warmer surface even warmer. But it is possible for the sun to warm the surface, and the surface to warm the atmosphere, and for the process of warming the atmosphere to include some microscopic (molecular) interactions which do not in and of themselves violate the second law of thermodynamics but do slow the rate of heat loss from the surface in such a way that the surface and atmosphere both achieve higher temperatures than they would in the absence of the greenhouse effect.

    If 240 W/m^2 is the sole radiative input and the various interactions in the system (reflection, absorption, greenhouse effect, etc. ) only slow the rate of heat loss from the surface, how is it the colder atmosphere cannot make the warmer surface even warmer?

    Slowing the rate of loss…in other words insulation. Are you suggesting insulation allows a radiating body to increase it’s own temperature?

    You asked
    “Now you can answer my question: Are you comfortable with the temperature of the earth’s surface being 255˚K (since the actual average temperature of the earth’s surface is 288˚K)?”

    I never said the earth’s actual average temp is 255K. Rather, I am contending solar radiation alone, can only achieve 255K. I am contending, contrary to your self conflicting statements, insulation can NOT increase a radiating bodies temperature. I am contending back radiation is spurious. I am contending non-radiative inputs must be increasing the temperature. However, I am NOT contending earth’s average temp is 255K.

    00

  • #
    L.J. Ryan

    Quick correction to 476.

    I am contending back radiation atmospheric forcing is spurious.

    00

  • #
    KR

    Paul @ 472, cohenite @ 474

    I am truly not making up these figures – 240 W/m^2 insolation, 396 W/m^2 surface IR, 333 W/m^2 backradiation, 18 W/m^2 convection, 80 W/m^2 latent heat, and so on…

    Those are the best estimates of the averages over the entire surface of the Earth, over the course of the year, night and day.

    I recommend you look at Trenberth 2009 and Trenberth 1997 (covers some numbers just summarized in the 2009 paper) – they are really summaries of multiple researchers works.

    Seriously, folks – measuring these numbers has taken a lot of work, by a lot of people. If you wish to disagree with the numbers, take it upon yourself to go to the original papers (lots of references in the Trenberth papers) and check the logic there. Don’t assume that your “common sense” (please, look at the link) is up to the task of determining what is going on in a complex system that you haven’t studied.

    00

  • #
    L.J. Ryan

    Smoking Frog: 473

    About -18 C., I think. So what?

    The “what” is, the maximum temperature by solar radiation alone. Slowing the radiative loss (insulating) -18 C can not, and does not get you to 15 C. Slowing radiative loss, like slowing conductive loss, only prevents temperature change.

    00

  • #
    KR

    Paul @ 475

    I was speaking specifically of the radiative greenhouse effect, not of warming caused by the sun, by ‘natural cycles’, by volcanic activity, meteor strike, or overzealous politicians pumping out hot air.

    The physics of greenhouse gas absorption/emission is well known, testable and tested in the lab. The fingerprints of the GHE – nights warming faster than days (which can only be caused by reduced loss to space at night), cooling stratosphere, winters warming more than summers (all of which contradict solar or volcanic warming), etc – all measured, observed, and match the physics.

    There’s all kinds of evidence, Paul – all pointing to or not contradicting radiative greenhouse warming. To take it to a crime analogy: The accused left fingerprints, was found standing over the body with the gasoline and matches, had a motive, and ran from the cops. Sure, he might be innocent – would you bet on it?

    00

  • #
    KR

    L.J. Ryan @ 479

    Slowing the radiative loss (insulating) -18 C can not, and does not get you to 15 C. Slowing radiative loss, like slowing conductive loss, only prevents temperature change.

    Insulate that heater core yet, L.J.? Wrap it in some aluminum foil to reflect IR back to it?

    I’ll be curious if you do, and what kind of temperature change in the heater core you see.

    00

  • #
    RW

    Paul (RE: 441)

    “The answer to that final question is very simple : the second law of thermodynamics refers to the macro-behaviour of the system, not to the behaviour of a single element of that system. And it applies to radiation just as much as it applies to conduction.

    The second law, in effect says, ‘on average more energy will flow from the warmer to the cooler region’ so that even supposing some photons are emitted from the cooler atmosphere towards the surface of the earth, on average their number will be less than the number of photons emitted from the surface of the earth and absorbed within the cooler atmosphere. On average, the flow of heat will be only in the direction of hot to cold.”

    I think you’re forgetting that the Sun is constantly adding energy to the system. The colder atmosphere is not heating the surface – it’s reducing the rate the incoming energy from the Sun, which is mostly transparent through the atmosphere, can leave the system.

    The GHE in principle is not much different than the interior of a car heating up inside on cold day from sitting out in the Sun. The Sun’s energy is mostly transparent through the windows. It’s then absorbed and re-radiated by the interior car components. The rate at which the energy is entering the interior is faster than the rate the re-radiated energy can leave the interior; therefore, the interior has to heat up. The colder exterior components of the car are not heating the interior – they are slowing heat loss.

    00

  • #
    Mark D.

    KR @471:

    I could go on, but I believe that’s sufficient to prove the point.

    Well you could go on and on and on but it would still be AA.

    The facts ARE that none of the actual measurements fit Svante Arrhenius (1896) and his weak theory.

    00

  • #
    Paul

    Somehow I don’t think that anyone would go from being an alarmist to a sceptic if the science could be demonstrated by physical experiments instead of being propped up by computer model runs as a substitute for real-world experimental data.

    Former “alarmist” scientist says Anthropogenic Global Warming (AGW) based in false science
    Share304
    posted at 10:05 am on May 12, 2011 by Bruce McQuain

    David Evans is a scientist. He has also worked in the heart of the AGW machine. He consulted full-time for the Australian Greenhouse Office (now the Department of Climate Change) from 1999 to 2005, and part-time 2008 to 2010, modelling Australia’s carbon in plants, debris, mulch, soils, and forestry and agricultural products. He has six university degrees, including a PhD in Electrical Engineering from Stanford University. The other day he said:

    “The debate about global warming has reached ridiculous proportions and is full of micro-thin half-truths and misunderstandings. I am a scientist who was on the carbon gravy train, understands the evidence, was once an alarmist, but am now a septic.”

    And with that he begins a demolition of the theories, premises and methods by which the AGW scare has been foisted on the public.

    I think that his description of the propagation of this conjecture as ‘settled science’ as “full of micro-thin half-truths and misunderstandings.” is a complete and succinct summary of the situation.

    In particular, the assumption that more heat from CO2 would increase atmospheric H2O with a consequent amplification of the warming, was demonstrated to be false in the 1980s, yet the models still retain that falsified assumption. Enough said?

    Paul

    00

  • #

    BobC (#468):

    In relation to your question of “Why have a carbon-dioxide tax,” your observation that “nobody is predicting a crisis” is apt. The case for a tax has been made through the use of a kind of double-talk that has confused a non-falsifiable “projection” with a falsifiable “prediction.” The double-talk has made it sound as though the case for a tax is scientific when this case has been unscientific.

    00

  • #
    Mark D.

    Terry Oldberg @ 392:

    Mark D (#382):
    I am unable to decode your message hence unable to reply. Are you saying that the IPCC models make predictions? Are you saying that the models make projections? Are you saying that there is not a meaningful distinction between a projection and a prediction? Please respond.

    Sorry for the delay in my response. Decode as follows: IPCC has made projections. In as much as the general public does not distinguish between “projection” and Prediction”, Either you or the IPCC is leveraging that difference.

    So which is it?

    00

  • #

    mkelly (#464):

    In the equation W/m^2 = e*(SB)*(T1^4-T2^4), the symbol “W/m^2” represents the radiative heat flux. Assume that T1 exceeds T2. The magnitude of the “back radiation” plays a role that is similar to that of e*(SB)*T2^4 in the sense that radiative energy is implied to be transported up a temperature gradient.

    00

  • #

    L.J. Ryan,

    I am contending non-radiative inputs must be increasing the temperature.

    And what exactly are these “non-radiative inputs”? Where does the extra 150 W/m2 — more than half again the input from the sun — actually come from?

    00

  • #

    KR (#471):

    Your claim to a “settled science” is a contradiction in terms.

    00

  • #
    Kevin

    Mr. Hammer, Sorry I did not respond to this sooner, regarding my comment #45 you wrote:

    ” This is not quite right. It is certainly absorbed but then new photons at the same wavelength are emitted due to the temperature of the absorber.”

    With respect the proper sequence is:
    1) the surface of the Earth emits an IR photon
    2) the surface of the Earth cools
    3) the photon travels at the speed of light and is absorbed by a “GHG” and thereafter ceases to exist
    4) the “GHG” warms
    5) the “GHG” emits a “new” photon (note, this photon replaces the photon which was deceased in step #3)
    6) the “GHG” cools
    7) the photon is absorbed by the Earth’s surface and the cycle continues

    I see no source for “new photons”, or “net energy gain”, or “radiative forcings”

    Cheers, Kevin.

    00

  • #
    L.J. Ryan

    KR 469

    I am quite serious. For the closest approximation to the radiative only transfer at the top of the atmosphere, do this with an IR heater. Measure the heater element with and without insulation and see what happens when heat loss from the element is reduced.

    In the case of the climate the heater (the Earth) is powered by sunlight, rather than electricity – but this is exactly the case I was talking about.

    Your interpretation of physics does not bode well for the freezer industry. A freezer set to -18 C will, by your supposition, will increase in to 15 C in short order.

    By your supposition, my geothermal passively cooled basement at 18C should be much much warmer.

    KR, a radiator wrapped in a thermal blanket will NOT increase beyond element temperature. Wrapping a cast iron, water filled will not act as a satellite boiler for a larger heating system.

    I realize you’re “all in” with this foolishness, but KR, GHG physics works only in climate models. ..surely you realize this truth.

    00

  • #
    Paul

    RW:
    May 20th, 2011 at 12:40 pm

    Paul (RE: 441)

    “The answer to that final question is very simple : the second law of thermodynamics refers to the macro-behaviour of the system, not to the behaviour of a single element of that system. And it applies to radiation just as much as it applies to conduction.

    The second law, in effect says, ‘on average more energy will flow from the warmer to the cooler region’ so that even supposing some photons are emitted from the cooler atmosphere towards the surface of the earth, on average their number will be less than the number of photons emitted from the surface of the earth and absorbed within the cooler atmosphere. On average, the flow of heat will be only in the direction of hot to cold.”

    I think you’re forgetting that the Sun is constantly adding energy to the system. The colder atmosphere is not heating the surface – it’s reducing the rate the incoming energy from the Sun, which is mostly transparent through the atmosphere, can leave the system.

    The GHE in principle is not much different than the interior of a car heating up inside on cold day from sitting out in the Sun. The Sun’s energy is mostly transparent through the windows. It’s then absorbed and re-radiated by the interior car components. The rate at which the energy is entering the interior is faster than the rate the re-radiated energy can leave the interior; therefore, the interior has to heat up. The colder exterior components of the car are not heating the interior – they are slowing heat loss.

    So how do CO2 molecules that are spaced at intervals of one for every several thousand molecules of nitrogen or oxygen act as a barrier to the flow of heat through the atmosphere?

    If you say that they do so by radiating some of the long-wave radiation back to the surface and that slows the rate at which it can cool, then I think that your explanation ignores the fact that this will simply increase the radiation from the surface by a similar amount, so that the net radiation from the surface will be restored. The formula for the radiation of a blackbody at a given temperature does not allow for the flow of heat to be reduced by the presence of another body in close proximity, as far as I am aware. It simply expresses the rate at which the body will radiate heat.

    Besides, if you consider the mass of the material that is in thermal conjunction, the earth’s mass is so much greater than that of the entire atmosphere that the amount of heat that is being radiated in each direction is proportionately greater from the earth to the atmosphere and space than anything being radiated from the atmosphere to the earth.

    Imagine, by way of simplification, that you run a hot bath with the doors and windows of the room closed. You will soon notice a decided increase in the temperature of the air and its moisture burden. On the other hand, if you run a cold bath and turn on a fan heater to blow hot air into the room and wait for the bath to warm up, you will be likely to have a problem with hygiene before you get your hot bath. So, to imagine that the atmosphere contains sufficient heat to radiate back to earth and warm the earth by 33 degrees is a bit lacking in credibility, don’t you think?

    I can almost imagine your response to that. You will say, no doubt, that the atmosphere doesn’t heat the earth it just slows the rate at which the earth loses heat, raising the temperature on the surface by those magical 33 degrees. Well even a blanket doesn’t do that without itself warming in the process. So far, the evidence from the tropics is that the atmosphere has not warmed at all, especially also it has not increased in its moisture content as was postulated. Hence, without that warming, the whole conjecture falls flat. No warming of the upper atmosphere in the tropics, no Global Warming, no Climate Change, no nothing, just the normal behaviour one would expect to find with a thermostatically controlled climate.

    And, by the way, the place where the radiation is escaping from the atmosphere is about 6 km above the surface of the earth where the temperature is close to that of a black-body exposed to the heat from the sun at that distance. Is that a co-incidence?

    How did the heat energy arrive at that height? By means of air circulation carrying warmer and moisture-laden air to the top of the troposphere [hence its name]. The moisture condenses into droplets, forming clouds and releasing the latent heat of vaporisation, and then the heat is radiated out to space.

    We know that there is negligible water vapour at that height. In fact there is negligible atmosphere at that height. Most of the mass of the atmosphere exists below that height, so the amount of atmospheric carbon dioxide that is above that height must be negligible also.

    One has to seriously wonder how most of the educated world has been convinced that atmospheric carbon dioxide is so powerfully bad!

    With regard to your illustration of a car gathering heat in the sun, I do not think that that has any connection with what happens in the open with a real atmosphere. True the sun’s rays penetrate the glass of the car and warm the interior. But take the glass away altogether and see if you do not get the same, if reduced, effect? It is not the one-way penetration of short-wave radiation through the glass coupled with the glass stopping the outgoing long-wave radiation that is heating the car. Rather it is the prevention of movement in the air that is trapping the heat, nothing else.

    Paul

    00

  • #

    RW (#482):

    Your hypothesis seems to be that the interior of an agricultural greenhouse warms because the glass is relatively transparent to short wave length electromagnetic radiation and relatively opaque to long wave length electromagnetic radiation. By convention, this hypothesis is called “the greenhouse effect. However, the “greenhouse effect” is refuted by the empirical findings of Wood. Wood’s data suggest that an agricultural greenhouse warms because its glass blocks convective heat transfer. The relative opacity of the glass to long wave length radiation is irrelevant.

    Climatologists have recycled the discredited “greenhouse effect” into an argument for anthropogenic (man made) global warming by drawing a parallel between the glass of an agricultural greenhouse and the “greenhouse gases” in Earth’s atmosphere. Supposedly, like the glass of an agricultural greenhouse the greenhouse gases trap heat by their relative opacity to long wave radiation.

    As an argument for the warming of Earth’s surface, the “greenhouse effect” suffers from a shortcoming that is similar to its shortcoming in the argument for the warming of an agricultural greenhouse by the “greenhouse effect.” If the “back radiation” from the emissions by the greenhouse gases were to increase the rate of decline of the atmospheric temperature with altitude above the adiabatic lapse rate, the air at Earth’s surface would be more buoyant than the air above it; that less buoyant air lay on top of more buoyant air would be an unstable state. A stable state would be restored by a rise in the more buoyant air below and a fall in the less buoyant air above. In this way, the adiabatic lapse rate would be re-established. With the re-establishment of a stable state, the magnitude of the downward energy flux of the “back radiation” would be counteracted by the magnitude of the upward energy flux from the convection. As in the case of an agricultural greenhouse, the effect of the “greenhouse effect” upon the transfer of heat would be nil.

    00

  • #

    Mark D (# 486):

    I report the results of a study that is related to this topic at http://judithcurry.com/2011/02/15/the-principles-of-reasoning-part-iii-logic-and-climatology/ . To summarize my findings, the methodology by which the IPCC reaches its conclusions in AR4 is illogical and unscientific.

    00

  • #
    Paul

    Proper scientific methodology usually requires four steps:

    (1) Observation. Objectivity is very important at this stage.

    (2) The inducement of general hypotheses or possible explanations for what has been observed. Here one must be imaginative yet logical. Occam’s Razor should be considered but need not be strictly applied: Entia non sunt multiplicanda, or as it is usually paraphrased, the simplest hypothesis is the best. Entities should not be multiplied unnecessarily.

    (3) The deduction of corollary assumptions that must be true if the hypothesis is true. Specific testable predictions are made based on the initial hypothesis.

    (4) Testing the hypothesis by investigating and confirming the deduced implications. Observation is repeated and data is gathered with the goal of confirming or falsifying the initial hypothesis.

    Pseudoscience often omits the last two steps above. The boundary between pathological science and outright pseudoscience is not distinct. Both are usually marked by a strenuous objection to allowing others to try to prove one’s fantastic theories to be wrong, while immediately meeting every objection with ad hoc hypotheses, denials of conflicting data, and ad hominem attacks. The importance of peer review is rejected wholesale and criticism is often discounted altogether or at best vaguely addressed.

    Applying that test to the Team, which category do they belong in?

    Paul

    00

  • #
    Bryan

    KR says

    ….”That has quite a bit to do with it, but given convection, ozone absorption of UV, and other things, the temperature profile looks like this. Simple adiabatic expansion/compression would give a monotonic temperature profile (continuously decreasing with altitude) to the atmosphere.”….

    But KR you know perfectly well that 90% of the mass of the atmosphere is in troposphere.
    Above this point convection has stopped and “greenhouse enthusiasts” have never claimed this area as part of the “greenhouse effect”
    The adiabatic lapse rate formula is given by Postma on page 21 – nothing to do with radiation “back” or otherwise.
    Scienceofdoom accepts this.
    Why do you post redundant rubbish about radiating shells what you know don’t exist.
    Are you posting as a propaganda exercise to mislead others?

    00

  • #
    co2isnotevil

    KR,

    You keep referring to CO2 as insulation. The nominal R value of the atmosphere is only about 1.62, including the effects of clouds. This isn’t enough to trap heat for very long, but this is a misleadinganalogy anyway. The GHG effect recirculates energy between the atmosphere and the surface. It’s a flux loop, not an isolated source of flux. In fact everything called back radiation is part of a flux loop where it’s made to seem worse by only counting the half of the cycle whose effect is inure to the benefit of CAGW ‘theory’.

    The intrinsic 3.7 W/m^2 of incremental atmospheric absorption arising from doubling CO2 (you may not have a modern MOTRAN equivalent, but I do) is only about 0.33C. Even if you make the incorrect assumption that the 3.7 W/m^2 of forcing is all directed to the surface, the intrinsic temperature increase is only about 0.7C. The 1.1C you quote already includes the effects of ‘feedback’, which is not really feedback, but the post feedback gain. This is the factor of 1.62 that you multiply the 3.7 W/m^2 of ‘surface forcing’ by to get your 1.1C. You can’t then go and claim there’s some nebulous feedback, positive or negative, further pushing temperatures. To get the 3C predicted by the IPCC, the factor of 1.62 must become 4.3, which is far too large. A reasonable change in gain from longer term feedback effects would be an increase (or decrease) of 5-10%, not an increase of 300%.

    George

    00

  • #
    Smoking Frog

    L.J. Ryan

    The “what” is, the maximum temperature by solar radiation alone. Slowing the radiative loss (insulating) -18 C can not, and does not get you to 15 C. Slowing radiative loss, like slowing conductive loss, only prevents temperature change.

    Even without the greenhouse effect, I don’t think the average temperature would be -18 C. That’s because the earth is rotating, and power is not proportional to temperature. However, this doesn’t contradict the attribution of 33 C. to the greenhouse effect, because the 240 w/m^2 is at top of atmosphere, while the 15 C. and the -18 C are at the surface. Which just brings us back to your challenge, of course. Don’t you agree that the exterior of the thermos bottle radiates less per cm^2 than the (all around) surface of the hot liquid radiates per cm^2?

    00

  • #
    Paul

    Even without the greenhouse effect, I don’t think the average temperature would be -18 C. That’s because the earth is rotating, and power is not proportional to temperature. However, this doesn’t contradict the attribution of 33 C. to the greenhouse effect, …

    That is totally illogical. You can’t attribute a false figure to anything without the attribution being false also. So why try to defend the indefensible?

    Paul

    00

  • #
    co2isnotevil

    Paul,

    The other part of the GHG effect is clouds, which are comprised of water, which is a far more potent GHG than CO2. Clouds increase the albedo of the planet from less than 0.1 to about 0.3. This has a cooling effect which offsets much of the warming. The net suface temperature increase, including the effects of clouds, is only about 18C, not 33C. Again, this is warmists only acknowledging the side of the picture that benefits their pet hypothesis.

    George

    .

    00

  • #
    L.J. Ryan

    Smoking Frog: 499

    Even without the greenhouse effect, I don’t think the average temperature would be -18 C. That’s because the earth is rotating, and power is not proportional to temperature. However, this doesn’t contradict the attribution of 33 C. to the greenhouse effect, because the 240 w/m^2 is at top of atmosphere, while the 15 C. and the -18 C are at the surface.

    I’m not sure what your saying…but 240 W/m^2 is radiation incident of earths surface.

    Don’t you agree that the exterior of the thermos bottle radiates less per cm^2 than the (all around) surface of the hot liquid radiates per cm^2?

    Yes the outer surface of a thermos does radiate less then the contents therein. That, however, does not mean the contents increase in temperature the longer enclosed in the thermos. GHG physics will have you believe the contents heat itself. Try this Smoking Frog, place an illuminate chrome flashlight in your thermos, at what point do you have a megawatt therein? Don’t dismiss analogy…GHG physics would have you believe light increases it’s energy by re-radiation and reflection.

    00

  • #

    This thread is an overflow of the first 500 (or so) comments from this post So make further comments there. Thanks

    00

  • #

    Cold warming warm eh ? Monty Pythons Black Knight rides again !( Or is he just rolled out ?)
    Repeat this slowly, …….cold …. warming ….warm…… If you haven’t cringed with embarrassment, you’ve probably got no shame !

    The last gasp desperate clinging to a “crap from the start ” hypothesis.
    CO2 cant heat anything, unless it is warmed and then conducts or radiates to a colder body, it’s not a source of heat and cold CO2 cant heat a warmer anything!

    This isn’t about science, its politics again. Some sceptics don’t want to further upset the mainstream scientific establishment.

    They think the sceptic movement will “lose credibility”, so they go along with, cold ….. warming ….. warm ?

    Joe Postma has brought a Tank to a fist fight and Joe’s logic shatters the illusion of any respectability in regard to the whole AGW argument and exposes it as the absurdity and scam it truly is.

    (Its cold tonight, so ill rug up in the fridge.)

    00

  • #
    Jose_X

    >>The last gasp desperate clinging to a “crap from the start ” hypothesis.
    CO2 cant heat anything, unless it is warmed

    You are probably right. I took a cold sweater out of my closet the other day but could not get my warm body to stay warm in the cool evening.

    Wait, I did stay warmer than I would have if I had no sweater.

    I get it. The cold CO2, like the cold sweater, doesn’t exactly “warm” the warmer body. It provides a mechanism (mostly convection for the sweater or radiation for the planet in space) where there is a lower loss of (net) heat by the body wearing the insulation.

    I get it!

    So the CO2 is warmed by the hotter earth which was warmed by the even hotter sun. And the net earth+CO2 stays at a higher temperature out in space than it would if there were less CO2 (say replaced with O2).

    I get it.

    Maybe the ” ‘crap from the start’ hypothesis” is not so crappy after all.

    Wow, maybe scientists have a clue about science after all!

    I’m so glad we are here to make sure the scientists know their science.

    00

    • #

      Hose, I notice you don’t try to defend the crappy notion of cold warming heat, you just imply CO2 as an insulator.
      But Hose if this crappy notion needs the jumper analogy and thats about as home spun and cheesy an analogy as you can get, then its doubly crap. (It also is a different issue as delaying cooling and insulating are not the same thing as heating and especially not “cold heating warm” !)
      Tell me how is CO2 like a jumper? A jumper is a physical barrier against convective cooling, so tell me what has an “insulator” (the analogous jumper of any temperature) got to do with the silly notion that a cold object can heat a warmer one? And by back radiation at that !
      A cold object, like a cold jumper, can physically contain heat but that is not the issue.

      An Eskimo igloo can keep you warm too by insulation but it cant make a warm object warmer and thats what this laughable back radiation nonsense proclaims.
      You don’t freeze in an Igloo because the bodies capacity to generate heat is higher than the ices capacity to freeze it but put a warm corpse in there and see what happens and if it really can make the objects inside hotter, it would melt !!!!
      A jumper doesn’t insulate by “back radiation” anymore than Tyndall’s original failed back radiated IR heating a greenhouse hypothesis did.
      As Woods proved, a greenhouse heats by the glass restricting cooling by conduction and convection.

      Atmosphere doesn’t make it hotter during the day, it doesn’t heat the ground, the Sun does, the ground heats the atmosphere but atmosphere slows cooling at night – all of atmosphere acts as insulator to slow cooling.

      If “Greenhouse gases” trapped radiation and back radiated it and miraculously made the source of the heat hotter, why then is the H2O filled tropical atmosphere cooler than humidity less deserts at the same or similar latitudes ? After all H20 is 40x a more potent “greenhouse gas’ and 400 times more plentiful.

      If cool objects can warm other objects, then why cant that be shown in a lab experiment?
      If it were true it must be known to engineers as they would have to factor that phenomenon with its profound implications into millions of applications.
      Can you enlighten please ?

      00

      • #
        Jose_X

        Peter Laux,
        I think you are confusing the view that a cold object can send energy to a warmer object via radiation with the incorrect view that a cold object will allow a warm object to gain heat overall. Cold sends energy in all directions, including to a warm object, but the warm object sends energy in all directions in a higher quantity/rate.

        As for CO2 as insulation, why do you call that a cheezy description?

        >> An Eskimo igloo can keep you warm too by insulation but it cant make a warm object warmer and thats what this laughable back radiation nonsense proclaims.

        “Back radiation” references by climate scientists don’t claim that. Please specify the source that made your claim because they could use some fix-upping. I might send them an email.

        Again, I think you are confusing that a warm object can get radiation that left a cold object with the notion that the warm object would gain heat from this overall exchange. It won’t because the warm object radiates away at a fast rate than it finds radiation from a cold object. Did the article here make the incorrect claim (I can’t remember)?

        00

        • #
          Jose_X

          I replied to the other version of this comment you made elsewhere. Recap: Radiation from sun to earth has different characteristics and interacts differently with the atmosphere than does the radiation from earth to sun. Google “line spectra”, “quantum mechanics”, and “planck distribution” for more insight. Consider taking university courses in these subjects to get someone to spend the time with you on experiments. Finally, not all heat cold cycles, say 100C and 0C exposures, are the same because one cycle that loses slower to the cold side will reach equilibrium at at higher temp than would a cycle that loses faster to the cold side.

          00

  • #
    blouis79

    You mean “greenhouse gases magically delay heat flow from earth to space at night and don’t have any impact of heat flow from sun to space during the day” or some such thing that climate scientists are into?

    Show me experimental proof of any of:
    a. thermalization of IR by IR absorbing/emitting gases
    b. magnitude of time delay of IR passing through IR absorbing/emitting gases compared to IR transparent gases (hours, minutes, seconds, msec, microseonds, nanoseconds)
    c. some other physical mechanism by which climate scientists think the greenhouse works

    Show me experimental proof that changing albedo (or material composition) has any effect at all on equilibrium temperature when exposing the material to alternating heating and cooling cycles.

    What goes up must come down. If it heats faster, it will cool faster. If it heats slower, it will cool slower.

    Hand wave all you want. I want to see a real laboratory experiment.

    The null hypothesis: changing albedo or material composition (without the addition of work) will make no difference.

    On the “snowball earth” hypothesis: if we consider the case of an earth with no sun and no ability to radiate. In such conditions, the earth would be isothermal. Isothermal earth would not be a snowball, it would be rather hotter because of the molten core. It is the ability to radiate and lose heat to space (there is no other mode of heat loss to space of note) that results in the real earth surface including a sun warming it being cooler than an isothermal radiatively insulated earth with no sun.

    Mainstream climate science operates in scientific fairyland. Their major defence is “the atmosphere is so complex we can’t possibly perform any valid experiments on parts of it”.

    00